Download as pdf or txt
Download as pdf or txt
You are on page 1of 432

Deeper Understanding, Faster Calc: SOA MFE

and CAS Exam 3F

Yufeng Guo

June 1, 2008
ii
Contents

Introduction vii

9 Parity and other option relationships 1


9.1 Put-call parity . . . . . . . . . . . . . . . . . . . . . . . . . . . . 1

9.1.1 Option on stocks . . . . . . . . . . . . . . . . . . . . . . . 1


9.1.2 Options on currencies . . . . . . . . . . . . . . . . . . . . 11

9.1.3 Options on bonds . . . . . . . . . . . . . . . . . . . . . . . 12


9.1.4 Generalized parity and exchange options . . . . . . . . . . 13
9.1.5 Comparing options with respect to style, maturity, and

strike . . . . . . . . . . . . . . . . . . . . . . . . . . . . . 19

10 Binomial option pricing: I 35

10.1 One-period binomial model: simple examples . . . . . . . . . . . 35


10.2 General one-period binomial model . . . . . . . . . . . . . . . . . 36

10.2.1 Two or more binomial trees . . . . . . . . . . . . . . . . . 49


10.2.2 Options on stock index . . . . . . . . . . . . . . . . . . . 64
10.2.3 Options on currency . . . . . . . . . . . . . . . . . . . . . 68
10.2.4 Options on futures contracts . . . . . . . . . . . . . . . . 72

11 Binomial option pricing: II 79


11.1 Understanding early exercise . . . . . . . . . . . . . . . . . . . . 79
11.2 Understanding risk-neutral probability . . . . . . . . . . . . . . . 80
11.2.1 Pricing an option using real probabilities . . . . . . . . . 81
11.2.2 Binomial tree and lognormality . . . . . . . . . . . . . . . 88
11.2.3 Estimate stock volatility . . . . . . . . . . . . . . . . . . . 91
11.3 Stocks paying discrete dividends . . . . . . . . . . . . . . . . . . 95
11.3.1 Problems with discrete dividend tree . . . . . . . . . . . . 97

11.3.2 Binomial tree using prepaid forward . . . . . . . . . . . . 99

iii
iv CONTENTS

12 Black-Scholes 105
12.1 Introduction to the Black-Scholes formula . . . . . . . . . . . . . 105
12.1.1 Call and put option price . . . . . . . . . . . . . . . . . . 105
12.1.2 When is the Black-Scholes formula valid? . . . . . . . . . 107
12.2 Applying the formula to other assets . . . . . . . . . . . . . . . . 107
12.2.1 Black-Scholes formula in terms of prepaid forward price . 107
12.2.2 Options on stocks with discrete dividends . . . . . . . . . 108
12.2.3 Options on currencies . . . . . . . . . . . . . . . . . . . . 108
12.2.4 Options on futures . . . . . . . . . . . . . . . . . . . . . . 110
12.3 Option the Greeks . . . . . . . . . . . . . . . . . . . . . . . . . . 110
12.3.1 Delta . . . . . . . . . . . . . . . . . . . . . . . . . . . . . 111
12.3.2 Gamma . . . . . . . . . . . . . . . . . . . . . . . . . . . . 111
12.3.3 Vega . . . . . . . . . . . . . . . . . . . . . . . . . . . . . . 112
12.3.4 Theta . . . . . . . . . . . . . . . . . . . . . . . . . . . . . 112
12.3.5 Rho . . . . . . . . . . . . . . . . . . . . . . . . . . . . . . 112
12.3.6 Psi . . . . . . . . . . . . . . . . . . . . . . . . . . . . . . . 112
12.3.7 Greek measures for a portfolio . . . . . . . . . . . . . . . 112
12.3.8 Option elasticity and volatility . . . . . . . . . . . . . . . 113
12.3.9 Option risk premium and Sharp ratio . . . . . . . . . . . 114
12.3.10 Elasticity and risk premium of a portfolio . . . . . . . . . 115
12.4 Profit diagrams before maturity . . . . . . . . . . . . . . . . . . . 115

12.4.1 Holding period profit . . . . . . . . . . . . . . . . . . . . . 115


12.4.2 Calendar spread . . . . . . . . . . . . . . . . . . . . . . . 118
12.5 Implied volatility . . . . . . . . . . . . . . . . . . . . . . . . . . . 119

12.5.1 Calculate the implied volatility . . . . . . . . . . . . . . . 119


12.5.2 Volatility skew . . . . . . . . . . . . . . . . . . . . . . . . 120

12.5.3 Using implied volatility . . . . . . . . . . . . . . . . . . . 121


12.6 Perpetual American options . . . . . . . . . . . . . . . . . . . . . 121
12.6.1 Perpetual calls and puts . . . . . . . . . . . . . . . . . . . 121
12.6.2 Barrier present values . . . . . . . . . . . . . . . . . . . . 126

13 Market-making and delta-hedging 129


13.1 Delta hedging . . . . . . . . . . . . . . . . . . . . . . . . . . . . . 129
13.2 Examples of Delta hedging . . . . . . . . . . . . . . . . . . . . . 129
13.3 Textbook Table 13.2 . . . . . . . . . . . . . . . . . . . . . . . . . 138
13.4 Textbook Table 13.3 . . . . . . . . . . . . . . . . . . . . . . . . . 140
13.5 Mathematics of Delta hedging . . . . . . . . . . . . . . . . . . . . 141
13.5.1 Delta-Gamma-Theta approximation . . . . . . . . . . . . 141
13.5.2 Understanding the market maker’s profit . . . . . . . . . 142
CONTENTS v

14 Exotic options: I 145


14.1 Asian option (i.e. average options) . . . . . . . . . . . . . . . . . 145
14.1.1 Characteristics . . . . . . . . . . . . . . . . . . . . . . . . 145
14.1.2 Examples . . . . . . . . . . . . . . . . . . . . . . . . . . . 146

14.1.3 Geometric average . . . . . . . . . . . . . . . . . . . . . . 146


14.1.4 Payoff at maturity T . . . . . . . . . . . . . . . . . . . . . 147
14.2 Barrier option . . . . . . . . . . . . . . . . . . . . . . . . . . . . . 147

14.2.1 Knock-in option . . . . . . . . . . . . . . . . . . . . . . . 147


14.2.2 Knock-out option . . . . . . . . . . . . . . . . . . . . . . . 147
14.2.3 Rebate option . . . . . . . . . . . . . . . . . . . . . . . . . 148
14.2.4 Barrier parity . . . . . . . . . . . . . . . . . . . . . . . . . 148
14.2.5 Examples . . . . . . . . . . . . . . . . . . . . . . . . . . . 148
14.3 Compound option . . . . . . . . . . . . . . . . . . . . . . . . . . 149
14.4 Gap option . . . . . . . . . . . . . . . . . . . . . . . . . . . . . . 151
14.4.1 Definition . . . . . . . . . . . . . . . . . . . . . . . . . . . 151
14.4.2 Pricing formula . . . . . . . . . . . . . . . . . . . . . . . . 151
14.4.3 How to memorize the pricing formula . . . . . . . . . . . 151
14.5 Exchange option . . . . . . . . . . . . . . . . . . . . . . . . . . . 152

20 Brownian motion and Ito’s Lemma 155


20.1 Introduction . . . . . . . . . . . . . . . . . . . . . . . . . . . . . . 155
20.1.1 Big picture . . . . . . . . . . . . . . . . . . . . . . . . . . 156
20.2 Brownian motion . . . . . . . . . . . . . . . . . . . . . . . . . . . 157

20.2.1 Stochastic process . . . . . . . . . . . . . . . . . . . . . . 157


20.2.2 Definition of Brownian motion . . . . . . . . . . . . . . . 157

20.2.3 Martingale . . . . . . . . . . . . . . . . . . . . . . . . . . 159


20.2.4 Properties of Brownian motion . . . . . . . . . . . . . . . 160
20.2.5 Arithmetic Brownian motion and Geometric Brownian
motion . . . . . . . . . . . . . . . . . . . . . . . . . . . . . 164
20.2.6 Ornstein-Uhlenbeck process . . . . . . . . . . . . . . . . . 165
20.3 Definition of the stochastic calculus . . . . . . . . . . . . . . . . . 166
20.4 Properties of the stochastic calculus . . . . . . . . . . . . . . . . 172
20.5 Ito’s lemma . . . . . . . . . . . . . . . . . . . . . . . . . . . . . . 173
20.5.1 Multiplication rules . . . . . . . . . . . . . . . . . . . . . 173
20.5.2 Ito’s lemma . . . . . . . . . . . . . . . . . . . . . . . . . . 173
20.6 Geometric Brownian motion revisited . . . . . . . . . . . . . . . 175
20.6.1 Relative importance of drift and noise term . . . . . . . . 175
20.6.2 Correlated Ito processes . . . . . . . . . . . . . . . . . . . 176
20.7 Sharpe ratio . . . . . . . . . . . . . . . . . . . . . . . . . . . . . . 180
20.8 Risk neutral process . . . . . . . . . . . . . . . . . . . . . . . . . 183

20.9 Valuing a claim on S a . . . . . . . . . . . . . . . . . . . . . . . . 183


vi CONTENTS

20.9.1 Process followed by S a . £. . . . ¤. . . . . . . . . . . . . . . 183


20.9.2 Formula for S A (t) and E S A (t) . . . . . . . . . . . . . . 184

20.9.3 Expected return of a claim on S A (t) . . . . . . . . . . . . 185


20.9.4 Lease rate δ ∗ . . . . . . . .£ . . . . ¤ . . . . . . . . . . . . . 185
P
20.9.5 Prepaid forward price F0,T S A (T ) . . . . . . . . . . . . 186
20.9.6 Specific examples . . . . . . . . . . . . . . . . . . . . . . . 186

24 Interest rate models 195


24.1 Market-making and bond pricing . . . . . . . . . . . . . . . . . . 195

24.1.1 Review of duration and convexity . . . . . . . . . . . . . . 195

24.1.2 Interest rate is not so simple . . . . . . . . . . . . . . . . 202

24.1.3 Impossible bond pricing model . . . . . . . . . . . . . . . 203


24.1.4 Equilibrium equation for bonds . . . . . . . . . . . . . . . 206
24.1.5 Delta-Gamma approximation for bonds . . . . . . . . . . 209
24.2 Equilibrium short-rate bond price models . . . . . . . . . . . . . 209
24.2.1 Arithmetic Brownian motion (i.e. Merton model) . . . . . 209

24.2.2 Rendleman-Bartter model . . . . . . . . . . . . . . . . . . 210


24.2.3 Vasicek model . . . . . . . . . . . . . . . . . . . . . . . . 211
24.2.4 CIR model . . . . . . . . . . . . . . . . . . . . . . . . . . 212
24.3 Bond options, caps, and the Black model . . . . . . . . . . . . . 213
24.3.1 Black formula . . . . . . . . . . . . . . . . . . . . . . . . . 213
24.3.2 Interest rate caplet . . . . . . . . . . . . . . . . . . . . . . 217

24.4 Binomial interest rate model . . . . . . . . . . . . . . . . . . . . 217


24.5 Black-Derman-Toy model . . . . . . . . . . . . . . . . . . . . . . 221
Introduction

This study guide is for SOA MFE and CAS Exam 3F. Before you start, make
sure you have the following items:

1. Derivatives Markets, the 2nd edition.


2. Errata of Derivatives Markets. You can download the errata at http://
www.kellogg.northwestern.edu/faculty/mcdonald/htm/typos2e_01.
html.
3. Download the syllabus from the SOA or CAS website.
4. Download the sample MFE problems and solutions from the SOA website.
5. Download the recent SOA MFE and CAS Exam 3 problems.

vii
viii INTRODUCTION
Chapter 9

Parity and other option


relationships

9.1 Put-call parity

9.1.1 Option on stocks


Notation

t=0 Current date (date when an option is sold or bought)


T Option expiration date (maturity date)
S0 Current price of the underlying asset
ST The price of the underlying asset at the option expiration date
K Strike price or exercise price
CEur (K, T ) Premium of a European call option with strike price K and T years to expiration
CEur (K, 0) Premium of a European call option on the expiration date
PEur (K, T ) Premium of a European put option with strike price K and T years to expiration
PEur (K, 0) Premium of a European put option on the expiration date
r The continuously compounded annual risk-free interest rate
δ The continuously compounded annual dividend rate
F0,T Delivery price in a forward contract expiring in T

Put-call parity
The textbook gives the following formula

CEur (K, T ) − PEur (K, T ) = P V0,T (F0,T − K) = e−rT (F0,T − K) (9.1)

The textbook explains the intuition behind Equation 9.1. If we set the
forward price F0,T as the common strike price for both the call and the put

1
2 CHAPTER 9. PARITY AND OTHER OPTION RELATIONSHIPS

(i.e. K = F0,T ) , then CEur (K, T ) − PEur (K, T ) = P V0,T (F0,T − K) = 0.


Buying a call and selling a put with K = F0,T synthetically creates a forward
contract and the premium for a forward contract is zero.
However, often I find that Equation 9.1 is not intuitive at all. In fact, it’s
annoyingly complex and hard to memorize. So I like to rewrite 9.1 as follows:

CEur (K, T ) + P V (K) = PEur (K, T ) + S0 (9.2)

Later I will explain the intuition behind 9.2. First, let’s prove 9.2. The proof
is extremely important.
Suppose at time zero we have two portfolios:

• Portfolio #1 consists of a European call option on a stock and P V (K) =


Ke−rT . P V (K) is the present value of the strike price K and r is the
continuously compounded risk free interest rate per year.

• Portfolio #2 consists of a European put option on the stock and one share
of the stock with current price S0 .

• Both the call and put have the same underlying stock, the same strike price
K, and the same expiration date T . Notice that at time zero Portfolio #1
is worthy CEur (K, T ) + P V (K); Portfolio #2 is worth PEur (K, T ) + S0 .

Since it’s difficult to compare the value of Portfolio #1 and the value of
Portfolio #2 at time zero, let’s compare them at the expiration date T . We’ll
soon see that the two portfolios have the same value at T .

Payoff of Portfolio #1 at the expiration date T


If ST < K If ST ≥ K
Call payoff is max (0, ST − K) 0 ST − K
Payoff of P V (K) K K
Total K ST

If you have P V (K) at t = 0, you’ll have K at T .

Payoff of Portfolio #2 at the expiration date T


If ST < K If ST ≥ K
Put payoff max (0, K − ST ) K − ST 0
Payoff of S0 ST ST
Total K ST

If you have one stock worth S0 at t = 0, you’ll have one stock at T worth
ST .
You see that Portfolio #1 and Portfolio #2 have identical payoffs of max (K, ST )
at T . If ST < K, both portfolios are worth the strike price K; if ST ≥ K, both
9.1. PUT-CALL PARITY 3

portfolios are worth the stock price ST . Since Portfolio #1 and #2 are worth
the same at T , to avoid arbitrage, they must be worth the same at any time
prior to T . Otherwise, anyone can make free money by buying the lower priced
portfolio and selling the higher priced one. So Portfolio #1 and #2 are worth
the same at time zero. Equation 9.2 holds.

I recommend that from this point now you throw Equation 9.1 away and use
Equation 9.2 instead.
How to memorize Equation 9.2:

Tip 9.1.1. Many candidates have trouble memorizing Equation 9.2. For exam-
ple, it’s very easy to write a wrong formula CEur (K, T ) + S0 = PEur (K, T ) +
P V (K). To memorize Equation 9.2, notice that for a call to work, a call must
go hand in hand with the strike price K. When exercising a call option, you
give the call seller both the call certificate and the strike price K. In return, the
call seller gives you one stock. Similarly, for a put to work, a put must go hand
in hand with one stock. When exercising a put, you must give the put seller both
the put certificate and one stock. In return, the put seller gives you the strike
price K.
Tip 9.1.2. Another technique that helps me memorize Equation 9.2 is the
phrase “Check (CK) Please (PS).” At expiration T , CEur (K, 0)+K = PEur (K, 0)+
ST . Discounting this equation to time zero, we get: CEur (K, T ) + P V (K) =
PEur (K, T ) + S0 .

Example 9.1.1. The price of a 6-month 30-strike European call option is 12.22.
The stock price is 35. The continuously compounded risk-free interest rate is 8%
per year. What”s the price of a 6-month 30-strike European put option on the
same stock?
Solution.

K = 30 T = 6/12 = 0.5 S0 = 35 r = 0.08 CEur = 12.22


CEur + P V (K) = PEur + S0
→ 12.22 + 30e−0.08(0.5) = PEur + 35, PEur = 6. 043

Parity if the stock pays discrete dividends

If the stock pays discrete dividend, the parity formula is

CEur (K, T ) + P V (K) = PEur (K, T ) + S0 − P V (Div) (9.3)

This is why we need to subtract the term P V (Div). At expiration, CEur (K, T )+
K = PEur (K, T ) + ST . If we discount ST from T to time zero, we’ll get
4 CHAPTER 9. PARITY AND OTHER OPTION RELATIONSHIPS

S0 − P V (Div). If you have one stock worth S0 at time zero, then during [0, T ],
you’ll receive dividend payments. Then at T , you not only have one share of
stock, you also have the accumulated value of the dividend. To get exactly one
stock at T , you need to have S0 − P V (Div) at time zero.
Discounting this equation back to time zero, we get Equation 9.3
Please note that Equation 9.3 assumes that both the timing and the amount
of each dividend are 100% known in advance.

Example 9.1.2. The price of a 9-month 95-strike European call option is 19.24.
The stock price is 100. The stock pays dividend of $1 in 3 months and $2 in 6
months. The continuously compounded risk-free interest rate is 10% per year.
What”s the price of a 9-month 95-strike European put option on the same stock?

Solution.

T = 9/12 = 0.75 K = 95 S0 = 100 r = 0.1


CEur = 19.24
P V (Div) = e−0.1(3/12) + 2e−0.1(6/12) = 2. 877 8

CEur (K, T ) + P V (K) = PEur (K, T ) + S0 − P V (Div)


19.24 + 95e−0.1(0.75) = PEur (K, T ) + 100 − 2. 877 8, PEur (K, T ) = 10. 25

Example 9.1.3. The price of a 9-month 83-strike European put option is 13.78.
The stock price is 75. The stock pays dividend of $1 in 3 months, $2 in 6 months,
$3 in 9 months, and $4 in 12 months. The continuously compounded risk-free
interest rate is 6% per year. What”s the price of a 9-month 95-strike European
call option on the same stock?

Solution.

T = 9/12 = 0.75 K = 83 S0 = 75 r = 0.06


PEur = 13.78
P V (Div) = e−0.06(3/12) + 2e−0.06(6/12) + 3e−0.06(9/12) = 5. 794

CEur (K, T ) + P V (K) = PEur (K, T ) + S0 − P V (Div)


CEur (K, T ) + 83e−0.06(0.75) = 13.78 + 75 − 5. 794 CEur (K, T ) = 3. 64

Tip 9.1.3. When calculating P V (Div) in Equation 9.3, discard any dividend
paid after the option expiration date. In this example, the $4 is paid in 12
months, which is after the expiration date of the option. This dividend is ignored
when we use Equation 9.3.
9.1. PUT-CALL PARITY 5

Parity if the stock pays continuous dividends

If the stock pays dividends at a continuously compounded rate of δ per year,


the parity formula is:

CEur (K, T ) + P V (K) = PEur (K, T ) + S0 e−δT (9.4)

At expiration, CEur (K, T ) + K = PEur (K, T ) + ST . If we discount ST from


T to time zero, we’ll get S0 e−δT . If you have e−δT share of a stock, by investing
dividends and buying additional stock, you’ll have exactly one stock at T . This
concept is called tailing. Refer to Derivatives Markets Section 5.2 about tailing.

Example 9.1.4. The price of a 6-month 90-strike European put option is 5.54.
The stock price is 110. The stock pays dividend at a continuously compounded
rate of 2% per year. The continuously compounded risk-free interest rate is 6%
per year. What”s the price of a 6-month 90-strike European call option on the
same stock?

Solution.

T = 6/12 = 0.5 K = 90 S0 = 110 r = 0.06 δ = 0.02


PEur = 5.54
CEur (K, T ) + P V (K) = PEur (K, T ) + S0 e−δT
CEur (K, T ) + 90e−0.06(0.5) = 5.54 + 110e−0.02(0.5) CEur (K, T ) = 27. 11

Example 9.1.5. The price of a 3-month 40-strike European call option is 6.57.
The stock price is 44. The stock pays dividend at a continuously compounded
rate of 5% per year. The continuously compounded risk-free interest rate is 8%
per year. What”s the price of a 3-month 40-strike European put option on the
same stock?

Solution.

T = 3/12 = 0.25 K = 40 S0 = 44 r = 0.08 δ = 0.08


CEur = 6.57
CEur (K, T ) + P V (K) = PEur (K, T ) + S0 e−δT
6.57 + 40e−0.08(0.25) = PEur (K, T ) + 44e−0.05(0.25) PEur (K, T ) = 2. 32
6 CHAPTER 9. PARITY AND OTHER OPTION RELATIONSHIPS

Synthetic stock
Rearranging Equation 9.3, we get:

S0 = CEur (K, T ) − PEur (K, T ) + P V (K) + P V (Div) (9.5)


To understand the meaning of Equation 9.5, notice

Symbol Meaning at t = 0
+CEur (K, T ) buy a K-strike call expiring in T
−PEur (K, T ) sell a K-strike put expiring in T
+P V (K) buy a zero-coupon bond that pays K at T
+P V (Div) buy a zero-coupon bond that pays Div at T

Next, notice CEur (K, T ) − PEur (K, T ) + P V (K) is worth ST . At T , you


always receive K from the zero-coupon bond seller. However, the call and put
values depend on whether ST ≥ K.
• If ST ≥ K, the sold put expires worthless; you exercise the call, paying K
and receiving one stock.
• If on the other hand, ST ≤ K, the purchased call expires worthless and the
sold put is exercised against you. You pay the put holder K and receive
one stock from him.
• Either way, if at time zero you buy a call, sell a put, and invest P V (K)
in a zero-coupon bond, then at T you are guaranteed to have one stock.
Once you understand CEur (K, T )−PEur (K, T )+P V (K) is worth one stock
at T , the meaning of Equation 9.5 is obvious: if you buy one stock at time zero,
then at time T , you’ll have one stock worth ST . In addition, you’ll receive the
future value of the dividends due to owning a stock.

Example 9.1.6. The price of a 9-month 52-strike European call option on a


non dividend-paying stock is 33.4420. The price of a 9-month 52-strike European
put option on the same stock is 15.1538. The continuously compounded risk-free
interest rate is 6% per year. How can you synthetically create one stock at time
zero? What’s the price of this synthetically created stock at time zero?
Solution.

The parity for a non-dividend paying stock is


CEur (K, T ) + P V (K) = PEur (K, T ) + S0
Rearranging this equation, we get:
S0 = CEur (K, T ) + P V (K) − PEur (K, T )

To synthetically create the ownership of one stock, you need to do the fol-
lowing at time zero:
9.1. PUT-CALL PARITY 7

• Buy a 9-month 52-strike European call option


• Sell a 9-month 52-strike European put option
• Invest 52e−0.06(0.75) = 49. 711 9 in an account earning risk-free interest
rate (i.e. buying a zero coupon bond that pays 52 in 9 months)

The current price of this synthetically created stock is:


S0 = 33.4420 + 49. 711 9 − 15.1538 = 68

Synthetic T-bill

Rearranging Equation 9.3, we get:

P V (Div) + P V (K) = S0 + PEur (K, T ) − CEur (K, T ) (9.6)


According to Equation 9.6, buying one stock, buying a K-strike put, and

selling a K-strike call synthetically creates a zero coupon bond with a present
value equal to P V (Div) + P V (K).

Creating synthetic T-bill by buying the stock, buying a put, and selling a
call is called a conversion. If we short the stock, buy a call, and sell a put, we
create a short position on T-bill. This is called a reverse conversion.
Example 9.1.7. Your mother-in-law desperately wants to borrow $1000 from
you for one year. She’s willing to pay you 50% interest rate for using your
money for one year. You really want to take her offer and earn 50% interest.
However, state anti-usury laws prohibits any lender from charging an interest
rate equal to or greater than 50%. Since you happen to know the put-call parity,
you decide to synthetically create a loan and circumvent the state anti-usury
law. Explain how you can synthetically create a loan and earn 50% interest.
You want to lend $1000 at time zero and receive $1000 (1.5) = 1500 at T = 1.
To achieve this, at time zero you can

• have your mother-in-law sell you an asset that’s worth $1000


• have your mother-in-law sell you a 1500-strike, 1-year to expiration put
option on the asset
• sell your mother-in-law a 1500-strike, 1-year to expiration call option on
the asset.

Let’s see what’s happens at T = 1.

• If ST ≥ 1500, you exercise the put and sell the asset to your mother-in-law
for 1500
8 CHAPTER 9. PARITY AND OTHER OPTION RELATIONSHIPS

• if ST ≤ 1500, your mother-in-law exercises the call and buys the asset
from you for 1500.

The net effect is that you give your mother-in-law $1000 at time zero and
receive $1500 from her at T = 1.
In this example, you have a long position on the synthetically created T-
bill. This is an example of conversion. In contrast, your mother-in-law has a
short position in the synthetically created T-bill. This is an example of reverse
conversion.

Synthetic call option

Rearranging Equation 9.3, we get:


CEur (K, T ) = PEur (K, T ) + S0 − P V (Div) − P V (K)
Example 9.1.8. The price of a 6-month 75-strike European put option on a
dividend-paying stock is 8.06. The stock price is 80. The continuously com-
pounded risk-free interest rate is 5% per year. The continuously compounded
dividend rate is 3% per year. Explain how you can create a synthetic 6-month
75-strike European call option on the stock. Calculate the premium for such a
synthetic call option.
Solution.

The put-call parity for a stock paying continuous dividend is


CEur (K, T ) + P V (K) = PEur (K, T ) + S0 e−δT

Rearranging this equation, we get:


CEur (K, T ) = PEur (K, T ) + S0 e−δT − P V (K)
= 8.06 + 80e−0.03(0.5) − 75e−0.05(0.5) = 13. 72
This is how to synthetically create a 6-month 75-strike European call
symbol at t = 0
+PEur (K, T ) buy a 6-month 75-strike European put
+S0 e−δT buy e−0.03(0.5) = 0.985 share of stock
−P V (K) sell a bond that pays 75 in 6 months

Let’s see why a call is synthetically created. 0.985 share of stock at t = 0


will grow into one stock if you reinvest the dividend and buy additional share of
stock. If at T = 0.5 the stock price is greater than 75 (i.e. ST > 75), then two
things happen: your purchased put expires worthless; the bond matures and
you need to pay the bond holder K = 75. So the net effect is that if ST > 75
then at T you pay K = 75 and own one stock. This is the same as if ST > 75
you exercise the call, paying K = 75 and receiving one stock.
If ST ≤ 75, then two things will happen. You exercise the put, selling your
stock for K = 75; the bond matures and you pay the bond holder K = 75. So
9.1. PUT-CALL PARITY 9

the net effect is that if ST ≤ 75 you net cash flow is zero and you don’t own a
stock. This is the same as if ST ≤ 75 you do nothing and let your call expire
worthless.

Example 9.1.9. The price of a 9-month 110-strike European put option on


a dividend-paying stock is 42.81. The stock price is 100. The continuously
compounded risk-free interest rate is 8% per year. The stock will pay $1 dividend
in 3 months and $1 dividend in 6 months. Explain how you can create a synthetic
9-month 110-strike European call option on the stock. Calculate the premium
for such a synthetic call option.

CEur (K, T ) = PEur (K, T ) + S0 − P V (Div) − P V (K)


= 42.81 + 100 − e−0.08(0.25) − e−0.08(0.5) − 110e−0.08(0.75) = 37. 27

How to synthetically create a 9-month 110-strike European call


symbol at t = 0
+PEur (K, T ) buy a 9-month 110-strike European put
+S0 buy one share of stock paying 100
−P V (Div) sell Bond #1 that pays $1 in 3 months and $1 in 6 months
−P V (K) sell Bond #2 that pays $110 in 6 months

Let’s see why a synthetic call is created.


If at expiration date ST > 110

• You’ll receive $1 dividend at the end of Month 3 and $1 dividend at the


end of Month 6. You use the dividends to pay off Bond #1

• Your purchased put expires worthless

• You have one stock at T = 0.75

• Your pay K = 110 at T = 0.75

The net effect is that you’ll pay K = 110 at T = 0.75 and own one stock.
This is the same as owning a call option and ST > 110.
If at expiration date ST ≤ 110

• You’ll receive $1 dividend at the end of Month 3 and $1 dividend at the


end of Month 6. You use the dividends to pay off Bond #1

• You have one stock at T = 0.75

• You exercise your put, surrendering one stock and receiving K = 110

• Your pay K = 110 at T = 0.75

The net effect is that you have zero cash left and don’t own one stock. This
is the same as owning a call option and ST ≤ 110.
10 CHAPTER 9. PARITY AND OTHER OPTION RELATIONSHIPS

Synthetic put option


Rearranging Equation 9.3, we get:
PEur (K, T ) = CEur (K, T ) − S0 + P V (Div) + P V (K)

If the stock pays continuous dividend, then

PEur (K, T ) = CEur (K, T ) + P V (K) − S0 e−δT


Example 9.1.10. The price of a 6-month 45-strike European call option on a
dividend-paying stock is 18.62. The stock price is 50. The continuously com-
pounded risk-free interest rate is 6% per year. The stock will pay $1 dividend in
3 months. Explain how you can create a 6-month 45-strike European put option
on the stock. Calculate the premium for such a synthetic put option.

Solution.
PEur (K, T ) = CEur (K, T ) − S0 + P V (Div) + P V (K)
= 18.62 − 50 + e−0.06(0.25) + 45e−0.06(0.5) = 13. 28

How to create a synthetic 6-month 45-strike European put option

symbol at t = 0
+CEur (K, T ) buy a 6-month 45-strike European call
−S0 short sell one share receiving 50 and invest in a savings account
+P V (Div) buy Bond #1 that pays $1 in 3 months
+P V (K) sell Bond #2 that pays $45 in 6 months

This is why CEur (K, T ) − S0 + P V (Div) + P V (K) behaves like a put.


If ST ≥ 45
• (1) You will receive K = 45 at T = 0.5 from Bond #2
• (2) You exercise the call at T = 0.5, paying K = 45 which you get from
(1) and receiving one stock
• (3) at T = 0.5 you give the stock you get from (2) to the broker from
whom you borrowed the stock for short sale
• (4) Bond #1 pays you $1 dividend at the end of Month 3. After receiving
this dividend, you Immediately pay this dividend to the original owner of
the stock you sold short
(2)+(3) will close out your short position on the stock
The net effect is that if ST ≥ 45 you keep the proceeds from the short sale,
which you can use to buy a stock. This is "keeping your asset (i.e. proceeds from
the short sale)." This is the same as if you own a 6-month 45-strike European
put and ST ≥ 45. If you own a 6-month 45-strike European put and ST ≥ 45,
you let the put option expire worthless and you still own a stock.
If ST < 45
9.1. PUT-CALL PARITY 11

• (1) You will receive K = 45 at T = 0.5 from Bond #2

• (2) You let the call expire worthless

• (3) At T = 0.5 you buy a stock from the open market using the proceeds
from the short sale; you give the stock to the broker from whom you
borrowed the stock for short sale

• (4) Bond #1 pays you $1 dividend at the end of Month 3. After receiving
this dividend, you Immediately pay this dividend to the original owner of
the stock you sold short

(3)+(4) will close out your short position on the stock.


The net effect is that if ST ≥ 45 you receive K = 45 and you spent the
proceeds from the short sale. This is "giving up an asset (proceeds from the
short sale) and getting the strike price K."

9.1.2 Options on currencies

The put-call parity when currencies are underlying is

CEur (K, T ) + P V (K) = x0 e−r€ + PEur (K, T ) (9.7)

In Equation 9.7, the underlying asset is 1 euro. The call holder has the
right, at T , to buy the underlying (i.e. 1 euro) by paying a fixed dollar amount
K. The premium of this call option is CEur (K, T ) dollars. Similarly, the put
holder has the right, at T , to sell the underlying (i.e. 1 euro) for a fixed dollar
amount K. The premium of this put option is PEur (K, T ) dollars. x0 is the
price, in dollars, of buying the underlying (i.e. 1 euro) at time zero. r€ is the
continuously compounded euro interest rate per year earned by the underlying
(i.e. 1 euro).
To understand the term x0 e−r€ in 9.7, notice that to have 1€ at T , you need
to have e−r€ € at time zero. Since the cost of buying 1€ at time zero is x0
dollars, the cost of buying e−r€ € at time zero is x0 e−r€ dollars.

Equation 9.7 is very similar to Equation 9.4. If you set S0 = x0 and δ = r€ ,


Equation 9.4 becomes 9.7. This shouldn’t surprise us. Both S0 and x0 refer to
the price of an underlying asset at time zero. S0 is the dollar price of a stock;
x0 is the dollar price of 1 euro. Both δ and r€ measure the continuous rate of
reward earned by an underlying asset.

Tip 9.1.4. How to memorize Equation 9.7. Just memorize Equation 9.4. Next,
set S0 = x0 and δ = r€ .
12 CHAPTER 9. PARITY AND OTHER OPTION RELATIONSHIPS

Tip 9.1.5. When applying Equation 9.7, remember that K, CEur (K, T ), PEur (K, T ),
and x0 are in U.S. dollars. To remember this, assume that you are living in the
U.S. (i.e. US dollar is your home currency); that your goal is to either buy or
sell the underlying asset (i.e.1€) with a fixed dollar amount. Also remember
that r€ is the euro interest rate on euro money.

Example 9.1.11. The current exchange rate is 1€= 1.33 US dollars. The
dollar-denominated 6-month to expiration $1.2-strike European call option on
one euro has a premium $0.1736. The continuously compounded risk-free in-
terest rate on dollars is 6% per year. The continuously compounded risk-free
interest rate on euros is 4% per year. Calculate the premium for the dollar-
denominated 6-month to expiration $1.2-strike European put option on one euro.

Solution.
CEur (K, T ) + P V (K) = PEur (K, T ) + S0 e−δT
K = 1.2 T = 0.5 CEur (K, T ) = $0.1736
r = 0.06 δ = 0.04 S0 = 1.33
0.1736 + 1.2e−0.06(0.5) = PEur (K, T ) + 1.33e−0.04(0.5)

PEur (K, T ) = 0.03 447


Example 9.1.12. The current exchange rate is $1 = 0.78€. The dollar-denominated
9-month to expiration $1-strike European put option on one euro has a premium
$0.0733. The continuously compounded risk-free interest rate on dollars is 7%
per year. The continuously compounded risk-free interest rate on euros is 3% per
year. Calculate the premium for the dollar-denominated 9-month to expiration
$1-strike European call option on one euro.
CEur (K, T ) + P V (K) = PEur (K, T ) + S0 e−δT
K=1 T = 0.5 PEur (K, T ) = 0.0733
1
r = 0.07 δ = 0.03 S0 = = 1. 282 05
0.78
1 −0.03(0.75)
CEur (K, T )+e−0.07(0.75) = 0.0733+ e CEur (K, T ) = $0.3780
0.78

9.1.3 Options on bonds


This is an option contract where the underlying asset is a bond. Other than
having different underlying assets, a stock option and a bond option have no
major differences. You buy a call option on stock if you think the stock price
will go up; you buy a put option if you think the stock price will drop. Similarly,
you buy a call option on a bond if you think that the market interest rate will
go down (hence the price of a bond will go up); you buy a put option on a bond
if you think that the market interest rate will go up (hence the price of the bond
will go down).
9.1. PUT-CALL PARITY 13

The coupon payments in a bond are like discrete dividends in a stock. Using

Equation 9.3, we get the

CEur (K, T ) + P V (K) = PEur (K, T ) + B0 − P V (Coupon) (9.8)


Please note that in Equation 9.8, in the term P V (Coupon), "coupon" refers
to the coupon payments during [0, T ]. In other words, only the coupons made
during the life of the options are used in Equation 9.8. Coupons made after T
are ignored.

Example 9.1.13. A 10-year $1,000 par bond pays 8% annual coupons. The
yield of the bond is equal to the continuously compounded risk-free rate of 6%
per year. A 15-month $1,000-strike European call option on the bond has a
premium $180. Calculate the premium for a 15-month $1,000-strike European
put option on the bond.

Solution.
The annual effective interest rate is i = e0.06 − 1 = 6. 184%

1 − 1.06184−10 ¡ ¢
B0 = 1000 (0.08) a10| +1000v 10 = 1000 (0.08)× +1000 1.06184−10 =
0.06184
1132. 50
We need to be careful about calculating P V (Coupon). The bond matures
in 10 years. There are 10 annual coupons made at t = 1, 2, ..., 10. However,
since the option expires at T = 15/12 = 1. 25, only the coupon paid at t = 1 is
used in Equation 9.8.

P V (Coupon) = e−0.06 (1000) (0.08) = 75. 34


CEur (K, T ) + P V (K) = PEur (K, T ) + B0 − P V (Coupon)
180 + 1000e−0.06(1.25) = PEur (K, T ) + 1132. 50 − 75. 34
PEur (K, T ) = 50. 59

9.1.4 Generalized parity and exchange options


General definition of call and put
Most times, the strike price is a constant and we can easily tell whether an
option is a call or a put. For example, an option that gives you the privilege of
buying one Google stock in 1 year for $35 is a call option; an option that gives
you the privilege of selling one Google stock in 1 year for $35 is a put option.
However, occasionally, the strike price is not a constant and it’s hard to
determine whether an option is a call or put. For example, you purchase an
option that gives you the privilege of receiving one Google stock by surrendering
one Microsoft stock in 1 year. If at T = 1, the price of a Google stock is higher
than that of a Micros0ft stock (i.e. STGoogle >.STMicrosof t ), it’s advantageous for
14 CHAPTER 9. PARITY AND OTHER OPTION RELATIONSHIPS

you to exercise the option. To exercise the call, you buy one Microsoft stock
from the open market for STMicrosof t , give it to the option writer. In return,
the option writer gives you one Google stock worth STGoogle . Your payoff is
STGoogle −.STMicrosof t . If, on the other hand, STGoogle ≤.STMicrosof t , you let your
option expires worthless and your payoff is zero. Is this option a call or a put?

It turns out that this option can be labeled as either a call or a put. If
you view the Google stock as the underlying asset and the Microsoft stock as
the strike asset, then it’s a call option. This option gives you the privilege of
buying, at T = 1, one Google stock by paying one Microsoft stock. If you view
the Microsoft stock as the underlying asset and the Google stock as the strike
asset, then it’s a put option. This option gives you the privilege of selling, at
T = 1, one Microsoft stock for the price of one Google stock.

Definition 9.1.1. An option gives the option holder the privilege, at T , of


surrendering an asset AT and receiving an asset BT (we denote the option as
AT → BT ). This is a call option if we view B as the underlying asset and A as
the strike asset (the option holder has the privilege of buying BT by paying AT ).
This is a put option if we view A as the underlying asset and B as the strike
asset (the option holder has the privilege of selling AT and receiving BT ).

Please note that the payoff of Option AT → BT is max (0, BT − AT ).

Example 9.1.14. An option gives the option holder the privilege, at T = 0.25
(i.e. 3 months later), of buying €1 with $1.25. Explain why this option can be
viewed (perhaps annoyingly) as either a call or a put.

Solution.

This option is $1.25 →€1. The option holder has the privilege, at T = 0.25,
of surrendering $1.25 and receiving €1 (i.e. give $1.25 and get €1).
This is a call option if we view €1 as the underlying asset. The option holder
has the privilege of buying €1 by paying $1.25.
This is also a put option if we view $1.25 as the underlying asset. The option
1
holder has the privilege of selling $1.25 for €1 (i.e. selling $1 for € =€
1.25
0.8).

Example 9.1.15. An option gives the option holder the privilege, at T = 0.25,
of buying one Microsoft stock for $35. Explain why this option can be viewed
(perhaps annoyingly) as either a call or a put.

This option is $35 → 1 M icrosof t stock (give $35 and get 1 Microsoft stock).
If we view the Microsoft stock as the underlying asset, this is a call option. The
option holder has the privilege of buying one Microsoft stock by paying $35.
This is also a put option if we view $35 as the underlying. The option holder
has the privilege of selling $35 for the price of one Microsoft stock.
9.1. PUT-CALL PARITY 15

Example 9.1.16. An option gives the option holder the privilege, at T = 0.25,
of selling one Microsoft stock for $35. Explain why this option can be viewed as
either a call or a put.
This option is 1 M icrosof t stock → $35 (give 1 Microsoft stock and get $35).
If we view the Microsoft stock as the underlying asset, this is a put option. The
option holder has the privilege of selling one Microsoft stock for $35. This is
also a call option if we view $35 as the underlying. The option holder has the
privilege of buying $35 by paying one Microsoft stock.

Generalized put and call parity


(AT → BT )0 + P V (AT ) = (BT → AT )0 + P V (BT ) (9.9)

In Equation 9.9, (AT → BT )0 is the premium paid at t = 0 for the privilege


of giving AT and getting BT at T . Similarly, (BT → AT )0 is the premium paid
at t = 0 for the privilege of giving BT and getting AT at T . P V (AT ) is the
present value of AT . If you have P V (AT ) at time zero and invest it from time
zero to T , you’ll have exactly AT . Similarly, if you have P V (BT ) at time zero
and invest it from time zero to T , you’ll have exactly BT .
This is the proof of Equation 9.9. Suppose we have two portfolios. Portfolio
#1 consists a European option AT → BT and the present value of the asset AT .
Portfolio #2 consists a European option BT → AT and the present value of the
asset BT .
Payoff of Portfolio #1 at T
If AT ≥ BT If AT < BT
Option AT → BT 0 BT − AT
P V (AT ) AT AT
Total AT BT

Payoff of Portfolio #2 at T
If AT ≥ BT If AT < BT
Option BT → AT AT − BT 0
P V (BT ) BT BT
Total AT BT

Portfolio #1 and Portfolio #2 have the identical payoff at T . To avoid


arbitrage, these two portfolios must cost us the same to set up at any time prior
to T . The cost of setting up Portfolio #1 is (AT → BT )0 + P V (AT ). The cost
of setting up Portfolio #2 is (BT → AT )0 +P V (BT ). Hence Equation 9.9 holds.
Example 9.1.17. Stock A currently sells for $30 per share. It doesn’t pay
any dividend. Stock B currently sells for $50 per share. It pays dividend at a
continuously compounded rate of 5% per year. The continuously compounded
risk-free interest rate is 6% per year. A European option gives the option holder
16 CHAPTER 9. PARITY AND OTHER OPTION RELATIONSHIPS

the right to surrender one share of Stock B and receive one share of Stock A at
the end of Year 1. This option currently sells for $8.54. Calculate the premium
for another European option that gives the option holder the right to surrender
one Stock A and receive one Stock B at the end of Year 1.

(AT → BT )0 + P V (AT ) = (BT → AT )0 + P V (BT )


(AT → BT )0 + 30 = 8.54 + 50e−0.05
(AT → BT )0 = 26. 1
Please note that the risk free interest rate 6% is not needed for solving the
problem. In addition, you don’t need to decide whether to call the option "give
Stock B and receive Stock A" or "give Stock A and receive Stock B" as a call or
put. Equation 9.9 holds no matter you call the option AT → BT or BT → AT
a call or put.

Example 9.1.18. Stock A currently sells for $55 per share. It pays dividend of
$1.2 at the end of each quarter. Stock B currently sells for $72 per share. It pays
dividend at a continuously compounded rate of 8% per year. The continuously
compounded risk-free interest rate is 6% per year. A European option gives the
option holder the right to surrender one share of Stock A and receive one share
of Stock B at the end of Year 1. This option currently sells for $27.64. Calculate
the premium for another European option that gives the option holder the right
to surrender one Stock B and receive one Stock A at the end of Year 1.

(AT → BT )0 + P V (AT ) = (BT → AT )0 + P V (BT )


P V (AT ) = A0 − 1.2a4|i = 55 − 1.2a4|i
i is the effective interest rate per quarter.
i = e0.25r − 1 = e0.25(0.06) − 1 = 1. 511
µ 3% ¶
1 − 1.01 511 3−4
P V (AT ) = 55 − 1.2a4|i = 55 − 1.2 = 50. 38
0.01 511 3
−δB T −0.08
P V (BT ) = B0 e = 72e = 66. 46
27.64 + 50. 38 = (BT → AT )0 + 66. 46
(BT → AT )0 = 11. 56

Currency options

Example 9.1.19. Let’s go through the textbook example. Suppose that a 1-year
dollar-denominated call option on €1 with the strike price $0.92 is $0.00337.
The current exchange rate is €1 = $0.9. What’s the premium for a 1-year
1
euro-denominated put option on $1 with strike price € = €1. 087?
0.92
First, let’s walk through the vocabulary. The phrase "dollar-denominated
option" means that both the strike price and the option premium are expressed
in U.S. dollars. Similarly, the phrase "euro-denominated option" means that
both the strike price and the option premium are expressed in euros.
9.1. PUT-CALL PARITY 17

Next, let’s summarize the information using symbols. "dollar-denominated


call option on €1 with the strike price $0.92" is $0.92 →€1. The premium for
this option is $0.00337. This is represented by ($0.92 → €1)0 =$0.0337.
1
"Euro-denominated put option on $1 with strike price € = €1. 087" can
0.92µ ¶
1 1
be represented by $1 →€ . The premium for this option is $1 → €
0.92 0.92 0
Now
µ the solution
¶ should µ be simple. ¶
1 1 1
$1 → € = $1 × 0.92 → € × 0.92
0.92 0 0.92 0.92 0
1 1
= ($0.92 → €1)0 = ×$0.0337
0.92 0.92
1
Since the exchange rate is €1 = $0.9 or $1 =€ , we have:
µ ¶ 0.9
1 1 1 1
$1 → € = ×$0.0337 = × 0.0337 × € = €0.04 07
0.92 0 0.92 0.92 0.9
18 CHAPTER 9. PARITY AND OTHER OPTION RELATIONSHIPS

General formula:
1
The current exchange rate is €1 = $x0 or $1=€
x0

A dollar-denominated $K strike call on €1 has a premium of $a


m
$
( $K → €1)0 = $a or ( $K → €1)0 = a

1
A euro-denominated € strike put on $1 has a premium of €b
K
m
µ ¶ µ ¶€
1 1
$1 → € =€b or $1 → € =b
K 0 K 0

It then follows:

µ ¶€ µ ¶$
1 1 1 $
$1 → € × x0 = $1 → € = × ( $K → €1)0 (9.10)
K 0 K 0 K
µ ¶€
1 1
In Equation 9.10, $1 → € is the euro-cost of "give $1, get € ." Since
K0 K
µ ¶€
1
the exchange rate is €1 = $x0 , $1 → € × x0 is the dollar cost of "give
K 0
1 $
$1, get ." Similarly, ( $K → €1)0 is the dollar-cost of "give $K, get €1."
K
1 1
Equation 9.10 essentially says that the dollar cost of "give $1, get " is of
K K
the dollar cost "give $K, get €1." This should make intuitive sense.
µ ¶
1 1
Tip 9.1.6. The textbook gives you the complex formula C$ (x0 , K, T ) = x0 KPf , ,T .
x0 K
Do not memorize this formula or Equation 9.10. Memorizing complex formulas
is often prone to errors. Just translate options into symbols. Then a simple
solution should emerge. See the next example.
9.1. PUT-CALL PARITY 19

Example 9.1.20. The current exchange rate is €0.9 per dollar. A European
euro-denominated call on 1 dollar with a strike price €0.8 and 6 months to
expiration has a premium €0.0892. Calculate the price of a European dollar-
denominated put option on 1 euro with a strike price $1.25.

Just translate the options into symbols. Then you’ll see a solution.

1
The current exchange rate is €0.9 per dollar. $1 =€0.9 or €1 = $
0.9

Euro-denominated call on 1 dollar with strike price €0.8 has a premium €0.0892
(€0.8 → $1)0 =€0.0892

Calculate the price of a dollar-denominated put on 1 euro with strike price $1.25
(€1 → $1.25)0 = $?
µ ¶
1
(€1 → $1.25)0 = 1.25 × € → $1 = 1.25 × (€0.8 → $1)0
1.25 0
1
= 1.25 × €0.0892 = 1.25 × 0.08928 × $ = $0.124
0.9

9.1.5 Comparing options with respect to style, maturity,

and strike
European vs American options
American options can be exercised at any time up to (and including) the matu-
rity. In contrast, European options can be exercised only at the maturity. Since
we can always convert an American option into a European option by exercising
the American option only at the maturity date, American options are at least
as valuable as an otherwise identical European option.

CAmer (K, T ) ≥ CEur (K, T ) (9.11)

PAmer (K, T ) ≥ PEur (K, T ) (9.12)

Equation 9.11 and Equation 9.12 are not earth-shaking observations. You
shouldn’t have trouble memorizing them.

Maximum and minimum price of a call

1. The price of a call option is always non-negative. CAmer (K, T ) ≥


CEur (K, T ) ≥ 0. Any option (American or European, call or put) is a
privilege with non-negative payoff. The price of a privilege can never be
negative. The worst thing you can do is to throw away the privilege.
20 CHAPTER 9. PARITY AND OTHER OPTION RELATIONSHIPS

2. The price of a call option can’t exceed the current stock price.
S0 ≥ CAmer (K, T ) ≥ CEur (K, T ). The best you can do with a call option
is to own a stock. So a call can’t be worth more than the current stock.

3. The price of a European call option must obey the put call
parity. For a non-dividend paying stock, the parity is CEur (K, T ) =
PEur (K, T ) + S0 − P V (K) ≥ S0 − P V (K)

Combining 1, 2, and 3, we have:


For a non-dividend paying stock

S0 ≥ CAmer (K, T ) ≥ CEur (K, T ) ≥ max [0, S0 − P V (K)] (9.13)

For a discrete-dividend paying stock

S0 ≥ CAmer (K, T ) ≥ CEur (K, T ) ≥ max [0, S0 − P V (Div) − P V (K)] (9.14)

For a continuous-dividend paying stock

£ ¤
S0 ≥ CAmer (K, T ) ≥ CEur (K, T ) ≥ max 0, S0 e−δT − P V (K) (9.15)

Tip 9.1.7. You don’t need to memorize Equation 9.13, 9.14, or 9.15. Just mem-
orize basic ideas behind these formulas and derive the formulas from scratch.

Maximum and minimum price of a put

1. The price of a put option is always non-negative. PAmer (K, T ) ≥


PEur (K, T ) ≥ 0.

2. The price of a European put option can’t exceed the present


value of the strike price. PEur (K, T ) ≤ P V (K). The best you can
do with a European put option is to get the strike price K at T . So a
European put can’t be worth more than the present value of the strike
price.

3. The price of an American put option can’t exceed the strike


price. K ≥ CAmer (K, T ). The best you can do with an American put
option is to exercise it immediately after time zero and receive the strike
price K. So an American put can’t be worth more than the strike price.

4. The price of a European put option must obey the put call
parity. For a non-dividend paying stock, the parity is PEur (K, T ) =
CEur (K, T ) + P V (K) − S0 ≥ P V (K) − S0
9.1. PUT-CALL PARITY 21

Combining 1, 2, and 3, we have:

K ≥ PAmer (K, T ) ≥ PEur (K, T ) ≥ 0 (9.16)


For a non-dividend paying stock

P V (K) ≥ PEur (K, T ) ≥ max [0, P V (K) − S0 ] (9.17)


For a discrete-dividend paying stock

P V (K) ≥ PEur (K, T ) ≥ max [0, P V (K) + P V (Div) − S0 ] (9.18)

For a continuous-dividend paying stock


£ ¤
P V (K) ≥ CEur (K, T ) ≥ max 0, P V (K) − S0 e−δT (9.19)

Tip 9.1.8. You don’t need to memorize Equation 9.16, 9.17, 9.18, or 9.19. Just
memorize the basic ideas behind these formulas and derive the formulas from
scratch.

Early exercise of American options


Suppose an American option is written at time zero. The option expires in date
T . Today’s date is t where 0 ≤ t < T . The stock price at the option expiration
date is ST . Today’s stock price is St . The continuously compounded risk-free
interest rate is r per year.

Proposition 9.1.1. It’s never optimal to exercise an American call early on a


non-dividend paying stock.

If at t you exercise an American call option , your payoff is St − K.


If at t you sell the remaining ¤ get at least CEur (St , K, T ) ≥
£ call option, you’ll
max [0, St − P V (K)] = max 0, St − Ke−r(T −t) , the premium for a European
call option written in t and expiring in T . £ ¤
Clearly, CEur (St , K, T ) ≥ max [0, St − P V (K)] = max 0, St − Ke−r(T −t) >
St − K for r > 0
It’s not optimal to exercise an American call option early if you can sell the
remaining call option.
What if you can’t sell the remaining call option? If you can’t sell the call
option written in t and expiring in T ., you can short sell one stock at t, receiving
St and accumulating to St er(T −t) . Then at T , if ST > K, you exercise your call,
paying K and receiving one stock. Next, you return the stock to the broker.
Your total profit is St er(T −t) − K > St − K for a positive r; if at T , ST < K,
you let your call option expire worthless, purchase a stock in the market, and
22 CHAPTER 9. PARITY AND OTHER OPTION RELATIONSHIPS

return it to the broker. Your profit is St er(T −t) − ST > St er(T −t) − K > St − K
for a positive r.
Intuition behind not exercising American call option early. If you exercise the
call option early at t, you pay the strike price K at t and gain physical possession
of the stock at t. You lose the interest you could have earned during [t, T ] had
you put K in a savings account, yet you gain nothing by physically owning a
stock during [t, T ] since the stock doesn’t pay any dividend. In addition, by
exercising the call option t, you throw away the remaining call option during
[t, T ].

Example 9.1.21. You purchase a 30-strike American call option expiring in 6


months on a non-dividend paying stock. 2 months later the stock reaches a high
price of $90. You are 100% sure that the stock will drop to $10 in 4 months.
You are attempted to exercise the call option right now (i.e. at t = 2/12 = 1/6
year) and receive 90 − 30 = 60 profit. The continuously compounded risk-free
interest rate is 6% per year. Explain why it’s not optimal to exercise the call
early.

Solution. The problem illustrates the pitfall in common thinking "If you I know
for sure that the stock price is going to fall, shouldn’t I exercise the call now and
receive profit right away, rather than wait and let my option expire worthless?"

Suppose indeed the stock price will be $10 at the call expiration date T =
6/12 = 0.5. If you exercise the call early at t = 2/12 = 1/6, you’ll gain
St − K = 90 − 30 = 60, which will accumulate to 60e0.06(4/12) = 61. 212 at
T = 0.5.
Instead of exercising the call early, you can short-sell the stock at t = 2/12 =
1/6. Then you’ll receive 90, which will accumulate to 90e0.06(4/12) = 91. 818 at
T = 0.5. Then at T = 0.5, you purchase a stock from the market for 10 and
return it to the brokerage firm where you borrow the stock for short sale. Your
profit is 91. 818 − 10 = 81. 818, which is the greater than 61. 212 by 81. 818 − 61.
212 = 20. 606.

Proposition 9.1.2. It might be optimal to exercise an American call option


early for a dividend paying stock.

Suppose the stock pays dividend at tD .


Time 0 ... ... tD ... ... T

Pro and con for exercising the call early at tD .

• +. If you exercise the call immediately before tD , you’ll receive dividend


and earn interest during [tD , T ]

• −. You’ll pay the strike price K at tD , losing interest you could have
earned during [tD , T ]
9.1. PUT-CALL PARITY 23

• −. You throw away the remaining call option during [tD , T ]. Had you
waited, you would have the call option during [tD , T ]
However, if the accumulated value of the dividend is big enough, then it can
optimal to exercise the stock at tD .
Proposition 9.1.3. If it’s optimal to exercise an American call early, then the
best time to exercise the call is immediately before the dividend payment.

Here is the proof. Suppose the dividend is paid at tD


Time 0 t1 ... ... tD t2 ... ... T

It’s never optimal to exercise an American call at t1 . If you exercise the call
at t1 instead of tD , you’ll
• lose interest that can be earned on K during [t1 , tD ]
• lose a call option during [t1 , tD ]
• gain nothing (there’s no dividend during [t1 , tD ])
It’s never optimal to exercise an American call at t2 . If you exercise the call
at t2 instead of tD , you’ll
• gain a tiny interest that can be earned during [tD , t2 ] but lose the dividend
that can be earned if the call is exercised at tD
So for a dividend paying stock, if it’s ever worthwhile to exercise an American
call early, you should exercise the call immediately before the dividend payment,
no sooner or later.
Combining these two proposition, we have:
Proposition 9.1.4. It’s only optimal to exercise an American call option either
at maturity or immediately before a dividend payment date. Any other time is
not optimal.
Proposition 9.1.5. It might be optimal to exercise an American put early.
Time 0 t1 ... ... t ... ... T
The pros and cons of exercising an American put at t instead of T
• +. You receive K at t and earn interest during [t, T ]
• −. You lose the remaining put option during [t, T ]. If you wait and
delay exercising the option, you’ll have a put option during [t, T ] and can
decide whether to exercise it or discarding it. This is especially painful if
ST > K. If ST > K and you exercise the put at t, you’ll get K at t, which
accumulates to Ker(T −t) at T . If you wait and ST > K, you’ll let the put
option expire worthless and have ST at T . If ST > Ker(T −t) , then you
lose money by exercising the put at t. So one danger of exercising the put
at t is that the stock might be worth more than K after t.
However, if the interest earned on K during [t, T ] is big enough, it can be
optimal to exercise the put at t instead of T .
24 CHAPTER 9. PARITY AND OTHER OPTION RELATIONSHIPS

Appendix 9.A: Parity bound for American options


Appendix 9.5A (and 9.5B) is not on the syllabus. However, I still recommend
that you study it.
Appendix 9.5A has two important ideas

1. Why the put-call parity doesn’t hold for American options


2. How to calculate the non-arbitrage boundary price for American options

Why the put-call parity doesn’t hold for American options The put-
call parity such as Equation 9.2 holds only for European options. It doesn’t hold
for American options. To understand why, let’s start from European options.
For Equation 9.2 to hold, among other things, the call and the put must be
exercised at the same time. Recall our proof of the put-call parity. At time t
we have two portfolios. Portfolio #1 consists a European call option on a stock
and P V (K), the present value of the strike price K. Portfolio #2 consists a
European put option on the stock and one share of the stock with current price
St . Both the call and put have the same underlying stock, have the same strike
price K, and the same expiration date T . We have found that Portfolio #1 and
Portfolio #2 have an identical payoff of max (K, ST ) at the common exercise
date T . To avoid arbitrage, the two portfolios must cost us the same at time
zero. Hence we have Equation 9.2.

Now suppose the call is exercised at T1 and the put is exercised at T2 where
T1 6= T2 . Portfolio #1 consists of a European call option and P V (K) = Ke−rT1 ;
Portfolio #2 consists an American put and one stock worth S0 . Then at
T1 , Portfolio #1 has a payoff of max (K, ST1 ); Portfolio #2 has a payoff of
max (K, ST2 ). Now the two payoffs differ in timing and the amount. As a
result, we don’t know whether the two portfolios have the same set-up cost.

Now you should understand why Equation 9.2 doesn’t hold for American
options. American options can be exercised at any time up to (and including)
the maturity. Even when an American call and an American put have the same
maturity T , the American call can be exercised at T1 where 0 ≤ T1 ≤ T ; the
American put can be exercised at T1 where 0 ≤ T2 ≤ T . Hence an American
call and an American put can be exercised at different times, they don’t follow
the put-call parity.

Non-arbitrage boundary price for American options For a stock that

pays discrete dividend, the key formula is

S0 − P V (K) ≥ CAme (K, T ) − PAme (K, T ) ≥ S0 − P V (Div) − K (9.20)

Many people find it hard to understand or memorize Equation 9.20. Here is


an intuitive proof without using complex math.
9.1. PUT-CALL PARITY 25

First, we establish the boundary of American options:

CEur (K, T ) + P V (Div) ≥ CAme (K, T ) ≥ CEur (K, T ) (9.21)

PEur (K, T ) + K − P V (K) ≥ PAme (K, T ) ≥ PEur (K, T ) (9.22)

This is why Equation 9.21 holds. Clearly, CAme (K, T ) ≥ CEur (K, T ). This
is because an American call option can always be converted to a European call
option.
To understand why CEur (K, T ) + P V (Div) ≥ CAme (K, T ), consider an
American call option and an otherwise identical European call option. Both
call options have the same underlying stock, the same strike price K, the same
expiration date T . The European call option is currently selling for CEur (K, T ).
How much more can the American call option sell for?
The only advantage of an American option over an otherwise identical Eu-
ropean call option is that the American call option can be exercised early. The
only good reason for exercising an American call early is to get the dividend.
Consequently, the value of an American call option can exceed the value of an
otherwise identical European call option by no more than the present value of
the dividend. So CEur (K, T ) + P V (Div) ≥ CAme (K, T ). A rational person
will pay no more than CEur (K, T ) + P V (Div) to buy the American call option.
So Equation 9.21 holds.

Similarly, an American put is worth at least as much as an otherwise identical


European put.
In addition, the value of an American put exceeds the value of an otherwise
identical European put by no more than K − P V (K). The only advantage of
an American put over an otherwise identical European put is that the American
put can be exercised early. The only good reason for exercising an American
put early is to receive the strike price K immediately at time zero (as opposed
to receiving K at T in a European put) and earn the interest on K from time
zero to T . The maximum interest that can be earned on K during [0, T ] is
K − P V (K) = K − Ke−rT . Consequently, PEur (K, T ) + K − P V (K) ≥
PAme (K, T ). Equation 9.22 holds.

Next, we are ready to prove Equation 9.20. CAme (K, T ) − PAme (K, T )
reaches its minimum value when CEur (K, T ) reaches it minimum value CEur (K, T )
and PAme (K, T ) reaches its maximum value PEur (K, T ) + K − P V (K):
CAme (K, T ) − PAme (K, T ) ≤ CEur (K, T ) − [PEur (K, T ) + K − P V (K)]
From the put-call parity, we have:
CEur (K, T ) + P V (K) = PEur (K, T ) + S0 − P V (Div)
→ CEur (K, T ) = PEur (K, T ) + S0 − P V (Div) − P V (K)
→ CEur (K, T ) − [PEur (K, T ) + K − P V (K)]
= PEur (K, T ) + S0 − P V (Div) − P V (K)
26 CHAPTER 9. PARITY AND OTHER OPTION RELATIONSHIPS

− [PEur (K, T ) + K − P V (K)]


= S0 − P V (Div) − K

→ CAme (K, T ) − PAme (K, T ) ≤ S0 − P V (Div) − K

Similarly, CAme (K, T )−PAme (K, T ) reaches it maximum value when CAme (K, T )
reaches its maximum value and PAme (K, T ) reaches it minimum value.
CAme (K, T ) − PAme (K, T ) ≤ CEur (K, T ) + P V (Div) − PEur (K, T )
CEur (K, T )+P V (Div)−PEur (K, T ) = [PEur (K, T ) + S0 − P V (Div) − P V (K)]
+P V (Div) − PEur (K, T )
= S0 − P V (Div)

Hence S0 − P V (K) ≥ CAme (K, T ) − PAme (K, T ) ≥ S0 − P V (Div) − K.


Equation 9.20 holds.
Tip 9.1.9. If you can’t memorize Equation 9.20, just memorize Equation 9.21,
Equation 9.22, and Equation 9.3. Then can derive Equation 9.20 on the spot.
Example 9.1.22. If the interest rate is zero. Is it ever optimal to exercise an
American put on a stock?
Solution. According to Equation 9.22, if the risk-free interest rate is zero, then
K = P V (K) and PEur (K, T ) ≥ PAme (K, T ) ≥ PEur (K, T ). This gives us
PAme (K, T ) = PEur (K, T ). So it’s never optimal to exercise an American put
early if the interest rate is zero.
Example 9.1.23. Is it ever optimal to exercise an American call on a non-
dividend paying stock?
Solution. According to Equation 9.21, if Div = 0, then CEur (K, T ) ≥ CAme (K, T ) ≥
CEur (K, T ) or CAme (K, T ) = CEur (K, T ). It’s never optimal to exercise the
American call option.
Example 9.1.24. An American call on a non-dividend paying stock with
exercise price 20 and maturity in 5 months is worth 1.5. Suppose that the current
stock price is 1.9 and the risk-free continuously compounded interest rate is 10%
per year. Calculate the non-arbitrage boundary price of the American put option
on the stock with strike price 20 and 5 months to maturity.
Solution.

If you can memorize Equation 9.20, then


S0 − P V (K) ≥ CAme (K, T ) − PAme (K, T ) ≥ S0 − P V (Div) − K

19 − 20e−0.1(5/12) ≥ 1.5 − PAme (K, T ) ≥ 19 − 0 − 20


−0.184 ≥ 1.5 − PAme (K, T ) ≥ −1
1 ≥ PAme (K, T ) − 1.5 ≥ 0.184
9.1. PUT-CALL PARITY 27

1 + 1.5 ≥ PAme (K, T ) ≥ 0.184 + 1.5


2. 5 ≥ PAme (K, T ) ≥ 1. 684
If you can’t memorize Equation 9.20, this is how to solve the problem using
basic reasoning.

The American put is worth at least the otherwise identical European put.
PAme (K, T ) ≥ PEur (K, T )
Using the put-call parity: PEur (K, T ) = CEur (K, T ) + P V (K) − S0
Since the stock doesn’t pay any dividend, CAme (K, T ) = CEur (K, T ) = 1.5
PEur (K, T ) = CEur (K, T ) + P V (K) − S0
= 1.5 + 20e−0.1(5/12) − 19 = 1. 684

So PAme (K, T ) ≥ PEur (K, T ) = 1.684


The value of an American put can exceed the value of an otherwise identical
European put by no more the early-exercise value. Since the only possible reason
to exercise an American put early is to receive K and earn interest K − P V (K)
during [0, T ], so the maximum early-exercise value is
K − P V (K) = 20 − 20e−0.1(5/12) = 0.816
→ PAme (K, T ) ≤ PEur (K, T ) + K − P V (K) = 1.684 + 0.816 = 2. 5

Time to expiration

American option An American option (call or put) has more time to expi-
ration is at least as valuable as an otherwise identical American option with less
time to expiration. If options are on the same stock and T1 > T2 , we have:

CAmer (K, T1 ) > CEur (K, T2 ) (9.23)


PAmer (K, T1 ) > PEur (K, T2 ) (9.24)

European option A European call on a non-dividend paying stock will be at

least as valuable as an otherwise identical European call option with a shorter


time to expiration. This is because for a non-dividend paying stock, an European
call option is worth the same as an otherwise identical American call option.
And an American call option with a longer time to expiration is more valuable
than an otherwise identical American option with a shorter time to expiration.
If both options are on the same non-dividend paying stock and T1 > T2 , we
have:
CEur (K, T1 ) > CEur (K, T2 ) (9.25)

If the stock pays dividend, then a longer-lived European option may be less
valuable than an otherwise identical but shorter-lived European option. The
textbook gives two good examples.
28 CHAPTER 9. PARITY AND OTHER OPTION RELATIONSHIPS

In Example #1, a stock is valuable only because of its dividend. The stock
pays a dividend at the end of Week 2. Once the dividend is paid, the stock dies
and is worth nothing. In this case, if T > 2 weeks, CEur (K, T ) = 0. If if T ≤ 2
weeks, CEur (K, T ) might be worth something depending on how high the strike
price K is.

In Example #2, a put is written on the asset of a bankrupt company. The


asset of the bankrupt company is a constant c. The put is worth P V (c) = ce−rT .
If the risk-free interest rate r > 0, ce−rT will decrease if time to expiration T
increases.

European options when the strike price grows over time Typically,
a call or put has a fixed strike price K. However, there’s nothing to prevent
someone from inventing a European option whose strike price changes over time.
Consider a European option whose strike price grows with the risk-free in-
terest rate. That is, KT = KerT . What can we say about the price of such a
European option?
For a European option with strike price KT = KerT , a longer-lived European
option is at least as valuable as an otherwise identical but shorter-lived European
option.
¡ ¢ ¡ ¢
CEur KerT1 , T1 ≥ CEur KerT2 , T2 if T1 > T2 (9.26)

¡ ¢ ¡ ¢
PEur KerT1 , T1 ≥ PEur KerT2 , T2 if T1 > T2 (9.27)

This is why Equation 9.26 holds. Suppose at time zero we buy two European
calls on the same stock. The first call expires at T1 and has a strike price KerT1 .
The second call expires at T2 and has a strike price KerT2 , where T1 > T2 .
Let’s choose a common time T1 and compare¡ the payoffs of ¢these two calls
at T1 . The payoff of the longer-lived call is max 0, ST1 − KerT1 .
The payoff of the shorter-lived call is calculated as follows. First, we calculate
its payoff at T2 . Next, we accumulate this payoff from ¡ T2 to T1 . rT ¢
The payoff of the shorter-lived
¡ call at T 2¢ is max 0, ST2 − Ke 2 . Next, we
rT2
accumulate this payoff max 0, ST2 − Ke from T2 to T1 .
As we’ll soon see, it’s much harder for a short-lived call to have a positive
payoff at T1 .
The longer-lived call will have a positive payoff at ST1 − KerT1 at T1 if
ST1 > KerT1 ; all else, the payoff is zero.
On the other hand, the shorter-lived call will have a positive payoff ST1 −
KerT1 at T1 only if the following two conditions are met

• ST2 > KerT2

• ST1 > KerT1


9.1. PUT-CALL PARITY 29

If ST2 ≤ KerT2 , the shorter-lived call will expire worthless, leading to zero
payoff at T2 , which accumulates to zero payoff at T1 .
If ST2 > KerT2 , we’ll receive a positive payoff ST2 − KerT2 at T2 . If we
accumulate ST2 − KerT2 from T2 to T1 , ST2 will accumulate to ST1 and KerT2
to KerT2 er(T1 −T2 ) = KerT1 , leading to a total amount ST1 − KerT1 at T1 . The
total payoff amount ST1 − KerT1 is positive if ST1 > KerT1 .

In summary, both calls can reach the common positive payoff ST1 −KerT1 at
T1 . The longer-lived call will reach this payoff if ST1 > KerT1 . The shorter-lived
call will reach this payoff if both ST2 > KerT2 and ST1 > KerT1 . Consequently,
the long-lived call has a better payoff and should be at least as valuable as the
shorter-lived call. Hence Equation 9.26 holds.
If you still have trouble understanding why the longer-lived call has a richer
payoff, you can draw the following payoff table:
The accumulated payoff of the shorter-lived call at T1
If ST1 ≤ KerT1 If ST1 > KerT1
rT2
If ST2 ≤ Ke P ayof f = 0 P ayof f = 0
If ST2 > KerT2 P ayof f = ST1 − KerT1 ≤ 0 P ayof f = ST1 − KerT1 > 0

The payoff of the longer-lived call at T1


If ST1 ≤ KerT1 If ST1 > KerT1
rT2
If ST2 ≤ Ke P ayof f = 0 P ayof f = ST1 − KerT1 > 0
rT2
If ST2 > Ke P ayof f = 0 P ayof f = ST1 − KerT1 > 0

You can see that the longer-lived call has a slightly better payoff than the
shorter-lived payoff. To avoid arbitrage, the longer-lived call can’t sell for less
than the shorter-lived call. Hence Equation 9.26 holds. Similarly, you can prove
Equation 9.27.

Different strike price

Proposition 9.1.6. A call (European or American) with a low strike price


is at least as valuable as an otherwise identical call with a higher strike price.
However, the excess premium shouldn’t exceed the excess strike price.

0 ≤ C (K1 , T ) − C (K2 , T ) ≤ K2 − K1 if K1 < K2 (9.28)


Equation 9.28 should make intuitive sense. The lower-strike call allows the
call holder to buy the underlying asset by the guaranteed lower strike price.
Clearly, the payoff of a lower-strike call can never be less than the payoff of
an otherwise identical but higher-strike call. Consequently, 0 ≤ C (K1 , T ) −
C (K2 , T ). And this is why C (K1 , T ) − C (K2 , T ) ≤ K2 − K1 . The only advan-
tage of a K1 -strike call over the K2 -strike call is that the guaranteed purchase
price of the underlying asset is K2 − K1 lower in the K1 -strike call; to buy the
asset, the K1 -strike call holder can pay K1 yet the K2 -strike call holder will pay
30 CHAPTER 9. PARITY AND OTHER OPTION RELATIONSHIPS

K2 . Consequently, no rational person will pay more than C (K2 , T ) + K2 − K1


to buy the K1 -strike call.

Proposition 9.1.7. A European call with a low strike price is at least as valu-
able as an otherwise identical call with a higher strike price. However, the excess
premium shouldn’t exceed the present value of the excess strike price.

0 ≤ CEur (K1 , T ) − CEur (K2 , T ) ≤ P V (K2 − K1 ) if K1 < K2 (9.29)

The only advantage of a K1 -strike European call over the K2 -strike European
call is that the guaranteed purchase price of the underlying asset is K2 − K1
lower in the K1 -strike call at T . Consequently, no rational person will pay more
than CEur (K2 , T ) + P V (K2 − K1 ) to buy the K1 -strike call.
Please note that CEur (K1 , T )−CEur (K2 , T ) ≤ P V (K2 − K1 ) doesn’t apply
to American call options because two American options can be exercised at
different dates.

Proposition 9.1.8. A put (European or American) with a higher strike price


is at least as valuable as an otherwise identical put with a lower strike price.
However, the excess premium shouldn’t exceed the excess strike price.

0 ≤ P (K2 , T ) − P (K1 , T ) ≤ K2 − K1 if K1 < K2 (9.30)

Clearly, a higher strike put is at least as valuable as an otherwise identical put


with a lower strike price. Since the only advantage of a K2 -strike put over the
K1 -strike put is that the guaranteed sales price of the underlying asset is K2 −K1
high in the K2 -strike put, no rational person will pay C (K1 , T ) + K2 − K1 to
buy the K2 -strike put.

Proposition 9.1.9. A European put with a higher strike price is at least as


valuable as an otherwise identical put with a lower strike price. However, the
excess premium shouldn’t exceed the present value of the excess strike price.

0 ≤ PEur (K2 , T ) − PEur (K1 , T ) ≤ P V (K2 − K1 ) if K1 < K2 (9.31)

Please note that PEur (K2 , T ) − PEur (K1 , T ) ≤ P V (K2 − K1 ) won’t apply
to two American put options because they can be exercised at two different
dates.

Proposition 9.1.10. A diversified option portfolio is at least as valuable as one


undiversified option. For K1 < K2 and 0 < λ < 1
9.1. PUT-CALL PARITY 31

C [λK1 + (1 − λ) K2 ] ≤ λC (K1 ) + (1 − λ) C (K2 ) (9.32)

P [λK1 + (1 − λ) K2 ] ≤ λP (K1 ) + (1 − λ) P (K2 ) (9.33)

Please note that Equation 9.32 and Equation 9.33 apply to both European
options and American options.
A call portfolio consists of λ portion of K1 -strike call and (1 − λ) portion
of K2 -strike call. The premium of this portfolio, λC (K1 ) + (1 − λ) C (K2 ),
can be no less than the premium of a single call with a strike price λK1 +
(1 − λ) K2 . Similarly, a call portfolio consists of λ portion of K1 -strike put and
(1 − λ) portion of K2 -strike put. The premium of this portfolio, λP (K1 ) +
(1 − λ) P (K2 ), can be no less than the premium of a single call with a strike
price λK1 + (1 − λ) K2 .
Before proving Equation 9.32, let’s look at an example.
Example 9.1.25. (Textbook example 9.8 K2 and K3 switched) K1 = 50, K2 =
65, K3 = 0.4 (50) + 0.6 (65) = 59. C (K1 , T ) = 14, C (K2 , T ) = 5. Explain why
C (59) ≤ 0.4C (50) + 0.6C (65).
Payoff ST < 50 50 ≤ ST < 59 59 ≤ ST < 65 ST ≥ 65
59-strike call (a) 0 0 ST − 59 ST − 59

50-strike call (b) 0 ST − 50 ST − 50 ST − 50


65-strike call (c) 0 0 0 ST − 65
0.4b + 0.6c 0 0.4 (ST − 50) 0.4 (ST − 50) ST − 591

(0.4b + 0.6c) − a 0 0.4 (ST − 50) ≥ 0 0.6 (65 − ST ) > 02 0

The above table says the following:


If we buy 0.4 unit of 50-strike call, buy 0.6 unit of 65-strike call, and sell
1 unit of 59-strike call, our initial cost is (0.4b + 0.6c) − a and our payoff at
T is always non-negative. To avoid arbitrage, the position of always having a
non-negative payoff at expiration T surely has a non-negative cost at t = 0.
Imagine what happens otherwise. For example, you always have a non-negative
payoff at T and it costs you −$10 (i.e. you receive $10) to set up this position
at t = 0. Then you’ll make at least $10 free money.

Hence we have (0.4b + 0.6c) − a ≥ 0 no matter what ST is.


Clearly, the call portfolio consisting of 40% 50-strike call and 60% 65-strike
call is at least as good as the 59-strike call. Consequently, the portfolio is at
least as valuable as the 59-strike call. C (59) ≤ 0.4C (50) + 0.6C (65) .
1 0.4 (S − 50) + 0.6 (ST − 65) = ST − 59
T
2 0.4 (S − 50) − (ST − 59) = 0.6 (65 − ST )
T
32 CHAPTER 9. PARITY AND OTHER OPTION RELATIONSHIPS

Example 9.1.26. Prove Equation 9.32.

Let K3 = λK1 + (1 − λ) K2 . Clearly, K1 < K3 < K2


Payoff ST < K1 K1 ≤ ST < K3 K3 ≤ ST < K2 ST ≥ K2
K3 -strike call (a) 0 0 ST − K3 ST − K3

K1 -strike call (b) 0 ST − K1 ST − K1 ST − K1


K2 -strike call (c) 0 0 0 ST − K2
λb + (1 − λ) c 0 λ (ST − K1 ) λ (ST − K1 ) ST − K3

λb + (1 − λ) c − a 0 λ (ST − K1 ) ≥ 0 (1 − λ) (K2 − ST ) > 0 0

Please note
λ (ST − K1 ) + (1 − λ) (ST − K2 ) = ST − [λK1 + (1 − λ) K2 ] = ST − K3
In addition,
λ (ST − K1 ) − (ST − K3 ) = K3 − λK1 − (1 − λ) ST
= λK1 + (1 − λ) K2 − λK1 − (1 − λ) ST = (1 − λ) K2 − (1 − λ) ST
= (1 − λ) (K2 − ST )
If we buy λ unit of K1 -strike call, buy (1 − λ) unit of K2 call, and sell 1 unit

of K3 = λK1 + (1 − λ) K2 strike call, we’ll have a non-negative payoff at T . To


avoid arbitrage, the initial cost must be non-negative. Hence λb+(1 − λ) c−a >
0.
Anyway, the call portfolio consisting of λ portion of K1 -strike call and 1 − λ
portion of K2 -strike call is at least as good as the call with the strike price
K3 = λK1 + (1 − λ) K2 . To avoid arbitrage, C [λK1 + (1 − λ) K2 ] ≤ λC (K1 ) +
(1 − λ) C (K2 ).

Example 9.1.27. (Textbook example 9.9 K2 and K3 switched) K1 = 50, K2 =


70, K3 = 0.75 (50) + 0.25 (70) = 55. P (K1 , T ) = 4, P (K2 , T ) = 16. Explain
why P (55) ≤ 0.75P (50) + 0.25P (70).

Payoff ST < 50 50 ≤ ST < 55 55 ≤ ST < 70 ST ≥ 70


55-strike put (a) 55 − ST 55 − ST 0 0

50-strike put (b) 50 − ST 0 0 0


70-strike put (c) 70 − ST 70 − ST 70 − ST 0
0.75b + 0.25c 55 − ST 0.25 (70 − ST ) 0.25 (70 − ST ) 0

(0.75b + 0.25c) − a 0 0.75 (ST − 50) > 0 0.25 (70 − ST ) > 03 0

Please note
0.75 (50 − ST ) + 0.25 (70 − ST ) = 55 − ST
0.25 (70 − ST ) − (55 − ST ) = 0.75 (ST − 50)
3 0.4 (S − 50) − (ST − 59) = 0.6 (65 − ST )
T
9.1. PUT-CALL PARITY 33

(0.75b + 0.25c) − a ≥ 0 no matter what ST is. Clearly, the put portfolio


consisting of 75% 50-strike put and 25% 70-strike put is at least as good as
the 55-strike put. Consequently, to avoid arbitrage, the portfolio is at least as
valuable as the 55-strike put. P (55) ≤ 0.75P (50) + 0.25P (70).

Example 9.1.28. Prove Equation 9.33.


Let K3 = λK1 + (1 − λ) K2 . Clearly, K1 < K3 < K2
Payoff ST < K1 K1 ≤ ST < K3 K3 ≤ ST < K2 ST ≥ K2
K3 -strike put (a) K3 − ST K3 − ST 0 0

K1 -strike put (b) K1 − ST 0 0 0


K2 -strike put (c) K2 − ST K2 − ST K2 − ST 0
λb + (1 − λ) c K3 − ST (1 − λ) (K2 − ST ) λ (K2 − ST ) ST − K3

λb + (1 − λ) c − a 0 λ (ST − K1 ) ≥ 0 λ (K2 − ST ) > 0 0

Please note
λ (K1 − ST ) + (1 − λ) (K2 − ST ) = K3 − ST
In addition,

(1 − λ) (K2 − ST ) − (K3 − ST )
= (1 − λ) (K2 − ST ) − λK1 − (1 − λ) K2 + ST = λ (ST − K1 )
λ (ST − K1 ) − (ST − K3 ) = K3 − λK1 − (1 − λ) ST
= λK1 + (1 − λ) K2 − λK1 − (1 − λ) ST = (1 − λ) K2 − (1 − λ) ST
= (1 − λ) (K2 − ST )
The payoff is always non-negative. Consequently, the call portfolio consisting

of λ portion of K1 -strike put and 1 − λ portion of K2 -strike put is at least as


good as the put with the strike price K3 = λK1 +(1 − λ) K2 . To avoid arbitrage,
P [λK1 + (1 − λ) K2 ] ≤ λP (K1 ) + (1 − λ) P (K2 ).

Exercise and moneyness


Proposition 9.1.11. If it’s optimal to exercise an option, it’s also optimal to
exercise an otherwise identical option that’s more in-the-money.
This is just common sense. The textbook gives an example. Suppose it’s
optimal to exercise a 50-strike American call on a dividend paying stock. The
current stock price is 70. If it’s optimal to exercise the American a 50-strike
American call, then it must also be optimal to exercise an otherwise identical
call but with a strike price 40.
34 CHAPTER 9. PARITY AND OTHER OPTION RELATIONSHIPS
Chapter 10

Binomial option pricing: I

This chapter is one of the easiest chapters in Derivatives Markets. The textbook
did a good job explaining the mechanics of how to calculate the option price in
a one-period binomial model. Besides learning the mechanics of option pricing,
you should focus on understanding two basic ideas: the non-arbitrage pricing
and the risk-neutral probabilities.

10.1 One-period binomial model: simple exam-


ples
Example 10.1.1. Suppose we want to find the price of a 12-month European
call option on a stock with strike price $15. The stock currently sells for $20. In
12 months, the stock can either go up to $30 or go down to $10. The continuously
compounded risk-free interest rate per year is 10%.
30 15
20 ?
10 0
Time 0 T =1 Time 0 T =1

Stock price binomial tree Call payoff binomial tree

It’s hard to directly calculate the price of the call option. So let’s build
something that behaves like a call, something that has the same payoff pattern
as the call. Suppose at time zero we create a portfolio by buying X stocks and
putting Y dollars in a savings account. We want this portfolio to have the exact
payoff as the call.

30X Y e0.1 15
20X + Y = ?
10X Y e0.1 0
Time 0 T =1 Time 0 T =1 Time 0 T =1

35
36 CHAPTER 10. BINOMIAL OPTION PRICING: I

In the above diagram, 2X stocks at time zero are worth either 30X or 10X
at T = 1. Putting Y dollars in a savings account at time zero will produce Y e0.1
at T = 1.
We want our portfolio to behave like a call. So the payoff of our portfolio
should
½ the same 0.1as the payoff of the call. We set up the following equation:
30X + Y e = 15
X = 0.75 Y = −10 (0.75) e−0.1 = −6. 786
10X + Y e0.1 = 0
Y = −6. 786 means that we borrow 6. 786 at t = 0 and pays 6. 786e0.1 = 7.
5 at T = 1
If at t = 0 we buy 0.75 share of a stock and borrow $6. 786, then at T = 1,
this portfolio will have the same payoff as the call. To avoid arbitrage, the
portfolio and the call should have the same cost at t = 0.
C = 20X + Y = 20 (0.75) − 6. 786 = 8. 214

Example 10.1.2. Find the price of a 12-month European put option on a stock
with strike price $15. The stock currently sells for $20. In 12 months, the
stock can either go up to $30 or go down to $10. The continuously compounded
risk-free interest rate per year is 10%.

Suppose at time zero we create a replicating portfolio by buying X stocks


and investing Y dollars in a savings account. We want this portfolio to have the
exact payoff as the put.

30X Y e0.1 0
20X + Y = ?
10X Y e0.1 5
Time 0 T =1 Time 0 T =1 Time 0 T =1
½
30X + Y e0.1 = 0
X = −0.25 Y = 6. 786
10X + Y e0.1 = 5

So the replicating portfolio is at t = 0 by short-selling 0.25 stock and invest-


ing 6. 786 in a savings account.
The price of the put at t = 0 is:
P = 20X + Y = 20 (−0.25) + 6. 786 = 1. 786
Let’s check whether the put-call parity holds.
C + P V (K) = 8. 214 + 15e−0.1 = 21. 787
P + S0 = 1. 786 + 20 = 21. 786
Ignoring the rounding difference, we get: C + P V (K) = P + S0

10.2 General one-period binomial model

Suppose we have two points in time, t = 0 (today) and t = h (some point in


the future). The continuously compounded risk-free interest rate per year is r
10.2. GENERAL ONE-PERIOD BINOMIAL MODEL 37

( a positive constant). We have two assets: a stock that pays zero dividend and
a savings account. The savings account is the same as a zero-coupon bond. At
time h, the stock price is Sh ; the bond price is Bh .
The bond price is deterministic: B0 = 1 Bh = erh

The stock price is½stochastic:


Su
S0 = S Sh =
Sd
So at h the stock price either goes up to Su ("up state") or goes down to Sd
("down state").

Su
S
Sd
Time 0 Time h

We assume there’s no tax, no transaction cost, or margin requirements; one


is allowed to short sell any security and receive full proceeds. In addition, we
assume that anyone can buy or sell any number of securities without affecting
the market price.
We assume that the market is an invisible cop automatically enforcing Sd <
Serh < Su . For example, if Su > Sd > Serh , then the stock’s return is guar-
anteed to be higher than the risk-free interest rate. If this is the case, then the
risk-free interest rate was set too low and stock’s return is too good. Everyone
will jump on this opportunity, withdraw all his money from his savings account,
and invest it in the stock. This will instantaneously bid up the price of the stock
and the risk-free interest rate, forcing Sd < Serh < Su to hold.

What’s the non-arbitrage condition is the stock pays dividend continuously


at the yield δ?
Suppose you buy e−δh share of a stock at t = 0 (thus you pay Se−δh ) , buy
reinvesting dividend in the stock, you’ll have exactly one e−δh eδh = 1 stock at
time h, which is worth either Su or Sd .

Su
Se−δh
Sd
Time 0 Time h

Suppose
¡ −δhyou invest Se−δh in a savings account, then your wealth at h is
¢ rh
simply Se e = Se(r−δ)h .
Se(r−δ)h
−δh
Se
Se(r−δ)h
Time 0 Time h

To avoid arbitrage, the following condition needs to be met:


38 CHAPTER 10. BINOMIAL OPTION PRICING: I

• At good times (i.e. when the stock goes up), the return you earn from the
stock should exceed the risk free interest rate. So Su > Se(r−δ)h

• At bad times (i.e. when the stock goes down), the return you earn from
the stock should be less than the risk free interest rate. So Sd < Se(r−δ)h

If the condition is not met, we’ll end up in a weird situation where the stock
is always better off than the savings account or the savings account is always
better off than the stock. Then everyone will invest his money in the better
performing asset, instantly bidding up the price of the lower-performing asset
and forcing the above condition to be met.
To avoid arbitrage, Su , Sd , and r need to satisfy the following condition:

Su > Se(r−δ)h > Sd (10.1)

If Su = uS and Sd = dS, then Equation 10.1 becomes:

u > e(r−δ)h > d (10.2)

Equation 10.2 is the textbook Equation 10.4.

Let’s continue.
Let C represent the option price at time zero. Let Cu and Cd represent the
payoff at time h of an option in the up state and down state respectively:

Cu
C
Cd
Time 0 Time h

Our task is to determine C by setting a portfolio that replicates the option


payoff. We build the replicating portfolio by buying 4 shares of the stock and
investing $B in a zero-coupon bond. So we set up the following equation:
4Su Berh Cu
4S + B = C
4Sd Berh Cd
t=0 t=h t=0 t=h t=0 t=h
½
4Su + Berh = Cu
4Sd + Berh = Cd

Solving these equations, we get:

Cu − Cd
4= (10.3)
Su − Sd
10.2. GENERAL ONE-PERIOD BINOMIAL MODEL 39

Su Cd − Sd Cu
B = e−rh (10.4)
Su − Sd

Rearranging Equation 10.3, we get 4Su − Cu = 4Sd − Cd . This equation


is critical as we’ll soon see.
The value of the portfolio at the up state is
4Su + Berh = 4Su + e−rh (Cu − 4Su ) erh = Cu

The value of the portfolio at the down state is


4Sd + Berh = 4Sd + e−rh (Cu − 4Sd ) erh = Cd

Using Equation 10.3 and Equation 10.4, we get (verify this for yourself):
µ rh ¶
−rh Se − Sd Su − Serh
C = 4S + B = e Cu + Cd (10.5)
Su − Sd Su − Sd
Define

Serh − Sd
π u = p∗ = (10.6)
Su − Sd
rh
Su − Se
πd = q∗ = (10.7)
Su − Sd

Because market automatically enforces Sd < Serh < Su , we have

0 < πu < 1 (10.8)


0 < πp < 1 (10.9)
πu + πd = 1 (10.10)

According to Equation 10.8 , we can pretend that π u and π d are probabilities.


Then Equation 10.5 becomes

C = e−rh (π u Cu + π d Cd ) (10.11)
Doing some algebra, we also get (verify this for yourself):

µ ¶
Serh − Sd Su − Serh
S = e−rh Su + Sd = e−rh (π u Su + π d Sd ) (10.12)
Su − Sd Su − Sd
40 CHAPTER 10. BINOMIAL OPTION PRICING: I

According to Equation 10.11 and 10.12, once we set up faked probabilities


πu and πd , the call price at t = 0 is simply the expected present value of the
call payoffs discounted at the risk-free interest rate; the stock price at t = 0 is
simply the expected present value of the future stock prices discounted at the
risk-free interest rate. Since the discounting rate is risk-free interest rate, π u
and πd are called risk neutral probabilities.
Please note that π u and πd are not real probabilities. They are artificially
created probabilities so that Equation 10.11 and 10.12 have simple and intuitive
explanations.

Example 10.2.1. Using the risk-neutral probabilities, find the price of a 12-
month European call option on a stock with strike price $15. The stock currently
sells for $20. In 12 months, the stock can either go up to $30 or go down to
$10. The continuously compounded risk-free interest rate per year is 10%.

Solution.

Stock price tree Option terminal payoff

Time 0 T Time 0 T
Su = 30 Cu = max (0, 30 − 15) = 15
S = 20 C =?
Sd = 10 Cd = max (0, 10 − 15) = 0
Serh − Sd 20e0.1(1) − 10
πu = = = 0.605
Su − Sd 30 − 10
πd = 1 − 0.605 = 0.395
Cu = 30 − 15 = 15 Cd = 0
C = e−rh (π u Cu + π d Cd ) = e−0.1(1) (0.605 × 15 + 0.395 × 0) = 8. 211

Example 10.2.2. Using the risk-neutral probabilities, find the price of a 12-
month European put option on a stock with strike price $15. The stock currently
sells for $20. In 12 months, the stock can either go up to $30 or go down to
$10. The continuously compounded risk-free interest rate per year is 10%.

Stock price tree Option terminal payoff

Time 0 T Time 0 T
Su = 30 Cu = max (0, 15 − 30) = 0
S = 20 C =?
Sd = 10 Cd = max (0, 15 − 10) = 5

Serh − Sd 20e0.1(1) − 10
πu = = = 0.605
Su − Sd 30 − 10
πd = 1 − 0.605 = 0.395
10.2. GENERAL ONE-PERIOD BINOMIAL MODEL 41

Cu = 0 Cd = 5
C = e−rh (πu Cu + πd Cd ) = e−0.1(1) (0.605 × 0 + 0.395 × 5) = 1. 787

Suppose the stock pays dividends at a continuously compounded rate δ per


year. At time zero, our replicating portfolio consists of 4 shares of stocks and
$B in a bond (or a savings account). If we continuously reinvest dividends and
buy additional stocks during [0, h], our 4 shares at time zero will grow into
4eδh shares at time h. Our 4eδh shares will be worth either 4eδh Su in the up
state or 4eδh Sd in the down state. We want our replicating portfolio to have
the same payoff as the option at time h.
½
4eδh Su + Berh = Cu
4eδh Sd + Berh = Cd
Solving the equations, we get:

Cu − Cd
4 = e−δh (10.13)
Su − Sd

Su Cd − Sd Cu
B = e−rh (10.14)
Su − Sd
Notice whether the stock pays dividend or not, at time zero, we always need
Su Cd − Sd Cu Su Cd − Sd Cu
to have e−rh in a savings account, which grows into
Su − Sd Su − Sd
Cu − Cd
dollars at t = h. If the stock doesn’t pay dividend, at t = 0 we hold
Su − Sd
Cu − Cd
shares of stock, which is shares of stock at t = h ; if the stock pays
Su − Sd
Cu − Cd
dividend at a continuously compounded rate δ, at t = 0 we hold e−δh ,
Su − Sd
Cu − Cd
which grows shares of stock at t = h.
Su − Sd
Cu − Cd Su Cd − Sd Cu
So at time h we need to have units of stocks and
Su − Sd Su − Sd
dollars in a savings account (or a bond), regardless of whether the stock pays
dividend or not.

To see why, suppose our replicating portfolio at t = h (not t = 0) consists


of U shares of stocks and V dollars in a savings account. Then regardless of
whether the stock pays dividend or not, we need to have:
½
U Su + V = Cu
U Sd + V = Cd

This gives us:


Cu − Cd Su Cd − Sd Cu
U= V =
Su − Sd Su − Sd
42 CHAPTER 10. BINOMIAL OPTION PRICING: I

To have V dollars in a savings account at t = h, we need to have V e−rh =


Su Cd − Sd Cu Cu − Cd
e−rh at t = 0. To have U = shares of stocks at t = h,
Su − Sd Su − Sd
if the stock pays dividend at a continuously compounded rate of δ, we need to
Cu − Cd
have U e−δh = e−δh shares of stocks at t = 0. If we use the dividends
Su − Sd
received to buy additional stocks, then we’ll have U shares of stock at t = h.

Now let’s find the cost of the option on a stock that pays dividends at a
continuously compounded rate δ per year. The option cost at time zero is

Cu − Cd Su Cd − Sd Cu
4S + B = e−δh × S + e−rh
S − S
µ u (r−δ)h
d Su − Sd ¶
(r−δ)h
−rh Se − Sd Su − Se
=e Cu + Cd
Su − Sd Su − Sd

C = 4S + B = e−rh (πu Cu + πd Cd ) (10.15)

where

Se(r−δ)h − Sd
πu = (10.16)
Su − Sd

Su − Se(r−δ)h
πd = (10.17)
Su − Sd

If Su = Su and Sd = Sd, then Equation 10.16 and 10.17 becomes:

e(r−δ)h − d
πu = (10.18)
u−d

u − e(r−δ)h
πd = (10.19)
u−d
Tip 10.2.1. If you don’t want to memorize Equation 10.12, 10.15, 10.16, 10.17,
just set up the replication portfolio and calculate 4 and B from scratch.

Example 10.2.3. Find the price of a 12-month European call option on a stock
with strike price $15. The stock pays dividends at a continuously compounded
rate 6% per year. The stock currently sells for $20. In 12 months, the stock can
either go up to $30 or go down to $10. The continuously compounded risk-free
interest rate per year is 10%.
10.2. GENERAL ONE-PERIOD BINOMIAL MODEL 43

Stock price tree Option terminal payoff

Time 0 T Time 0 T
Su = 30 Cu = max (0, 30 − 15) = 15
S = 20 C =?
Sd = 10 Cd = max (0, 10 − 15) = 0

Replicating portfolio
Time 0 T
(4u , Bu ) = (0.75, −7. 50)
(4, B) = (0.706 32, −6. 786 28)
(4d , Bd ) = (0.75, −7. 50)
Our replicating portfolio at t = 0 consists of 4 shares of stocks and $B in a
savings account.
Cu − Cd 15 − 0
4 = e−δh = e−0.06(1) = 0.706 32
Su − Sd 30 − 10
Su Cd − Sd Cu 30 (0) − 10 (15)
B = e−rh = e−0.1(1) = −6. 786 28
Su − Sd 30 − 10
C = 4S + B = 0.706 3 (20) − 6. 786 3 = 7. 34

The replicating portfolio at T is:


4u = 4d = 4eδh = 0.706 32e0.06(1) = 0.75
Bu = Bd = Berh = −6. 786 28e0.1(1) = −7. 5

Verify that the replicating portfolio and the option have the same value:
The value of the replicating portfolio at the up state:
4u Su + Bu = 0.75 (30) − 7. 50 = 15 = Cu
The value of the replicating portfolio at the down state:
4d Sd + Bd = 0.75 (10) − 7. 50 = 0 = Cd

The value of the portfolio at t = 0 :


C = 4S + B = 0.706 32 (20) − 6. 78628 = 7. 34
Alternatively,
Se(r−δ)h − Sd 20e(0.1−0.06)1 − 10
πu = = = 0.540 81
Su − Sd 30 − 10
π d = 1 − πu = 1 − 0.540 81 = 0.459 19
C = e−rh (πu Cu + πd Cd ) = e−0.1(1) (0.540 81 × 15 + 0.459 19 × 0) = 7. 34

Example 10.2.4. Find the price of a 12-month European put option on a stock
with strike price $15. The stock pays dividends at a continuously compounded
rate 6% per year. The stock currently sells for $20. In 12 months, the stock can
either go up to $30 or go down to $10. The continuously compounded risk-free
interest rate per year is 10%.

Solution.
44 CHAPTER 10. BINOMIAL OPTION PRICING: I

Stock price tree Option terminal payoff

Time 0 T Time 0 T
Su = 30 Cu = max (0, 15 − 30) = 0
S = 20 C =?
Sd = 10 Cd = max (0, 15 − 10) = 5

Replicating portfolio
Time 0 T
(4u , Bu ) = (−0.25, 7. 50)
(4, B) = (−0.235 4, 6. 786 3)
(4d , Bd ) = (−0.25, 7. 50)
Our replicating portfolio at t = 0 consists of 4 shares of stocks and $B in a
savings account.
Cu − Cd 0−5
4 = e−δh = e−0.06(1) = −0.25e−0.06(1) = −0.235 4
Su − Sd 30 − 10
Su Cd − Sd Cu 30 (5) − 10 (0)
B = e−rh = e−0.1(1) = 6. 786 3
Su − Sd 30 − 10
C = 4S + B = −0.235 4 (20) + 6. 786 3 = 2. 078
The replicating portfolio at T is:
4u = 4d = 4eδh = −0.235 4e0.06(1) = −0.25
Bu = Bd = Berh = 6. 786 3e0.1(1) = 7. 5
Verify that the replicating portfolio and the option have the same value:
The value of the replicating portfolio at the up state:
4u Su + Bu = −0.25 (30) + 7. 50 = 0 = Cu
The value of the replicating portfolio at the down state:
4d Sd + Bd = −0.25 (10) + 7. 50 = 5 = Cd

The value of the portfolio at t = 0 :


C = 4S + B = −0.235 4 (20) + 6. 786 3 = 2. 078
Alternatively,
Se(r−δ)h − Sd 20e(0.1−0.06)1 − 10
πu = = = 0.540 8
Su − Sd 30 − 10
πd = 1 − π u = 1 − 0.540 8 = 0.459 2
C = e−rh (π u Cu + π d Cd ) = e−0.1(1) (0.540 8 × 0 + 0.459 2 × 5) = 2. 078

Arbitrage a mispriced option

If an option sells for more or less than the price indicated by Equation 10.15,
we can make money by "buy low, sell high."
Example 10.2.5. A 12-month European call option on a stock has strike price
$15. The stock pays dividends at a continuously compounded rate 6% per year.
The stock currently sells for $20. In 12 months, the stock can either go up to
$30 or go down to $10. The continuously compounded risk-free interest rate per
year is 10%. This call currently sells for $8. Design an arbitrage strategy.
10.2. GENERAL ONE-PERIOD BINOMIAL MODEL 45

Solution.

Stock price tree Option terminal payoff

Time 0 T Time 0 T
Su = 30 Cu = max (0, 30 − 15) = 15
S = 20 C =?
Sd = 10 Cd = max (0, 10 − 15) = 0

Replicating portfolio
Time 0 T
(4u , Bu ) = (0.75, −7. 50)
(4, B) = (0.706 32, −6. 786 28)
(4d , Bd ) = (0.75, −7. 50)

There are two calls. One is in the market selling for $8 at t = 0. The other is
a synthetic call, which consists, at t = 0, of holding 0.706 3 stock and borrowing
$6. 786 3 at risk-free interest rate. The synthetic call sells for $7. 34 at t = 0.
These two calls have identical payoffs at t = 1.
To make a riskless profit, we buy low and sell high. At t = 0, we sell a
call for $8 (sell high). Then at t = 1, if the stock price is $30, the call holder
exercises the call and our payoff is 15 − 30 = −15; if the stock is $10, the call
expires worthless and our payoff is zero.

−15
8
0
Time 0 Time t = 1
Payoff of a written call

Simultaneously, at t = 0 we buy 0.706 3 stock and borrow 6. 786 3 dollars


at risk-free interest rate (buy low). This costs us 0.706 3 (20) − 6. 786 3 = 7.
34 at t = 0. At t = 1, our initial 0.706 3 stock becomes 0.706 3e0.06(1) stock
and our initial debt 6. 786 3 grows into 6. 786 3e0.1(1) . Our portfolio is worth
0.706 3e0.06(1) S1 − 6. 786 3e0.1(1) .
If S1 = 30, our portfolio is worth
0.706 3e0.06(1) (30) − 6. 786 3e0.1(1) = 15
If S1 = 10, our portfolio is worth
0.706 3e0.06(1) (10) − 6. 786 3e0.1(1) = 0

15
−7.34
0
Time 0 Time t = 1
46 CHAPTER 10. BINOMIAL OPTION PRICING: I

Payoff of a replicating portfolio


So at t = 0, we gain 8 − 7. 34 = 0.66. At t = 1, the portfolio exactly offsets
our payoff in the call. We earn 0.66 sure profit at t = 0.

Example 10.2.6. A 12-month European call option on a stock has strike price
$15. The stock pays dividends at a continuously compounded rate 6% per year.
The stock currently sells for $20. In 12 months, the stock can either go up to
$30 or go down to $10. The continuously compounded risk-free interest rate per
year is 10%. This call currently sells for $7. Design an arbitrage strategy.

Stock price tree Option terminal payoff

Time 0 T Time 0 T
Su = 30 Cu = max (0, 30 − 15) = 15
S = 20 C =?
Sd = 10 Cd = max (0, 10 − 15) = 0

Replicating portfolio
Time 0 T
(4u , Bu ) = (0.75, −7. 50)
(4, B) = (0.706 32, −6. 786 28)
(4d , Bd ) = (0.75, −7. 50)

There are two calls. One is in the market selling for $7 at t = 0. The other is
a synthetic call, which consists, at t = 0, of holding 0.706 3 stock and borrowing
6. 786 3 dollars at risk-free interest rate. The synthetic call sells for $7. 34 at
t = 0. These two calls have identical payoffs at t = 1.
To make a riskless profit, we buy low and sell high. At t = 0, we buy a
call for $7 (buy low). Then at t = 1, if the stock price is $30, the call holder
exercises the call and our payoff is 30 − 15 = 15; if the stock is $10, the call
expires worthless and our payoff is zero.

15
−7
0
Time 0 Time t = 1
Payoff of a purchased call

Simultaneously, at t = 0 we sell the replicating portfolio. We short sell


0.706 3 stock and lend 6. 786 3 dollars at risk-free interest rate (sell high). We
gain 0.706 3 (20) − 6. 786 3 = 7. 34 at t = 0.
At t = 1, our initially borrowed 0.706 3 stock becomes 0.706 3e0.06(1) stock
and our lent principal 6. 786 3 grows into 6. 786 3e0.1(1) . Our portfolio is worth
10.2. GENERAL ONE-PERIOD BINOMIAL MODEL 47

6. 786 3e0.1(1) − 0.706 3e0.06(1) S1 .

If S1 = 30, our portfolio is worth


6. 786 3e0.1(1) − 0.706 3e0.06(1) (30) = −15

If S1 = 10, our portfolio is worth


6. 786 3e0.1(1) − 0.706 3e0.06(1) (10) = 0

−15
7.34
0
Time 0 Time t = 1

Payoff of a replicating portfolio


So at t = 0, we gain7.34 − 7 = 0.34. At t = 1, the call payoff exactly offsets
our liabilities in the replicating portfolio We earn 0.34 sure profit at t = 0.

Tip 10.2.2. An option and its replicating portfolio are exactly the same in terms
of payoff and cost. If an option in the market sells for more than the fair price
indicated in Equation 10.15, we can make a sure profit by buying the replicating
portfolio and selling the option. If an option in the market sells for less than
the fair price indicated in Equation 10.15, we can make a sure profit by selling
the replicating portfolio and buying the option.

Risk neutral probability and forward price

If we use π u and π d to calculate the undiscounted stock price, we get:

Se(r−δ)h − Sd Su − Se(r−δ)h
π u Su + π d Sd = Su + Sd
Su − Sd Su − Sd
(r−δ)h
= Se = F0,h

πu Su + πd Sd = Se(r−δ)h = F0,h (10.20)


F0,h is the delivery price at t = h of a forward contract signed at t = 0.
The textbook uses the symbol Ft,t+h to indicate that the forward contract is
signed at time t and the asset is to be delivered at t + h. If we treat the contract
initiation date t as time zero, then the asset deliver date is time h. So the
notation doesn’t matter. If you want to use the textbook notation, you’ll have

πu Su + πd Sd = Se(r−δ)h = Ft,t+h (10.21)


If you want to use my notation, you’ll get Equation 10.20.
Suppose you enter into a forward contract as a seller agreeing to deliver one
stock for a guaranteed price F0,h at t = h. To ensure you indeed can deliver
one stock at time h, you’ll want to buy e−δh stock at time zero. If you reinvest
48 CHAPTER 10. BINOMIAL OPTION PRICING: I

dividend and buy additional stocks, your initial e−δh stock will grow into exactly
one stock at time h. Your cost of buying e−δh stock at time zero is Se−δh . Since

you’ll tie up your money Se−δh during [0, h], you’ll want the forward price to
include the interest you could otherwise earn on Se−δh . So the forward price is
just the future value of Se−δh :
F0,h = Se−δh erh = Se(r−δ)h

According to Equation 10.20, the undiscounted stock price equals the for-
ward price under the risk neutral probability. If a problem gives you a forward
price, you can use Equation 10.20 to calculate the risk-neutral probability.

Constructing a binomial tree

Suppose we are standing at time t. If we can be 100% certain about the stock
price at time t + h, then investing in stocks doesn’t have any risk. Then stocks
must earn a risk-free interest rate. Since the stock already pays dividends at
rate δ, to earn a risk free interest rate, the stock price just needs to grow at
the rate of r − δ. Hence the stock price at t + h is Se(r−δ)h , which is just the
forward price Ft,t+h because Ft,t+h is also equal to Se(r−δ)h .
However, the stock price is a random variable and we generally can’t be
100% certain about a stock’s future price. To incorporate uncertainty, we use
Formula 11.16 in Derivatives Markets:

St+h √
ln = (r − δ) h ± σ h (Textbook 11.16)
St
St+h
In the above equation, ln is the continuously compounded rate of return
St
during [t, t√+ h]. This return consists of a known element (r − δ) h and a random
element σ h, where σ is the annualized standard deviation of the continuously
compounded stock return. The variance of the stock return in one year is σ 2 .
The variance during the interval [t, t + h] (which is h year long) is σ 2 h and this
is why. [t, t + h] can be broken down into h intervals, with each interval being
one year long. Assume stock return during each year is independent identically
distributed. The total return during [t, t + h] is the sum of the returns over h
intervals. Then the total variance is just the sum of the variance over h intervals,
σ 2 h.
√ √ √
So St+h = St e(r−δ)h±σ h = Se(r−δ)h±σ h = Ft,t+h √
e±σ h . In the

binomial
(r−δ)h+σ h σ h
model, the stock price

either goes √
up to Se = Ft,t+h e or goes
(r−δ)h−σ h −σ h
down to Se = Ft,t+h e . So we have


uSt = Ft,t+h eσ h
(10.22)
10.2. GENERAL ONE-PERIOD BINOMIAL MODEL 49


dSt = Ft,t+h e−σ h
(10.23)

If we set volatility σ to zero, then Equation 10.22 and 10.23 becomes uSt =
dSt = Ft,t+h . This means that if the stock price is 100% certain, then the stock
price is just the forward price.
Apply Equation 10.21 to Equation 10.22 and 10.23, we get:

u = e(r−δ)h+σ h
(10.24)

d = e(r−δ)h−σ h
(10.25)

10.2.1 Two or more binomial trees


Example 10.2.7. Let’s reproduce Derivatives Markets Figure 10.4. Here is the
recap of the information on a European call option. The current stock price is 41.
The strike price K = 40. The annualized standard deviation of the continuously
compounded stock return is σ = 30%. The continuously compounded risk-free
rate per year is r = 8%. The continuously compounded dividend rate per year
is δ = 0%.The option expiration date is T = 2 years. Use a 2-period binomial
tree to calculate the option premium.
T
Each period is h = = 1 year long.
2
Step 1 Draw a stock price tree.
√ √
u = e(r−δ)h+σ√ h = e(0.08−0)1+0.3√1 = 1. 462 28
d = e(r−δ)h−σ h = e(0.08−0)1−0.3 1 = 0.802 52
Stock price
Period 0 1 2 Period 0 1 2
Su2 Suu = 87.6693
Su Su = 59.9537
=⇒
S Sud S = 41 Sud = 48.1139
Sd Sd = 32.9033
Sd2 Sdd = 26.4055

Step 2 Draw the terminal payoffs at T = 2.


47.6693 = max (0, 87.6693 − 40)
8.1139 = max (0, 48.1139 − 40)

Option Payoff
Period 0 1 2
Cuu = 47.6693
Cu
C =? Cud = 8.1139
Cd
Cdd = 0
50 CHAPTER 10. BINOMIAL OPTION PRICING: I

Step 3 Work backward from right to left (called backwardization). Cal-


culate the premium using the formula
C = e−rh (π u Cu + π d Cd )
e(r−δ)h − d e(0.08−0)1 − 0.802 52
πu = = = 0.425 56
u−d 1. 462 28 − 0.802 52
πd = 1 − π u = 1 − 0.42556 = 0.574 44

Option premium
Period 0 1 2
Cuu = 47.6693
Cu = 23.0290
C = 10.7369 Cud = 8.1139
Cd = 3.1875
Cdd = 0

Cu = e−rh (π u Cuu + πd Cud )


= e−0.08(1) (0.425 56 × 47.6693 + 0.574 44 × 8.1139) = 23. 029 0

Cd = e−rh (π u Cud + π d Cud )


= e−0.08(1) (0.425 56 × 8.1139 + 0.574 44 × 0) = 3. 187 5
The call premium is
C = e−rh (π u Cu + π d Cd )

= e−0.08(1) (0.425 56 × 23.0290 + 0.574 44 × 3.1875) = 10.7369


10.2. GENERAL ONE-PERIOD BINOMIAL MODEL 51

Step 4 Calculate 4 and B using the formula


−δh Cu − Cd Su Cd − Sd Cu
4=e B = e−rh
Su − Sd Su − Sd
To avoid errors, put the stock price tree and the premium tree side by side:

Period 0 1 2 Period 0 1 2
Suu = 87.6693 Cuu = 47.6693
Su = 59.9537 Cu = 23.0290
S = 41 Sud = 48.1139 C = 10.7369 Cud = 8.1139
Sd = 32.9033 Cd = 3.1875
Sdd = 26.4055 Cdd = 0

Period 0 1

(4u , Bu ) = (1.0, −36.9247)


(4, B) = (0.7335, −19.3367)
(4d , Bd ) = (0.3738, −9.1107)

Cuu − Cud 47.6693 − 8.1139


4u = e−δh = e−0(1) = 1.0
Suu − Sdd 87.6693 − 48.1139
Cud − Cdd 8.1139 − 0
4d = e−δh = e−0(1) = 0.373 8
Sud − Sdd 48.1139 − 26.4055
Cu − Cd 23.0290 − 3.1875
4 = e−δh = e−0(1) = 0.733 5
Su − Sd 59.9537 − 32.9033

Suu Cud − Sud Cuu


Bu = e−rh
Suu − Sud
−0.08(1) 87.6693 (8.1139) − 48.1139 (47.6693)
=e = −36.9247
87.6693 − 48.1139

−36.9247 means that $36.9247 needs to be borrowed at a risk-free rate.

Sdu Cdd − Sdd Cdu


Bd = e−rh
Sdu − Sdd
−0.08(1) 48.1139 (0) − 26.4055 (8.1139)
=e = −9. 110 7
48.1139 − 26.4055
Su Cd − Sd Cu
B = e−rh
Su − Sd
59.9537 (3.1875) − 32.9033 (23.0290)
= e−0.08(1) = −19. 3367
59.9537 − 32.9033
52 CHAPTER 10. BINOMIAL OPTION PRICING: I

Step 5 Verify that the portfolio replicates the premium tree. Here is
recap of the information:
Period 0 1 2 0 1 2
Suu = 87.6693 Cuu = 47.6693
Su = 59.9537 Cu = 23.0290
S = 41 Sud = 48.1139 C = 10.7369 Cud = 8.1139
Sd = 32.9033 Cd = 3.1875
Sdd = 26.4055 Cdd = 0

Period 0 1

(4u , Bu ) = (1.0, −36.9247)


(4, B) = (0.7335, −19.3367)
(4d , Bd ) = (0.3738, −9.1107)

The value of (4, B) at time zero:


4S + B = 0.7335 (41) − 19.3367 = 10. 736 8 = C

The value of(4u , Bu ) at Node u (up state):


4u Su + Bu = 1.0 (59.9537) − 36.9247 = 23. 029 = Cu
The value of(4d , Bd ) at Node d (down state):
4d Sd + Bd = 0.3738 (32.9033) − 9.1107 = 3. 188 = Cd

Finally, let’s verify that the portfolio replicates the terminal payoff. First,
we need to find the replicating portfolio at the expiration date.
Period 1 2
(4uu , Buu ) = (1.0, −40)
(4u , Bu ) = (1.0, −36.9247)
(4ud , Bud ) = (1.0, −40) = (0.3738, −9. 869 5)
(4d , Bd ) = (0.3738, −9.1107)
(4dd , Bdd ) = (0.3738, −9. 869 5)

If we reinvest dividends, 4u stocks grows into 4u eδh after h years; Bu grows


into Bu erh after h¡ years. ¢ ¡ ¢
(4uu , Buu ) = 4u eδh , Bu erh = 1.0e0(1) , −36.9247e0.08(1) = (1.0, −40)
4uu Suu + Buu = 1.0 (87.6693)
¡ − 40 = 47.
¢ 669¡ 3 = Cuu ¢
Similarly, (4dd , Bdd ) = 4d eδh , Bd erh = 0.3738e0(1) , −9.1107e0.08(1) =
(0.373 8, −9. 869 5)
4dd Sdd + Bdd = 0.3738 (26.4055) − 9. 869 5 = 0.00 = Cdd
Finally, (4ud , Bud ) = (1.0, −40) = (0.3738, −9. 869 5)
Portfolios (1.0, −40) and (0.3738, −9. 869 5) have equal values at Node ud.
1.0 (48.1139) − 40 = 8. 113 9 = Cud
0.3738 (48.1139) − 9. 869 5 = 8. 115 = Cud (ignore rounding difference)
10.2. GENERAL ONE-PERIOD BINOMIAL MODEL 53

Tip 10.2.3. For a multi-binomial tree, using the risk neutral probability to find
the premium is faster than using the replicating portfolio. The risk neutral prob-
abilities π u and π d are constant cross nodes. However, the replicating portfolio
(4, B) varies by node.
Tip 10.2.4. For European options, you can calculate the premium using the
terminal payoffs. This is how to quickly find the premium for this problem.
Node at T = 2 Payoff at T Risk neutral probability of reaching this node1
uu Cuu = 47.6693 π 2u = 0.425 562 = 0.181 1
ud Cud = 8.1139 2π u πd = 2 (0.425 56) (0.574 44) = 0.488 9 2
dd Cdd = 0 π 2d = 0.574 442 = 0.3300
2
Total (πu + π d ) = 0.181 1 + 0.488 9 + 0.3300 = 1 3
The premium is the expected present value of the terminal payoff using the
risk neutral probability.
¡ ¢
C = e−rT π2u Cuu + 2π u πd Cud + π 2d Cdd
¡ ¢
= e−0.08(2) 0.425 562 × 47.6693 + 0.488 9 × 8.1139 + 0.33 × 0 = 10. 736 9

Tip 10.2.5. If you purchased the textbook Derivatives Markets, you should see
a CD attached to the back cover of the book. Install the CD in your computer.
Run the spreadsheet titled "optall2’" or "optbasic2." These two spreadsheets can
calculate European and American option prices. When you solve a practice prob-
lem, you can use either of these two spreadsheets to double check you answer.
Please note that these two spreadsheets don’t calculate the replicating portfo-
lio (4, B). So you can’t use them to verify your calculation of the replicating
portfolio.

Example 10.2.8. Let’s reproduce Derivatives Markets Figure 10.5. Here is


the recap of the information on a European call. The current stock price is 41.
The strike price K = 40. The annualized standard deviation of the continuously
compounded stock return is σ = 30%. The continuously compounded risk-free
rate per year is r = 8%. The continuously compounded dividend rate per year
is δ = 0%.The option expiration date is T = 1 year. Use a 3-period binomial
tree to calculate the option premium.

Solution.

1
Each period is h = year long.
3
√ √
u = e(r−δ)h+σ h
= e(0.08−0)1/3+0.3 1/3 = 1. 221 246
1 The probabilities in this column are the 3 terms of (π u + πd )2 = π 2d + 2π d π u + π 2u
2 There are two ways of reaching Node ud: up and dow or down and up.
3 The total probability is one. Otherwise, you made an error.
54 CHAPTER 10. BINOMIAL OPTION PRICING: I

√ √
d = e(r−δ)h−σ h = e(0.08−0)1/3−0.3 1/3
= 0.863 693
Stock price
Period 0 1 2 3
Su3 = 74.6781
Su2 = 61.1491
Su = 50.0711 Su2 d = 52.8140
S = 41 Sud = 43.2460
Sd = 35.4114 Sd2 u = 37.3513
2
Sd = 30.5846
Sd3 = 26.4157
Calculate the premium by working backward from right to left.

Period 0 1 2 3
Cu3 = max (0, 74.6781 − 40) = 34. 678 1
Cu2 = 22. 201 6
Cu = 12. 889 5 Cu2 d = max (0, 52.8140 − 40) = 12. 814
C = 7. 073 9 Cud = 5. 699 5
Cd = 2. 535 1 Cd2 u = max (0, 37.3513 − 40) = 0
Cd2 = 0
Cd3 = max (0, 26.4157 − 40) = 0
−rh
C=e (π u Cu + π d Cd )
e(r−δ)h − d e(0.08−0)1/3 − 0.863 693
πu = = = 0.456 806
u−d 1. 221 246 − 0.863 693
πd = 1 − π u = 1 − 0.456 806 = 0.543 194

Cu2 = e−rh (π u Cu3 + πd Cu2 d )


= e−0.08(1/3) (0.456 806 × 34. 678 1 + 0.543 194 × 12. 814) = 22. 201 7
Cud = e−rh (π u Cu2 d + π d Cd2 u )
= e−0.08(1/3) (0.456 806 × 12. 814 + 0.543 194 × 0) = 5. 699 5
Cd2 = e−rh (πCd2 u + πd Cd3 ) = 0

Cu = e−rh (π u Cu2 + π d Cud )


= e−0.08(1/3) (0.456 806 × 22. 201 6 + 0.543 194 × 5. 699 5) = 12. 889 5
Cd = e−rh (π u Cud + π d Cd2 )
= e−0.08(1/3) (0.456 806 × 5. 699 5 + 0.543 194 × 0) = 2. 535 1

C = e−rh (πu Cu + πd Cd )
= e−0.08(1/3) (0.456 806 × 12. 889 5 + 0.543 194 × 2. 535 1) = 7. 073 9
10.2. GENERAL ONE-PERIOD BINOMIAL MODEL 55

Now I’m going to tell you a calculator shortcut I used when I was preparing
for the old Course 6. The above calculations are intense and prone to errors.
To quickly and accurately calculate the premium at each node, use TI-30X IIS
calculator because TI-30X IIS allows you to modify formulas easily.
For example, to calculate Cu2 = e−rh (π u Cu3 + π d Cu2 d ), enter

eˆ(−0.08/3)(0.456 806 × 34. 678 1 + 0.543 194 × 12. 814


Please note for TI-30 IIS, the above expression is the same as
eˆ(−0.08/3)(0.456 806 × 34. 678 1 + 0.543 194 × 12. 814)

In other words, you can omit the ending parenthesis ")". I tell you this
because occasionally people emailed me saying they discovered a typo. This is
not a typo.

Now you have entered the formula


eˆ(−0.08/3)(0.456 806 × 34. 678 1 + 0.543 194 × 12. 814.
Press "=" and you should get: 22.20164368

Next, to calculate Cud = e−rh (πud Cu2 d + πd Cd2 u ), you don’t need to enter
a brand new formula. Just reuse the formula
eˆ(−0.08/3) (0.456 806 × 34. 678 1 + 0.543 194 × 12. 814)

Change 34. 678 1 to 12. 8140 (so 0.456 806 ×34. 678 1 becomes 0.456 806 × 12.
8140).
Change 12. 814 into 00.000 (so 0.543 194×12. 814 becomes 0.543 194×00.000).
Now the modified formula is
eˆ(−0.08/3) (0.456 806 × 12. 8140 + 0.543 194 × 00.000)
Press "=" and you should get: 5.6994813
To calculate Cu = e−rh (πu Cu2 + π d Cud ), once again reuse a previous for-
mula. Change the formula
eˆ(−0.08/3) (0.456 806 × 12. 8140 + 0.543 194 × 00.000)

into
eˆ(−0.08/3) (0.456 806 × 22. 201 6 + 0.543 194 × 05. 699 5)
Press "=" and you should get:12.8894166

Reusing formulas avoids the need to retype e−rh , π u , and π d (these three
terms are constant across all nodes) and increases your speed and accuracy. By
reusing formulas, you should quickly find C = 7. 073 9.
56 CHAPTER 10. BINOMIAL OPTION PRICING: I

Next, we calculate the replicating portfolio.


Cu − Cd Su Cd − Sd Cu
4 = e−δh B = e−rh
Su − Sd Su − Sd
To avoid errors, put the stock price tree and the premium tree side by side:

Period 0 1 2 3
Su3 = 74.6781
Su2 = 61.1491
Su = 50.0711 Su2 d = 52.8140
S = 41 Sud = 43.2460
Sd = 35.4114 Sd2 u = 37.3513
2
Sd = 30.5846
Sd3 = 26.4157

Period 0 1 2 3
Cu3 = 34. 678 1
Cu2 = 22. 201 6
Cu = 12. 889 5 Cu2 d = 12. 814
C = 7. 073 9 Cud = 5. 699 5
Cd = 2. 535 1 Cd2 u = 0
Cd2 = 0
Cd3 = 0

Period 0 1 2

(4, B)u2 = (1, −38. 947 4)


(4, B)u = (0.921 8, −33. 263 6)
(4, B) = (0.706 3, −21. 885 2) (4, B)ud = (0.828 7, −30. 138 6)
(4, B)d = (0.450 1, −13. 405 2)
(4, B)d2 = (0, 0)

Cu3 − Cu2 d 34. 678 1 − 12. 814


4u2 = e−δh = e−0(1/3) =1
Su3 − Su2 d 74.6781 − 52.8140
Su3 Cu2 d − Su2 dCu2 d 74.6781 (12. 814) − 52.8140 (34. 678 1)
Bu2 = e−rh = e−0.08(1/3) =
Su3 − Su2 d 74.6781 − 52.8140
−38. 947 4

Cu2 d − Cd2 u 12. 814 − 0


4ud = e−δh = e−0(1/3) = 0.828 7
Su2 d − Sd2 u 52.8140 − 37.3513
2 2
Su dCd2 u − Sd uCu2 d 52.8140 (0) − 37.3513 (12. 814)
Bud = e−rh = e−0.08(1/3) =
Su2 d − Sd2 u 52.8140 − 37.3513
−30. 138 6
10.2. GENERAL ONE-PERIOD BINOMIAL MODEL 57

Cd2 u − Cd3
4d2 = e−δh =0
Su2 d − Sd3
Su dCd3 − Sd3 Cd2 u
2
Bd2 = e−rh =0
Su2 d − Sd3

Cu2 − Cud 22. 201 6 − 5. 699 5


4u = e−δh = e−0(1/3) = 0.921 8
Su2 − Sud 61.1491 − 43.2460
Su2 Cud − SudCu2 61.1491 (5. 699 5) − 43.2460 (22. 201 6)
Bu = e−rh = e−0.08(1/3) =
Su2 − Sud 61.1491 − 43.2460
−33. 263 6

Cud − Cd2 5. 699 5 − 0


4d = e−δh 2
= e−0(1/3) = 0.450 1
Sud − Sd 43.2460 − 30.5846
2
SudCd2 − Sd Cud 43.2460 (0) − 30.5846 (5. 699 5)
Bd = e−δh 2
= e−0.08(1/3) =
Sud − Sd 43.2460 − 30.5846
−13. 405 2

Cu − Cd 12. 889 5 − 2. 535 1


4 = e−δh = e−0(1/3) = 0.706 3
Su − Sd 50.0711 − 35.4114
Cu − Cd 50.0711 (2. 535 1) − 35.4114 (12. 889 5)
B = e−rh = e−0.08(1/3) =
Su − Sd 50.0711 − 35.4114
−21. 885 2

If our goal is to calculate the premium without worrying about the replicating
portfolio, then we just need to know the risk neutral probability and the terminal
payoff.

Node Payoff Risk neutral prob of reaching this node4


u3 Cu3 = 34. 678 1 π 3u¡= 0.456 806
¢
3

u2 d Cu2 d = 12. 814 3π 2u πd = 3 0.456 8062 0.543 194


ud2 Cd2 u = 0 3π u π 2d = 3 (0.456 806) 0.543 1942
d3 Cd3 = 0 π 3d = 0.543 1943

The option premium is just the expected present value of the terminal payoffs
using the risk¡neutral probability. ¢
C = e−rT Cu3 π 3u + Cu2 d × 3π 2u πd + Cd2 u × 3πu π 2d + Cd3 π 3d
¡ ¢
= e−0.08(1) 34. 678 1 × 0.456 8063 + 12. 814 × 3 × 0.456 8062 × 0.543 194
= 7. 073 9

Example 10.2.9. Let’s reproduce Derivatives Markets Figure 10.6. Here is


the recap of the information on a European put. The current stock price is 41.
The strike price K = 40. The annualized standard deviation of the continuously
4 The probabilities of this column is just the four terms in (π + π )3 = π 3 + 3π π 2 +
u d u d u
3π 2d π u + π 3d = 1.
58 CHAPTER 10. BINOMIAL OPTION PRICING: I

compounded stock return is σ = 30%. The continuously compounded risk-free


rate per year is r = 8%. The continuously compounded dividend rate per year
is δ = 0%.The option expiration date is T = 1 year. Use a 3-period binomial
tree to calculate the option premium.

Solution.

1
Each period is h = year long.
3
√ √
u = e(r−δ)h+σ√ h = e(0.08−0)1/3+0.3√1/3 = 1. 221 246
d = e(r−δ)h−σ h = e(0.08−0)1/3−0.3 1/3 = 0.863 693
Stock price
Period 0 1 2 3
Su3 = 74.6781
Su2 = 61.1491
Su = 50.0711 Su2 d = 52.8140
S = 41 Sud = 43.2460
Sd = 35.4114 Sd2 u = 37.3513
2
Sd = 30.5846
Sd3 = 26.4157

Calculate the premium by working backward from right to left.

Period 0 1 2 3
Cu3 = max (0, 40 − 74.6781) = 0
Cu2 = 0
Cu = 0.740 9 Cu2 d = max (0, 40 − 52.8140) = 0
C = 2. 998 5 Cud = 1. 400 9
Cd = 5. 046 2 Cd2 u = max (0, 40 − 37.3513) = 2. 648 7
Cd2 = 8. 362 9
Cd3 = max (0, 40 − 26.4157) = 13. 584 3

C = e−rh (π u Cu + π d Cd )
e(r−δ)h − d e(0.08−0)1/3 − 0.863 693
πu = = = 0.456 806
u−d 1. 221 246 − 0.863 693
πd = 1 − π u = 1 − 0.456 806 = 0.543 194

Cu2 = e−rh (π u Cu3 + πd Cu2 d ) = e−0.08(1/3) (0.456 806 × 0 + 0.543 194 × 0) =


0
Cud = e−rh (π u Cu2 d + π d Cd2 u ) = e−0.08(1/3) (0.456 806 × 0 + 0.543 194 × 2. 648 7) =
1. 400 9
10.2. GENERAL ONE-PERIOD BINOMIAL MODEL 59

Cd2 = e−rh (π u Cd2 u + πd Cd3 ) = e−0.08(1/3) (0.456 806 × 2. 648 7 + 0.543 194 × 13. 584 3) =
8. 362 9

Cu = e−rh (πu Cu2 + π d Cud ) = e−0.08(1/3) (0.456 806 × 0 + 0.543 194 × 1. 400 9) =
0.740 9
Cd = e−rh (π u Cud + π d Cd2 ) = e−0.08(1/3) (0.456 806 × 1. 400 9 + 0.543 194 × 8. 362 9) =
5. 046 2

C = e−rh (πu Cu + π d Cd ) = e−0.08(1/3) (0.456 806 × 0.740 9 + 0.543 194 × 5. 046 2) =


2. 998 5

Next, we calculate the replicating portfolio.


Cu − Cd Su Cd − Sd Cu
4 = e−δh B = e−rh
Su − Sd Su − Sd
Period 0 1 2

(4, B)u2 = (0, 0)


(4, B)u = (−0.07 82, 4. 658 9)
(4, B) = (−0.293 7, 15. 039 5) (4, B)ud = (−0.171 3, 8. 808 8)
(4, B)d = (−0.549 9, 24. 517 3)
(4, B)d2 = (1, 38. 947 4)

Cu3 − Cu2 d 0−0


4u2 = e−δh = e−0(1/3) =0
Su3 − Su2 d 74.6781 − 52.8140
3 2
Su Cu2 d − Su dCu2 d 74.6781 (0) − 52.8140 (0)
Bu2 = e−rh = e−0.08(1/3) =0
Su3 − Su2 d 74.6781 − 52.8140

Cu2 d − Cd2 u 0 − 2. 648 7


4ud = e−δh 2 2
= e−0(1/3) = −0.171 3
Su d − Sd u 52.8140 − 37.3513
2 2
Su dCd2 u − Sd uCu2 d 52.8140 (2. 648 7) − 37.3513 (0)
Bud = e−rh 2 2
= e−0.08(1/3) =
Su d − Sd u 52.8140 − 37.3513
8. 808 8
Cd2 u − Cd3 2. 648 7 − 13. 584 3
4d2 = e−δh = e−0(1/3) = −1.0
Su2 d − Sd3 37.3513 − 26.4157
Su2 dCd3 − Sd3 Cd2 u 37.3513 (13. 584 3) − 26.4157 (2. 648 7)
Bd2 = e−rh 2 3
= e−0.08(1/3) =
Su d − Sd 37.3513 − 26.4157
38. 947 4
C 2 − Cud 0 − 1. 400 9
4u = e−δh u2 = e−0(1/3) = −0.07 82
Su − Sud 61.1491 − 43.2460
2
Su Cud − SudCu2 61.1491 (1. 400 9) − 43.2460 (0)
Bu = e−rh = e−0.08(1/3) =
Su2 − Sud 61.1491 − 43.2460
4. 659
Cud − Cd2 1. 400 9 − 8. 362 9
4d = e−δh = e−0(1/3) = −0.549 9
Sud − Sd2 43.2460 − 30.5846
60 CHAPTER 10. BINOMIAL OPTION PRICING: I

SudCd2 − Sd2 Cud 43.2460 (8. 362 9) − 30.5846 (1. 400 9)


Bd = e−δh = e−0.08(1/3) =
Sud − Sd2 43.2460 − 30.5846
24. 517 3
Cu − Cd 0.740 9 − 5. 046 2
4 = e−δh = e−0(1/3) = −0.293 7
Su − Sd 50.0711 − 35.4114
Cu − Cd 50.0711 (5. 046 2) − 35.4114 (0.740 9)
B = e−rh = e−0.08(1/3) = 15.
Su − Sd 50.0711 − 35.4114
039 5

If our goal is to calculate the premium without worrying about the replicating
portfolio, then we just need to know the risk neutral probability and the terminal
payoff.

Node Payoff Risk neutral prob of reaching this node5


u3 Cu3 = 0 π3u¡= 0.456 806¢
3

u2 d Cu2 d = 0 2 2
3πu π d = 3 0.456 806 0.543 194
ud2 Cd2 u = 2. 648 7 3π u π2d = 3 (0.456 806) 0.543 1942
d3 Cd3 = 13. 584 3 π3d = 0.543 1943

The option premium is just the expected present value of the terminal payoffs
using the risk¡neutral probability. ¢
C = e−rT Cd2 u × 3π u π 2d + Cd3 π 3d
¡ ¢
= e−0.08(1) 2. 648 7 × 3 × 0.456 806 × 0.543 1942 + 13. 584 3 × 0.543 1943 =
2. 998 5

Example 10.2.10. Let’s reproduce Derivatives Markets Figure 10.7. Here is


the recap of the information on an American put. The current stock price is 41.
The strike price K = 40. The annualized standard deviation of the continuously
compounded stock return is σ = 30%. The continuously compounded risk-free
rate per year is r = 8%. The continuously compounded dividend rate per year
is δ = 0%.The option expiration date is T = 1 year. Use a 3-period binomial
tree to calculate the option premium.

Solution.

1
Each period is h = year long.
3
√ √
u = e(r−δ)h+σ√ h = e(0.08−0)1/3+0.3√1/3 = 1. 221 246
d = e(r−δ)h−σ h = e(0.08−0)1/3−0.3 1/3 = 0.863 693
5 The probabilities of this column is just the four terms in (π + π )3 = π 3 + 3π π 2 +
u d u d u
3π 2d π u + π3d = 1.
10.2. GENERAL ONE-PERIOD BINOMIAL MODEL 61

Step 1 Find the stock price tree


Period 0 1 2 3
Su3 = 74.6781
Su2 = 61.1491
Su = 50.0711 Su2 d = 52.8140
S = 41 Sud = 43.2460
Sd = 35.4114 Sd2 u = 37.3513
Sd2 = 30.5846
Sd3 = 26.4157

Step 2 Calculate the payoff at expiration (terminal payoff)

Period 0 1 2 3
Vu3 = max (0, 40 − 74.6781) = 0
Vu2
Vu Vu2 d = max (0, 40 − 52.8140) = 0
V Vud
Vd Vd2 u = max (0, 40 − 37.3513) = 2. 648 7
Vd2
Vd3 = max (0, 40 − 26.4157) = 13. 584 3

Step 3 Calculate the value of the American put one step left to the
expiration.
An American put can be exercised immediately. The value of an American
option if exercised immediately is called the exercise value (EV ) or intrinsic
value. At Period 2, we compare the value calculated using backwardization and
the exercise value. We take the greater of the two as the value of the American
put.

The backwardized values at Period 2 are:


Cu2 = e−rh (πu Cu3 + π d Cu2 d ) = e−0.08(1/3) (0.456 806 × 0 + 0.543 194 × 0) =
0
Cud = e−rh (πu Cu2 d + πd Cd2 u ) = e−0.08(1/3) (0.456 806 × 0 + 0.543 194 × 2. 648 7) =
1. 400 9
Cd2 = e−rh (π u Cd2 u + πd Cd3 ) = e−0.08(1/3) (0.456 806 × 2. 648 7 + 0.543 194 × 13. 584 3) =
8. 362 9

The exercise values at Period 2 are:


¡ ¢
EVu2 = max 0, K − Su2 = max (0, 40 − 61.1491) = 0
EVud = max ¡(0, K − Sud)
¢ = max (0, 40 − 43.2460) = 0
EVd2 = max 0, K − Sd2 = max (0, 40 − 30.5846) = 9. 415 4

We take the greater of the two as the value of the American put.
62 CHAPTER 10. BINOMIAL OPTION PRICING: I

Vu2 = max (Cu2 , EVu2 ) = max (0, 0) = 0


Vud = max (Cud , EVud ) = max (1. 400 9, 0) = 1. 400 9
Vd2 = max (Cd2 , EVd2 ) = max (8. 362 9, 9. 415 4) = 9. 415 4

Now we have:
Period 0 1 2 3
Vu3 = 0
Vu2 = 0
Vu Vu2 d = 0
V Vud = 1. 400 9
Vd Vd2 u = 2. 648 7
Vd2 = 9. 415 4
Vd3 = 13. 584 3

Step 3 Move one step to the left. Repeat Step 2. Compare the back-
wardized value and the exercise value. Choose the greater.
The backwardized values are:
Cu = e−0.08(1/3) (0.456 806 × 0 + 0.543 194 × 1. 400 9) = 0.740 9

Cu = e−0.08(1/3) (0.456 806 × 1. 400 9 + 0.543 194 × 9. 415 4) = 5. 602 9


The exercise value at Period 1 is:

EVu = max (0, K − Su ) = max (0, 40 − 50.0711) = 0


EVd = max (0, K − Sd ) = max (0, 40 − 35.4114) = 4. 588 6

Vu = max (Cu , EVu ) = max (0.740 9, 0) = 0.740 9


Vud = max (Cd , EVd ) = max (5. 602 9, 4. 588 6) = 5. 602 9
Hence we have:

Period 0 1 2 3
Vu3 = 0
Vu2 = 0
Vu = 0.740 9 Vu2 d = 0
V Vud = 1. 400 9
Vd = 5. 602 9 Vd2 u = 2. 648 7
Vd2 = 9. 415 4
Vd3 = 13. 584 3
Step 4 Repeat Step 3. Move one step left. Compare the backwardized
value and the exercise value. Choose the greater value.
The backwardized value at t = 0 is:
C = e−0.08(1/3) (0.456 806 × 0.740 9 + 0.543 194 × 5. 602 9) = 3. 292 9
The exercise value is:
EV = max (0, K − S) = max (0, 40 − 41) = 0
Hence the premium for the American put is:
V = max (C, EV )= max (3. 292 9, 0) = 3. 292 9
10.2. GENERAL ONE-PERIOD BINOMIAL MODEL 63

Now we have:

Period 0 1 2 3
Vu3 = 0
Vu2 = 0
Vu = 0.740 9 Vu2 d = 0
V = 3. 292 9 Vud = 1. 400 9
Vd = 5. 602 9 Vd2 u = 2. 648 7
Vd2 = 9. 415 4
Vd3 = 13. 584 3

Step 5 Calculate the replicating portfolio at each node.

Cu − Cd Su Cd − Sd Cu
4 = e−δh B = e−rh
Su − Sd Su − Sd
Period 0 1 2

(4, B)u2 = (0, 0)


(4, B)u = (−0.07 82, 4. 658 9)
(4, B) = (−0.331 66, 16. 890 9) (4, B)ud = (−0.171 3, 8. 808 8)
(4, B)d = (−0.632 99, 28. 017 8)
(4, B)d2 = (1, 38. 947 4)

Vu3 − Vu2 d 0−0


4u2 = e−δh 3 2
= e−0(1/3) =0
Su − Su d 74.6781 − 52.8140
3 2
Su Vu2 d − Su dVu2 d 74.6781 (0) − 52.8140 (0)
Bu2 = e−rh 3 2
= e−0.08(1/3) =0
Su − Su d 74.6781 − 52.8140

Vu2 d − Vd2 u 0 − 2. 648 7


4ud = e−δh = e−0(1/3) = −0.171 3
Su2 d − Sd2 u 52.8140 − 37.3513
Su2 dVd2 u − Sd2 Vu2 d 52.8140 (2. 648 7) − 37.3513 (0)
Bud = e−rh = e−0.08(1/3) =
Su2 d − Sd2 u 52.8140 − 37.3513
8. 808 8

Vd2 u − Vd3 2. 648 7 − 13. 584 3


4d2 = e−δh = e−0(1/3) = −1.0
Su2 d − Sd3 37.3513 − 26.4157
Su2 dVd3 − Sd3 Vd2 u 37.3513 (13. 584 3) − 26.4157 (2. 648 7)
Bd2 = e−rh = e−0.08(1/3) =
Su2 d − Sd3 37.3513 − 26.4157
38. 947 4
V 2 − Vud 0 − 1. 400 9
4u = e−δh u2 = e−0(1/3) = −0.07 82
Su − Sud 61.1491 − 43.2460
2
Su Vud − SudVu2 61.1491 (1. 400 9) − 43.2460 (0)
Bu = e−rh = e−0.08(1/3) =
Su2 − Sud 61.1491 − 43.2460
4. 659
64 CHAPTER 10. BINOMIAL OPTION PRICING: I

Vud − Vd2 1. 400 9 − 9. 415 4


4d = e−δh 2
= e−0(1/3) = −0.632 99
Sud − Sd 43.2460 − 30.5846
2
SudVd2 − Sd Vud 43.2460 (9. 415 4) − 30.5846 (1. 400 9)
Bd = e−δh = e−0.08(1/3) =
Sud − Sd2 43.2460 − 30.5846
28. 017 8
Vu − Vd 0.740 9 − 5. 602 9
4 = e−δh = e−0(1/3) = −0.331 66
Su − Sd 50.0711 − 35.4114
Vu − Vd 50.0711 (5. 602 9) − 35.4114 (0.740 9)
B = e−rh = e−0.08(1/3) = 16.
Su − Sd 50.0711 − 35.4114
890 9

Please note that for an American option, you can’t use the following ap-
proach to find the option price:
Node Payoff Risk neutral prob of reaching this node
u3 Vu3 = 0 π3u¡= 0.456 806
¢
3

u2 d Vu2 d = 0 3π2u π d = 3 0.456 8062 0.543 194


2
ud Vd2 u = 2. 648 7 3π u π2d = 3 (0.456 806) 0.543 1942
3
d Vd3 = 13. 584 3 π3d = 0.543 1943
−rT
¡ 2 3
¢
V =e Cd2 u × 3π u πd + Cd3 πd
¡ ¢
= e−0.08(1) 2. 648 7 × 3 × 0.456 806 × 0.543 1942 + 13. 584 3 × 0.543 1943 =
2. 998 5

This approach is wrong because it ignores the possibility that the American
option can be exercised early.

10.2.2 Options on stock index


The price of an option on stock index can be calculated the same way as the
price of an option on a stock is calculated.
Example 10.2.11. Let’s reproduce Derivatives Markets Figure 10.8. Here is
the recap of the information on an American call. The current stock index
is 110. The strike price K = 100. The annualized standard deviation of the
continuously compounded stock index return is σ = 30%. The continuously
compounded risk-free rate per year is r = 5%. The continuously compounded
dividend rate per year is δ = 3.5%.The option expiration date is T = 1 year.
Use a 3-period binomial tree to calculate the option premium.

Solution.

1
Each period is h = year long.
3
√ √
u = e(r−δ)h+σ√ h = e(0.05−0.035)1/3+0.3√1/3 = 1. 195 07
d = e(r−δ)h−σ h = e(0.05−0.035)1/3−0.3 1/3 = 0.845 18
e(r−δ)h − d e(0.05−0.035)1/3 − 0.845 18
πu = = = 0.456 807
u−d 1. 195 07 − 0.845 18
10.2. GENERAL ONE-PERIOD BINOMIAL MODEL 65

π d = 1 − πu = 1 − 0.456 807 = 0.543 193


Step 1 Find the stock price tree
Period 0 1 2 3
Su3 = 187.7471
Su2 = 157.1013
Su = 131.4577 Su2 d = 132.7789
S = 110 Sud = 111.1055
Sd = 92.9699 Sd2 u = 93.9042
Sd2 = 78.5763
Sd3 = 66.4112

Step 2 Calculate the payoff at expiration (terminal payoff)

Period 0 1 2 3
Vu3 = max (0, 187.7471 − 100) = 87. 747 1
Vu2
Vu Vu2 d = max (0, 132.7789 − 100) = 32. 778 9
V Vud
Vd Vd2 u = max (0, 93.9042 − 100) = 0
V d2
Vd3 = max (0, 66.4112 − 100) = 0

Step 3 Calculate the value of the American call at Period 2 by taking


the greater of the backwardized value and the exercise value at each node.

The backwardized values at Period 2 are:


Cu2 = e−rh (πu Vu3 + π d Vu2 d ) = e−0.05(1/3) (0.456 807 × 87. 747 1 + 0.543 193 × 32. 778 9) =
56. 931 9
Cud = e−rh (πu Vu2 d + πd Vd2 u ) = e−0.05(1/3) (0.456 807 × 32. 778 9 + 0.543 193 × 0) =
14. 726 1
Cd2 = e−rh (π ud Vd2 u + π d Vd3 ) = e−0.05(1/3) (0.456 807 × 0 + 0.543 193 × 0) =
0

The exercise values at Period 2 are:


¡ ¢
EVu2 = max 0, Su2 − K = max (0, 157.1013 − 100) = 57. 101 3
EVud = max ¡(0, Sud − K)
¢ = max (0, 111.1055 − 100) = 11. 105 5
EVd2 = max 0, Sd2 − K = max (0, 78.5763 − 100) = 0

We take the greater of the two as the values.


Vu2 = max (Cu2 , EVu2 ) = max (56. 931 9, 57. 101 3) = 57. 101 3
Vud = max (Cud , EVud ) = max (14. 726 1, 11. 105 5) = 14. 726 1
Vd2 = max (Cd2 , EVd2 ) = max (0, 0) = 0

Now we have:
66 CHAPTER 10. BINOMIAL OPTION PRICING: I

Period 0 1 2 3
Vu3 = 87. 747 1
Vu2 = 57. 101 3
Vu Vu2 d = 32. 778 9
V Vud = 14. 726 1
Vd Vd2 u = 0
Vd2 = 0
Vd3 = 0

Step 4 Calculate the value of the American call at Period 1 by taking


the greater of the backwardized value and the exercise value at each node.
Cu = e−rh (π u Vu2 + π d Vud ) = e−0.05(1/3) (0.456 807 × 57. 101 3 + 0.543 193 × 14. 726 1) =
33. 520 02
Cd = e−rh (π u Vud + π d Vd2 ) = e−0.05(1/3) (0.456 807 × 14. 726 1 + 0.543 193 × 0) =
6. 615 8

EVu = max (0, Su − K) = max (0, 131.4577 − 100) = 31. 457 7


EVd = max (0, Sd − K) = max (0, 92.9699 − 100) = 0
Vu = max (Cu , EVu ) = max (33. 520 02, 31. 457 7) = 33. 520 02
Vd = max (Cd , EVd ) = max (6. 615 8, 0) = 6. 615 8

Now we have:
Period 0 1 2 3
Vu3 = 87. 747 1
Vu2 = 57. 101 3
Vu = 33. 520 02 Vu2 d = 32. 778 9
V Vud = 14. 726 1
Vd = 6. 615 8 Vd2 u = 0
Vd2 = 0
Vd3 = 0

Step 5 Calculate the value of the American call at Period 0 by taking


the greater of the backwardized value and the exercise value at each node.

C = e−rh (π u Vu + π d Vd ) = e−0.05(1/3) (0.456 807 × 33. 520 02 + 0.543 193 × 6. 615 8) =


18. 593 35
EV = max (0, S − K) = max (0, 110 − 100) = 10
V = max (C, EV ) = max (18. 593 35, 10) = 18. 593 35

Step 6 Calculate the replicating portfolio


Our goal is to replicate the following values:
10.2. GENERAL ONE-PERIOD BINOMIAL MODEL 67

Period 0 1 2 3
Vu3 = 87. 747 1
Vu2 = 57. 101 3
Vu = 33. 520 02 Vu2 d = 32. 778 9
V = 18. 593 35 Vud = 14. 726 1
Vd = 6. 615 8 Vd2 u = 0
Vd2 = 0
Vd3 = 0

Cu − Cd Su Cd − Sd Cu
4 = e−δh B = e−rh
Su − Sd Su − Sd
Period 0 1 2

4u2 = 0.988 401


4u = 0.910 598
4 = 0.690 923 4ud = 0.988 401
4d = 0.447 45
4d2 = 0

Period 0 1 2

Bu2 = −98. 347 1


Bu = −86. 185 1
B = −57. 408 1 Bud = −77. 870 8
Bd = −34. 983 9
Bd2 = 0

Vu3 − Vu2 d 87. 747 1 − 32. 778 9


4u2 = e−δh = e−0.035(1/3) = 0.988 401
Su3 − Su2 d 187.7471 − 132.7789
Su3 Vu2 d − Su2 dVu3 187.7471 (32. 778 9) − 132.7789 (87. 747 1)
Bu2 = e−rh = e−0.05(1/3) =
Su3 − Su2 d 187.7471 − 132.7789
−98. 347 1

Vu2 d − Vud2 32. 778 9 − 0


4ud = e−δh 2 2
= e−0.035(1/3) = 0.988 401
Su d − Sud 187.7471 − 132.7789
3 2
Su dVu2 d − Su dVu3 187.7471 (0) − 132.7789 (32. 778 9)
Bud = e−rh 3 2
= e−0.05(1/3) =
Su − Su d 187.7471 − 132.7789
−77. 870 8

Vd2 u − Vd3
4d2 = e−δh =0
Sd2 u − Sd3
Sd uVd3 − Sd3 Vd2 u
2
Bd2 = e−rh =0
Sd2 u − Sd3
68 CHAPTER 10. BINOMIAL OPTION PRICING: I

Vu2 − Vud 57. 101 3 − 14. 726 1


4u = e−δh 2
= e−0.035(1/3) = 0.910 598
Su − Sud 157.1013 − 111.1055
2
Su Vud − SudVu2 157.1013 (14. 726 1) − 111.1055 (57. 101 3)
Bu = e−rh = e−0.05(1/3) =
Su2 − Sud 157.1013 − 111.1055
−86. 185 1

Vud − Vd2 14. 726 1 − 0


4d = e−δh = e−0.035(1/3) = 0.447 45
Sud − Sd2 111.1055 − 78.5763
2
Su Vud − SudVu2 111.1055 (0) − 78.5763 (14. 726 1)
Bd = e−rh = e−0.05(1/3) =
Su2 − Sud 111.1055 − 78.5763
−34. 983 9

Vu − Vd 33. 520 02 − 6. 615 8


4 = e−δh = e−0.035(1/3) = 0.690 923
Su − Sd 131.4577 − 92.9699
SuVd − SdVu 131.4577 (6. 615 8) − 92.9699 (33. 520 02)
B = e−rh = e−0.05(1/3) =
Su − Sd 131.4577 − 92.9699
−57. 408 1

10.2.3 Options on currency

Now let’s find the price of a European call option on €1. The underlying asset is
€1. The option expires in h years. The current dollar value of the underlying is
S (so €1 = $S at t = 0). In h years the dollar value of the underlying asset €1
can go up to $Su or go down to $Sd . The strike price is $K. The continuously
compounded risk-free interest rate on dollars is r per year (so dollars earn a
return r). The continuously compounded interest rate on the underlying asset
of €1 is δ per year (so euros earn a return δ).
Let’s calculate the call price using one-period binomial tree. The asset price
tree and the option payoff tree are as follows:
Asset price tree (in dollars) Option payoff (in dollars)

time 0 h time 0 h
Su Cu = max (0, Su − K)
S C
Sd Cd = max (0, Sd − K)
To replicate the payoff, at t = 0 we’ll buy ∆ units of the underlying (i.e.
buy ∆ euros) and simultaneously invest B dollars in a savings account. Since
the underlying asset €∆ earns interest at a continuous interest δ, it will grow
into €∆eδh at T , which is worth $∆eδh Su in the up state and $∆eδh Sd in the
down state.

We want our replicating portfolio to match the option payoff. So we have:


∆eδh Su Berh Cu
4S + B = C
∆eδh Sd Berh Cd
t=0 t=h t=0 t=h t=0 t=h
10.2. GENERAL ONE-PERIOD BINOMIAL MODEL 69

½
∆eδh Su + Berh = Cu
∆eδh Sd + Berh = Cd
Solving these equations, we get:

Cu − Cd
4 = e−δh (10.26)
Su − Sd
Su Cd − Sd Cu
B = e−rh (10.27)
Su − Sd

Equation 10.26 and 10.27 are exactly the same as Equation 10.13 and 10.14.
This tells us that if we treat the currency as a stock and treat the euro return
δ as the stock’s dividend rate, we can find the currency option’s price and the
replicating portfolios using all the formulas available for a stock option.

Example 10.2.12. Reproduce Derivatives Markets Figure 10.9. Here is the re-
cap of the information on an American put option on €1. The current exchange
rate is S = $1.05/€. The strike price is K = $1.10. The annualized standard
deviation of the continuously compounded return on dollars is σ = 10%. The
continuously compounded risk-free rate on dollars is r = 5% per year. The
continuously compounded return on euros is δ = 3.1% per year. The option
expiration date is T = 0.5 year. Use a 3-period binomial tree to calculate the
option premium.

Solution.
T 1
The length period is h = =
3 6
√ √
u = e(r−δ)h+σ√ h = e(0.055−0.031)1/6+0.1√1/6 = 1. 045 845
d = e(r−δ)h−σ h = e(0.055−0.031)1/6−0.1 1/6 = 0.963 845
e(r−δ)h − d e(0.055−0.031)1/6 − 0.963 845
πu = = = 0.489 795
u−d 1. 045 845 − 0.963 845
π d = 1 − πu = 1 − 0.489 795 = 0.510 205

Step 1 Find the underlying asset price tree


Period 0 1 2 3
Su3 = 1.2011
Su2 = 1.1485
Su = 1.0981 Su2 d = 1.1070
S = 1.05 Sud = 1.0584
Sd = 1.0120 Sd2 u = 1.0202
2
Sd = 0.9754
Sd3 = 0.9402

Step 2 Calculate the payoff at expiration (terminal payoff)


70 CHAPTER 10. BINOMIAL OPTION PRICING: I

Period 0 1 2 3
Vu3 = max (0, 1.10 − 1.2011) = 0
Vu2
Vu Vu2 d = max (0, 1.10 − 1.1070) = 0
V Vud
Vd Vd2 u = max (0, 1.10 − 1.0202) = 0.079 8
Vd2
Vd3 = max (0, 1.10 − 0.9402 ) = 0.159 8

Step 3 Calculate the value of the American put at Period 2 by taking


the greater of the backwardized value and the exercise value at each node.

The backwardized values at Period 2 are:


Cu2 = e−rh (π u Vu3 + π d Vu2 d ) = e−0.055(1/6) (0.489 795 × 0 + 0.510 205 × 0) =
0
Cud = e−rh (π u Vu2 d + π d Vd2 u ) = e−0.055(1/6) (0.489 795 × 0 + 0.510 205 × 0.079 8) =
0.04 034
Cd2 = e−rh (πud Vd2 u + πd Vd3 ) = e−0.055(1/6) (0.489 795 × 0.079 8 + 0.510 205 × 0.159 8) =
0.119 5

The exercise values at Period 2 are:


¡ ¢
EVu2 = max 0, K − Su2 = max (0, 1.10 − 1.1485) = 0
EVud = max ¡(0, K − Sud)
¢ = max (0, 1.10 − 1.0584) = 0.041 6
EVd2 = max 0, K − Sd2 = max (0, 1.10 − 0.9754) = 0.124 6

We take the greater of the two as the values.


Vu2 = max (Cu2 , EVu2 ) = max (0, 0) = 0
Vud = max (Cud , EVud ) = max (0.04 034 , 0.041 6) = 0.041 6
Vd2 = max (Cd2 , EVd2 ) = max (0.119 5, 0.124 6) = 0.124 6

Now we have:
Period 0 1 2 3
Vu3 = 0
Vu2 = 0
Vu Vu2 d = 0
V Vud = 0.041 6
Vd Vd2 u = 0.079 8
Vd2 = 0.124 6
Vd3 = 0.159 8

Step 4 Calculate the value of the American put at Period 1 by taking


the greater of the backwardized value and the exercise value at each node.
Cu = e−rh (π u Vu2 + π d Vud ) = e−0.055(1/6) (0.489 795 × 0 + 0.510 205 × 0.041 6) =
0.02 10
10.2. GENERAL ONE-PERIOD BINOMIAL MODEL 71

Cd = e−rh (π u Vud + π d Vd2 ) = e−0.055(1/6) (0.489 795 × 0.041 6 + 0.510 205 × 0.124 6) =
0.08 31

EVu = max (0, K − Su ) = max (0, 1.10 − 1.0981) = 0.001 9


EVd = max (0, K − Sd ) = max (0, 1.10 − 1.0120 ) = 0.088
Vu = max (Cu , EVu ) = max (0.02 10 , 0.001 9) = 0.021
Vd = max (Cd , EVd ) = max (0.088 , 0.021) = 0.088

Now we have:
Period 0 1 2 3
Vu3 = 0
Vu2 = 0
Vu = 0.021 Vu2 d = 0
V Vud = 0.041 6
Vd = 0.088 Vd2 u = 0.079 8
Vd2 = 0.124 6
Vd3 = 0.159 8

Step 5 Calculate the value of the American put at Period 0 by taking


the greater of the backwardized value and the exercise value at each node.

C = e−rh (πu Vu + πd Vd ) = e−0.055(1/6) (0.489 795 × 0.021 + 0.510 205 × 0.088) =


0.05 47
EV = max (0, K − S ) = max (0, 1.10 − 1.05) = 0.05
V = max (C, EV ) = max (0.05 47, 0.05) = 0.054 7

Now we have:
Period 0 1 2 3
Vu3 = 0
Vu2 = 0
Vu = 0.021 Vu2 d = 0
V = 0.054 7 Vud = 0.041 6
Vd = 0.088 Vd2 u = 0.079 8
Vd2 = 0.124 6
Vd3 = 0.159 8

You can verify that the replicating portfolios are as follows:


Period 0 1 2

4u2 = 0.0000
4u = −0.4592
4 = −0.7736 4ud = −0.9151
4d = −0.9948
4d2 = −0.9948
72 CHAPTER 10. BINOMIAL OPTION PRICING: I

Period 0 1 2

Bu2 = $0.0000
Bu = $0.5253
B = $0.8669 Bud = $1.0089
Bd = $1.0900
Bd2 = $1.0900

10.2.4 Options on futures contracts


Suppose we want to find the price of a European call option on a stock futures
contract. The underlying asset is futures. The option expires in h years. The
current price of the underlying asset is F0,h = F , where F0,h is the price of a
futures contract signed at t = 0 and expiring on date h.In h years the futures
price can go up to Fh,h = Fu = F u or go down to Fh,h = Fd = F d, where Fh,h
is the price of a futures contract signed at t = h and expiring on date h years
(i.e. expiring immediately). Fh,h is equal to the spot price Sh . The fact that
Fh,h can be either Fu or Fd is the same as fact that the stock price at t = h is
either Fu or Fd . The continuously compounded risk-free interest rate is r per
year. Stocks pay dividend at a continuously compounded rate of δ per year.
The strike price is K.
The find the European call price on the futures, we draw the price tree of
the underlying asset and the payoff tree.
Asset price tree Option payoff

time 0 h time 0 h
Fu = F u Cu = max (0, Fu − K)
F C
Fd = F d Cd = max (0, Fd − K)
We form a replicating portfolio at t = 0 by entering ∆ futures contracts as
a buyer and simultaneously putting $B in the savings account. Assume that
no margin account is needed before one enters a futures contract. Then the
cost of entering a futures contracts is zero. At the contract expiration date h,
the ∆ futures contracts are settled in cash. If the futures price at expiration is
Fu > F , then the seller in the futures pays ∆ (Fu − F ) to us, the buyer.
If on the other hand, the futures price at h is Fd < F , then we pay the seller
∆ (F − Fd ). Paying ∆ (F − Fd ) is the same as receiving ∆ (Fd − F ).
We assume that Fd < F < Fu holds so there’s no arbitrage.

So the cash flow of the underlying asset (futures) is:


time 0 h
$∆ (Fu − F )
$0
$∆ (Fd − F )
10.2. GENERAL ONE-PERIOD BINOMIAL MODEL 73

We want the replicating portfolio and the option have the same payoff.
∆ (Fu − F ) Berh Cu
0 + B = C
∆ (Fd − F ) Berh Cd
t=0 t=h t=0 t=h t=0 t=h
½
∆ (Fu − F ) + Berh = Cu
∆ (Fd − F ) + Berh = Cd

Solving these equations, we get:


Cu − Cd Cu − Cd
4= = (10.28)
Fu − Fd F (u − d)
µ ¶
1−d u−1
B = e−rh Cu + Cd (10.29)
u−d u−d
µ ¶
1−d u−1
V = 4 × 0 + B = B = e−rh Cu + Cd (10.30)
u−d u−d
Define

F − Fd 1−d
πu = = (10.31)
Fu − Fd u−d
Fu − 1 u−1
πd = = (10.32)
Fu − Fd u−d

Then
V = e−rh (Cu πu + Cd π d ) (10.33)

Equation 10.30 is the same as Equation 10.16 if we set r − δ = 0 and S = F .


Consequently, we can find the price of a futures option by using Equation 10.16
Cu − Cd
if we set δ = r. We just need to remember that 4 = instead of
µ Fu − Fd¶
Cu − Cd 1−d u−1
4 = e−rδ and that B = V = e−rh Cu + Cd instead of
Fu − Fd u−d u−d
Su Cd − Sd Cu
B = e−rh .
Su − Sd
How we can specify u or d?

up price of Fh,h down price of Fh,h


u= u=
F0,h F0,h
( √
S0 e(r−δ)h+σ√h
We know that Fh,h = Sh = . In addition, F0,h = S0 e(r−δ)h .
S0 e(r−δ)h−σ h
74 CHAPTER 10. BINOMIAL OPTION PRICING: I


up price of Fh,h S0 e(r−δ)h+σ h √
σ h
u= = = e
F0,h S0 e(r−δ)h √
down price of Fh,h S0 e(r−δ)h−σ h √
−σ h
d= = = e
F0,h S0 e(r−δ)h

u = eσ h
(10.34)

d = e−σ h
(10.35)

Equation 10.34 and 10.35 are the same as Equation 10.24 and 10.25 if we
set δ = r. We can use the stock option’s formula on u and d for futures options.
Tip 10.2.6. To find the price of a futures option, just use the price formula for
a stock option and set δ = r. However, remember that for aµ futures option, 4 = ¶
Cu − Cd Cu − Cd 1−d u−1
instead of 4 = e−rh and B = V = e−rh Cu + Cd
Fu − Fd Fu − Fd u−d u−d
Su Cd − Sd Cu
instead of B = e−rh .
Su − Sd
Example 10.2.13. Let’s reproduce Derivatives Markets Figure 10.10. Here is
the recap of the information on an American call on a futures contract. The
current futures price is S = 300. The strike price K = 300. The annualized
standard deviation of the continuously compounded stock index return is σ =
10%. The continuously compounded risk-free rate per year is r = 5%.The option
expiration date is T = 1 year. Use a 3-period binomial tree to calculate the
option premium.

Solution.
We’ll reuse the stock option formula and set δ = r.
1
Each period is h = year long.
3
√ √ √
u = e(r−δ)h+σ√ h = eσ √h
= e0.1 1/3
√ = 1. 059 434
d = e(r−δ)h−σ h = e−σ h = e−0.1 1/3 = 0.943 900
e(r−δ)h − d 1−d 1 − 0.943 900
πu = = = = 0.485 57
u−d u−d 1. 059 434 − 0.943 900
πd = 1 − π u = 1 − 0.485 57 = 0.514 43

Step 1 Find the underlying asset price tree


Period 0 1 2 3
Su3 = 356.7330
Su2 = 336.7203
Su = 317.8303 Su2 d = 317.8303
F = 300 Sud = 300.0000
Sd = 283.1700 Sd2 u = 283.1700
2
Sd = 267.2842
Sd3 = 252.2895
10.2. GENERAL ONE-PERIOD BINOMIAL MODEL 75

Step 2 Calculate the payoff at expiration (terminal payoff)

Period 0 1 2 3
Vu3 = max (0, 356.7330 − 300) = 56. 733
Vu2
Vu Vu2 d = max (0, 317.8303 − 300) = 17. 830 3
V Vud
Vd Vd2 u = max (0, 283.1700 − 300) = 0
Vd2
Vd3 = max (0, 252.2895 − 300) = 0

Step 3 Calculate the value of the American call at Period 2 by taking


the greater of the backwardized value and the exercise value at each node.

The backwardized values at Period 2 are:


Cu2 = e−rh (πu Vu3 + π d Vu2 d ) = e−0.05(1/3) (0.485 57 × 56. 733 + 0.514 43 × 17. 830 3) =
36. 113 3
Cud = e−rh (πu Vu2 d + πd Vd2 u ) = e−0.05(1/3) (0.485 57 × 17. 830 3 + 0.514 43 × 0) =
8. 514 7
Cd2 = e−rh (π ud Vd2 u + π d Vd3 ) = e−0.05(1/3) (0.485 57 × 0 + 0.514 43 × 0) =
0

The exercise values at Period 2 are:


¡ ¢
EVu2 = max 0, Su2 − K = max (0, 336.7203 − 300) = 36. 720 3
EVud = max ¡(0, Sud − K)
¢ = max (0, 300.0000 − 300) = 0
EVd2 = max 0, Sd2 − K = max (0, 267.2842 − 300) = 0

We take the greater of the two as the values.


Vu2 = max (Cu2 , EVu2 ) = max (36. 113 3, 36. 720 3) = 36. 720 3
Vud = max (Cud , EVud ) = max (8. 514 7, 0) = 8. 514 7
Vd2 = max (Cd2 , EVd2 ) = max (0, 0) = 0

Now we have:
Period 0 1 2 3
Vu3 = 56. 733
Vu2 = 36. 720 3
Vu Vu2 d = 17. 830 3
V Vud = 8. 514 7
Vd Vd2 u = 0
Vd2 = 0
Vd3 = 0

Step 4 Calculate the value of the American call at Period 1 by taking


the greater of the backwardized value and the exercise value at each node.
76 CHAPTER 10. BINOMIAL OPTION PRICING: I

Cu = e−rh (π u Vu2 + π d Vud ) = e−0.05(1/3) (0.485 57 × 36. 720 3 + 0.514 43 × 8. 514 7) =


21. 843 4
Cd = e−rh (π u Vud + π d Vd2 ) = e−0.05(1/3) (0.485 57 × 8. 514 7 + 0.514 43 × 0) =
4. 066 2

EVu = max (0, Su − K) = max (0, 317.8303 − 300) = 17. 830 3


EVd = max (0, Sd − K) = max (0, 283.1700 − 300) = 0
Vu = max (Cu , EVu ) = max (21. 843 4, 17. 830 3) = 21. 843 4
Vd = max (Cd , EVd ) = max (4. 066 2, 0) = 4. 066 2
Now we have:
Period 0 1 2 3
Vu3 = 56. 733
Vu2 = 36. 720 3
Vu = 21. 843 4 Vu2 d = 17. 830 3
V Vud = 8. 514 7
Vd = 4. 066 2 Vd2 u = 0
Vd2 = 0
Vd3 = 0

Step 5 Calculate the value of the American call at Period 0 by taking


the greater of the backwardized value and the exercise value at each node.

C = e−rh (π u Vu + π d Vd ) = e−0.05(1/3) (0.485 57 × 21. 843 4 + 0.514 43 × 4. 066 2) =


12. 488 4
EV = max (0, S − K) = max (0, 300 − 300) = 0
V = max (C, EV ) = max (12. 488 4, 0) = 12. 488 4
Now we have:
Period 0 1 2 3
Vu3 = 56. 733
Vu2 = 36. 720 3
Vu = 21. 843 4 Vu2 d = 17. 830 3
V = 12. 488 4 Vud = 8. 514 7
Vd = 4. 066 2 Vd2 u = 0
Vd2 = 0
Vd3 = 0

Next, we need to find the replicating portfolio. Our goal is to replicate the
following values:
Period 0 1 2 3
Vu3 = 56. 733
Vu2 = 36. 720 3
Vu = 21. 843 4 Vu2 d = 17. 830 3
V = 12. 488 4 Vud = 8. 514 7
Vd = 4. 066 2 Vd2 u = 0
Vd2 = 0
Vd3 = 0
10.2. GENERAL ONE-PERIOD BINOMIAL MODEL 77

Using Equation 10.29 and 10.29, you should get:


Period 0 1 2

4u2 = 1
4u = 0.7681
4 = 0.5129 4ud = 0.5144
4d = 0.2603
4d2 = 0

Period 0 1 2

Bu2 = $36. 720 3


Bu = $21. 843 4
B = $12. 488 4 Bud = $8. 514 7
Bd = $4. 066 2
Bd2 = $0

For example,
V 3 − Vu2 d 56. 733 − 17. 830 3
4u2 = u = = 1.0
Su3 − Su2 d 356.7330 − 317.8303
Bu2 = Vu2 = $36. 720 3

Vu − Vd 21. 843 4 − 4. 066 2


4= = = 0.512 9
Su − Sd 317.8303 − 283.1700
B = V = $12. 488 4
I recommend that you reproduce my replicating portfolio in each node.

Options on commodities

The textbook is brief on this topic. So you don’t need to spend lot of time on it.
This is the main idea: we can price commodity options using the same framework
for pricing stock options if we can build a replicating portfolio using commodities
and bonds with zero transaction cost. In reality, it’s hard to build a replicating
portfolio using commodities. Unlike stocks, commodities such as corn may incur
storage cost or other cost. It may be impossible to short sell commodities.
As such, our ability to build a replicating portfolio is limited. However, if
we can build any replicating portfolios using commodities instantaneously and
effortlessly, commodity options and stock options are conceptually the same.
We can use the same framework to calculate the price of a commodity option
and the price of a stock option.

Example 10.2.14. Here is the information on an American call on a com-


modity. The current commodity price is 110. The strike price is K = 100.
78 CHAPTER 10. BINOMIAL OPTION PRICING: I

The annualized standard deviation of the continuously compounded return on


the commodity is σ = 30%. The continuously compounded risk-free rate per
year is r = 5%. The continuously compounded lease rate of the commodity is
δ = 3.5% per year. The option expiration date is T = 1 year. Use a 3-period
binomial tree to calculate the option premium. Assume that you can effortlessly
and instantaneously build any replicating portfolio using the commodity and the
bond.

We just treat the commodity as a stock. The lease rate δ = 3.5% is the same
as the stock dividend rate. We can use the framework for pricing stock options
to price this commodity option. The solution to this problem is in the textbook
Figure 10.8.

Options on bonds
The textbook points out two major differences between bonds and stocks:

1. A bond’s volatility decreases over time as the bond approaches its matu-
rity. A stock’s volatility doesn’t have this pattern.
2. When pricing a stock option, we assume that the interest rate is constant
over time. The random variable is the stock price under a fixed interest.
However, when pricing a bond, we can’t assume that the interest rate is
constant any more. If the interest rate is constant, then the bond’s price
is known with 100% certainty. If the bond price doesn’t change randomly,
an option on the bond has zero value. Who wants to buy a call or put on
an asset whose price is known with 100% certainty?

Because of these differences, options on bonds should be treated differently


from options on stocks. This is all you need to know about bond options right
now. Chapter 24 will cover more on options on bonds.
Chapter 11

Binomial option pricing: II

11.1 Understanding early exercise


Pros and cons of exercising an American call option early
Pro:

• Receive the stock and future dividends

Cons

• Pay the strike price early and lost interest that could have earned on the
strike price.

• Lose the insurance implicit in the call. If you hold the option, the stock
price might be below the strike price at expiration, in which case you
would not exercise the option. However, if you exercise the American call
early, you lose the privilege of not exercising it. To understand this point,
suppose you go to a garage sale and find a book you like that sells for
only $1. You think "How cheap the book is. I must buy it." You pay $1
and buy the book. You think you get a good deal. However, if you resist
the temptation to buy the book immediately and wait till the end of the
garage sale, the book’s price may drop to $0.25. Better yet, you may even
get the book for free. That same thing may happen when you exercise
an American call early. At the moment, the stock price is high and you
might be attempted to exercise the call. However, if we wait for a while,
the stock price may drop below the strike price.

Next, the textbook gives us a rule to determine when it’s optimal to exercise
a perpetual 1 American call early. For a perpetual American call with zero
volatility, it’s optimal to exercise a perpetual American call early if the dividend
to be received exceeds the interest savings on the strike price:
1 The textbook errata say the formulas work for an infinitely-lived American call option.

79
80 CHAPTER 11. BINOMIAL OPTION PRICING: II

δST > rK
¡ δ ¢
µ ¶ gain if you exercise the call early is ST e − 1 =
The annual dividend you
1
ST 1 + δ + δ 2 + ... − 1 ≈ δST for a small δ. The annual interest earned on
2 µ ¶
1
K is K (e − 1) = K 1 + r + r2 + ... − 1 ≈ rK for a small r. If you early
r
2
exercise, you’ll pay K and receive ST ; during a year you’ll receive δST dividend
but you will lose rK interest. Hence early exercise is optimal if the annual
dividend exceeds the annual interest, δST > rK.
And it’s optimal to defer exercising a perpetual American call if the interest
savings on the strike price exceeds dividends lost:
δST < rK

Example 11.1.1. A perpetual American call option on a stock has a strike


price $50. The stock pays dividend at a continuously compounded rate of 8%
per year. The continuously compounded risk-free interest rate is 6% per year.
The volatility of the stock price is zero. When is it optimal to exercise this
American call option early? When is it optimal to defer exercise?

Sδ > rK
rK 0.06 (50)
S> = = 37. 5
δ 0.08
Once the stock price becomes greater than 37. 5, then it’s optimal to exercise
this perpetual American call option early.
If the stock price is less than 37. 5, then it’s optimal to defer exercising this
perpetual American call option early.
Please note that zero volatility doesn’t mean that the stock price is a con-
stant. It means that the stock price is known in advance with 100% certainty.

Next, the textbook says that the decision to exercise a perpetual American
call option early is complex if the volatility of the stock price is greater than
zero. In this case, the insurance in the call option is greater than zero. For
each non-zero volatility, there’s a lowest stock price at which the early exercise
is optimal.

11.2 Understanding risk-neutral probability


Risk neutral probability is explained in the previous chapter. There’s not much
new information about the risk neutral probability in this chapter. The key
point to remember is that risk neutral probability is a shortcut or a math trick
that enables us to quickly find the price of an option. We can use risk neutral
probability to find the correct price of an option whether consumers are really
risk neutral or not.
The risk neutral probability is similar to the moment generating function in
Exam P. The moment generating function (M GF ) is merely a math trick that
11.2. UNDERSTANDING RISK-NEUTRAL PROBABILITY 81

allows us to quickly find the mean and variance of a random variable (hence
the name "moment generating"). If we don’t use GM F , we can still find the
mean and variance, but we have to work a lot harder. With the help of GM F ,
we can quickly find the mean and variance. Similarly, if we don’t use the risk
neutral probability, we can still find the option price, but we have to work a lot
harder. Once we use risk-neutral probability, we can quickly find an option’s
price. Risk neutral probability is merely a math risk.
By the way, one investment consultant told me that risk neutral probability
is often hard to non-technical clients to understand. If you tell a non-technical
client that an option price is calculated using risk-neutral probability and that
the risk neutral probability not real, the client often immediately ask "So the
price you calculated is wrong then?" It may take the consultant a while to
explain why the risk neutral probability is not real yet the price is still correct.

11.2.1 Pricing an option using real probabilities


Next, the textbook answers a frequently asked question: Can we calculate the
option price as the expected payoff using real probabilities of the stock price?
The answer is "Yes if you know the discount rate."
Let p and q = 1−p represent the real probability of stock going up and down
respectively. Let γ represent the real discount rate (instead of the risk-free rate,
which is used to discount payoff in the risk neutral world).

uS with real probability p Cu with real probability p


S C =?
dS with real probability q Cd with real probability q
t=0 t=h t=0 t=h

Then the price of a European option expiring in h years is:


C = e−γh (pCu + qCd )

How can we find the real probability p and the real discount rate γ? Suppose
we know that the expected return on the stock during [0, h] is α. Assume that
the continuously compounded dividend rate is δ per year. If we have one stock
at t = 0, then at t = h we’ll have eδh stocks. The value at t = 0 is the expected
value at t = h discounted at rate α.

uSeδh with real probability p


S
dSeδh with real probability q
Value t = 0 Value t = h
¡ ¢ e(α−δ)h − d
S = e−αh puSeδh + qdSeδh →p=
u−d

e(α−δ)h − d
p= (11.1)
u−d
82 CHAPTER 11. BINOMIAL OPTION PRICING: II

u − e(α−δ)h
q= (11.2)
u−d

Tip 11.2.1. If we set r = α Equation 10.16 and 10.17 become Equation 11.1
and 11.2. So you just need to memorize Equation 10.16 and 10.17. To get the
formulas for the real probability, just set r = α.

We can use the replicating portfolio to find γ. Suppose the replicating port-
folio at t = 0 consists of ∆ shares of stock and putting $B in a savings account.
We already know that we can calculate ∆ and B using Equation 10.13 and
10.13.
At t = 0, the replicating portfolio is worth ∆S + B. At t = h, the replicating
portfolio consists of ∆eδh shares of stock and $Berh in a savings account, which
is worth ∆eδh dS + Berh in the up state and ∆eδh dS − Berh in the down state.

uS∆eδh + Berh with real probability p


∆S + B
dS∆eδh + Berh with real probability q
Value t = 0 Value t = h

The value at
£ t¡ = 0 is the expected
¢ value
¡ at t = h discounted
¢¤ at rate γ:
∆S + B = p uS∆eδh + Berh + q dS∆eδh + Berh e−γh
£ ¡ ¢ ¤
= ∆ puSeδh + qdSeδh + Berh (p + q) e−γh
¡ ¢
= ∆Seαh + Berh e−γh

∆S + B
e−γh = (11.3)
∆Seαh + Berh

∆S + B
C = e−γh (pCu + qCd ) = (pCu + qCd )
∆Seαh + Berh
Since C = ∆S + B, we just need to prove that pCu + qCd = ∆Seαh + Berh .

pCu + qCd
e(α−δ)h − d u − e(α−δ)h
= Cu + C
u−d
∙ (r−δ)h u−d ¸ ∙ ¸
(α−δ)h
e −d e − e(r−δ)h u − e(r−δ)h e(r−δ)h − e(α−δ)h
= Cu + Cu + Cd + Cd
u−d u−d u−d u−d
∙ (r−δ)h ¸
e −d u − e(r−δ)h e(α−δ)h − e(r−δ)h
= Cu + Cd + (Cu − Cd )
u−d u−d u−d

According to Equation 10.15:


e(r−δ)h − d u − e(r−δ)h
Cu + Cd = erh (∆S + B)
u−d u−d
11.2. UNDERSTANDING RISK-NEUTRAL PROBABILITY 83

Cu − Cd
According to Equation 10.13, = ∆eδh S
u−d
pCu + qCd £ ¤
= erh (∆S + B) + ∆eδh¡ S e(α−δ)h¢ − e(r−δ)h
= erh (∆S + B) + ∆S eαh − erh = eαh ∆S + Berh

∆S + B
→C= (pCu + qCd ) = ∆S + B
∆Seαh + Berh
The above derivation tell us that

• Real probabilities lead to the same answer as the risk neutral probability
• Any consistent pair of (α, γ) will produce the correct answer. The above
derivation doesn’t require that α has to be reasonable or precise. Any α
will generate a corresponding γ. Together, α and γ will produce the option
price correctly.
• The simplest calculation is to set α = γ = r. Setting α = γ = r means
using risk neutral probabilities.

Example 11.2.1. Reproduce the textbook Figure 11.3 (which is the same as the
textbook Figure 10.5). A European call option has strike price K = 40. The
current price is S = 41. The annualized standard deviation of the continuously
compounded stock return is σ = 30%. The continuously compounded risk-free
rate per year is r = 8%. The continuously compounded dividend rate per year
is δ = 0%. The continuously compounded expected return on the stock per year
is α = 15%. The option expiration date is T = 1 year. Use a 1-period binomial
tree and real probabilities to calculate the option premium.

Solution.

√ √
u = e(r−δ)h+σ √h = e(0.08−0)1+0.3√1 = 1. 462 3
u = e(r−δ)h−σ h = e(0.08−0)1−0.3 1 = 0.802 5
1. 462 3 (41) = 59. 954 with real probability p
41
0.802 5 (41) = 32. 9023 with real probability q
t=0 t=h=1

Cu = max (0, 59. 954 − 40) = 19. 954 with real probability p
C =?
Cd = max (0, 32. 9023 − 40) = 0 with real probability q
t=0 t=h=1

Calculate the real probabilities:


84 CHAPTER 11. BINOMIAL OPTION PRICING: II

41e0.15(1) = 59. 954p + 32. 9023 (1 − p)


p = 0.544 6 q = 1 − p = 1 − 0.544 6 = 0.455 4

Calculate the replicating portfolio:


Cu − Cd 19. 954 − 0
4 = e−δh = e−0(1) = 0.737 6
Su − Sd 59. 954 − 32. 9023
Su Cd − Sd Cu 59. 954 (0) − 32. 9023 (19. 954)
B = e−rh = e−0.08(1) = −22.
Su − Sd 59. 954 − 32. 9023
403 6

Calculate the
¡ discounting rate:
¢
∆S + B = ∆Seαh + Berh e−γh
(∆S + B) eγh = ∆Seαh + Berh
(0.737 6 × 41 − 22. 403 6) eγ(1) = 0.737 6 × 41e0.15(1) − 22. 403 6e0.08(1)
eγ(1) = 1. 386 γ = 0.326 4

Calculate the option price:


C = e−γh (pCu + qCd ) = e−0.326 4(1) (0.544 6 × 19. 954 + 0.455 4 × 0) = 7. 84
This option price is the same as the price in the textbook Figure 10.3.

Example 11.2.2. Reproduce the textbook Figure 11.4 (the risk neutral solution
to an otherwise identical European call is in the textbook Figure 10.5). Here is
the recap of the information on an American call. The current stock price is 41.
The strike price K = 40. The annualized standard deviation of the continuously
compounded stock return is σ = 30%. The continuously compounded risk-free
rate per year is r = 8%. The continuously compounded expected return on the
stock per year is α = 15%.The continuously compounded dividend rate per year
is δ = 0%.The option expiration date is T = 1 year. Use a 3-period binomial
tree and real probabilities to calculate the option premium.

Solution.

1
Each period is h = year long.
3
√ √
u = e(r−δ)h+σ√ h = e(0.08−0)1/3+0.3√1/3 = 1. 221 246
d = e(r−δ)h−σ h = e(0.08−0)1/3−0.3 1/3 = 0.863 693
Stock price
Period 0 1 2 3
Su3 = 74.6781
Su2 = 61.1491
Su = 50.0711 Su2 d = 52.8140
S = 41 Sud = 43.2460
Sd = 35.4114 Sd2 u = 37.3513
2
Sd = 30.5846
Sd3 = 26.4157
11.2. UNDERSTANDING RISK-NEUTRAL PROBABILITY 85

The replicating portfolios are copied over from the textbook Figure 10.5.

Period 0 1 2

(4, B)u2 = (1, −38. 947 4)


(4, B)u = (0.921 8, −33. 263 6)
(4, B) = (0.706 3, −21. 885 2) (4, B)ud = (0.828 7, −30. 138 6)
(4, B)d = (0.450 1, −13. 405 2)
(4, B)d2 = (0, 0)

In addition, we need to calculate the discount rate for each node. We put
the stock price table and the replicating portfolio table side by side:
Stock price (4, B)
Period 0 1 2 3 Period 0 1 2
74.6781
61.1491 (1, −38. 947 4)
50.0711 52.8140 (0.921 8, −33. 263 6)
41 43.2460 (0.706 3, −21. 885 2) (0.828 7, −30. 138 6)
35.4114 37.3513 (0.450 1, −13. 405 2)
30.5846 (0, 0)
26.4157

Calculate the common discounting factor Node u →Node 0 and Node d →Node
0:

∆S + B 0.706 3 (41) − 21. 885 2


e−γ(1/3) = = =
∆Seαh + Berh 0.706 3 (41) e0.15(1/3) − 21. 885 2e0.08(1/3)
0.887 87
e−γ(1/3) = 0.887 87 γ = 0.356 8

Calculate the common discounting factor Node u2 →Node u and Node


ud →Node u :

∆S + B 0.921 8 1 (50.0711) − 33. 263 6


e−γ(1/3) = αh rh
= =
∆Se + Be 0.921 8 (50.0711) e0.15(1/3) − 33. 263 6 2e0.08(1/3)
0.897 84
e−γ(1/3) = 0.897 84 γ = 0.323 3

Calculate the common discounting factor Node ud →Node d and Node


d2 →Node d :

∆S + B 0.450 1 (35.4114) − 13. 405 2


e−γ(1/3) = = =
∆Seαh + Berh 0.450 1 (35.4114) e0.15(1/3) − 13. 405 2 64e0.08(1/3)
0.847 79
86 CHAPTER 11. BINOMIAL OPTION PRICING: II

e−γ(1/3) = 0.847 79 γ = 0.495 4

Calculate the common discounting factor Node u3 →Node u2 and Node


u d →Node u2 :
2

∆S + B 1 (61.1491) − 38. 947 4


e−γ(1/3) = = =
∆Seαh + Berh 1 (61.1491) e0.15(1/3) − 38. 947 4e0.08(1/3)
0.914 24
e−γ(1/3) = 0.914 24 γ = 0.2690

Calculate the common discounting factor Node u2 d →Node ud and Node


2
ud →Node ud :

∆S + B 0.828 7 (43.2460) − 30. 138 6


e−γ(1/3) = αh rh
= =
∆Se + Be 0.828 7 (43.2460) e0.15(1/3) − 30. 138 6e0.08(1/3)
0.847 83
e−γ(1/3) = 0.847 83 γ = 0.495 2

Calculate the common discounting factor Node ud2 →Node d2 and Node
d →Node d2 :
3

∆S + B 0 (30.5846) − 0
e−γ(1/3) = αh rh
= = N/A
∆Se + Be 0 (30.5846) e0.15(1/3) − 0 6e0.08(1/3)
e−γ(1/3) = N/A γ = N/A

We calculate the premium by working backward from right to left. At each


node, we take the greater of the backwardized value and the exercise value.
Calculate the common real probability of stock going up and down at each
node.
41e0.15(1/3) = 50.0711p + 35.4114 (1 − p)
p = 0.524 6 q = 1 − 0.524 6 = 0.475 4

Period 0 1 2 3
Cu3 = max (0, 74.6781 − 40) = 34. 678 1
γ = 0.2690
γ = 0.323 3 Cu2 d = max (0, 52.8140 − 40) = 12. 814
γ = 0.356 8 γ = 0.495 2
γ = 0.495 4 Cd2 u = max (0, 37.3513 − 40) = 0
γ = N/A
Cd3 = max (0, 26.4157 − 40) = 0
11.2. UNDERSTANDING RISK-NEUTRAL PROBABILITY 87

Period 0 1 2 3
Su3 = 74.6781
Su2 = 61.1491
Su = 50.0711 Su2 d = 52.8140
S = 41 Sud = 43.2460
Sd = 35.4114 Sd2 u = 37.3513
2
Sd = 30.5846
Sd3 = 26.4157
Cu2 = (34. 678 1 × 0.524 6 + 12. 814 × 0.475 4) e−0.2690(1/3) = 22. 201 2
EVu2 = max (0, 61.1491 − 40) = 21. 149 1

Vu2 = max (22. 201 2, 21. 149 1) = 22. 201 2

Cud = (12. 814 × 0.524 6 + 0 × 0.475 4) e−0.495 2(1/3) = 5. 699 4


EVud = max (0, 43.2460 − 40) = 3. 246

Vud = max (5. 699 4, 3. 246) = 5. 699 4


Cd2 = 0
EVd2 = max (0, 30.5846 − 40) = 0

Vd2 = max (0, 0) = 0


Now we have:
Period 0 1 2 3
Vu3 = 34. 678 1
Vu2 = 22. 201 2
γ = 0.323 3 Vu2 d = 12. 814
γ = 0.356 8 Vud = 5. 699 4
γ = 0.495 4 Vud2 = 0
Vd2 = 0
Vd3 = 0

Similarly,
Cu = (22. 201 2 × 0.524 6 + 5. 699 4 × 0.475 4) e−0.323 3(1/3) = 12. 889 6
EVu = max (0, 50.0711 − 40) = 10. 071 1

Vu = max (12. 889 6, 10. 071 1) = 12. 889 6

Cd = (5. 699 4 × 0.524 6 + 0 × 0.475 4) e−0.495 4(1/3) = 2. 534 8


EVd = max (0, 35.4114 − 40) = 0

Vd = max (2. 534 8, 0) = 2. 534 8


Now we have:
88 CHAPTER 11. BINOMIAL OPTION PRICING: II

Period 0 1 2 3
Vu3 = 34. 678 1
Vu2 = 22. 201 2
Vu = 12. 889 6 Vu2 d = 12. 814
γ = 0.356 8 Vud = 5. 699 4
Vd = 2. 534 8 Vud2 = 0
Vd2 = 0
Vd3 = 0
Finally,
C = (12. 889 6 × 0.524 6 + 2. 534 8 × 0.475 4) e−0.356 8(1/3) = 7. 0734
EV = max (0, 41 − 40) = 1

V = max (7. 0734, 1) = 7. 073 4


So we have:
Period 0 1 2 3
Vu3 = 34. 678 1
Vu2 = 22. 201 2
Vu = 12. 889 6 Vu2 d = 12. 814
V = 7. 073 4 Vud = 5. 699 4
Vd = 2. 534 8 Vud2 = 0
Vd2 = 0
Vd3 = 0

Tip 11.2.2. Real probability pricing requires intensive calculation. Not only
do we need to find the real probability of up and down, we also need to find
the replicating portfolio at each node. In contrast, in risk neutral pricing, we
either use risk neutral probabilities or use replicating portfolio but not both. In
comparison, risk neutral pricing is more efficient than real probability pricing.

11.2.2 Binomial tree and lognormality

Random Walk model

Here’s a brief review of the random walk model. There are 3 schools of thoughts:
the chartist approach (or technical analysis), fundamental analysis, and the
random walk model. Those who use the chartist approach draw charts to predict
stock future prices. They believe that history repeats itself and that past stock
prices help predict future stock prices. Fundamental analysis believes that at
any point the stock has an intrinsic value that depends on the earning potential
of the stock. Random Walk model, on the other hand, believes that the price of
a stock is purely random and that past price can’t help predict the stock price
in the future.
Is the random walk model true? Is stock price purely random? Some scholars
challenged the random walk theory. If interested, you can look into the book
11.2. UNDERSTANDING RISK-NEUTRAL PROBABILITY 89

A Non-Random Walk Down Wall Street at Amazon.com http://www.amazon.


com/
Now let’s look at the random walk math model. Imagine that an object zero
starts from point zero and travels along a straight line (such as Y axis). At each
step, the object either move up by 1 unit with probability p or move down by 1
unit with probability 1 − p. An equivalent description of the movement is this:
at each move a coin is tossed. If we get a head, the object moves up by 1 unit;
if we get a tail, the object moves down by 1 unit.
Let Y½ i represent the movement in the i-th step. Then
1 with probability p
Yi =
−1 with probability 1 − p
Let Zn represent the position of the article after n steps. Then
Zn = Y1 + Y2 + ... + Yn
Y1 , Y2 , ...Yn are independent identically distributed.

To apply the random walk model to stock prices, we can use Yi to represent
the price movement during an interval of time. We can use Zn to represent the
ending price of a stock after n equal intervals.

To get a good feel of the random variable Zn , check out the simulation of
the random walk model at
http://math.furman.edu/~dcs/java/rw.htmll

Just type in the url in your web browser.

Modeling stock as a random walk

There are 3 problems if we use Zn = Y1 + Y2 + ... + Yn to model the price of a


stock:

1. Zn can be negative yet the price of a stock can’t be negative.


2. The incremental change of any stock price Yi is either 1 or −1. Though
$1 change might be OK for modeling the change of a low priced stock, it
may be inappropriate to model the change of a high priced stock.
3. The expected return on stock whose price following a random walk is zero,
that is E (Zn ) = nE (Y ) = n × 0 = 0. However, stock on average should
have a positive return.

Continuously compounded returns


Let rt,t+h represent the continuously compounded return earned during the time
interval [t, t + h]. Let St and St+h represent the stock price at time t and t + h
respectively. Then
St+h St+h
St ert,t+h = St+h ert,t+h = rt,t+h = ln
St St
90 CHAPTER 11. BINOMIAL OPTION PRICING: II

Continuously compounded returns are additive. Consider the time interval


[t, t + nh]. We have:
St+nh St+h St+2h St+3h St+nh
ert,t+nh = = × × × ... ×
St St St+h St+2h St+(n−1)h
= ert,t+h ert+h,t+2h ert+2h,t+3h ...ert+(n−1)h,t+nh
⇒ rt,t+nh = rt,t+h +rt+h,t+2h +rt+2h,t+3h + ... + rt+(n−1)h,t+nh
Continuously compounded returns can be negative. Even if r < 0, we still
have er > 0. Thus if ln S follows a random walk, S can’t be negative.

Standard deviation of returns


The annual return is the sum of the returns in each of the 12 months:
rannual = rJan + rF eb + ... + rDec
Assume each monthly return is independent identical distributed with com-
mon variance σ 2Monthly . Let σ 2 represent the variance of the return over 1 year
period.
V ar (rannual ) = V ar (rJan + rF eb + ... + rDec ) = 12σ 2Monthly
σ
σ 2 = 12σ2M onthly σMonthly = √
12
Suppose we split one year into n intervals with each interval being h long (so
nh = 1). Let σ h represent the standard deviation of the return over an interval
of h long, then
rannual = r0,h + rh,2h + r2h,3h
£ + ... + r(n−1)h,nh ¤
σ 2 = V ar (rannual ) = V ar r0,h + rh,2h + r2h,3h + ... + r(n−1)h,nh = n σ 2h
σ
σh = √
n
σ √
However, nh = 1. So we have: σ h = √ = σ h
n

Binomial model
√ √
Previously, we set u = e(r−δ)h+σ h and d = √
e(r−δ)h−σ h . Now let’s

see why do-
(r−δ)h+σ h
ing so is reasonable. Setting u = e and d = e(r−δ)h−σ h is equivalent
to setting √
St+h = St e(r−δ)h±σ h , which is equivalent to

rt,t+h = (r − δ) h ± σ h (11.4)
Let’s see why Equation 11.4 solves the three problems in the random walk
model:

1. Even when rt,t+h is negative, the stock price St+h is always positive.
2. The change
h in stock priceiis proportional to the stock price. ∆S = St+h −

St = St e(r−δ)h±σ h − 1 .

3. The expected return during [t, t + h] is largely driven by the constant term
(r − δ) h. Hence the expected return is no long always zero.
11.2. UNDERSTANDING RISK-NEUTRAL PROBABILITY 91

Alternative binomial tree

The Cox-Ross-Rubinstein binomial tree


u = eσ h
(11.5)


d = e−σ h
(11.6)

The lognormal tree (also called the Jarrow-Rudd binomial model)

2

u = e(r−δ−0.5σ )h+σ h
(11.7)

2

d = e(r−δ−0.5σ )h−σ h
(11.8)

11.2.3 Estimate stock volatility


Formulas
n+1 number of stock prices observed: S0 , S1 ,...,Sn−1 ,and Sn
n number of stock returns observed. Continuously compounded
return per period is estimated as
S1 S2 S3 Sn
r1 = ln r2 = ln r3 = ln ...rn = ln
S0 S1 S2 Sn−1
Since our focus is stock returns not stock price, the number of observations
is n (i.e. the number of stock prices observed minus one). Remember this point.
The expected return is:
∧ r1 + r2 + ... + rn
r=
n

The v
estimated standard deviation is:
u³ ´2 ³ ´
∧ 2
³ ´
∧ 2
u r −∧ r + r − r + ... + r − r
∧ t 1 2 n
σ=
n−1

Example 11.2.3. Reproduce the textbook Figure 11.1 and estimate the standard
deviation of the continuously compounded return per year earned by S&P 50
index.
92 CHAPTER 11. BINOMIAL OPTION PRICING: II

St ³ ´
∧ 2
Week S&P 500 price rt =ln rt − r
St−1
0 829.85
1 804.19 −0.0314 0.001846
2 874.02 0.0833 0.005143
3 869.95 −0.0047 0.000263
4 880.9 0.0125 0.000001
5 865.99 −0.0171 0.000819
6 879.91 0.0159 0.000019
7 919.02 0.0435 0.001020
8 916.92 −0.0023 0.000191
9 929.62 0.0138 0.000005
10 939.28 0.0103 0.000001
11 923.42 −0.0170 0.000817
12 953.22 0.0318 0.000409
Total 0.1386 0.010534

The expected continuously compounded return per week is estimated as:


∧ r1 + r2 + ... + r12 0.1386
r= = = 0.011 55
n 12

The standard deviation of the continuously compounded return per week is


estimated
r as:
∧ 0.010534
σ= = 0.03 095
12 − 1
1 Year = 52 Weeks

Let Y represent the continuously compounded return per year and Xi rep-
resent the continuously compounded return earned in the i-th week.
Then Y = X1 + X2 + ... + X52
Where X1 , X2 , ..., X52 are assumed to be independent identically distributed.
√ 1 + X2 + ... + X52 ) = 52V ar (X)
V ar (Y√) = V ar (X
σ Y = 52σ Y = 52 (0.03 095) = 0.223 18

Please note my calculation was done using Excel. If you can’t perfectly
reproduce my numbers, that’s fine.
By the way, in Excel, the formula for the mean is AVERAGE; the formula
for the sample variance is VAR

This is how to use Excel to calculate the continuously compounded return


per year earned by S&P 500 index. Suppose the stock prices are entered in
Column C (from C3 to C15) and the weekly returns are calculated in Column
D (from D4 to D15).
11.2. UNDERSTANDING RISK-NEUTRAL PROBABILITY 93

1 Column B Column C Column D


St
2 Week S&P 500 price rt =ln
St−1
3 0 829.85
4 1 804.19 −0.0314
5 2 874.02 0.0833
6 3 869.95 −0.0047
7 4 880.9 0.0125
8 5 865.99 −0.0171
9 6 879.91 0.0159
10 7 919.02 0.0435
11 8 916.92 −0.0023
12 9 929.62 0.0138
13 10 939.28 0.0103
14 11 923.42 −0.0170
15 12 953.22 0.0318
The expected continuously compounded return per week is:

r =AVERAGE(D4:D15)= 0.01155006
The sample variance of the continuously compounded return per week is
estimated as:
∧2
σ =VAR(D4:D15)= 0.000957682
∧ √
σ = 0.000957682 = 0.030946434
Make sure you don’t use the population variance formula:
VARP(D4:D15)= 0.000877875
In Excel, VARP is:
³ ´
∧ 2
³ ´
∧ 2
³ ´
∧ 2
r1 − r + r2 − r + ... + rn − r
V ARP =
n

While VAR is³calculated


´ as ³ ´ ³ ´
∧ 2 ∧ 2 ∧ 2
r1 − r + r2 − r + ... + rn − r
V AR =
n−1

n
So V AR = V ARP ×
n−1

Finally, I’ll give you a BA II Plus (or BA II Plus Professional) calculator


shortcut for quickly finding the sample mean and the sample standard deviation.
BA II Plus and BA II Plus Professional have a 1-V Statistics Worksheet. This
worksheet can calculate the sample mean, the sample standard deviation, the
population mean (which is the same as the sample mean), and the population
standard deviation.
In 1-V Statistics Worksheet, enter:
94 CHAPTER 11. BINOMIAL OPTION PRICING: II

1 Column B Column C Column D


St
2 Week S&P 500 price rt =ln
St−1
3 0 829.85
4 1 804.19 −0.0314
5 2 874.02 0.0833
6 3 869.95 −0.0047
7 4 880.9 0.0125
8 5 865.99 −0.0171
9 6 879.91 0.0159
10 7 919.02 0.0435
11 8 916.92 −0.0023
12 9 929.62 0.0138
13 10 939.28 0.0103
14 11 923.42 −0.0170
15 12 953.22 0.0318
804.19
X01 = ln = −0.03 140 940
829.85
Y 01 = 1 (we observed X01 only once)
874.02
X02 = ln = 0.08 326 770
804.19
Y 02 = 1 (we observed X02 only once)

So on and so forth. The final entry is:


953.22
X12 = ln = 0.03 176 156
923.42
Y 12 = 1 (we observed X12 only once)

Press 2nd STAT of the calculator key, you should get:


n = 12
X = 0.01155006
This is the sample mean (or population mean) of the continuously com-
pounded return per week.
SX = 0.03094643
This is the estimated standard deviation (or sample standard deviation) of
the continuously compounded return per week.
σ X = 0.02962895
This is the estimated standard deviation (or population standard deviation)
of the continuously compounded return per week.

You should use SX and discard σ X when estimating the stock volatility.

Example 11.2.4. Reproduce the textbook Figure 11.1 and estimate the standard
deviation of the continuously compounded return per year earned by IBM.
11.3. STOCKS PAYING DISCRETE DIVIDENDS 95

St
Week S&P 500 price rt =ln
St−1
0 77.73
1 75.18 −0.0334
2 82 0.0868
3 81.55 −0.0055
4 81.46 −0.0011
5 78.71 −0.0343
6 82.88 0.0516
7 85.75 0.0340
8 84.9 −0.0100
9 86.68 0.0207
10 88.7 0.0230
11 86.18 −0.0288
12 87.57 0.0160

In BA II Plus (or BA II Plus Professional) 1-V Statistics Worksheet, enter:


X01 = −0.0334 Y 01 = 1
X02 = 0.0868 Y 02 = 1
X03 = −0.0055 Y 03 = 1
......
X13 = 0.0160 Y 12 = 1
You should get: SX = 0.0365
So the estimated standard deviation (or sample standard deviation) of the
continuously compounded return per week is:
σ X = 0.0365

The
√ standard deviation of the continuously compounded return per year is:
52 (0.0365) = 0.263 2

11.3 Stocks paying discrete dividends

Previously, we assume that dividends are paid continuously at rate δ. However,


in reality, dividends are paid discretely (such as quarterly or annually). Now
let’s build a binomial tree to calculate the price of a stock that pays discrete
dividends.
Suppose we have a European option on a stock that pays a discrete dividend.
The option is written at t (today) and expires in t+h. The stock pays a dividend
or several dividends during [t, t + h]. The future value of the dividends at t + h
is D. The continuously compounded risk-free interest rate per year is r ( a

positive constant). The stock price today is St . At t + h, the stock price either
goes up to Stu = uSt or goes down to Std = dSt . The standard deviation of the
96 CHAPTER 11. BINOMIAL OPTION PRICING: II

continuously compounded return earned by the stock per year is σ. We want


to calculate the option price.

Using Equation 10.22√ and 10.23, we have: √


Stu = uSt = Ft,t+h eσ h Std = dSt = Ft,t+h e−σ h
However, Ft,t+h = St erh − D
This gives us:

¡ ¢ √
Stu = St erh − D eσ h (11.9)

¡ ¢ √
Std = St erh − D e−σ h (11.10)
To find the price of the European option, we calculate the cost of the repli-
cating portfolio. We have two assets: the stock and a savings account. The
savings account is the same as a zero-coupon bond. At time t + h, the stock
price is Sh ; the bond price is Bt+h . The bond price is deterministic:
Bt = 1 Bt+h = erh

The stock
( price at¡ t + h is stochastic:
¢ √
Stu = St erh − D eσ √h
St+h = ¡ ¢
Std = St erh − D e−σ h
So at t + h the stock price either goes up to Stu ("up state") or goes down
to Std ("down state").

Stu
St
Std
Time t Time t + h
Our task is to determine C by setting a portfolio that replicates the option
payoff of Cu in the up state and Cd in the down state. We build the replicating
portfolio by buying 4 stocks and investing $B in a zero-coupon bond.
If we own one stock at t, then at t + h our total wealth is St+h + D. We
not only can sell the stock in the market for St+h , we’ll also have D, the future
value of the dividend earned during [t, t + h].
So we need to set up the following equation:
4 (Stu + D) Berh Cu
4St ¡ ¢ + B = C
4 Std + D Berh Cd
t t+h t t+h t t+h
½
4 ¡(Stu + D)¢ + Berh = Cu
4 Std + D + Berh = Cd
Solving these equations, we get:
11.3. STOCKS PAYING DISCRETE DIVIDENDS 97

Cu − Cd
4= (11.11)
Stu − Std
µ ¶
Stu Cd − Std Cu
B = e−rh − 4D (11.12)
Stu − Std

11.3.1 Problems with discrete dividend tree

One major problem with the stock price tree using Equation 11.9 and 11.10 is
that the tree doesn’t complete recombine after the discrete dividend.

Example 11.3.1. Reproduce the textbook Figure 11.1 but just for the 2 periods.
Show that the stock price tree doesn’t recombine at Period 2. This is the recap
of the information. The current stock price is 41. The stock pays a dividend
during Period 1 and Period 2. The future value of the dividend accumulated
at the risk-free interest rate r from Period 1 to Period 2 is 5. Other data are:
r = 0.08, σ = 0.3, t = 1,and h = 1/3.

Period 0 1 2 3
Stuuu = 67. 417 15
Stuu = 55. 203
Stu = 50. 071 Stuud = Studu = 47. 678 91
Stud = 39. 041
Studd = 33. 719 59
St = 41
Stduu = 45. 553 05
Stdu = 37. 300
Std = 35. 411 Stdud = Stddu = 32. 216 14
Stdd = 26. 380
Stddd = 22. 783 97

No dividend is paid during Period 0 and Period 1.


¡ ¢ √ ¡ ¢ √
Stu = St erh − D eσ h = 41e0.08×1/3 − 0 e0.3 1/3 = 50. 071 09
¡ ¢ √ ¡ ¢ √
Std = St erh − D e−σ h = 41e0.08×1/3 − 0 e−0.3 1/3 = 35. 411 39

Dividend is paid during Period 1 to Period 2, whose value


√ at Period 2 is 5.
¡ ¢ √ ¡ ¢
Stuu = Stu erh − D eσ h = 50. 071 09e0.08×1/3 − 5 e0.3 1/3 = 55. 203 57
¡ ¢ √ ¡ ¢ √
Stud = Stu erh − D e−σ h = 50. 071 09e0.08×1/3 − 5 e−0.3 1/3 = 39. 041 20
¡ ¢ √ ¡ ¢ √
Stdu = Std erh − D eσ h = 35. 411 39e0.08×1/3 − 5 e0.3 1/3 = 37. 300 47
¡ ¢ √ ¡ ¢ √
Stdd = Std erh − D e−σ h = 35. 411 39e0.08×1/3 − 5 e−0.3 1/3 = 26. 379 73
98 CHAPTER 11. BINOMIAL OPTION PRICING: II

Now you see that Stud 6= Stdu , . The tree doesn’t recombine.
No dividend is paid during Period 2 and Period 3. √
¡ ¢ √ ¡ ¢
Stuuu = Stuu erh − D eσ h = 55. 203 57e0.08×1/3 − 0 e0.3 1/3 = 67. 417 15
¡ ¢ √ ¡ ¢ √
Stuud = Stuu erh − D e−σ h = 55. 203 57e0.08×1/3 − 0 e−0.3 1/3 = 47.
678 91 ¡ ¢ √ ¡ ¢ √
Studu = Stud erh − D eσ h = 39. 041 20e0.08×1/3 − 0 e0.3 1/3 = 47. 678 91
¡ ¢ √ ¡ ¢ √
Studd = Stud erh − D e−σ h = 39. 041 20e0.08×1/3 − 0 e−0.3 1/3 = 33.
719 59

¡ ¢ √ ¡ ¢ √
Stduu = Stdu erh − D eσ h = 37. 300 47e0.08×1/3 − 0 e0.3 1/3 = 45. 553 05
¡ ¢ √ ¡ ¢ √
Stdud = Stdu erh − D e−σ h = 37. 300 47e0.08×1/3 − 0 e−0.3 1/3 = 32.
216 14 ¡ ¢ √ ¡ ¢ √
Stddu = Stdd erh − D eσ h = 26. 379 73e0.08×1/3 − 0 e0.3 1/3 = 32. 216 14
¡ ¢ √ ¡ ¢ √
Stddd = Stdd erh − D e−σ h = 26. 379 73e0.08×1/3 − 0 e−0.3 1/3 = 22.
783 97

Please note that in this example Stuud = Studu and Stdud = Stddu . This is not
a coincidence.
Let’s verify that Stuud = Studu .
¡ ¢ √ √
Stu = St erh − 0 eσ h = St erh eσ h
¡ ¢ √ ³ √ ´ √
Stuu = Stu erh − 5 eσ h = St erh eσ h − 5 eσ h
¡ ¢ √ ³³ √ ´ √ ´ √
Stuud = Stuu erh − 0 e−σ h = St erh eσ h − 5 eσ h erh e−σ h
³ √ ´
= St erh eσ h − 5 erh

¡ ¢ √ ³ √ ´ √
Stud = Stu erh − 5 e−σ h = St erh eσ h − 5 e−σ h

¡ ¢ √ ³³ √ ´ √ ´ √
Studu = Stud erh − 0 eσ h = St erh eσ h − 5 e−σ h erh − 0 eσ h
³ √ ´
= St erh eσ h − 5 erh
Clearly, Stuud = Studu . Similarly, you can verify for yourself that Stdud =
Stddu .

In this problem, Period 2 had 4 prices. If the stock pays continuous dividend,
Period 2 will have only 3 prices.
Similarly, in this problem, Period 3 has 5 distinct prices. In contrast, if the
stock pays continuous dividend, Period 3 will have only 4 prices.
In addition to non-combining stock prices, the above method may produce
negative stock prices.
11.3. STOCKS PAYING DISCRETE DIVIDENDS 99

11.3.2 Binomial tree using prepaid forward

Schroder presents a method that overcomes the two shortcomings of the above
method.

This is the idea behind Schroder’s method. Instead of directly building a


stock price tree (which proves to be non-combining after the discrete dividend is
paid), we’ll build a tree of a series of prepaid forward prices on the same stock.
Hopefully, the prepaid forward
³ price
´ tree is recombining and looks like this:
uu
P
Ft+2h,T
³ ´u
P
Ft+h,T
³ ´ud
P P
Ft,T Ft+2h,T
³ ´d
P
Ft+h,T
³ ´dd
P
Ft+2h,T

In the above table,


P
• Ft,T is the prepaid price of a forward contract signed at t expiring on date
T
P
• Ft+h,T is the prepaid price of a forward contract signed at t + h expiring
on date T
P
• Ft+2h,T is the prepaid price of a forward contract signed at t + 2h expiring
on date T

Since there’s a one-to-one relationship between a prepaid forward price and


the stock price, we can change the (recombining) prepaid forward price tree into
a (recombining) stock price tree. Once we have a recombining stock price tree,
we’ll can easily find the price of the option using either risk-neutral probability
or the replicating portfolio.
First, let’s find the relationship between the stock price and the prepaid
forward price on the stock. Suppose today is time t . At t we enter into a
forward contract agreeing to buy a stock on date T . The stock will pay dividend
D on date TD where TD < T .

Suppose we want to prepay the seller at t. The price of this prepaid forward
contract is the current stock price
½ St −r(T
minus the present value of the dividend:
D −t)
P De if TD ≥ t
Ft,T = St − P Vt (D) = St −
0 if TD < t
½
P De−r(TD −t) if TD ≥ t
→ St = Ft,T +
0 if TD < t
100 CHAPTER 11. BINOMIAL OPTION PRICING: II

Similarly, ½
P De−r(TD −t−h) if TD ≥ t + h
Ft+h,T= St+h − P Vt+h (D) = St+h −
0 if TD < t + h
½ −r(TD −t−h)
P De if TD ≥ t + h
→ St+h = Ft+h,T +
0 if TD < t + h

So there’s a one-to-one mapping between the prepaid forward price and the
stock price.
Next, let’s find out how to build a prepaid forward price tree. We need to
know how the prepaid forward price changes over time. Suppose today is time
t . At t + h we enter into a forward contract agreeing to buy a stock at date T
where T > t + h. The stock will pay dividend D in date TD where t < TD < T .
If the stock volatility is zero (meaning that the future stock price is known today
with 100% certainty), then the price of the prepaid forward contact at t + h is

P P rh
Ft+h,T = Ft,T e (11.13)

P
Ft+h,T
P
= erh (11.14)
Ft,T

P
This is why Equation 11.13 holds. If we pay Ft,T at t, we’ll receive one stock
P −r(T −t) P
at T . This gives us Ft,T = ST e . Similarly, if we pay Ft+h,T at t + h, we’ll
P −r(T −t−h)
also receive a stock at T . This gives us Ft+h,T = ST e . Then Equation
11.13 holds.
Now suppose that the forward price has a volatility of σ F per year. Then
it’s reasonable
( to assume that

P rh σ h P
P Ft,T e e √ = Ft,T u in the up state
Ft+h,T = P rh −σ h P
Ft,T e e = Ft,T d in the down state

√ √
where u = erh+σF h
and d = erh−σF h
½ P
P Ft+h,T u in the up state
Similarly, Ft+2h,T = P
Ft+h,T d in the down state

Typically, we know the volatility of the stock, σ S . We need to calculate σ F


using the following approximation:

St
σF = σS × P
Ft,T
Now the prepaid forward price tree is:
11.3. STOCKS PAYING DISCRETE DIVIDENDS 101
³ ´uu
P P
Ft+2h,T = Ft,T u2
³ ´u
P P
Ft+h,T = Ft,T u
³ ´ud ³ ´du
P P P P
Ft,T Ft+2h,T = Ft+2h,T = Ft,T ud
³ ´d
P P
Ft+h,T = Ft,T d
³ ´dd
P P 2
Ft+2h,T = Ft,T d

Once we have the prepaid forward price tree, we’ll transform it into the stock
price tree:

³ ´uu
uu P
St+2h = Ft+2h,T + P Vt+2h (D)
³ ´u
u P
St+h = Ft+h,T + P Vt+h (D)
³ ´ud
P ud P
St = Ft,T + P Vt (D) St+2h = Ft+2h,T + P Vt+2h (D)
³ ´d
d P
St+h = Ft+h,T + P Vt+h (D)
³ ´dd
dd P
St+2h = Ft+2h,T + P Vt+2h (D)

Example 11.3.2. Let’ s reproduce the textbook Figure 11.11. Here is the recap
of the information on an American call option. The stock pays a dividend of $5
in 8 months. Current stock price is $41. The strike price K = $40. The stock
volatility is σ S = 0.3. The continuously compounded risk-free rate is r = 0.08.
The option expires in T = 1 year. Use a 3-period binomial tree to calculate the
option price.

First, we’ll build a prepaid forward price tree.

St
σF = σS × P
Ft,T
P
Ft,T = St − P Vt (D) = 41 − 5e−0.08(8/12) = 36. 26
St 41
σ F = σ S × P = 0.3 × = 0.339 2
Ft,T 36. 26
√ √
u = erh+σF √ h = e0.08(1/3)+0.3392 √1/3 = 1. 249 20
d = erh−σF h = e0.08(1/3)−0.3392 1/3 = 0.844 36

Prepaid forward price tree:


102 CHAPTER 11. BINOMIAL OPTION PRICING: II
³ ´uuu
P
Ft+3h,T
³ ´uu
P
Ft+2h,T
³ ´u ³ ´uud
P P
Ft+h,T Ft+3h,T
³ ´ud
P P
Ft,T Ft+2h,T
³ ´d ³ ´udd
P P
Ft+h,T Ft+3h,T
³ ´dd
P
Ft+2h,T
³ ´ddd
P
Ft+3h,T
³ ´u
P P
Ft+h,T = Ft,T u = 36. 26 × 1. 249 20 = 45. 296
³ ´d
P P
Ft+h,T = Ft,T d = 36. 26 × 0.844 36 = 30. 616
³ ´uu ³ ´u
P P
Ft+2h,T = Ft,T u2 = Ft+h,T
P
u = 45. 296 × 1. 249 20 = 56. 584
³ ´ud ³ ´u
P P
Ft+2h,T = Ft+h,T d = 45. 296 × 0.844 36 = 38. 246
³ ´dd ³ ´d
P P 2 P
Ft+3h,T = Ft,T d = Ft+h,T d = 30. 616 × 0.844 36 = 25. 851
³ ´uuu ³ ´uu
P P
Ft+3h,T = Ft+2h,T u = 56. 584 × 1. 249 20 = 70. 685
³ ´uud ³ ´uu
P P
Ft+3h,T = Ft+2h,T d = 56. 584 × 0.844 36 = 47. 777
³ ´udd ³ ´ud
P P
Ft+3h,T = Ft+2h,T d = 38. 246 × 0.844 36 = 32. 293
³ ´ddd ³ ´dd
P P
Ft+3h,T = Ft+2h,T d = 25. 851 × 0.844 36 = 21. 828

Prepaid forward price tree:

70. 685
56. 584
45. 296 47. 777
36. 26 38. 246
30. 616 32. 293
25. 851
21. 828
Next, we change the prepaid forward price tree into a stock price tree. The
one-to-one mapping between the prepaid forward price and the stock price is
½
P De−r(TD −t−∆t) if TD ≥ t + ∆t
St+∆t = Ft+∆t,T +
0 if TD < t + ∆t
where 0 ≤ ∆t ≤ T − t
11.3. STOCKS PAYING DISCRETE DIVIDENDS 103
¡ ¢uuu
¡ ¢uu St+3/3
St+2/3
¡ ¢u ¡ ¢uud
St+1/3 St+3/3
¡ ¢ud
St St+2/3
¡ ¢d ¡ ¢udd
St+1/3 St+3/3
¡ ¢dd
St+2/3
¡ ¢ddd
St+3/3

8
In this problem, TD = + t, D = 5,T = t + 1
12
P −r(TD −t)
St = Ft,T + De = 36. 26 + 5e−0.08(8/12) = 41. 000

¡ ¢u ³ P ´u
St+1/3 = Ft+1/3,T + De−r(TD −t−1/3) = 45. 296 + 5e−0.08(8/12−1/3) =
50. 164
¡ ¢d ³ P ´d
St+1/3 = Ft+1/3,T + De−r(TD −t−1/3) = 30. 616 + 5e−0.08(8/12−1/3) =
35. 484
¡ ¢uu ³ P ´uu
St+2/3 = Ft+2/3,T +De−r(TD −t−2/3) = 56. 584+5e−0.08(8/12−2/3) =
61. 584
¡ ¢ud ³ P ´ud
St+2/3 = Ft+2/3,T +De−r(TD −t−2/3) = 38. 246+5e−0.08(8/12−2/3) =
43. 246
¡ ¢dd ³ P ´dd
St+2/3 = Ft+2/3,T + De−r(TD −t−2/3) = 25. 851 + 5e−0.08(8/12−2/3) =
30. 851
¡ ¢uuu ³ P ´uuu
St+3/3 = Ft+3/3,T = 70. 685 (because TD < t + 3/3)
Similarly,
¡ ¢uud ³ P ´uud
St+3/3 = Ft+3/3,T = 47. 777
¡ ¢udd ³ ´udd
P
St+3/3 = Ft+3/3,T = 32. 293
¡ ¢ddd ³ P ´ddd
St+3/3 = Ft+3/3,T = 21. 828

So the stock price tree is:


70. 685
61. 584
50. 164 47. 777
41. 000 43. 246
35. 484 32. 293
30. 851
21. 828

After getting the stock price tree, we calculate the price of the American
call option as usual. We work backward from right to left. At each node, we
104 CHAPTER 11. BINOMIAL OPTION PRICING: II

compare the backwardized value with the exercise value, taking the maximum
of the two.

The risk neutral probabilities are:


erh − d e0.08(1/3) − 0.844 36
πu = = = 0.451 2
u−d 1. 249 20 − 0.844 36
πd = 1 − 0.451 2 = 0.548 8

Payoff tree:
Vuuu = 30. 685
Vuu = 21. 584
Vu = 11. 308 Vuud = 7. 777
V = 5.770 Vud = 3. 417
Vd = 1. 501 Vudd = 0
Vdd = 0
Vddd = 0

Vuuu = Cuuu = max (0, 70. 685 − 40) = 30. 685


Vuud = Cuud = max (0, 47. 777 − 40) = 7. 777
Vudd = Cudd = max (0, 32. 293 − 40 ) = 0
Vddd = Cddd = max (0, 21. 828 − 40) = 0
Cuu = (30. 685 × 0.451 2 + 7. 777 × 0.548 8) e−0.08(1/3) = 17. 636
EVuu = max (0, 61. 584 − 40) = 21. 584
Vuu = max (17. 636 , 21. 584) = 21. 584

Cud = (7. 777 × 0.451 2 + 0 × 0.548 8) e−0.08(1/3) = 3. 417


EVud = max (0, 43. 246 − 40) = 3. 246
Vud = max (3. 417 , 3. 246) = 3. 417

Cdd = 0
EVdd = max (0, 30. 851 − 40) = 0
Vud = 0

Cu = (21. 584 × 0.451 2 + 3. 417 × 0.548 8) e−0.08(1/3) = 11. 308


EVu = max (0, 50. 164 − 40) = 10. 164
Vu = max (11. 308, 10. 164) = 11. 308

Cd = (3. 417 × 0.451 2 + 0 × 0.548 8) e−0.08(1/3) = 1. 501


EVd = max (0, 35. 484 − 40) = 0
Vd = max (1. 501 , 0) = 1. 501

C = (11. 308 × 0.451 2 + 1. 501 × 0.548 8) e−0.08(1/3) = 5. 770


EV = max (0, 41. 000 − 40) = 1.0
V = max (5. 770, 1.0) = 5. 77
Chapter 12

Black-Scholes

Except the option Greeks and the barrier option price formula, this chapter is
an easy read.

12.1 Introduction to the Black-Scholes formula


12.1.1 Call and put option price
The price of a European call option is:

C (S, K, σ, r, T, δ) = Se−δT N (d1 ) − Ke−rT N (d2 ) (12.1)

The price of a European put option is:

P (S, K, σ, r, T, δ) = −Se−δT N (−d1 ) + Ke−rT N (−d2 ) (12.2)

µ ¶
S 1 2
ln + r−δ+ σ T
K 2
d1 = √ (12.3)
σ T

d2 = d1 − σ T (12.4)
Notations used in Equation 12.1, 12.3, and 12.4:

• S, the current stock price (i.e. the stock price when the call option is
written)

• K, the strike price

• r, the continuously compounded risk-free interest rate per year

• δ, the continuously compounded dividend rate per year

105
106 CHAPTER 12. BLACK-SCHOLES

• σ, the annualized standard deviation of the continuously compounded


stock return (i.e. stock volatility)

• T , option expiration time

• N (d) = P (z ≤ d) where z is a standard normal random variable

• C (S, K, σ, r, T, δ), the price of a European call option with parameters


(S, K, σ, r, T, δ)

• P (S, K, σ, r, T, δ), the price of a European put option with parameters


(S, K, σ, r, T, δ)

Tip 12.1.1. To help memorize Equation 12.2, we can rewrite Equation 12.2
similar to Equation 12.1 as P (S, K, σ, r, T, δ) = (−S) e−δT N (−d1 )+(−K) e−rT N (−d2 ).
In other words, change S,K,d1 ,and d2 in Equation 12.1 and you’ll get Equation
12.2.

Example 12.1.1. Reproduce the textbook example 12.1. This is the recap of
the information. S = 41, K = 40,r = 0.08, σ = 0.3, T = 0.25 (i.e. 3 months),
and δ = 0. Calculate the price of the price of a European call option.
µ ¶
S 1 2
ln + r−δ+ σ T
K 2
d1 = √
µ σ T ¶
41 1
ln + 0.08 − 0 + × 0.32 0.25
40 2
= √ = 0.3730
√ 0.3 0.25 √
d2 = d1 − σ T = 0.3730 − 0.3 0.25 = 0.2230
N (d1 ) = 0.645 4 N (d2 ) = 0.588 2
−0(0.25)
C = 41e 0.645 4 − 40e−0.08(0.25) 0.588 2 = 3. 399

Example 12.1.2. Reproduce the textbook example 12.2. This is the recap of
the information. S = 41, K = 40,r = 0.08, σ = 0.3, T = 0.25 (i.e. 3 months),
and δ = 0. Calculate the price of the price of a European put option.

N (−d1 ) = 1 − N (d1 ) = 1 − 0.645 4 = 0.354 6

N (−d2 ) = 1 − N (d2 ) = 1 − 0.588 2 = 0.411 8


P = −41e−0(0.25) 0.354 6 + 40e−0.08(0.25) 0.411 8 = 1. 607
12.2. APPLYING THE FORMULA TO OTHER ASSETS 107

12.1.2 When is the Black-Scholes formula valid?


Assumptions under the Black-Scholes formula:

Assumptions about the distribution of stock price:


1. Continuously compounded returns on the stock are normally distributed
(i.e. stock price is lognormally distributed) and independent over time
2. The volatility of the continuously compounded returns is known and con-
stant
3. Future dividends are known, either as a dollar amount (i.e. D and TD are
known in advance) or as a fixed dividend yield (i.e. δ is a known constant)
Assumptions about the economic environment
1. The risk-free rate is known and fixed (i.e. r is a known constant)
2. There are no transaction costs or taxes
3. It’s possible to short-sell costlessly and to borrow at the risk-free rate

12.2 Applying the formula to other assets


12.2.1 Black-Scholes formula in terms of prepaid forward
price

P
The prepaid forward price for the stock is: F0,T (S) = Se−δT
P
The prepaid forward price for the strike asset is: F0,T (K) = P V (K) =
−rT
Ke √
Define V (T ) = σ T
The price of a European call option in terms of repaid forward is:

¡ P P
¢ P P
C F0,T (S) , F0,T (K) , V (T ) = F0,T (S) N (d1 ) − F0,T (K) N (d2 ) (12.5)

The price of a European put option in terms of prepaid forward is:

¡ P P
¢ P P
P F0,T (S) , F0,T (K) , V (T ) = −F0,T (S) N (−d1 ) + F0,T (K) N (−d2 ) (12.6)

P
F0,T (S) 1
ln P
+ V 2 (T )
F0,T (K) 2
d1 = (12.7)
V (T )

d2 = d1 − V (T ) (12.8)
108 CHAPTER 12. BLACK-SCHOLES

12.2.2 Options on stocks with discrete dividends


When the stock pays discrete dividends, the prepaid forward price is:
P
F0,T (S) = S − P V0,T (Div)
P
Apply F0,T (S) in Equation 12.5 and 12.6, you should get the price of the
European call and put where the stock pays discrete dividends.

Example 12.2.1. Reproduce the textbook example 12.3. Here is the recap of
the information. S = 41, K = 40, σ = 0.3, r = 0.08, and T = 0.25 (i.e. 3
months). The stock pays dividend of 3 in 1 month, but makes no other payouts
over the life of the option (so δ = 0). Calculate the price of the European call
and put.
P
F0,T (S) = S − P V0,T (Div) = 41 − 3e−(0.08)1/12 = 38. 020
P
F0,T (K) = P V (K) = Ke−rT = 40e−0.08(0.25) = 39. 208
√ √
V (T ) = σ T = 0.3 0.25 = 0.15
P
F0,T (S) 1
ln P + V 2 (T ) 38. 020 1
F0,T (K) 2 ln + 0.152
d1 = = 39. 208 2 = −0.130 1
V (T ) 0.15
d2 = d1 − V (T ) = −0.130 1 − 0.15 = −0.280 1
N (d1 ) = 0.448 2 N (d2 ) = 0.389 7
N (−d1 ) = 1 − 0.448 2 = 0.551 8
N (−d2 ) = 1 − 0.389 7 = 0.610 3
P P
C = F0,T (S) N (d1 ) − F0,T (K) N (d2 )
= 38. 020 (0.448 2) − 39. 208 (0.389 7) = 1. 76

P P
P = −F0,T (S) N (−d1 ) + F0,T (K) N (−d2 )
= −38. 020 (0.551 8) + 39. 208 (0.610 3) = 2. 95

12.2.3 Options on currencies


Notation

• x, the current dollar value of €1

• K, the strike price in dollars of €1

• r, the continuously compounded risk-free rate earned by $1

• rf , the continuously compounded risk-free rate earned by €1

• σ, the annualized standard deviation of the continuously compounded


return on dollars

• T , the option expiration date

• C (x0 , K, σ, r, T, rf ), the price of a European call option with parameters


(x0 , K, σ, r, T, rf )
12.2. APPLYING THE FORMULA TO OTHER ASSETS 109

• P (x0 , K, σ, r, T, rf ), the price of a European put option with parameters


(x0 , K, σ, r, T, rf )

The price of a European call option is:

C (x, K, σ, r, T, rf ) = xe−rf T N (d1 ) − Ke−rT N (d2 ) (12.9)

The price of a European put option is:

P (x, K, σ, r, T, rf ) = −xe−rf T N (−d1 ) + Ke−rT N (−d2 ) (12.10)

µ ¶
x 1
ln + r − rf + σ2 T
K 2
d1 = √ (12.11)
σ T

d2 = d1 − σ T (12.12)
Tip 12.2.1. For currency options, just set S = x and δ = rf and apply the
Black-Scholes formulas on European call and put. The same thing happened in
Equation 10.26 and 10.27.
Example 12.2.2. Reproduce the textbook example 12.4. Here is the recap of the
information. The current dollar price of €1 is $0.92. The strike dollar price of
€1 is $0.9. The annualized standard deviation of the continuously compounded
return on dollars is σ = 0.1. The continuously compounded risk-free rate earned
by dollars is r = 6%. The the continuously compounded risk-free rate earned
by €1 is rf = 3.2%. The option expires in 1 year. Calculate the price of the
European call and put on €1.
µ ¶
x 1 2
ln + r − rf + σ T
K 2
d1 = √
µ σ T ¶
0.92 1
ln + 0.06 − 0.032 + × 0.12 1
0.9 2
= √ = 0.549 8
√ 0.1 1 √
d2 = d1 − σ T = 0.549 8 − 0.1 1 = 0.449 8
N (d1 ) = 0.708 8
N (−d1 ) = 1 − N (d1 ) = 1 − 0.708 8 = 0.291 2
N (d2 ) = 0.673 6
N (−d2 ) = 1 − N (d2 ) = 1 − 0.673 6 = 0.326 4
C = xe−rf T N (d1 ) − Ke−rT N (d2 )
= 0.92e−0.032(1) 0.708 8 − 0.9e−0.06(1) 0.673 6 = 0.06 06
P = −xe−rf T N (−d1 ) + Ke−rT N (−d2 )
= −0.92e−0.032(1) 0.291 2 + 0.9e−0.06(1) 0.326 4 = 0.017 2
110 CHAPTER 12. BLACK-SCHOLES

12.2.4 Options on futures


For a futures contract, the prepaid price is just the present value of the futures
P
price. Set F0,T (F ) = F e−rT and F0,T
P
(K) = Ke−rT , we get:

P
F0,T (F ) 1
ln P
+ σ2T
F0,T (K) 2
d1 = √ (12.13)
σ T

d2 = d1 − σ T (12.14)

C (F, K, σ, r, T ) = F e−rT N (d1 ) − Ke−rT N (d2 ) (12.15)

P (F, K, σ, r, T ) = −F e−rT N (−d1 ) + Ke−rT N (−d2 ) (12.16)

Example 12.2.3. Reproduce the textbook example 12.5. Here is the recap of
the information about the European option on a 1-year futures contract. The
current futures price for natural gas is $2.10. The strike price is K = 2.10. The
volatility is σ = 0.25. r = 0.055, T = 1. Calculate the price of the European
call and put.

F 1 2.10 1
ln + σ2T ln + × 0.252 (1)
d1 = K √ 2 = 2.10 2¡√ ¢ = 0.125
σ T 0.25 1
√ ¡√ ¢
d2 = d1 − σ T = 0.125 − 0.25 1 = −0.125
N (d1 ) = 0.549 7 N (d2 ) = 0.450 3
N (−d1 ) = 1 − N (d1 ) = 1 − 0.549 7 = 0.450 3
N (−d2 ) = 1 − N (d2 ) = 1 − 0.450 3 = 0.549 7
C = F e−rT N (d1 ) − Ke−rT N (d2 )
= 2.10e−0.055(1) 0.549 7 − 2.10e−0.055(1) 0.450 3 = 0.197 6
P = −F e−rT N (−d1 ) + Ke−rT N (dd2 )
= −2.10e−0.055(1) 0.450 3 + 2.10e−0.055(1) 0.549 7 = 0.197 6

12.3 Option the Greeks


The learning objectives in the SOA’s syllabus is to Interpret the option Greeks.
The learning objective in CAS Exam 3 Financial Economics is to Interpret
the option Greeks and elasticity measures. How to derive the option Greeks
is explained in the textbook Appendix 12.B, but Appendix 21.B is excluded
from the SOA MFE and CAS FE. So you might want to focus on the learning
objective of the exam.
Of all the Greeks, delta and gamma are the most important.
12.3. OPTION THE GREEKS 111

12.3.1 Delta

Cu − Cd
Delta ∆. You already see ∆ = when we try to find the replicating
Su − Sd
portfolio of a European call or put. It’s the number of stocks you need to
own at time zero to replicate the discrete payoff of a European call or put at
∂C ∂C
expiration date T . If the payoff is continuous, then ∆ = . Here ∆ =
∂S ∂S
is the number of stocks you need to have now to replicate the payoff of the
next instant (i.e. the payoff one moment later). The European call price is
∂C
C = Se−δT N (d1 ) − Ke−rT N (d2 ) and the delta for a call is ∆call = =
∂S
−δT
e N (d1 ).
One less visible thing to know is that d1 is also a function of S. So it’ll tame
∂C
some work to derive ∆call = = e−δT N (d1 ). One naive approach is treat
∂S
∂C
N (d1 ) as a constant and get ∆call = = e−δT N (d1 ). Interestingly, this gives
∂S
the correct answer!
Since deriving delta is not on the syllabus, you don’t need to go through the
∂C
messy math and prove ∆call = = e−δT N (d1 ). Just memorize that ∆call =
∂S
e−δT N (d1 ) for a European call and ∆put = −e−δT N (−d1 ) = −e−δT [1 − N (d1 )] =
∆call − e−δT for a European put.
Other results you might want to memorize:
0 ≤ ∆call ≤ 1
−1 ≤ ∆put ≤ 0

Example 12.3.1. Calculate the delta of the following European call and put.
The information is: S = 25,K = 20, σ = 0.15, r = 6%, δ = 2%, and T = 1
year.
µ ¶ µ ¶
S 1 2 25 1 2
ln + r−δ+ σ T ln + 0.06 − 0.02 + × 0.15 1
K 2 20 2
d1 = √ = √ =
σ T 0.15 1
1. 829 3
N (d1 ) = 0.966 3
N (−d1 ) = 1 − 0.966 3 = 0.033 7
∆call = e−δT N (d1 ) = e−0.02(1) 0.966 3 = 0.947 2
∆put = −e−δT N (−d1 ) = −e−0.02(1) 0.033 7 = −0.03 30

12.3.2 Gamma
Gamma is a measure of the change in delta regarding change in the underlying
stock price.
∂∆ ∂2C
Γ= =
∂S ∂S 2
112 CHAPTER 12. BLACK-SCHOLES

If gamma is too large a small change in stock price will cause a big change
in ∆. The bigger Γ, the more often you need to adjust your holding of the
underlying stocks.
∂∆put ∂∆call
Since ∆put = ∆call − e−δT , then = or Γcall = Γput .
∂S ∂S

12.3.3 Vega
Vega is the change of option price for 1% change of stock volatility (you can
think that the letter V stands for volatility).
∂C
V ega =
100∂σ

12.3.4 Theta
Theta is the change of option price regarding change in time when the option
is written (you can think that the letter T represents time). Let t represent the
time when the option is written and T the expiration date. Then
∂C (T − t)
θ=
∂t

12.3.5 Rho
Rho is a measure of the change in option value regarding a 1% change in the
risk free interest rate (you can think the letter R represent r)
∂C
ρ=
100∂r

12.3.6 Psi
Psi is a measure of the change in option value regarding a 1% change in the
dividend yield.
∂C
Ψ=
100∂δ

12.3.7 Greek measures for a portfolio


The portfolio’s Greek is just the sum of the individual Greek.

Example 12.3.2. A portfolio consists of 10 European calls and 50 otherwise


identical puts. The information about the European call and put is as follows:
S = 60,K = 65, σ = 0.25, r = 6%, δ = 4%, and T = 0.75 year. Calculate the
portfolio delta.
µ ¶ µ ¶
S 1 2 60 1 2
ln + r−δ+ σ T ln + 0.06 − 0.04 + × 0.25 0.75
K 2 65 2
d1 = √ = √ =
σ T 0.25 0.75
−0.192 2
N (d1 ) = N (−0.192 2) = 1 − N (0.192 2)
12.3. OPTION THE GREEKS 113

N (0.192 2) = 0.576 2
N (d1 ) = 1 − 0.576 2 = 0.423 8
N (−d1 ) = N (0.192 2) = 0.576 2
∆call = e−δT N (d1 ) = e−0.04(0.75) 0.423 8 = 0.411 3
∆put = −e−δT N (−d1 ) = −e−0.04(0.75) 0.576 2 = −0.559 2
∆portf olio = 10 (0.411 3) + 50 (−0.559 2) = −23. 847

Example 12.3.3. You buy 20 European calls and simultaneously write 35 Eu-
ropean puts on the same stock. The call expires in 3 months. The put both
expires in 9 months. The current stock price is 40. The call strike price is 35.
The put strike price is 45. The volatility is 20%. r = 8%, δ = 3%. Calculate
the delta of your portfolio.

−δT
Calculate ∆call
µ=e N (d¶
1) µ ¶
S 1 2 40 1
ln + r−δ+ σ T ln + 0.08 − 0.03 + × 0.22 0.25
K 2 35 2
d1 = √ = √ =
σ T 0.2 0.25
1. 510 3
N (d1 ) = 0.934 5
∆call = e−δT N (d1 ) = e−0.03(0.25) 0.934 5 = 0.927 5

−δT
Calculate ∆put µ = −e N (−d
¶ 1) µ ¶
S 1 2 40 1 2
ln + r−δ+ σ T ln + 0.08 − 0.03 + × 0.2 0.75
K 2 45 2
d1 = √ = √ =
σ T 0.2 0.75
−0.376 9
N (−d1 ) = 0.646 9
∆put = −e−0.03(0.75) 0.646 9 = −0.632 5
∆portf olio = 20 (0.927 5) − 35 (−0.632 5) = 40. 687 5
Since your write 35 puts, the delta of 35 puts is −35 (−0.632 5).

12.3.8 Option elasticity and volatility


The elasticity is
% change in option price
Ω=
% change in stock price
Suppose the stock price increase by where can be positive or negative.
Then the option price will change by ∆.
% change in stock price=
S

% change in option price=
C

∆S
Ω= C = (12.17)
C
S
114 CHAPTER 12. BLACK-SCHOLES

The option volatility is

σ option = σ stock × |Ω| (12.18)


Example 12.3.4. Calculate the elasticity and the volatility of a European call
option and an otherwise identical European put option. The information is:
S = 80,K = 70, σ = 0.35, r = 5%, δ = 3%, and T = 0.5.
µ ¶ µ ¶
S 1 2 80 1 2
ln + r−δ+ σ T ln + 0.05 − 0.03 + × 0.35 0.5
K 2 70 2
d1 = √ = √ =
σ T 0.35 0.5
0.703 7
N (d1 ) = 0.759
√ 2 N (−d1 ) = √ 1 − 0.759 2 = 0.240 8
d2 = d1 − σ T = 0.703 7 − 0.35 0.5 = 0.456 2
N (d2 ) = 0.675 9 N (−d2 ) = 1 − 0.675 9 = 0.324 1
C = Se−δT N (d1 )−Ke−rT N (d2 ) = 80e−0.03(0.5) 0.759 2−70e−0.05(0.5) 0.675 9 =
13. 687
P = −Se−δT N (−d1 )+Ke−rT N (−d2 ) = −80e−0.03(0.5) 0.240 8+70e−0.05(0.5)
0.324 1 = 3. 150
∆call = e−δT N (d1 ) = e−0.03(0.5) 0.759 2 = 0.747 9
∆put = −e−δT N (−d1 ) = −e−0.03(0.5) 0.240 8 = −0.237 2
∆call S 0.747 9 × 80
Ωcall = = = 4. 371
C 13. 687
∆put S −0.237 2 × 80
Ωput = = = −6. 024
P 3. 150
σ call = σ stock × |Ωcall | = 0.35 (4. 371) = 1. 530
σ put = σstock × |Ωput | = 0.35 (6. 024) = 2. 108

12.3.9 Option risk premium and Sharp ratio

At t = 0 the option is worth C = ∆S + B. Suppose the option expires in h


years. Let α represent the expected annual return on the stock, r the contin-
uously compounded risk-free rate per year, and γ the expected continuously
compounded return earned on the option per year. According to Equation 11.3,
we have
∆S + B
e−γh =
∆Seαh + Berh
However, ∆S + B = C µ ¶
γh ∆Seαh + Berh ∆S αh ∆S
e = = e + 1− erh
C C C
eγh = Ωeαh + (1 − Ω) erh (12.19)

Equation 12.19 holds for any h. Using the Taylor’s expansion, we have:
1
1 + γh + (γh)2 + ...
2
12.4. PROFIT DIAGRAMS BEFORE MATURITY 115

∙ ¸ ∙ ¸
1 1
= Ω 1 + ah + (ah)2 + ... + (1 − Ω) 1 + rh + (rh)2 + ...
2 2

For the above equation to hold for any h, it seems reasonable to assume that
1 = Ω + (1 − Ω)

γh = Ωah + (1 − Ω) rh
∙ ¸ ∙ ¸
1 2 1 2 1 2
(γh) + ... = Ω (ah) + ... + (1 − Ω) (rh) + ...
2 2 2
So we have γh = Ωah + (1 − Ω) rh or

γ − r = Ω (α − r) (12.20)
The Sharp ratio of an asset is the asset’s risk premium divided by the asset’s
volatility:
α−r
Sharp Ratio = (12.21)
σ
The Sharp ratio of an option is

Ω (α − r) α−r
Sharp Ratiooption = = = Sharp Ratiostock (12.22)
Ωσ stock σ stock

So the Sharp ratio of an option equals the Sharp ratio of the underlying
stock.

12.3.10 Elasticity and risk premium of a portfolio


The elasticity of a portfolio is the weighted average of the elasticities of the
portfolio components. In contrast, the Greek of a portfolio is just the sum of
the Greeks of the portfolio components.
The risk premium of a stock portfolio is just the portfolio’s elasticity times
the stock’s risk premium:

(γ − r)portf olio = Ωportf olio (α − r) (12.23)

12.4 Profit diagrams before maturity

12.4.1 Holding period profit

Example 12.4.1.
116 CHAPTER 12. BLACK-SCHOLES

You buy a European call option that expires in 1 year and hold it for one
day. Calculate your holding profit. Information is:

• The stock price is 40 when you buy the option.


• The stock price is still 40 one day later.
• K = 40
• r = 0.08
• δ=0
• σ = 30%

Solution.
At time zero, you buy a 1-year European option. Your purchase price is the
call price. µ ¶
S 1 2
ln + r−δ+ σ T
K 2
d1 = √
µ σ T ¶
40 1
ln + 0.08 − 0 + × 0.32 1
40 2
= √ = 0.416 7
√ 0.3 1 √
d2 = d1 − σ T = 0.416 7 − 0.3 1 = 0.116 7
N (d1 ) = 0.661 6 N (d2 ) = 0.546 5
C = 40e−0(1) 0.661 6 − 40e−0.08(1) 0.546 5 = 6. 285

One day later,µyour option is worth:¶


40 1 2 364
ln + 0.08 − 0 + × 0.3
40 2 365
d1 = r = 0.416 1
364
0.3
365 r
√ 364
d2 = d1 − σ T = 0.416 1 − 0.3 = 0.116 5
365
N (d1 ) = 0.661 33
N (d2 ) = 0.546 37
C = 40e−0(364/365) 0.661 33 − 40e−0.08(364/365) 0.546 37 = 6. 274

Suppose you buy the option at t = 0 by paying 6. 285 and sell the option
one day later for 6. 274 . Your holding period profit is:
6. 274 − 6. 285e0.08(1/365) = −0.01 2
You buy a European call option that expires in 1 year and hold it for 6
months. Calculate your holding profit. Information is:

• The stock price is 40 when you buy the option.


12.4. PROFIT DIAGRAMS BEFORE MATURITY 117

• The stock price is 40 after 6 months.

• K = 40

• r = 0.08

• δ=0

• σ = 30%

At time zero, you buy a 1-year European option. Your purchase price is the
call price. As calculated before, the call price is 6. 285

6 months later,µ the call is worth: ¶


40 1
ln + 0.08 − 0 + × 0.32 0.5
40 2
d1 = √ = 0.294 6
√ 0.3 0.5 √
d2 = d1 − σ T = 0.294 6 − 0.3 0.5 = 0.082 5
N (d1 ) = 0.615 9 N (d2 ) = 0.532 9
C = 40e−0(0.5) 0.615 9 − 40e−0.08(0.5) 0.532 9 = 4. 156
Your holding profit is:
4. 156 − 6. 285e0.08(0.5) = −2. 385
You buy a European put option that expires in 1 year and hold it for 6
months. Calculate your holding profit. Information is:

• The stock price is 40 when you buy the option.

• The stock price is 42 after 6 months.

• K = 40

• r = 0.08

• δ = 0.02

• σ = 30%

At time zero, you buy a 1-year European option. Your purchase price is the
call price. µ ¶
S 1 2
ln + r−δ+ σ T
K 2
d1 = √
µ σ T ¶
40 1
ln + 0.08 − 0.02 + × 0.32 1
40 2
= √ = 0.35
√ 0.3 1 √
d2 = d1 − σ T = 0.35 − 0.3 1 = 0.05
N (−d1 ) = 0.363 2 N (−d2 ) = 0.480 1
P = −40e−0.02(1) 0.363 2 + 40e−0.08(1) 0.480 1 = 3. 487
118 CHAPTER 12. BLACK-SCHOLES

One day later, your


µ option is worth: ¶
42 1
ln + 0.08 − 0.02 + × 0.32 0.5
40 2
d1 = = √ = 0.477 5
√ 0.3 0.5√
d2 = d1 − σ T = 0.477 5 − 0.3 0.5 = 0.265 4
N (−d1 ) = 0.316 5 N (−d2 ) = 0.395 4
P = −42e−0.02(0.5) 0.316 5 + 40e−0.08(0.5) 0.395 4 = 2. 035

Suppose you buy the option at t = 0 by paying 3. 487 and sell the option 6
months later for 2. 035. Your holding period profit is:
2. 035 − 3. 487e0.08(0.5) = −1. 59

12.4.2 Calendar spread

The textbook has an intimidating diagram (Figure 12.14). Don’t worry about
this diagram. Just focus on understanding what a calendar spread is.
A calendar spread (also called time spread or horizontal spread) is an option
strategy that takes advantage of the deteriorating time value of options. A
calendar spread involves selling one option that has a shorter expiring date and
simultaneously buying another option that has a longer expiration date, with
both options on the same stock and having the same strike price.

Suppose that Microsoft is trading for $40 per share. To have a calendar
spread, you can sell a $40-strike call on a Microsoft stock with option expiring
in 2 month. Simultaneously, you buy a $40-strike call on a Microsoft stock
with option expiring in 3 months. Suppose the price of a $40-strike 2-month to
expiration call is $2; the price of a $40-strike 3-month to expiration call is $5.
So your net cost of having a calendar spread at time zero is $3.
Then as time goes by, suppose the stock price doesn’t move much and is still
around $40, then the value of your sold call and purchased call both deteriorate
but at a different deteriorating speed. The value of the $40-strike 2-month to
expiration call deteriorates much faster. With each day passing, this option has
less and less value left. If there are only several days left before expiration, the
value of the sold call will be close to zero.
With each day passing, the value of the $40-strike 3-month to expiration call
also decreases but at a slower speed.

For example, one month later, the sold call has 1 month to expiration and
is worth only $1. The purchased call has 2-month to expiration and is worth
$4.5. Now the calendar spread is worth $3.5. You can close out your position by
buying a $40-strike 1-month to expiration call (price: $1) and sell a $40-strike
2-month to expiration call (price: $4.5). If you close out your position, you’ll
get $3.5.
12.5. IMPLIED VOLATILITY 119

At time zero, you invest $3 to set up a calendar spread. One month later,
you close out your position and get $3.5. Your profit (assuming no transaction
cost) is $0.5.
Time zero: your cost is $3
$40-strike call $40-strike call
Value $2 $5
Time to expiration 2 months 3 months

One month later: your total value is $3.5


$40-strike call $40-strike call
Value $1 $4.5
Time to expiration 1 month 2 months

A calendar spread can create value because as time passes the sold option
(which is your liability) can quickly become worthless yet the purchased option
(your asset) is still worth something.
For more examples, please refer to

• http://www.optionsxpress.com/educate/strategies/calendarspread.
aspx
• http://www.highyieldstrategy.com/artclndrsprds.htm.

12.5 Implied volatility

12.5.1 Calculate the implied volatility


Volatility cannot be observed. One approach to estimating volatility is use past
returns to calculate the historical volatility. However, past volatility may be a
poor estimate of future volatility because the market condition may change.
Another approach to estimating volatility is to calculate the implied volatil-
ity. The call and put values depend on (S, K, T, r, δ, σ). Given (S, K, T, r, δ)
and the option price, we can calculate σ. This is called the implied volatility.
Calculate the implied volatility given the following information about a Eu-
ropean call.

• CEuropean = 7.25
• S = 60
• K = 55
• T = 0.75 (i.e. 9 months)
• r = 0.06
• δ = 0.02
120 CHAPTER 12. BLACK-SCHOLES

Solution.
This is a difficult problem to solve manually. However, the calculation pro-
cedure is conceptually simple.
Implied volatility is solved by trial and error. You use a trial σ and see
whether the computed option price under the trial σ reproduces the actual
option price. If the computed option price is lower than the observed option
price, use a higher trial σ and try again; if the computed option price is higher
than the observed option price, use a lower trial σ and try again. Keep doing
this until you find a σ such the computed option price equals the observed option
price.
First, let’s tryµσ = 10% ¶
60 1 2
ln + 0.06 − 0.02 + 0.1 0.75
55 2
d1 = √ = 1. 3944
√ 0.1 0.75 √
d2 = d1 − σ T = 1. 3944 − 0.1 0.75 = 1. 307 8
N (d1 ) = 0.918 4 N (d2 ) = 0.904 5
C = 60e−0.02(0.75) 0.918 4 − 55e−0.06(0.75) 0.904 5 = 6. 725
6. 725 < CEuropean = 7.25. So increase σ and try again.
Try σ = 20% µ ¶
60 1
ln + 0.06 − 0.02 + 0.22 0.75
55 2
d1 = √ = 0.762 2
√ 0.2 0.75 √
d2 = d1 − σ T = 0.762 2 − 0.2 0.75 = 0.5890
N (d1 ) = 0.777 0 N (d2 ) = 0.722 1
C = 60e−0.02(0.75) 0.777 0 − 55e−0.06(0.75) 0.722 1 = 7. 958
7. 958 > CEuropean = 7.25. So decrease σ and try again

Try σ = 15% µ ¶
60 1 2
ln + 0.06 − 0.02 + 0.15 0.75
55 2
d1 = √ = 0.965 7
√ 0.15 0.75 √
d2 = d1 − σ T = 0.965 7 − 0.15 0.75 = 0.835 8
N (d1 ) = 0.832 9 N (d2 ) = 0.798 4
C = 60e−0.02(0.75) 0.832 9 − 55e−0.06(0.75) 0.798 4 = 7. 25
7. 25 = CEuropean = 7.25.
So the implied σ is 15%.

12.5.2 Volatility skew

Volatility skew refers to


• Options on the same stock with different strike price and expiration date
should have the same implied volatility. However, in reality, options on
the same stock with different strike price and expiration date don’t have
the same implied volatility.
12.6. PERPETUAL AMERICAN OPTIONS 121

• In addition, the different implied volatilities often form a pattern such as


"smiles," "frowns," and "smirks." (You don’t need to know what "smiles,"
"frowns," and "smirks" mean.)

Volatility is explained more in Derivatives Markets Chapter 23. However,


Chapter 23 is excluded from both SOA MFE and CAS Exam 3.

12.5.3 Using implied volatility


Implied volatility is important because it helps us

• We can generate option price that’s consistent with the price of other
similar options
• We can quote the option in terms of volatility rather than a dollar price
• Volatility skew helps us see how well an option pricing formula works.
Volatility skew shows that the Black-Scholes formula and assumptions are
not perfect.

This is all you need to know about how to use the implied volatility.

12.6 Perpetual American options


Perpetual American options are excluded from the exam syllabus. Please ignore
this chapter.
I included this section for completeness, but you don’t need to read it.

12.6.1 Perpetual calls and puts


Our task here is to derive the price formula for a perpetual American call with
strike price K.
A perpetual American option never expires and the option holder can exer-
cise the option at any time. It’s a typical American option where the expiration
date T = +∞.
The theoretical framework behind the perpetual option formula is the Black-
Scholes partial differential equation (called the Black-Scholes PDE). The Black-
Scholes PDE is Derivatives Market Equation 21.11 (page 682):
1
Vt + σ 2 S 2 VSS + (r − δ) SVS − rV = 0 (Textbook 20.11)
2

The above equation is also presented in Derivatives Market Equation 13.10


(page 430):
1
θ + σ 2 St2 Γt + rSt ∆t − rV (St ) = 0 (Textbook 13.10)
2
122 CHAPTER 12. BLACK-SCHOLES

Please note that Equation 13.10 assumes δ = 0. In addition,it uses C (in-


stead of V ) to represent the option price.
The Black-Scholes PDE is commonly written as:

∂V (t, St ) 1 2 2 ∂ 2 V (t, St ) ∂V (t, St )


+ σ St 2 + (r − δ) St − rV (t, St ) = 0 (12.24)
∂t 2 ∂St ∂St

In the above equation, V (t, St ) is the option price at time t where the stock
price is St .
If you are interested in learning how to derive the Black-Scholes PDE, refer
to the textbook. For now let’s accept Equation 12.24.
∂V
For a perpetual option, its value doesn’t depends on time. Hence = 0.
∂t
The Black-Scholes PDE becomes an ordinary differential equation:

1 2 2 d2 V (t, St ) dV (t, St )
σ St 2 + (r − δ) St − rV (t, St ) = 0 (12.25)
2 dSt dSt
To find the solution to Equation 12.25, let’s simplify the equation as
d2 V (t, St ) dV (t, St )
St2 2 + St − V (t, St ) = 0
dSt dSt
We can guess the solution is in the form of V (t, St ) = Sth . Then
dV (t, St ) d2 V (t, St )
= hSth−1 = h (h − 1) Sth−2
dSt dSt2
d2 V (t, St ) dV (t, St )
St2 + St − V (t, St )
dSt2 dSt
= St2 h (h − 1) Sth−2 + St hSth−1¡ − Sth ¢
= Sth [h (h − 1) + h − 1] = Sth h2 − 1
¡ ¢ d2 V (t, St ) dV (t, St )
So as long as h2 − 1 = 0, or h = ±1, Equation St2 +St −
dSt2 dSt
V (t, St ) = 0 has a solution.
Of course, if Sth is a solution, aSth must also be a solution.
Similarly, we can guess that the solution to Equation 12.25 is in the form of
V (t, St ) = Sth . Some brilliant thinker guessed the following solution:
µ ¶h
St H∗ − K h
V (t, St ) = (H ∗ − K) ∗
= h
St = aSth
H (H ∗ )
Here H ∗ the stock price where exercise is optimal (H ∗ is a constant). H ∗ −K
µ ¶h
St
is the terminal payoff at exercise time. is an indicator telling us how
H∗

close the stock price approaches H .
dV (t, St ) d ¡ h¢ d2 V (t, St )
= aSt = ahSth−1 = ah (h − 1) Sth−2
dSt dS dSt2
Equation 12.25 becomes:
1 2 2
σ St ah (h − 1) Sth−2 + (r − δ) St ahSth−1 − raSth = 0
2
12.6. PERPETUAL AMERICAN OPTIONS 123

1 2
σ h (h − 1) + (r − δ) h − r = 0
2 µ ¶
1 2 2 1
σ h + r − δ − σ2 h − r = 0
2 2 s
µ ¶2
1 2 1 2
σ − (r − δ) ± r−δ− σ + 2σ2 r
2 2
h=
sµ σ2
¶2
1 r−δ r−δ 1 2r
= − 2 ± − + 2
2 σ σ2 2 σ
sµ ¶2
1 r−δ 1 r−δ 2r
= − 2 ± − 2 + 2
2 σ 2 σ σ

But how can we find H ∗ ? Since the perpetual American option can be ex-
ercised at any time, the option holder will choose H ∗ such that V (t, St ) =
µ ¶h
St dV (t, St )
(H ∗ − K) ∗
reaches its maximum value. This requires setting =
H dH ∗
0. " µ ¶h #
dV (t, St ) d ∗ St
= (H − K)
dH ∗ dH ∗ H∗
d h i
∗ 1−h ∗ −h
= Sth (H ) − K (H )
hdH ∗ i
= St (1 − h) (H ∗ )−h − K (−h) (H ∗ )−h−1 = 0
h

(1 − h) (H ∗ )−h − K (−h) (H ∗ )−h−1 = 0


(1 − h) H ∗ − K (−h) = 0
K (−h) h
H∗ = = K
1−h h−1
h 1
H∗ − K = K −K = K
h−1 h−1
µ ¶h µ ¶h
∗ St K h − 1 St
So V (t, St ) = (H − K) =
H∗ h−1 h K

Set t = 0. Let S represent the stock price at time zero (i.e. S = S0 ). Then
the option value at time zero is
µ ¶h µ ¶h
S K h−1 S
V (0, S) = (H ∗ − K) =
H∗ h−1 h K

But how do we choose h since there are two possible value of h? H ∗ − K =


1
K. To avoid h − 1 becoming negative, we choose the bigger h:
h−1 sµ ¶2
1 r−δ 1 r−δ 2r
hcall = − 2
+ − 2 + 2
2 σ 2 σ σ
124 CHAPTER 12. BLACK-SCHOLES

A similar logic can be applied to a perpetual American put. We can guess


the put value is:
µ ¶h
∗ St K − H∗ h
V (t, St ) = (K − H ) = St = bSth
H∗ (H ∗ )h

Once again, we get the following equation:


1 2
σ h (h − 1) + (r − δ) h − r = 0
2 µ ¶
1 2 2 1 2
σ h + r−δ− σ h−r =0
2 s2µ ¶2
1 r−δ 1 r−δ 2r
h= − 2 ± − + 2
2 σ 2 σ2 σ

dV (t, St )
Set = 0:
dH ∗ " µ ¶h #
dV (t, St ) d St h i
∗ h d ∗ ∗ −h
= (K − H ) = (St ) (K − H ) (H )
dH ∗ dH ∗ H∗ dH ∗
h i
h d −h 1−h
= (St ) K (H ∗ ) − (H ∗ )
hdH ∗ i
= (St )h K (−h) (H ∗ )−h−1 − (1 − h) (H ∗ )−h = 0

−h−1 −h
K (−h) (H ∗ ) − (1 − h) (H ∗ ) =0
h
K (−h) − (1 − h) H ∗ = 0 → H∗ = K
µ ¶ h − 1
∗ h 1
→K −H = 1− K= K
h−1 1−h

To avoid 1 − h becoming
sµ negative, ¶
we choose the smaller h:
2
1 r−δ 1 r−δ 2r
hput = − 2
− − 2
+ 2
2 σ 2 σ σ

Summary of the formulas for perpetual American calls and perpetual Amer-
ican puts:

µ ¶hcall µ ¶hcall
∗ S K hcall − 1 S
Cperpetual = (HCall − K) ∗ =
HCall hcall − 1 hcall K
(12.26)

∗ hcall
HCall = K (12.27)
hcall − 1
µ ¶ sµ ¶2
1 r−δ 1 r−δ 2r
hcall = − 2
+ − 2
+ 2 (12.28)
2 σ 2 σ σ
12.6. PERPETUAL AMERICAN OPTIONS 125

µ ¶hput µ ¶hput
¡ ∗
¢ S K hput − 1 S
Pperpetual = K − Hput ∗ = (12.29)
Hput 1 − hput hput K

∗hput
Hput = K (12.30)
hput − 1
µ ¶ sµ ¶2
1 r−δ 1 r−δ 2r
hput = − 2 − − 2 + 2 (12.31)
2 σ 2 σ σ
1 2
hcall and hput satisfies: σ h (h − 1) + (r − δ) h − r = 0 (12.32)
2

Example 12.6.1. Calculate the price of a perpetual American call and the price
of an otherwise identical perpetual American put. The information is as follows.
The current stock price is S = 50. The strike price is K = 45. The continuously
compounded risk-free rate is r = 6%. The continuously compounded dividend
yield is 2%. The stock volatility is σ = 25%.

Solution.
Solve for h.
1 2
σ h (h − 1) + (r − δ) h − r = 0
2
1
× 0.252 h (h − 1) + (0.06 − 0.02) h − 0.06 = 0
2
h1 = 1. 252 7 h2 = −1. 532 7
Use the bigger h for call and the smaller h for put.

Next, calculate the stock price where exercising the option is optimal.
∗ hcall 1. 252 7
HCall = K= × 45 = 223. 08
hcall − 1 1. 252 7 − 1
∗ hput −1. 532 7
Hput = K= × 45 = 27. 23
hput − 1 −1. 532 7 − 1
µ ¶hcall µ ¶1. 252 7
∗ S 50
Cperpetual = (HCall − K) ∗ = (223. 08 − 45) =
HCall 223. 08
27. 35 µ ¶hput µ ¶−1. 532 7
¡ ∗
¢ S 50
Pperpetual = K − Hput ∗ = (45 − 27. 23 ) = 7.
Hput 27. 23
00
Tip 12.6.1. The CD attached to the textbook Derivatives Markets has a spread-
sheet that calculates the price of a perpetual American call and a perpetual Amer-
ican put. The spreadsheet is titled "optbasic2." You can use this spreadsheet to
double check your solution.
126 CHAPTER 12. BLACK-SCHOLES

12.6.2 Barrier present values


Consider the following barrier option. If the stock price first reaches a preset
price H from below, then the payoff of $1 is received. This is a special case of a

perpetual American call option by setting the terminal payoff HCall − K as $1

and by setting HCall = H.
The value at time zero of $1 received when the stock price first reaches H
from below (i.e. the stock first rises to H) is
µ ¶h1
S
(12.33)
H

µ ¶ sµ ¶2
1 r−δ 1 r−δ 2r
h1 = − 2 + − 2 + (12.34)
2 σ 2 σ σ2

Similarly, the value at time zero of $1 received when the stock price first
reaches H from above (i.e. the stock first falls to H) is
µ ¶h2
S
(12.35)
H

µ ¶ sµ ¶2
1 r−δ 1 r−δ 2r
h2 = − 2 − − 2 + (12.36)
2 σ 2 σ σ2

1 2
h1 and h2 satisfies: σ h (h − 1) + (r − δ) h − r = 0 (12.37)
2

Example 12.6.2.

Calculate the value of a $1 paid if the stock price first reaches $100 and $60
respectively. The information is:

• S = 80

• r = 6%

• δ = 2%

• σ = 30%

Solution.

• calculate the value of a $1 paid if the stock price first reaches $100
12.6. PERPETUAL AMERICAN OPTIONS 127

H = 100 > S. So we need to calculate the price of $1 payoff when the stock
price first rises to H from below
1 2
σ h (h − 1) + (r − δ) h − r = 0
2
1 2
0.3 h (h − 1) + (0.06 − 0.02) h − 0.06 = 0
2
h1 = 1. 211 6 h2 = −1. 100 5
Use the bigger h
µ ¶h1 µ ¶1. 211 6
S 80
= = 0.763
H 100

• calculate the value of a $1 paid if the stock price first reaches $90

H = 60 < S.So we need to calculate the price of $1 payoff when the stock
price first falls to H from above.
h1 = 1. 211 6 h2 = −1. 100 5
Use the smaller
µ ¶h2 µ ¶−1. 100 5h
S 80
= = 0.729
H 60
Tip 12.6.2. The CD attached to the textbook Derivatives Markets has a spread-
sheet that calculates the price of a barrier option. The spreadsheet is titled "opt-
basic2." You can use this spreadsheet to double check your solution.
128 CHAPTER 12. BLACK-SCHOLES
Chapter 13

Market-making and
delta-hedging

13.1 Delta hedging


∂V
Here is the main idea behind delta hedging. Delta of an option is ∆ = .
∂S
Hence for a small change in stock price, the change of the option value is ap-
proximately V1 − V0 ≈ ∆ (S1 − S0 ). Suppose you sell one European call option.
If you hold ∆ shares of stock, you are immunized against a small change of the
stock price.
If the stock price goes up from S0 to S1 , then the call will be more valuable
to the buyer and your are exposed to more risk. Suppose the value of the call
goes up from V0 to V1 as the stock price goes up from S0 to S1 , then your
liability will increase by V1 − V0 . At the same time, the value of your ∆ shares
of stock will go up by ∆ (S1 − S0 ). Because V1 − V0 ≈ ∆ (S1 − S0 ), the increase
of your liability will be roughly offset by the increase of your asset.
However, under delta hedging, you are immunized against only a small
change of the stock price (just like immunization by duration matching assets
and liabilities is only good for a small change of interest rate). If a big change
of stock price knocks off your hedging, you’ll need to rebalance your hedging.
However, in the real world, continuously rebalancing the hedging portfolio
is impossible. Traders can only do discrete rebalancing.

13.2 Examples of Delta hedging


Make sure you can reproduce the textbook calculation of delta-hedging for 2
days. In addition, make sure you can reproduce the textbook table 13.2 and
13.3
Exam problems may ask you to outline hedging transactions or calculate the
hedging profit.

129
130 CHAPTER 13. MARKET-MAKING AND DELTA-HEDGING

The major difficulty many candidates face is not knowing how to hedge.
They wonder "Should the market-maker buy stocks? Should he sell stocks?"
To determine how to hedge a risk, use the following ideas:

• The goal of hedging is to break even. If a trader makes money


on option, he must lose money on stock; if he loses money on
option, he must make money on stock.

• To determine whether a trader should buy stocks or sell stocks,


ask "If the stock price go up (or down), will the trader make
money or lose money on the option?"

• If a trader loses money on the option as the stock price goes up,
then the trader needs to initially own (i.e. buy) stocks. This
way, the value of the trader’s stocks will go up and the trader
will make money on his stocks. He can use this profit to offset
his loss in the option.

• If a trader makes money on the option as the stock price goes


up, then the trader needs to initially short sell stocks. This way,
as the stock price goes up, the trader will lose money on the
short sale (because he needs to buy back the stocks at a higher
price). His loss in short sale can offset his profit in option.

Example 13.2.1. The trader sells a call. How can he hedge his risk,?

If the stock price goes up, the call payoff is higher and the trader will lose
money. To hedge this risk, the trader should buy stocks. This way, if the stock
price goes up, the trader makes money in the stocks. This profit can be used to
offset the trader’s loss in the written call.
You can also ask the question "If the stock goes down, will the trader make
money or lose money on the option?" If the stock goes down, the call payoff is
lower and the trader will make money. To eat up his profit in the option, the
trader needs to buy stocks. This way, as the stock price goes down, the value
of the trader’s stocks will go down too and the trader will lose money in his
stocks. This loss will offset the trader’s profit in the written call.

Example 13.2.2. The trader sells a put. How can he hedge his risk?

If the stock price goes down, the put payoff is higher and the trader will
make money. To hedge his risk, the trader should short sell stocks. This way,
if the stock price goes down, the trader can buy back stocks at lower price,
making a profit on stocks. This profit can be used to offset the trader’s loss in
the written put.
You can also ask the question "If the stock goes up, will the trader make
money or lose money on the option?" If the stock goes up, the put payoff is
lower and the trader will make money on the written put. To eat up his profit
in the option, the trader needs to short sell stocks. This way, as the stock price
13.2. EXAMPLES OF DELTA HEDGING 131

goes up, the trader needs to buy back stocks at a higher price. The trader will
lose money in his stocks. This loss will offset the trader’s profit in the written
put.

Example 13.2.3. The trader buys a call. How can he hedge his risk?

If a trader buys a call, the most he can lose is his premium and there’s no
need to hedge. This is different from selling a call, where the call seller has
unlimited loss potential.
However, if a trader really wants to hedge his limited risk, he can short sell
stocks.

Example 13.2.4. The trader buys a put. How can he hedge his risk?

If a trader buys a put, the most he can lose is his premium and there’s no
need to hedge. This is different from selling a put, where the call seller has a
big loss potential.
However, if a trader really wants to hedge his limited risk, he can buy stocks.

Example 13.2.5.

Reproduce the textbook example of delta-hedging for 2 days. Here is the


recap of the information. At time zero the market maker sells 100 European
call options on a stock. The option expires in 91 days.

• K = $40

• r = 0.08

• δ=0

• σ = 0.3

• The stock price at t = 0 is $40

• The stock price at t = 1/365 (one day later) is $40.50

• The stock price at t = 2/365 (two days later) is $39.25

The market-maker delta hedges its position daily. Calculate the market-
maker’s daily mark-to-market profit

Solution.

First, let’s calculate the call premium and delta at Day 0, Day 1, and Day 2.
I used my Excel spreadsheet to do the following calculation. If you can’t fully
match my numbers, it’s OK.
132 CHAPTER 13. MARKET-MAKING AND DELTA-HEDGING

Time t Day 0 (t = 0) Day 1 Day 2


Expiry T (Yrs) 91/365 90/365 89/365
µ ¶ St 40 40.50 39.25
St 1
ln + 0.08 + × 0.32 T
40 2
d1 = √ 0.208048 0.290291 0.077977
0.3 T
N (d
√1 ) 0.582404 0.614203 0.531077
d2 = d1 − 0.3 T 0.058253 0.141322 −0.070162
N (d2 ) 0.523227 0.556192 0.472032
C = St N (d1 ) − 40e−0.08T N (d2 ) 2.7804 3.0621 2.3282
∆ = e−δ(T −t) N (d1 ) 0.58240 0.61420 0.53108

The above table can be simplified as follows:


Time t Day 0 t = 0 Day 1 t = 1/365 Day 2 t = 2/365
Expiry T T0 = 91/365 T1 = 90/365 T2 = 89/365
St S0 = 40 S1 = 40.50 S2 = 39.25
Ct C0 = 100 × 2.7804 = 278. 04 C1 = 306.21 C2 = 232.82
∆t ∆0 = 100 × 0.58240 = 58. 24 ∆1 = 61.420 ∆2 = 53.108

Beginning of Day 0
Trader #0 goes to work
Day 0 t = 0
T0 = 91/365
S0 = 40
C0 = 278.04
∆0 = 58.240

Trader #0 goes to work. The brokerage firm (i.e. the employer of Trader
#0) gives Trader #0 C0 = $278. 04. This is what the call is worth today.
Trader #0 needs to hedge the risk of the written call throughout Day 0.
To hedge the risk, Trader #0 buys ∆0 stocks, costing ∆0 S0 = 58. 24×40 = $
2329. 6. Since Trader #0 gets $278. 04 from the brokerage firm, he needs to
borrow:
∆0 S0 − C0 = 2329. 6 − 278. 04 = $2051. 56.
The trader can borrow $2051. 56 from a bank or use this own money. Either
way, this amount is borrowed. The borrowed amount earns a risk free interest
rate.
Now Trader #0’s portfolio is:
Component Value
∆0 = 58.24 stocks 2487. 51
call liability −278. 04
borrowed amount 2051. 56
Net position 0

End of Day 0 (or Beginning of Day 1)


13.2. EXAMPLES OF DELTA HEDGING 133

Mark to market without rebalancing the portfolio


Day 1 t = 1/365
T1 = 90/365
S1 = 40.50
C1 = 306.21

Method 1
To cancel out his position, Trader #0 can at t = 0

• buy a call (we call this the 2nd call) from the market paying C1 = $306.
21. At expiration, the payoff of this 2nd call will exactly offset the payoff
of the 1st call. The 1st call is the call sold by the brokerage firm at t = 0
to the customer who bought the call. For example, if at expiration the
stock price is ST = 100, then both calls are exercised. The trader gets
ST − K = 100 − 40 = 60 from the 2nd call. The liability of the first call
is also $60. These two calls cancel each other out.

• sell out ∆0 = 58. 24 stocks for ∆0 S1 = 58. 24 × 40.5 = $2358. 72

• pay off the loan. The payment is (∆0 S0 − C0 ) erh = 2051. 56e0.08×1/365 =


2052. 01

At the end of Day 0, the trader’s profit is −C1 + ∆0 S1 − (∆0 S0 − C0 ) erh =


−306. 21 + 2358. 72 − 2052. 01 = 0.5
Trader #0 hands in $0.5 profit to his employer and goes home.

Method 2 We consider the change between Day 0 and Day 1.

• In the beginning of Day 0, the trader’s stock is worth ∆0 S0 = 58. 24 (40) =


2329. 6. In the end of Day 0 (or the beginning of Day 1), the trader’s stock
is worth ∆0 S1 = 58. 24 (40.5) = 2358. 72. The value of the trader’s 58. 24
stocks goes up by 58. 24 (40.5 − 40) = 29. 12. This is good for the trader.

• In the beginning of Day 0, the call is worth C0 = $278. 04. In the end of
Day 0 (or the beginning of Day 1), the call is worth C1 = 306.21. The call
value is the trader’s liability. Now the trader’s liability increases by 306.
21 − 278. 04 = 28. 17. So the trader has a loss 28. 17 (or a gain of −28.75).
Recall under Method 1, the trader has to buy a call for 306. 21 to cancel
out the call he sold. So call value increase is bad for the trader.

• In the beginning of Day 0, the borrowed amount is ∆0 S0 − C0 = 2051. 56.


In the end of Day 0 (or the beginning of Day 1), this borrowed amount
grows to (∆0¡S0 − C0 ¢) erh = 2051. 56e0.08×1/365 = 2052. 01. The increase
(∆0 S0 − C0 ) erh − 1 = 2051. 56e0.08×1/365 − 2051. 56 = 0.45. This is
the interest paid on the amount borrowed. No matter the trader borrows
money from a bank or uses his own money, the borrowed money needs to
earn a risk free interest rate.
134 CHAPTER 13. MARKET-MAKING AND DELTA-HEDGING

At the end of Day 0, the trader’s profit is:


¡ ¢
∆0 S1 − ∆0 S0 − (C1 − C0 ) − (∆0 S0 − C0 ) erh − 1 = 29. 12 − 28. 17 − 0.45 =
0.5
You can verify that ¡ ¢
∆0 S1 −∆0 S0 −(C1 − C0 )−(∆0 S0 − C0 ) erh − 1 = −C1 +∆0 S1 −(∆0 S0 − C0 ) erh

Method 3

On Day 0, the trader owns ∆0 = 58.240 stocks to hedge the call liability
C0 = $278. 04. The trader’s net asset is
M V (0) = ∆0 S0 − C0 = 58. 24 × 40 − 278. 04 = $2051. 56

In the end of Day 0 (or the beginning of Day 1) before the trader rebalances
his portfolio, the trader’s asset is:
M V BR (1) = ∆0 S1 − C1 = 58. 24 × 40.5 − 100 × 3.0621 = 2052. 51
BR stands for before rebalancing.
The trader’s profit at the end of Day 0 is:
M V BR (1) − M V (0) e0.08×1/365 = (∆0 S1 − C1 ) − (∆0 S0 − C0 ) erh = 2052.
51 − 2051. 56e0.08×1/365 = 0.50

Please note that Trader #0 doesn’t need to rebalance the portfolio. The
portfolio is rebalanced by the next trader.

Beginning of Day 1
Trader #1 goes to work
Day 1 t = 1/365
T1 = 90/365
S1 = 40.50
C1 = 306.21
∆1 = 61.420

Trader #1 goes to work. He starts from a clean slate. The portfolio is


automatically rebalanced since Trader #1 starts from scratch.

The brokerage firm gives Trader #1 C1 = $306.21.


Trader #1 needs to hedge the risk of the written call throughout Day 1.
To hedge the risk, Trader #1 buys ∆1 = 61.420 stocks, costing ∆1 S1 =
61.42 × 40.5 = $2487. 51. Since Trader #1 gets $306.21 from the brokerage firm,
he needs to borrow:
∆1 S1 − C1 = 2487. 51 − 306.21 = 2181. 3
The trader can borrow $2181. 3 from a bank or use this own money. Either
way, this amount is borrowed. The borrowed amount earns a risk free interest
rate.
Now Trader #1’s portfolio is:
13.2. EXAMPLES OF DELTA HEDGING 135

component value
∆1 = 61.42 stocks 2487. 51
call liability −306.21
borrowed amount 2181. 3
Net position 0

One question arises, "What if Trader #1 doesn’t start from a clean slate?"
Next, we’ll answer this question.
Instead of starting from scratch, Trader #1 can start off with Trader #0’s
portfolio. At the end of Day 0, Trader #0 has

• ∆0 = 58.24 stocks
• a borrowed amount (∆0 S0 − C0 ) erh = 2051. 56e0.08×1/365 = 2052. 01
• 0.5 profit

At the end of Day 0 or the beginning of Day 1, ∆1 = 61.420. So Trader #1


buys additional shares:
∆1 − ∆0 = 61.42 − 58.24 = 3. 18
The cost of these additional shares is (∆1 − ∆0 ) S1 = 3. 18 × 40.5 = 128. 79
Since these additional shares are bought at the current market price, Trader
#1 can sell these shares at the same price he bought them. This doesn’t affect
the mark-to-market profit.
Trader #1 can borrow 128. 79 to pay for the purchase of 3. 18 stocks.
Now Trader #1 has a total of ∆1 = 61.42 shares worth ∆1 S1 = 61.42×40.5 =
2487. 51
His liability is now C1 = 306.21
The borrowed amount is now 2052. 01 + 128. 79 + 0.5 = 2181. 3. The 0.5 (the
profit made by Trader #0) is the amount of money Trader #1 borrows from
Trader #0.

Now Trader #1’s portfolio is:


component value
∆1 = 61.42 stocks $2487. 51
call liability −306.21
borrowed amount 2181. 3
Net position 0
The portfolio is the same as the portfolio if Trader #1 starts from scratch.
Calculation is simpler and cleaner if we start from scratch.

End of Day 1 (or Beginning of Day 2)


Mark to market without rebalancing the portfolio
Day 2 t = 2/365
T2 = 89/365
S2 = 39.25
C2 = 232.82
136 CHAPTER 13. MARKET-MAKING AND DELTA-HEDGING

Method 1
To cancel out his position, the trader can

• buy a call from the market paying C2 = 232.82. The payoff of this call
will exactly offset the payoff of the call sold by the brokerage firm to the
customer. These two call have the common expiration date T1 = 89/365
and the same payoff. They will cancel each other out.
• sell out ∆1 = 61.420 stocks for ∆1 S2 = 61.420 × 39.25 = 2410. 735
• pay off the loan. The payment is (∆1 S1 − C1 ) erh = 2181. 3e0.08×1/365 =
2181. 778

The trader’s profit at the end of Day 1 is


−C2 + ∆1 S2 − (∆1 S1 − C1 ) erh = 232.82 + 2410. 735 − 2181. 778 = −3. 863
Trader #1 hands in −3. 863 profit to his employer and goes home.

Method 2 We consider the change between the beginning of Day 1 and


the end of Day 1 (or the beginning of Day 2).

• In the beginning of Day 1, the trader’s stock is worth ∆1 S1 ; In the end of


Day 1 (or the beginning of Day 2), the trader’s stock is worth ∆1 S2 . The
value of the trader’s stocks goes up by ∆1 S2 − ∆1 S1
• In the beginning of Day 1, the call is worth C1 ; In the end of Day 1 (or the
beginning of Day 2), the call is worth to C2 . The call value is the trader’s
liability. Now the trader’s liability increases by C2 − C1
• In the beginning of Day 1, the borrowed amount is ∆1 S1 − C1 . In the
end of Day 1 (or the beginning of Day 2), this borrowed
¡ ¢amount grows
to (∆1 S1 − C1 ) erh . The increase is (∆1 S1 − C1 ) erh − 1 . This is the
interest paid on the amount borrowed. No matter the trader borrows
money from a bank or uses his own money, the borrowed money needs to
earn a risk free interest rate.

The trader’s profit at the end of Day 1: ¡ ¢


(∆1 S2 − ∆1 S1 ) − (C2 − C1 ) − (∆1 S1 − C1 ) erh − 1
= −C2 + ∆1 S2 − (∆1 S1 − C1 ) erh = −3. 863

Method 3

In the beginning of Day 1, the trader’s net asset is


M V (1) = ∆1 S1 − C1 = 61.420 × 40.5 − 306.21 = 2181. 3
In the end of Day 1 before the trader rebalances his portfolio, the trader’s
asset is:
M V BR (2) = ∆1 S2 − C2 = 61.420 × 39.25 − 232.82 = 2177. 915
The trader’s profit at the end of Day 1 is:
M V BR (2) − M V (1) erh = 2177. 915 − 2181. 3e0.08×1/365 = −3. 863
13.2. EXAMPLES OF DELTA HEDGING 137

You can verify that


M V BR (2) − M V (1) erh ¡ ¢
= (∆1 S2 − ∆1 S1 ) − (C2 − C1 ) − (∆1 S1 − C1 ) erh − 1
= −C2 + ∆1 S2 − (∆1 S1 − C1 ) erh
Method 3 is often faster. Under this method,
Profit during a day=Asset at the end of Day before rebalancing the portfolio
- Future value of the asset at the beginning of the day

Asset at the end of Day before rebalancing the portfolio = delta at the
beginning of the day × stock price at the end of the day
Profit at the end of Day t = M V BR (t + 1)−M V (t) erh = (∆t St+1 − Ct+1 )−
(∆t St − Ct ) erh

Now this completes the textbook example on page 417 to 418.


Next, let’s do additional calculations and calculate the profit at the end of
Day 2, 3, and 4 (or the profit at the beginning of Day 3,4,5)

Day Day 2 Day 3 Day 4 Day 5


Time t t = 2/365 t = 3/365 t = 4/365 t = 5/365
Expiry T T2 = 89/365 T3 = 88/365 T4 = 87/365 T5 = 86/365
St S2 = 39.25 S3 = 38.75 S4 = 40 S5 = 40
Ct C2 = 232.82 C3 = 205. 46 C4 = 271.04 C5 = 269.27
∆t ∆2 = 53.108 ∆3 = 49. 564 ∆4 = 58.06 ∆5 = 58.01
Profit end of Day 0.40 −4. 0 1. 32

Profit at the end of Day 2 (or beginning of Day 3).

• Asset at the beginning of Day 2: M V (2) = ∆2 S2 − C2 = 53.108 × 39.25 −


232.82 = 1851. 669

• Asset at the end of Day 2 before rebalancing: M V BR (3) = ∆2 S3 − C3 =


53.108 × 38.75 − 205. 46 = 1852. 475

• Profit at the end of Day 2: M V BR (3) − M V (2) erh = 1852. 475 − 1851.
669e0.08/365 = 0.400 1

Profit at the end of Day 3 (or beginning of Day 4).

• Asset at the beginning of Day 3: M V (3) = ∆3 S3 − C3 = 49. 564 × 38.75 −


205. 46 = 1715. 145

• Asset at the end of Day 3 before rebalancing: M V BR (4) = ∆3 S4 − C4 =


49. 564 × 40 − 271.04 = 1711. 52

• Profit at the end of Day 3: M V BR (4) − M V (2) erh = 1711. 52 − 1715.


145e0.08/365 = −4. 000 96
138 CHAPTER 13. MARKET-MAKING AND DELTA-HEDGING

Profit at the end of Day 4 (or beginning of Day 5).

• Asset at the beginning of Day 4: M V (4) = ∆4 S4 − C4 = 58.06 × 40 −


271.04 = 2051. 36

• Asset at the end of Day 4 before rebalancing: M V BR (3) = ∆4 S5 − C5 =


58.06 × 40 − 269.27 = 2053. 13

• Profit at the end of Day 4: M V BR (5) − M V (4) erh = 2053. 13 − 2051.


36e0.08/365 = 1. 320 3

You can verify that the profit at the end of Day 2, 3, 4 calculated above
matches Derivatives Markets Table 13.2. However, in Table 13.2, the profit at
the end of Day 0 is posted in Day 1 column. Similarly, the profit at the end of
Day 1 is posted in Day 1 column. So on and so forth.

13.3 Textbook Table 13.2


Next, we want to reproduce the textbook Table 13.2.
Profit at the end of Day t (or the beginning of Day t + 1) can be broken
down into two parts:
= M V BR (t + 1) − M V (t) erh = M V BR (t + 1) − M V (t) − M V (t) erh +
M V (t) ¡ ¢
= M V BR (t + 1) − M V (t) − M V (t) erh − 1
¡ ¢
= M V BR (t + 1) − M V (t) + −M V (t) erh − 1
| {z } | {z }
capital gain at end of Day t Interest earned at the end of Day t

Define M V BR (t + 1) − M V (t) = CapitalGain earned at the end of Day t


CapitalGain (t) is also equal to:
M V BR (t + 1)−M V (t) = (∆t St+1 − Ct+1 )−(∆t St − Ct ) = ∆t (St+1 − St )−
(Ct+1 − Ct )
¡ ¢
Define −M V (t) erh − 1 as the interest earned at the end of Day t. If the
trader invests
¡ money
¢ at the beginning of Day t, then M V (t) is positive and
−M V (t) erh − 1 is negative. The negative interest earned is just the interest
expense incurred by the trader.

Define M V (t) as the investment made at the beginning of Day t. If M V (t)


is negative, then it means that the trader receives money.

Example 13.3.1.

Reproduce the textbook Table 13.2


We already reproduced the daily profit in Table 13.2. We just need to
reproduce the investment, interest, and the capital gain.
13.3. TEXTBOOK TABLE 13.2 139

Day 0 1 2
Time t t=0 t = 1/365 t = 2/365
Expiry T T0 = 91/365 T1 = 90/365 T2 = 89/365
St S0 = 40 S1 = 40.5 S2 = 39.25
Ct C0 = 278.04 C1 = 306.21 C2 = 232.82
∆t ∆0 = 58.2404 ∆1 = 61.42 ∆2 = 53.108
Investment (beginning of the day) 2051. 56 2051. 56 1851. 669
interest earned during the day −0.45 −3. 385 −0.41
Capital gain (end of the day) 0.95 −0.478 0.81
Profit (end of the day) 0.50 −3. 863 0.40

Day 0

We already know:
M V (0) = ∆0 S0 − C0 = 58. 24 × 40 − 278. 04 = $2051. 56
M V BR (1) = ∆0 S1 − C1 = 58. 24 × 40.5 − 100 × 3.0621 = 2052. 51
The trader’s profit at the end of Day 0 is:
M V BR (1) − M V (0) erh = 2052. 51 − 2051. 56e0.08×1/365 = 0.50

To find the capital gain and the interest earned at the end of Day 0, we just
need to break down the profit M V BR (1) − M V (0) erh into two parts:
M V BR (1) − M V (0) erh
= M V BR (1) − M V (0) − M V (0) erh + M V (0)
¡ rh ¢
BR
= MV (1) − M V (0) + −M V (0) e − 1
| {z } | {z }
capital gain interest earned

The capital gain at the end of Day 0:


M V BR (1) − M V (0) = 2052. 51 − 2051. 56 = 0.95

The interest
¡ credited
¢ at the end¡of Day 0: ¢
−M V (0) erh − 1 = −2051. 56 e0.08×1/365 − 1 = −0.449 7 = −0.45
Investment at the beginning of Day 0:
M V (0) = 2051. 56

Please note the textbook shows the interest credited at the end of Day 0,
capital gain earned at the end of Day 0, and daily profit at the end of Day 0 in
Day 1 column.

Day 1
M V (1) = ∆1 S1 − C1 = 61.420 × 40.5 − 306.21 = 2181. 3
M V BR (2) = ∆1 S2 − C2 = 61.420 × 39.25 − 232.82 = 2177. 915
The trader’s profit at the end of Day 1 is:
M V BR (2) − M V (1) erh = 2177. 915 − 2181. 3e0.08×1/365 = −3. 863
140 CHAPTER 13. MARKET-MAKING AND DELTA-HEDGING

To find the capital gain and the interest earned at the end of Day 0, we just
need to break down the profit M V BR (1) − M V (0) erh into two parts:
¡ ¢
M V BR (2) − M V (1) erh = M V BR (2) − M V (1) + −M V (1) erh − 1
| {z } | {z }
capital gain interest earned

Capital gain at the end of Day 1: M V BR (2) − M V (1) = 2177. 915 − 2181.
3 = −3. 385
¡ ¢ ¡ ¢
Interest earned at the end of Day 1: −M V (1) erh − 1 = −2181. 3 e0.08×1/365 − 1 =
−0.478
Investment at the beginning of Day 1:
M V (1) = ∆1 S1 − C1 = 61.420 × 40.5 − 306.21 = 2181. 3

You should be able to reproduce Table 13.2 for the other days.
Day 3 4 5
Time t t = 3/365 t = 4/365 t = 5/365
Expiry T T3 = 88/365 T4 = 87/365 T5 = 86/365
St S3 = 38.75 S4 = 40 S5 = 40
Ct C3 = 205. 46 C4 = 271.04 C5 = 269.27
∆t ∆3 = 49. 564 ∆4 = 58.06 ∆5 = 58.01
Investment (beginning of the day) 1715. 15 2051. 36
interest earned during the day −0.38 −0.45
Capital gain (end of the day) −3. 63 1. 77
Profit (end of the day) −4. 01 1. 32

13.4 Textbook Table 13.3


The stock price of Table 13.3 follows the binomial tree with σ = 0.3.
On Day 0, the stock price is S0 = 40
On Day 1 the√stock moves up 1 σ√
S1 = S0 erh+σ h = 40e0.08/365+0.3 1/365 = 40.642
Day 2 the stock√
moves down 1 σ √
S2 = S1 e rh−σ h
= 40.642e0.08/365−0.3 1/365 = 40. 018
Day 3 the stock√
moves down 1 σ √
S3 = S2 erh−σ h = 40. 018e0.08/365−0.3 1/365 = 39. 403
Day 4 the stock√
moves down 1 σ √
S4 = S3 e rh−σ h
= 39. 403e0.08/365−0.3 1/365 = 38. 797
Day 5 the stock√
moves up 1 σ √
S5 = S4 erh+σ h = 38. 797e0.08/365+0.3 1/365 = 39. 420

If you use the same method for reproducing Table 13.2, you should be able
to reproduce Table 13.3. When reading Table 13.3, remember the interest, the
capital gain, and the daily profit on Day 1 is the interest, the capital gain, and
the daily profit at the end of Day 0 (or the beginning of Day 1). Similarly, the
13.5. MATHEMATICS OF DELTA HEDGING 141

interest, the capital gain, and the daily profit on any other day is the interest,
the capital gain, and the daily profit at the end of the previous day or the
beginning of that day.
The author of the textbook uses Table 13.3 to show us that if the stock price
moves up or down 1 σ daily, then the trader’s profit is zero.

13.5 Mathematics of Delta hedging


13.5.1 Delta-Gamma-Theta approximation
First, let’s understand the textbook Equation 13.6:

1 2
V (St+h , T − t − h) ≈ V (S0 , T − t) + ∆t + θh + Γt (Textbook 13.6)
2

Let’s consider an option written at time t expiring on date T (i.e. expiring


in T − t years). The value of this option is V (St , T − t), where St is the stock
price at t. Suppose a tiny time interval h (such as 0.00001 second) has passed
and we are now standing at t + h. Now the option has a remaining life T − t − h
years and is worth V (St+h , T − t − h), where St+h is the stock price at t + h.
Suppose St+h = St + .
Time t t+h T
Stock price St St+h = St +
Option value V (St , T − t) V (St+h , T − t − h)

We want to estimate V (St+h , T − t − h). By Taylor series, we have:


∂f (x0 , y0 ) ∂f (x0 , y0 )
f (x0 + x , y0 + y ) ≈ f (x0 , y0 ) + x+ y
∂x ∂y
2 2
1 ∂ f (x0 , y0 ) 2 1 ∂ f (x0 , y0 ) 2
+ x+ y
2 ∂x2 2 ∂y 2
Similarly,
V (St+h , T − t − h) = V [St+h , − (t + h) + T ]
∂V (St , T − t) ∂V (St , T − t)
≈ V (St , T − t) + + h
∂S ∂t
2 2

1 V (St , T − t) 2 1 ∂ V (St , T − t) 2
+ + h
2 ∂S 2 2 ∂t2
However,
∂V (St , T − t) ∂V (St , T − t) ∂ 2 V (St , T − t)
= ∆t =θ = Γt
∂S ∂t ∂S 2

1 ∂ 2 V (St , T − t) 2
We decide to ignore h since it’s close to zero. However,
2 ∂t2
1 ∂ 2 V (St , T − t) 2 1 ∂ 2 V (St , T − t) 2
is not close to zero. The reason that h
2 ∂S 2 2 ∂t2
142 CHAPTER 13. MARKET-MAKING AND DELTA-HEDGING

1 ∂ 2 V (St , T − t) 2
is close to zero but is not close to zero will be explained
2 ∂S 2
in Derivatives Markets Chapter 20 when we derive Ito’s Lemma. For now just
accept it.

Now we have:
1 2
V (St+h , T − t − h) ≈ V (St , T − t) + ∆t + θh + Γt
2

13.5.2 Understanding the market maker’s profit

Suppose a trader sets up a hedging portfolio at time t. The trader’s profit after
a short interval h (i.e. at time t + h) is:

P rof it (t + h)
= M V BR (t + h) − M V (t) erh
= (∆t St+h − Ct+h ) − (∆t St − Ct ) e¡rh ¢
= (∆t St+h − Ct+h ) − (∆t St − Ct ) erh − 1 + (∆ t St − ¢
¡ rh Ct )
= ∆t (St+h − St ) − (Ct+h − Ct ) − (∆t St − Ct ) e − 1
For a small h, using Taylor series, we get erh ≈ 1 + rh
P rof it (t + h) = ∆t (St+h − St ) − (Ct+h − Ct ) − rh (∆t St − Ct )

According to the textbook Equation 13.6,


1
Ct+h − Ct = ∆t + θh + Γt 2
2 µ ¶
1
P rof it (t + h) = ∆t (St+h − St ) − ∆t + θh + Γt 2 − rh (∆t St − Ct )
2
Since St+h = St + , we have:
P rof it (tµ+ h) ¶
1 2
= ∆t − ∆t + θh + Γt − rh (∆t St − Ct )
2
µ ¶
1 2
= − θh + Γt − rh (∆t St − Ct )
2
µ ¶
1 2
P rof it (t + h) ≈ − Γt + θh − rh [∆t St − Ct ] (Textbook 13.7)
2

However, St+h = St erh+σ h

√ ³ √ ´ 1³ √ ´2
Using the Taylor series, we have: erh+σ h
= 1+ rh + σ h + rh + σ h +
2
...
1³ √ ´2
As h → 0, rh + σ h and higher order terms all approach zero.
2 √
√ In addition, as h approaches 0, h is much larger than h. For example,
0.0001 = 0.01 is much larger than 0.0001.Hence, we can discard rh but keep

σ h.
13.5. MATHEMATICS OF DELTA HEDGING 143
µ ³ ¶
√ √ ´ 1³ √ ´2 ³ √ ´
→ St+h = St erh+σ h
= St 1 + rh + σ h + rh + σ h + ≈ St 1 + σ h
√ 2
→ = St+h − St ≈ St σ h
→ 2 ≈ St2 σ2 h
Plug the above equation in Textbook Equation 13.7, we get:

µ ¶
1 2 2
P rof it (t + h) ≈ − St σ hΓt + θh − rh [∆t St − Ct ] (Textbook 13.9)
2

From the textbook Table 13.3, we know that if the stock price moves up or
down µ
by 1 σ, the trader’s profit is zero. ¶So
1 2 2
− S σ hΓt + θh − rh [∆t St − Ct ] = 0
2 t
This gives us the Black-Scholes PDE:
1 2 2
S σ Γt + θ − r [∆t St − Ct ] = 0 (Textbook 13.10)
2 t

Next, the textbook has the following topics:

• Delta-hedging of American options


• The advantage to frequent rehedging
• Delta-hedging in practice
• Market making as insurance

These topics are minor ideas. I recommend that you skip them.
144 CHAPTER 13. MARKET-MAKING AND DELTA-HEDGING
Chapter 14

Exotic options: I

Any option that is not a plain vanilla call or put is called an exotic option.
There are usually no markets in these options and they are purely bought OTC
(over-the-counter). They are much less liquid than standard options. They often
have discontinuous payoffs and can have huge deltas near expiration which make
them difficult to hedge.
Before studying this chapter, make sure you understand the learning objec-
tive.
SOA’s learning outcome for this chapter:

• Explain the cash flow characteristics of the following exotic options: Asian,
barrier, compound, gap and exchange

If you want to cut corners, you can skip the pricing formula for exotic options
because calculating the exotic option price is out of the scope of the learning
outcome or learning objective.

14.1 Asian option (i.e. average options)


14.1.1 Characteristics
Asian options (also called average options) have payoffs that are based on the
average price of the underlying asset over the life of the option. The average
price can be the average stock price or the average strike price. The average
can be arithmetic or geometric.

• The unique characteristic of an average price option is that the underlying


asset prices are averaged over some predefined time interval.

• Averaging dampens the volatility and therefore average price options are
less expensive than standard options

145
146 CHAPTER 14. EXOTIC OPTIONS: I

• Average price options are path-dependent, meaning that the value of the
option at expiration depends on the path by which the stock arrives at its
final price. The price path followed by the underlying asset is crucial to
the pricing of the option.

• Average price options are useful in situations where the trader/hedger


is concerned only about the average price of a commodity which they
regularly purchase.

14.1.2 Examples

1
Examples are based on

• A 9-month European average price contract calls for a payoff equal to the
difference between the average price of a barrel of crude oil and a fixed
exercise price of USD18. The averaging period is the last two months of
the contract. The impact of this contract relative to a standard option
contract is that the volatility is dampened by the averaging of the crude oil
price, and therefore the option price is lower. The holder gains protection
from potential price manipulation or sudden price spikes.

• A Canadian exporting firm doing business in the U.S. is exposed to Can$/US$


foreign exchange risk every week. For budgeting purposes the treasurer
must pick some average exchange rate in which to quote Can$ cash flows
(derived from US$ revenue) for the current quarter. Suppose the treasurer
chooses an average FX rate of Can$1.29/US$1.00. If the US$ strength-
ens, the cash flows will be greater than estimated, but if it weakens, the
company’s Can$ cash flows are decreased.

Arithmetic average
We record the stock price every h periods from time 0 to time T . There are

N = T /h periods. The arithmetic average is:


n
1 X
A (T ) = Sih
N i=1

14.1.3 Geometric average

G (T ) = (Sh S2h ...SN h )1/N


1 Based on

http://www.fintools.com/doc/exotics/exoticsAbout_Average_Options.html
14.2. BARRIER OPTION 147

14.1.4 Payoff at maturity T


Let AV G (T ) represent the average stock price. For arithmetic average, AV G (T ) =
n
1 X
A (T ) = Sih . For geometric average, AV G (T ) = (Sh S2h ...SN h )1/N .
N i=1
Option Payoff
average price call max [0, AV G (T ) − K]
average price put max [0, K − AV G (T )]
average strike call max [0, ST − AV G (T )]
average strike put max [0, AV G (T ) − ST ]

14.2 Barrier option

Barrier options are similar to standard options except that they are extinguished
or activated when the underlying asset price reaches a predetermined barrier or
boundary price. As with average options, a monitoring frequency is defined as
part of the option which specifies how often the price is checked for breach of
the barrier. The frequency is normally continuous but could be hourly, daily,
etc.

14.2.1 Knock-in option


• "In" option starts its life worthless and becomes active only if the barrier
price is reached
• Technically, this type of contract is not an option until the barrier price
is reached. So if the barrier price is never reached it is as if the contract
never existed.
• down-and-in (spot price starts above the barrier level and has to move
down for the option to become activated.)
• up-and-in (spot price starts below the barrier level and has to move up
for the option to become activated.)

14.2.2 Knock-out option


• "Out" option starts its life active and becomes null and void if the barrier
price is reached
• The option will expire worthless if the asset price exceeds the barrier price
• down-and-out (spot price starts above the barrier level and has to move
down for the option to become null and void)
• up-and-out (spot price starts below the barrier level and has to move up
for the option to be knocked out)
148 CHAPTER 14. EXOTIC OPTIONS: I

14.2.3 Rebate option


• Barrier options are sometimes accompanied by a rebate, which is a payoff
to the option holder in case of a barrier event.

• Rebates can either be paid at the time of the event or at expiration.

14.2.4 Barrier parity


”Knock − in” option + ”Knock − out” option = Ordinary Option (14.1)

Example 14.2.1.

Explain why
up-and-in option + up-and-out option = Ordinary option.

T =option expiration time


tH =time when the stock price first reaches the barrier, where the barrier
price is greater than the current stock price
During the time interval [0, tH )

• the up-and-out option is alive

• the up-and-in option is dead

During the time interval [tH , T ]

• the up-and-out option is dead (because the barrier is reached)

• the up-and-in option is alive (because the barrier is reached)

If we have a down-and-in option and a down-and-out option with two options


on the same stock, having the same barrier price and the same strike price, then
at any moment in the interval [0, T ], we’ll always have exactly one ordinary
option alive. Hence down-and-in option + down-and-out option = Ordinary
option.
Similarly, down-and-in option + down-and-out option = Ordinary option.

14.2.5 Examples
1. A European call option is written on an underlying with spot price $100,
and a knockout barrier of $120. This option behaves in every way like
a vanilla European call, except if the spot price ever moves above $120,
the option "knocks out" and the contract is null and void. Note that the
option does not reactivate if the spot price falls below $120 again. Once
it is out, it’s out for good.
14.3. COMPOUND OPTION 149

2. A bank purchases an at-the-money 9-month Nikkei call option struck at


17,000 with a down-and-out barrier price of 16,000. If the price of the
Nikkei falls to 16,000 or below, during the 9-month period, the bank will
no longer have the benefit of Nikkei price appreciation since the call option
will have been knocked out.
3. An airline is concerned that events in the Middle East might drive up
the price of fuel. An up-and-in call would allow the airline to buy crude
oil futures at a fixed price if some knock-in boundary price is reached.
The price of the U&I call would be less than a standard call with the
same expiration and exercise price so it might be viewed as a cost effective
hedging instrument.

14.3 Compound option


Definition
• With a compound option one has the right to buy an ordinary option at
a later date
• Compound options are more expensive to purchase than the underlying
option, as the purchaser has received a price guarantee and effectively
extended the life of the option

Make sure you understand the textbook Figure 14.2.


For a compound call (i.e. call on call) to be valuable, the following two
conditions need to be met:

• St1 > S ∗
• St1 > K

Compound option parity

The key formula is DM 14.12:

CallOnCall − P utOnCall + xe−rt1 = BSCall (DM 14.12)

This formula looks scary but is actually easy to remember. We know the
put-call parity C + Ke−rT = P + S0 . If we treat the standard BSCall as the
underlying asset, then applying the standard put-call parity, we get:
CallOnCall + xe−rt1 = P utOnCall + BSCall

This is DM 14.12.
Similarly, we have:
CallOnP ut + xe−rt1 = P utOnP ut + BSP ut
150 CHAPTER 14. EXOTIC OPTIONS: I

Options on dividend-paying stocks

This section is a minor part of Exam MFE. I recommend that you skip it.
However, if you don’t want to skip, here is the main idea.
At t1 , the value of an American call option CA (St1 , T − t1 ) is (see the text-
book’s explanation for this formula):

CA (St1 , T − t1 ) = max [CE (St1 , T − t1 ) , St1 + D − K] (DM 14.13)

Here CE (St1 , T − t1 ) is a European call option. Under the put-call parity,


we get:
CE (St1 , T − t1 ) = PE (St1 , T − t1 ) + St1 − Ke−r(T −t)
So £ ¤
CA (St1 , T − t1 ) = max PE (St1 , T − t1 ) + St1 − Ke−r(T −t) , St1 + D − K
£ ¤
= St1 +D−K+max PE (St1 , T − t1 ) + St1 − Ke−r(T −t) − (St1 + D − K) , 0
£ ¡ ¡ ¢¢ ¤
= St1 + D − K + max PE (St1 , T − t1 ) − D − K 1 − e−r(T −t) , 0
h ³ ³ ´´ i
CA (St1 , T − t1 ) = St1 +D−K+max PE (St1 , T − t1 ) − D − K 1 − e−r(T −t) , 0
£ ¡ ¡ ¢¢ ¤ (DM 14.14)
max PE (St1 , T − t1 ) − D − K 1 − e−r(T −t) , 0 is payoff of a call on put
¡ ¢
with strike price equal to D − K 1 − e−r(T −t) .

Next, the author made two points:


Exercising call on put at t1 is the same as not exercising the American call
at
¡ t1 . If you
¡ exercise the
¢¢ call on put, then it must be true that PE (St1 , T − t1 )−
D − K 1 − e−r(T −t) ≥ 0 (otherwise you won’t exercise it). Then DM 14.14
becomes ¡ ¡ ¢¢
CA (St1 , T − t1 ) = St1 + D − K + PE (St1 , T − t1 ) − D − K 1 − e−r(T −t)
= St1 + PE (St1 , T − t1 ) − Ke−r(T −t) = CE (St1 , T − t1 )
So exercising the call on put at t1 means that not early exercising the Amer-
ican call option at t1 (so the American call option becomes a European call
option).

The second point. If you don’t¡ exercise


¡ the call on
¢¢ put at t1 , then it must
be true that PE (St1 , T − t1 ) − D − K 1 − e−r(T −t) < 0.
This gives us CA (St1 , T − t1 ) = St1 + D − K. This equation means that we
receive dividend D at t1 and immediately exercise the call at t1 (so the American
call option at t1 is equal to St1 + D − K). So not exercising the call on put at
t1 is the same as early exercising the American call option at t1 .
This is all you need to know about this section.

DM Example 14.2 shows you how to use DM 14.14 to calculate the price of
an American call option with a single dividend. When reading this example,
please note two things:
14.4. GAP OPTION 151

(1) DM Example 14.2 has errors. Make sure you download the errata. The
DM textbook errata can be found at the SOA website.
(2) The call on put price is calculated using the Excel spreadsheet (so don’t
worry about how to manually calculate the call on put).

14.4 Gap option


14.4.1 Definition
• An option in which one strike price K1 determines the size of the payoff
and another strike price K2 determines whether or not the payoff is made.

• Example. A call option pays ST − 5 if ST > 8 and pays zero otherwise.


The strike price K1 = 5 determines the size of the payoff but a different
strike price K2 = 8 determines whether or not the payoff is made. K2 is
called the payment trigger.
• Example. A put option pays 8 − ST if ST > 5 and pays zero otherwise.
The strike price K1 = 8 determines the size of the payoff but a different
strike price K2 = 8 determines whether or not the payoff is made.
• As the footnote in Page 457 of the textbook indicates, a gap call or put
option is really not a option; once the payment trigger is satisfied, the
owner of a gap option MUST exercise the option. Hence the premium of
a gap option can be negative.

14.4.2 Pricing formula


To find the price of a gap option, you can modify a standard option’s formula
by changing d1 :
C (K1 , K2 ) = Se−δT N (d1 ) − K1 e−rT N (d2 )
−rT −δT
P (K1 , K2 ) = K
µ1 e N (−d¶
2 ) − Se N (−d1 )
S 1 2
ln + r−δ+ σ T
K2 2
d1 = √
√ σ T
d2 = d1 − σ T

14.4.3 How to memorize the pricing formula


• The call payoff is 0 if ST ≤ K2 and ST − K1 if ST > K2 . The call option
price is just the risk-neutral expected discounted value of the payoff. So
the call price is Se−δT N (d1 ) − K1 e−rT N (d2 ). Similarly, the put price is
K1 e−rT N (−d2 ) − Se−δT N (−d1 ).

You are given the following information on a gap option on a stock

• The gap call expires in 6 months


152 CHAPTER 14. EXOTIC OPTIONS: I

• The stock is currently selling for $40

• The payment trigger is $50

• The strike price is $60

• The continuously compounded risk-free interest rate is 6% per year

• The continuously compounded dividend yield is 2% per year

• The stock volatility σ = 30%

Calculate

• the price of the gap call option.

• the price of the gap put option

S = 40 K1 = 60 K2 = 50 r = 0.06
δ = 0.02 T = 0.5
µ ¶ µ ¶
S 1 40 1
ln + r − δ + σ2 T ln + 0.06 − 0.02 + × 0.32 0.5
K2 2 50 2
d1 = √ = √ =
σ T 0.3 0.5
−0.851 6 √ √
d2 = d1 − σ T = −0.851 6 − 0.3 0.5 = −1. 063 7
N (d1 ) = 0.197 2 N (d2 ) = 0.143 7
N (−d2 ) = 0.856 3 N (−d1 ) = 0.802 8
C = Se−δT N (d1 ) − K1 e−rT N (d2 ) = 40e−0.02×0.5 0.197 2 − 60e−0.06×0.5
0.143 7 = −0.56

P = K1 e−rT N (−d2 ) − Se−δT N (−d1 ) = 60e−0.06×0.5 0.856 3 − 40e−0.02×0.5


0.802 8 = 18. 07

14.5 Exchange option


• Allows the holder of the option to exchange one asset for another

• Pricing
p formula. Use the standard Black-Scholes formula except changing
σ to σ 2S + σ 2K − 2ρσ S σ K . Also, make sure you know which is the stock
asset and which is the strike asset. If you give up Asset 2 and receive
Asset 1, Asset 1 is the stock and Asset 2 is the strike asset.

Let’s walk through DM Example 14.3.


S = 95.92 K = 3.5020 × 27.39 = 95. 92 = S (i.e. at-the-money call)
δ S = 0.75% δ K = 1.17% σ = 0.1694 T =1
14.5. EXCHANGE OPTION 153

95.92e−0.0075×1 1
ln −0.0117×1
+ × 0.16942 × 1
d1 = 95.92e √2 = 0.109 5
√ 0.1694 1 √
d2 = d1 − σ T = 0.109 5 − 0.1694 1 = −0.059 9
N (d1 ) = 0.543 6 N (d2 ) = 0.476 1
C = Se−δS T N (d1 ) − K1 e−δK T N (d2 )
= 95.92e−0.0075×1 × 0.543 6 − 95.92e−0.0117×1 × 0.476 1 = 6. 616

This is slightly different from the textbook call price 6.6133 due to rounding.
You can verify the call price using the Excel worksheet "Exchange."

Inputs:
Underlying Asset
Price 95.92
Volatility 20.300%
Dividend Yield 0.750%

Strike Asset
Price 95.92
Volatility 22.270%
Dividend Yield 1.170%

Other
Correlation 0.6869
Time to Expiration (years) 1

Output:
Exchange Option
Black-Scholes
Call Put
Price 6.6144 6.2154

So the exchange call price is 6.6144; the exchange put price is 6.2154.
154 CHAPTER 14. EXOTIC OPTIONS: I
Chapter 20

Brownian motion and Ito’s


Lemma

20.1 Introduction
According to Wikipedia, Brownian motion (named in honor of the botanist
Robert Brown) is either the random movement of particles suspended in a fluid
or the mathematical model used to describe such random movements, often
called a Wiener process.
In 1827, while examining pollen grains suspended in water under a micro-
scope, Brown observed minute particles in the pollen grains executing a contin-
uous jittery motion. He observed the same motion in particles of dust, enabling
him to rule out the hypothesis that the motion was due to pollen being alive.
Although he did not provide a theory to explain the motion, the phenomenon
is now known as Brownian motion in his honor.
Brownian motion is a useful tool for modeling the stock price. The price
of a stock is constantly hit by random events, just as a particle in the water is
constantly hit by water molecules. In fact, the Brownian motion is to stochastic
processes as the standard normal distribution to random variables.
Brownian motion is an abstract concept. The first step toward learning the
Brownian motion is to see it and experiment it. You can easily find Brownian
motion simulations in the internet. Here are some simulations in the internet:

• http://www.phy.ntnu.edu.tw/java/gas2d/gas2d.html
• http://www.aip.org/history/einstein/brownian.htm
• http://www.stat.umn.edu/~charlie/Stoch/brown.html
• http://www.matter.org.uk/Schools/Content/BrownianMotion

There’s a large body of knowledge on the internet about the Brownian mo-
tion. For example, you can check out Wikipedia’s explanation at:

155
156 CHAPTER 20. BROWNIAN MOTION AND ITO’S LEMMA

http://en.wikipedia.org/wiki/Brownian_motion

To help you experiment with the Brownian motion, I designed a spreadsheet


titled "Simulate Brownian Motion." Download this spreadsheet.
My spreadsheet uses the following Excel functions:

• Rand() returns a random number equal to or greater than 0 but less than 1.
In other words, Rand() simulates a random variable uniformly distributed
over [0, 1).

• NORMINV(probability,mean,standard_dev) returns the inverse of the nor-


mal cumulative distribution for the specified mean and standard deviation.

20.1.1 Big picture


When studying the Brownian motion and Ito’s lemma, remember the following
big picture. The Black-Scholes option pricing formula Equation 12.1 relies on
the Black-Scholes PDE (Equation 12.24). Equation 12.24 assumes the following
price model for a risk-free asset and a risky asset (i.e. stock):
The price of a risk free asset (i.e. the savings account or a bond) is

B (t) = ert (20.1)


The price of a risky asset (i.e. stock) at time t is:
# $
1 2 √
α− σ t+σ tY (t)
S (t) = S (0) e 2 (20.2)
In the above equation:

• S (0) is the stock price at time zero

• S (t) is the stock price at time t

• α is the expected (annualized continuously compounded) ∙ return


¸ of the
1 S (t)
stock. In other words, E [S (t)] = S (0) eαt or α = ln E
t S (0)
• r is the (annualized continuously compounded) risk-free interest rate.

• σ is the stock’s volatility.

• Y (t) is a random draw of a standard normal random variable. Y (t1 ) and


Y (t2 ) are independent for t1 6= t2

Equation 20.2 is another form of Equation 20.1 in the textbook:

dS (t)
= αdt + σdZ (t) (Textbook 20.1)
S (t)
20.2. BROWNIAN MOTION 157

20.2 Brownian motion

20.2.1 Stochastic process


A stochastic process is a family of random variables indexed by time. For
example, the temperature out side your house X (t) is a stochastic process. Let
t = 0 represent now and t = 1 represent the next time (such as next hour, next
day, next week). Then for a series of time points t = 1, 2, 3, ... there is a family
of random temperatures X (0), X (1), X (2), X (3) ....
This is the major difference between a stochastic process and a deterministic
process. In a stochastic process you’ll see a series of random variables; for
each time t there’s a corresponding random variable X (t). In contrast, in a
deterministic process, you’ll see only one random variable.

20.2.2 Definition of Brownian motion

Consider a particle that jumps, at discrete times, up or down along the vertical
line. At t = 0 the particle is at position zero. After each h-long time period,
the particle jumps up or down by a constant distance of k and with equal
probability of 0.5. That is, at t = h, 2h, 3h, ..., nh, the particle either moves
up by k or moves down by k, with up and down movements having an equal
probability of 0.5. Let Z (t) represent the height of the article from the position
zero at time t. Clearly Z (0) = 0. We like to find Z (T ), the height of the article
at time T = nh.

The particle’s height

4k
3k
2k 2k
k k
0 0 0
−k −k
−2k −k
−3k
−4k
time 0 h 2h 3h 4h

Let’s walk through the above table. At t = 0 the particle is at the position
zero. At t = h, the particle’s height is either k or −k. At t = 2h, the k node
either goes up to 2k or goes down to 0. Similarly, the −k node either goes up
to 0 or goes down to −2k. So on and so forth.
The jump at t = h is: Z (h) − Z (0) = Y (h) k
158 CHAPTER 20. BROWNIAN MOTION AND ITO’S LEMMA

Here Y (h) is a direction indicator. If Y (h) = 1, then the particle moves up


by k; if Y (h)
½ = −1, then the particle moves down by k:
1 Probability 0.5
Y (h) =
−1 Probability 0.5

The jump½at t = 2h is: Z (2h) − Z (h) = Y (2h) k


1 Probability 0.5
Y (2h) =
−1 Probability 0.5

The jump at t = nh = T is: Z (nh) − Z [(n − 1) h] = Y (nh) k


Here Y (nh)
½ is a direction indicator:
1 Probability 0.5
Y (nh) =
−1 Probability 0.5

The direction indicators Y (h), Y (2h), Y (3h), ..., Y (nh) are independent
identically distributed binomial random variables. For i = 1 to n,
E (ih)
h =i(1) 0.5 + (−1) 0.5 = 0
2 2 2
E (ih) = (1) 0.5 + (−1) 0.5 = 1
h i
V ar (ih) = E (ih)2 − E 2 (ih) = 1
The particle’s height at time T is:
Z (T ) = [ Z (h) − Z (0)]+[ Z (2h) − Z (h)]+[ Z (3h) − Z (2h)]+...+[ Z (nh = T ) − Z [(n − 1) h]]
= Y (h) k + Y (2h) k + Y (3h) k + ... + Y (nh) k
= [Y (h) + Y (2h) + Y (3h) + ... + Y (nh)] k

According to the central limit theorem, Z (T ) is approximately normal with


mean and variance as:

E [Z (T )] = kE [Y (h) + Y (2h) + Y (3h) + ... + Y (nh)] = 0


V ar [Z (T )] = k 2 V ar [Y (h) + Y (2h) + Y (3h) + ... + Y (nh)]
T k2
= k2 n = k2 = T
h h
2
k
We want V ar [Z (T )] = T to exist (i.e. not to become infinite) as n → ∞.
h
2
k k2
To achieve this, we set to a positive constant: = c. To make our
h h
model simple, we set c = 1. Hence
k2 √
=1 k= h
h
Now we have:

Z (ih) − Z [(i − 1) h] = Y (ih) h (20.3)

k2
Now Z (T ) is approximately normal with mean zero and variance T = T:
h
Z (T ) ∼ N (0, T )
20.2. BROWNIAN MOTION 159

Please note that another way to specify the model is treat Y (ih) as a random
draw of a standard normal random variable (instead of a binomial random
variable):


Z (ih) − Z [(i − 1) h] = Y (ih) h and Y (ih) ∼ N (0, 1) (20.4)

My spreadsheet for simulating the Brownian motion uses both Equation 20.3
and Equation 20.4.

The random process Z (t) as n → ∞ is called the Brownian motion or the


Wiener process.
Next, let’s formally define the Brownian motion.

Definition 20.2.1.
A stochastic process Z (t) is a Brownian motion or a Wiener process if

1. Z (0) = 0 . Brownian motion starts at zero (this is merely for our conve-
nience).
2. Z (t + h) − Z (t) is normally distributed with mean 0 and variance h. This
means that the increments over a time interval h is normally distributed
with mean 0 and variance h. This stands true no matter how small or big
h is.
3. Z (t + s1 ) − Z (t) is independent of Z (t) − Z (t − s2 ) where s1 , s2 > 0.
4. Z (t) is continuous.

20.2.3 Martingale
The following part is based on Wikipedia.
A martingale is a stochastic process (i.e., a sequence of random variables)
such that the conditional expected value of an observation at some time t, given
all the observations up to some earlier time s, is equal to the observation at that
earlier time s.
Originally, martingale referred to a betting strategy popular in 18th century
France. The rule of the game is that the gambler wins his stake if a coin comes
up heads and loses it if the coin comes up tails. The martingale strategy had
the gambler double his bet after every loss, so that the first win would recover
all previous losses plus win a profit equal to the original stake. Since eventually
a gambler will win at least once, the martingale betting strategy was thought
to be sure way of winning. In reality, however, the exponential growth of the
bets would eventually bankrupt the gambler.
A stochastic process (i.e., a sequence of random variables) X (t) is a martin-
gale if the following holds:
160 CHAPTER 20. BROWNIAN MOTION AND ITO’S LEMMA

E [X (t) |X (s)] = X (s) for t > s.

It can be shown that the Brownian motion Z (t) is martingale.


For t > s, we have:
E [Z (t) |Z (s)]
= E [Z (t) − Z (s) + Z (s) |Z (s)]
= E [Z (t) − Z (s) |Z (s)] + E [Z (s) |Z (s)]

E [Z (s) |Z (s)] = Z (s)


Z (t) − Z (s) is independent of Z (s).
→ E [Z (t) − Z (s) |Z (s)] = E [Z (t) − Z (s)] = 0
→ E [Z (t) |Z (s)] = E [Z (t) − Z (s) |Z (s)] + E [Z (s) |Z (s)] = Z (s)
Hence Z (t) is martingale.
E [Z (t) |Z (s)] = Z (s) means that the best estimate of the future value of a
Brownian motion is its current value.

20.2.4 Properties of Brownian motion


1. The Brownian motion is continuous everywhere yet differentiable nowhere.

2. The first-order variation is infinite: lim |Z (h)−Z (0) |+|Z (2h)−Z (h) |+
n→∞
... + |Z (nh) − Z [(n − 1) h] | → ∞

3. The second-order variation (called quadratic variation) is equal to the


2 2
length of the time interval: lim {[Z (h) − Z (0)] + [Z (2h) − Z (h)] +
n→∞
... + (Z (nh) − Z [(n − 1) h])2 } = T

4. Cov [Z (s) , Z (t)] = min (s, t). The covariance of two Brownian motions is
the shorter time interval.
½
n 0 if n is odd
5. The higher moments of Z (t) is: E [Z (t)] =
tn/2 (n − 1) (n − 3) ...1 if n is even

Let’s look at the first property.


If you look at the simulation of Brownian motion over the internet, you’ll
find that the Brownian motion is always continuous yet it’s not differentiable
anywhere. Can you imagine that a function is continuously anywhere yet differ-
entiable nowhere? If I hadn’t studied the Brownian motion, I would have never
thought that such a function exists.
We can explain the non-differentiality using the following equation (it’s text-
book Equation 20.4):

dZ (t) = Y (t) dt (20.5)

The above equation is another form of Equation 20.3 or 20.4:


20.2. BROWNIAN MOTION 161


dZ (t) = Z (t + dt) − Z (t) = Y (t) dt
The textbook explains in the footnote that you can treat Y (t) as a binomial

random variable or a standard normal random variable. Either way, dZ (t) is a


normal random variable.
From Equation 20.5, we have:
dZ (t) Y (t)
= √ → ∞ as dt → 0
dt dt
Let’s look at the 2nd property. The following is an intuitive but not rigorous
proof.

According to the definition of the Brownian motion, Z (h) − Z (0), Z (2h) −


Z (h), ..., Z (nh) − Z [(n − 1) h] are independent identically distributed normal
random variable with mean 0 and variance h. On average, |Z [(i + 1) h]−Z (ih) |
approaches E|X| where X ∼ N (0, h). Let f (x) represent the probability den-
sity functionR of X, then

E|X| = −∞ |x|f (x) dx > 0 (since |x| ≥ 0, it’s mean must be positive)
Then lim (|Z (h) − Z (0) | + |Z (2h) − Z (h) | + ... + |Z (nh) − Z [(n − 1) h] |)
n→∞

approaches nE|X|, which approaches ∞ (since n → ∞, a positive constant E|X|


times n must also approach infinity.) √
Property 2 should be easy to understand. Since ∆Z (t)√= Y (t) ∆t, for a
√ 0.000001
tiny interval ∆t, ∆t is much large than ∆t. For example, = 1, 000.
0.000001
Hence during any short interval, the Brownian motion can move up or down by
an infinitely large amount.
In contrast, for a continuously differentiable function y = f (t), we have:
∆y = f 0 (t) ∆t → 0 as ∆t → 0

Let’s get an intuitive


¡ feel¢ of the 3rd property. On average, {Z [(i + 1) h] −
2 2
Z (ih)}¡ ¢approaches E X .
E X 2 = E 2 (X) + V ar (X) = 02 + h = h
2 2 2
lim [Z (h) − Z (0)] + [Z (2h) − Z (h)] + ... + (Z (nh) − Z [(n − 1) h]) ap-
n→∞
proaches
h + h + ... + h = nh = T
In contrast, the 2nd order variation of a differentiable function is zero. For
example, we can find the 2nd order variation of the function y = x is zero.
Divide the interval [0, T ] into [0, h],[h, 2h],...,[(n − 1) h, nh = T ]
µ ¶2
2 2 T
lim {(h − 0) + (2h − h) +... +[nh − (n − 1) h]}2 = nh2 = lim n =
n→∞ n→∞ n
0
It can be proven that the quadratic variation of any continuously differen-
tiable function is zero. The quadratic variation of any continuously differentiable
function is zero because such a function is roughly linear at any point. For a
continuously differentiable function y = f (t), we have:
162 CHAPTER 20. BROWNIAN MOTION AND ITO’S LEMMA

2 2 2
∆y = f 0 (t) ∆t → (∆y) = [f 0 (t)] (∆t)
For a tiny interval ∆t, (∆t)2 → 0 must faster than ∆t → 0.√Hence (∆y)2 → 0
2
In contrast, for a Brownian motion we have ∆Z (t) = Y (t) ∆t and [∆Z (t)] =
P P
[Y (t)]2 ∆t → ∆t. Hence [∆Z (t)]2 → ∆t = T
Brownian motion Property 3 reconfirms the idea that Brownian motion is
not differentiable anywhere. If it’s differentiable, then its second order variation
would be zero.


Tip 20.2.1. Just memorize Equation 20.5 dZ (t) = Y (t) dt. This equation
tells you that the Brownian motion Z (t) is not differentiable anywhere, its sec-
ond order variation is t, and its first order variation is infinite.

Tip 20.2.2. To get an intuitive feel of the equation dZ (t) = Y (t) dt, imagine
you are looking at the Brownian motion under a magnifying class. If you zoom
in on the Brownian motion by shrinking the time interval dt, no matter how
much you reduce dt, you’ll see a jigsaw. In comparison, if you zoom in on a
continuously differentiable function such as y = t2 , you’ll see a straight line.

Please note that Derivatives Markets explains Property 2 and 3 using the
following formula: √
Z [(i + 1) h] − Z (ih) = Y [(i + 1) h] h
Since Y [(i + 1) h] is a√binomial random variable having a value of ±1, then
|Z [(i + 1) h] − Z (ih) | = h and {Z [(i + 1) h] − Z (ih)} = h. Hence √
lim (|Z (h) − Z (0) | + |Z (2h) − Z (h) | + ... + |Z (nh) − Z [(n − 1) h] |) = n h →
n→∞

2 2 2
lim {[Z (h) − Z (0)] + [Z (2h) − Z (h)] + ... + (Z (nh) − Z [(n − 1) h]) } =
n→∞
nh = T
The problem with this explanation is that it works if we treat Y [(i + 1) h]
as binomial random variable whose value is ±1. Such explanation won’t work if
we treat Y [(i + 1) h] as a random draw of a standard normal random variable.
The explanation I provided here works no matter if you treat Y [(i + 1) h] as a
binomial random variable or a standard normal random variable.
Let’s look at Property 4. Suppose s ≤ t
Cov [Z (s) , Z (t)] = Cov{Z (s) , Z (s) + [Z (t) − Z (s)]}
Using the formula Cov (a, b + c) = Cov (a, b) + Cov (a, c), we get:
Cov{Z (s) , Z (s)+[Z (t) − Z (s)]} = Cov{Z (s) , Z (s)}+Cov{Z (s) , [Z (t) − Z (s)]}
Cov{Z (s) , Z (s)} = V ar [Z (s)] = s
Since Z (s) and [Z (t) − Z (s)] are independent (Brownian motion definition
Point #3), Cov{Z (s) , [Z (t) − Z (s)]} = 0
→ Cov [Z (s) , Z (t)] = s = min (s, t)
Property 5 is based on the moment formula for a standard normal random
variable φ
½
0 if n is odd
E (φn ) = (20.6)
(n − 1) (n − 3) ...1 if n is even
20.2. BROWNIAN MOTION 163

We can find the n-th moment of a random variable X using the moment
generating function (MGF):
∙ n ¸
n d
E (X ) = MX (t) (20.7)
dtn t=0

The MGF of a normal random variable X with mean μ and the standard
deviation σ is:
1
¡ ¢ μt+ σ 2 t2
MX (t) = E etX = e 2 (20.8)
1 2
t
→ Mφ (t) = e 2
Using Equation 20.7, you can verify that Equation 20.6 holds.
Since Z (t) is a normal random variable with mean 0 and variance t, then
Z (t)
√ is a standard normal random variable. Hence
t
½
0 if n is odd
E [Z n (t)] = (20.9)
tn/2 (n − 1) (n − 3) ...1 if n is even
Example 20.2.1. Calculate P [Z (3) > 1]
Z (3) is a normal random variable with mean
µ 0 and¶variance 3.
1−0
P [Z (3) > 1] = 1 − P [Z (3) ≤ 1] = 1 − Φ √ = 1 − Φ (0.577 35) =
3
0.281 9
Example 20.2.2. Calculate P [Z (1) ≤ 0 ∩ Z (2) ≤ 0]
Z (1) is a normal random variable with mean 0 and variance 1. Let X = Z (1)

Z (2) = Z (1) + [Z (2) − Z (1)]


Z (2) − Z (1) is a normal random variable with mean 0 and variance 1.
Z (2) = Z (1) + [Z (2) − Z (1)] ≤ 0 → [Z (2) − Z (1)] ≤ −Z (1)
Let Y = Z (2) − Z (1).
X and Y are independent.
P [Z (1) ≤ 0 ∩ Z (2) ≤ 0] = P (X ≤ 0 ∩ Y ≤ −X)
To have X ≤ 0 ∩ Y ≤ −X, we first fix X at a tiny interval (x, x + dx)
where −∞ < x < 0. Next, we set Y < −x. Then we are guaranteed to have
X ≤ 0∩Y ≤ −X. Let f (x) and Φ (x) represent the probability density function
(pdf) and the cumulative density function (cdf) of a standard normal random
variable. R0 R0
P (X ≤ 0 ∩ Y ≤ −X) = −∞ P (x < X < x + dx) P (Y < −x) = −∞ [f (x) dx] P (Y < −x)
However, f (x) dx = dΦ (x) and P (Y < −x) = Φ (−x) = 1 − Φ (x)
R0 R0 R0
[f (x) dx] P (−x) = −∞ [dΦ (x)] [1 − Φ (x)] = −∞ [1 − Φ (x)] dΦ (x) =
R 0 −∞
−∞
[1 − Φ (x)] dΦ (x)
164 CHAPTER 20. BROWNIAN MOTION AND ITO’S LEMMA

∙ ¸0
1 2
= Φ (x) − Φ (x)
2 −∞
1£ 2 ¤
= [Φ (0) − Φ (−∞)] − Φ (0) − Φ2 (−∞)
∙ ¸ " µ ¶22 #
1 1 1 2
= −0 − −0
2 2 2
µ ¶2
1 1 1 3
= − =
2 2 2 8
Example 20.2.3. Calculate Cov [Z (5) , Z (2)]
Cov [Z (5) , Z (2)] = min (5, 2) = 2
£ ¤
Example 20.2.4. Calculate E Z 4 (t) .
£ ¤
E Z 4 (t) = t4/2 (4 − 1) = 3t2

20.2.5 Arithmetic Brownian motion and Geometric Brown-


ian motion
A standard Brownian motion is normally distributed with mean 0 and variance
1. Now we want to extend the Brownian motion to allow for non-zero mean and
an arbitrary variance. Define a stochastic
√ process:
X (t + h) − X (t) = αh + σY (t + h) h
Breaking down [0, T ] into small intervals [0, h],[h, 2h],...,[(n − 1) h, nh = T ],
we have: P h √ i P h √ i
X (T ) − X (0) = ni=1 αh + σY (ih) h = αT + σ ni=1 Y (ih) h
Pn h √ i
As n → ∞, i=1 Y (ih) h → Z (T )

X (T ) − X (0) = αT + σZ (T ) (20.10)

dX (t) = αdt + σdZ (t) (20.11)


Equation 20.10 and 20.11 are called arithmetic Brownian motion. α is the
instantaneous mean per unit of time; σ is the instantaneous standard deviation
per unit of time.
Equation 20.10 and 20.11 indicate that X (T )−X (0) is normally distributed.
Its mean and variance are:
E [X (T ) − X (0)] = E [αT + σZ (T )] = αT + σE [Z (T )] = αT + σ × 0 = αT
V ar [X (T ) − X (0)] = V ar [αT + σZ (T )] = V ar [σZ (T )] = σ2 V ar [Z (T )] =

σ2T

The textbook lists the major properties and weaknesses of Equation 20.11.
Major properties:
20.2. BROWNIAN MOTION 165

1. X (t) is normally distributed.

2. We can change the variance by changing the parameter σ.

3. We can change the mean by changing the parameter α. Now the mean is
no longer zero if α 6= 0. And we have E [X (T )] − E [X (0)] = αT . This
means that after time T , the stock price drifts away from the price at time
zero.

Major weaknesses:

1. The stock price X (t) can be negative. Since X (t) is normally distributed,−∞ <
X (t) < ∞. Equation 20.11 allows a negative stock price. Of course, the
stock price can’t become negative.

2. The expected change of the stock price does not depend on the stock price.
In reality, the expected change of the stock price should be proportional
to the stock price. The higher the stock price, the higher the expected
change. So we like to have E [dX (t)] = αX (t). We need to modify
Equation 20.11 to allow E [dX (t)] = αX (t).

3. The variance of the stock price does not depend on the stock price. In
reality, the variance should be proportional to the stock price. So we need
to modify Equation 20.11 to allow σ [X (t) , t] = σX (t).

Major Weakness #2 and #3 can also be stated this way. Equation 20.11 can
dX (t) α σ dX (t)
be rewritten as = dt + dZ (t). This indicates that ,
X (t) X (t) X (t) X (t)
the percentage return on the stock depends on the stock price X (t). However,
in reality, we think that the stock return on average shouldn’t depend on the
stock price. In other words, instead of Equation 20.11, we like to see

dX (t)
= αdt + σdZ (t) (20.12)
X (t)
or

dX (t) = X (t) αdt + σX (t) dZ (t) (20.13)


Equation 20.12 and 20.13 are called the geometric Brownian motion.

20.2.6 Ornstein-Uhlenbeck process


We can modify Equation 20.11 to allow for mean reversion. It’s reasonable for
us to assume that the stock price or the interest rate will revert to the mean.
For example, if the stock price is too high, then it might go down; if the stock
price is too low, it might go up. We modify the drift term in Equation 20.11:

dX (t) = λ [α − X (t)] dt + σdZ (T ) (20.14)


166 CHAPTER 20. BROWNIAN MOTION AND ITO’S LEMMA

If α = 0, 20.14 is called the Ornstein-Uhlenbeck process:

dX (t) = −λX (t) dt + σdZ (T ) (20.15)

The Ornstein-Uhlenbeck process is the most widely used mean reverting


stochastic process in financial modeling.

20.3 Definition of the stochastic calculus


We can rewrite Equation 20.12 as:
RT RT RT
0
dX (t) = 0 X (t) αdt + 0 σX (t) dZ (t)
RT RT RT RT
or X (T )−X (0) = 0 X (t) αdt+ 0 σX (t) dZt = α 0 X (t) dt+σ 0 X (t) dZt
RT
But what’s the meaning of 0 X (t) dZ (t)? Or generally, what’s the meaning
Rb
of a g (t) dZ (t)?
To answer this question, let’s
R 1take a step back and find out the meaning of
a simple deterministic calculus 0 x2 dx.
R1 2
0
x dx is the area of the function x2 bounded by x = 0 and x = 1. To find
this area, we divide the interval [0, 1] into n intervals [0, h], [h, 2h], [2h, 3h], ...,
[(n − 1) h, nh = 1]. Then we approximate the area with the sum of n rectangles.
The area of function x2 over the interval [(i − 1) h, ih] is roughly the area of the
rectangular with height [(i − 1) h]2 and width ih − (i − 1) h = h.
R ih
(i−1)h
x2 dx ≈ [(i − 1) h]2 h = (i − 1)2 h3
R1 2 Pn R ih 2
Pn 2 3 2
0
x dx = lim i=1 (i−1)h x dx = lim i=1 (i − 1) h = lim {0 h +
n→∞ n→∞ n→∞
2 2 2
h h + (2h) h + ... + [(n − 1) h] h}
02 h +hh2 h + (2h)2 h + ... + [(n − 1)i h]2 h
= h3 02 + 12 + 22 + ... + (n − 1)2
Using the famous formula:
n (n + 1) (2n + 1)
12 + 22 + 32 + ... + n2 =
h i6 (n − 1) (n) (2n − 1)
2
h 0 + 1 + 2 + ... + (n − 1) = h3
3 2 2 2
6
1
Since h = , we have:
n
R1 1 (n − 1) (n) (2n − 1)
0
x2 dx = lim 3
∙ n→∞
µ n¶ µ 6 ¶¸
1 1 1 1
= lim 1− (1) 2 − =
n→∞ 6 n n 6
R ih
There are other ways to approximate (i−1)h x2 dx. For example, the area of
function x2 over the interval [(i − 1) h, ih] is roughly the area of the rectangular
with height (ih)2 and width ih − (i − 1) h = h.
R ih 2
(i−1)h
x2 dx ≈ (ih) h = i2 h3
20.3. DEFINITION OF THE STOCHASTIC CALCULUS 167

R1 Pn R ih Pn n (n + 1) (2n + 1) 3
0
x2 dx = lim i=1 (i−1)h x2 dx = lim i=1 i2 h3 = lim h
n→∞ n→∞ n→∞ 6
n (n + 1) (2n + 1) 1 1
= lim =
n→∞ 6 n3 R 6
ih
The 3rd way to approximate (i−1)h x2 dx is to take the average height of
the rectangular. So the area of function x2 over the interval [(i − 1) h, ih] is
2 2
(i − 1) h2 + (ih)
roughly the area of the rectangular with height and width
2
ih − (i − 1) h = h.
2 2 2
(i − 1) h2 + (ih) (i − 1) + i2 2
= h
2 " 2 #
2
R ih 2 (i − 1) + i2 2
(i−1)h
x dx ≈ h h
2
" #
2
R1 2 Pn R ih 2
P n (i − 1) + i 2
0
x dx = lim i=1 (i−1)h x dx = n→∞ lim i=1 h2 h
n→∞ 2
1 Pn h 2 2
i 1 Pn £ 2 2 ¤
= lim i=1 (i − 1) h h + lim i h h
2µn→∞ ¶ µ ¶ 2 n→∞ i=1
1 1 1 1 1
= + =
2 6 2 6 6
It seems natural that we extend this logic of deterministic integration to a
define a stochastic integration.
Suppose we partition [a, b] into a = t0 < t1 < t2 < ... < tn = b. We can
make the partition intervals [t0, t1 ], [t1, t2 ], ..., [tn−1, tn ] have the same length
b−a
tk+1 − tk = . We can also have the partition intervals [t0, t1 ], [t1, t2 ], ...,
n
[tn−1, tn ] have different lengths.
Rb Pn−1
a
g (t) dZ (t) = lim k=0 g (tk ) [Z (tk+1 ) − Z (tk )]
n→∞

Definition 20.3.1.
Suppose g (t) is a simple process, meaning that g (t) is piecewise-constant
Rb £ ¤
but may have jumps at a = t0 < t1 < t2 < ... < tn = b. If a E g 2 (t) dt < ∞,
Rb
then the stochastic integral a g (t) dZ (t) is defined as
Z b n−1
X
g (t) dZ (t) = g (tk ) [Z (tk+1 ) − Z (tk )] (20.16)
a k=0
Rb £ ¤
In the above definition, a E g 2 (t) dt < ∞ is the sufficient condition for
Pn−1
k=0 g (tk ) [Z (tk+1 ) − Z (tk )] to exist.

RT
Example 20.3.1. Calculate 0
dZ (t)

Solution.
168 CHAPTER 20. BROWNIAN MOTION AND ITO’S LEMMA

Here g (t) = 1.
R T ¡ 2¢ RT
0
E 1 dt = T < ∞. So 0 dZ (t) exists.
RT Pn−1
0
dZ (t) = k=0 [Z (tk+1 ) − Z (tk )] = Z (T )

Example 20.3.2.

⎨ 1 if 0 ≤ t ≤ 1
X (t) = 2 if 1 < t ≤ 2

3 if 2 < t ≤ 3
R3
Calculate 0 X (t) dZ (t)

Solution.

Dividend [0, 3] into (0, 1), (1, 2), and (2, 3). Then X (t) is constant during
the Rinterval and jumps at t = 1 and t = 2. Hence
3
0
X (t) dZ (t)
= X (0) [Z (1) − Z (0)] + X (1) [Z (2) − Z (1)] + X (2) [Z (3) − Z (2)]
= 1 [Z (1) − Z (0)] + 2 [Z (2) − Z (1)] + 3 [Z (3) − Z (2)]
= 1Z (1) + 2 [Z (2) − Z (1)] + 3 [Z (3) − Z (2)]
= 3Z (3) − Z (2) − Z (1)

If g (tk ) is not a simple process, then we define the stochastic integral as


follows.
If E lim (X − a)2 = 0, we say that X approach a in mean square.
n→∞
Explain why the sample mean approaches the population mean in mean
square.
Suppose we take n random samples X1 , X2 ,...,Xn from the population X.
1 Pn
Let μ represent the population mean. Then the sample mean is Xk . It
n k=1
can be shown that
µ ¶2
1 Pn
E lim k=1 Xk − μ =0
n→∞ n
Toµsee why, notice¶
1 Pn 1 Pn 1
E Xk = E ( k=1 Xk ) = nμ = μ
n k=1 n n
µ ¶2
1 Pn
E Xk − μ
n µk=1 ¶
1 Pn 1 P
= V ar k=1 Xk = 2 V ar ( nk=1 Xk )
n n
1 V ar (X)
= 2 nV ar (X) =
n µ n ¶2
1 Pn
→ E lim k=1 X k − μ =0
n→∞ n
So the sample mean approaches the population mean in mean square.
20.3. DEFINITION OF THE STOCHASTIC CALCULUS 169

Definition 20.3.2.

Let a = t0 < t1 < t2 < ... < tn = b represent a partition of [a, b]. Define
P
random variable In = n−1
k=0 g (tk ) [Z (tk+1 ) − Z (tk )], where g (tk ) is a simple
or a complex process and g (tk ) and Z (tk+1 ) − Z (tk ) are independent. If
Rb £ 2 ¤
a
E g (t) dt < ∞ and the mean squared difference between In and U is zero
as n → ∞:

2
E lim (In − U ) = 0 (20.17)
n→∞

Then we say
Rb
• a
g (t) dZ (t) = U
Rb
• In converges to a
g (t) dZ (t) in mean square

The above definition holds whether g (t) is a simple or complex process.


Please note that in the term g (tk ) [Z (tk+1 ) − Z (tk )], g is evaluated at the
left of the interval [tk , tk+1 ]:
Rb Pn−1
a
g (t) dZ (t) = lim k=0 g (tk ) [Z (tk+1 ) − Z (tk )]
n→∞
Rb Pn−1
a
g (t) dZ (t) 6= lim k=0 g (tk+1 ) [Z (tk+1 ) − Z (tk )]
n→∞
Rb Pn−1 g (tk ) + g (tk+1 )
a
g (t) dZ (t) 6= lim k=0 [Z (tk+1 ) − Z (tk )]
n→∞ 2 R ih
This is different from the deterministic calculus (i−1)h x2 dx, which can be
approximated using 3 heights:

• The left height [(i − 1) h]2

• The right height (ih)2

(i − 1)2 h2 + (ih)2
• The average height
2

In addition, we require that g (tk ) and Z (tk+1 ) − Z (tk ) are independent. In


other words, we require that g (tk ) not depend on the future Brownian incre-
ment Z (tk+1 ) − Z (tk ). The requirements that we evaluate g at the left of the
interval [tk , tk+1 ] and that g (tk ) and Z (tk+1 ) − Z (tk ) are independent agree
with our intuition. We calculating g (tk ) [Z (tk+1 ) − Z (tk )], g (tk ) is based on
the information available to us during [0, tk ] and is independent of the future
Brownian increment Z (tk+1 ) − Z (tk ).
RT 2
Example 20.3.3. Calculate 0 [dZ (t)] .
170 CHAPTER 20. BROWNIAN MOTION AND ITO’S LEMMA

RT 2 RT 2
0
[dZ (t)] = 0 g (t) [dZ (t)] where g (t) = 1
Partition [0, T ] into [0, h], [h, 2h],...,[(n − 1) h, nh = T ].
P Pn
Let In = n−1 k=0 {Z [(k + 1) h] − [Z (kh)]} =
2
k=1 {Z (kh) − [Z (k − 1) h]}
2

Z (kh) − [Z (k − 1) h] is a normal random variable with mean 0 and variance


h. Using Equation 20.9, we have
2
E{Z (kh)
Pn − [Z (k − 1) h]} = h
In = k=1 {Z (kh) − [Z (k − 1) h]}2
RT 2
Since E (In ) = nh = T , we guess that 0 [dZ (t)] = T
The difficult part is to verify that the mean square error is zero as n → ∞:
hP i2
2
lim E ( nk=1 {Z (kh) − [Z (k − 1) h]}) − T = 0
n→∞

Let ∆k = Z (kh) − [Z (k − 1) h]
Pn 2
→ ( k=1 {Z (kh) − [Z (k − 1) h]}) = ∆21 + ∆22 + ... + ∆2n
hP i2
n 2
→ ( k=1 {Z (kh) − [Z (k − 1) h]}) − T
£¡ ¢ ¤2 ¡ ¢2 ¡ ¢
= ∆21 + ∆22 + ... + ∆2n − T = ∆21 + ∆22 + ... + ∆2n +T 2 −2T ∆21 + ∆22 + ... + ∆2n
¡ 2 ¢2
∆1 + ∆22 + ... + ∆2n = ∆41 + ∆42 + ...∆4n + 2∆21 ∆22 + 2∆21 ∆23 + ...
¡ ¢2 ¡ ¢ ¡ ¢
→ E ∆21 + ∆22 + ... + ∆2n = E ∆41 + ∆42 + ...∆4n +2E ∆21 ∆22 + ∆21 ∆23 + ...
∆k is¡normal
¢ with mean 0 and variance h
→ E ¡∆4k = 3h2 ¢
→ E ∆41 + ∆42 + ...∆4n = 3nh2
∆i and ∆j where i 6= j are two independent normal random variables (Point
3 of the Brownian
¡ ¢ motion
¡ ¢definition)
¡ ¢
→ E ∆2i ∆2j = E ∆2i E ∆2j = h × h = h2
1
There are n (n − 1) pairs of ∆i and ∆j where i 6= j
2
¡ ¢ 1
→ 2E ∆21 ∆22 + ∆21 ∆23 + ... = 2 × n (n − 1) h2 = n (n − 1) h2
¡ ¢2 2
E ∆21 + ∆22 + ... + ∆2n = 3nh2 + n (n − 1) h2

£ ¡ ¢¤ ¡ ¢
E 2T ∆21 + ∆22 + ... + ∆2n = 2T E ∆21 + ∆22 + ... + ∆2n = 2T (nh) = 2T 2
hP i2
n 2
→ E ( k=1 {Z (kh) − [Z (k − 1) h]}) − T
= 3nh2 + n (n − 1) h2 + T 2 − 2T = [3n + n (n − 1)] h2 − T 2

T
However, h =
n µ ¶2
2 2 T 3n + n (n − 1) 2
→ [3n + n (n − 1)] h −T = [3n + n (n − 1)] −T 2 = T −
n n2
T2
3n + n (n − 1) 3n + n (n − 1) 2
As n → ∞, →1 T − T2 → 0
n2 n2
20.3. DEFINITION OF THE STOCHASTIC CALCULUS 171

hP i2
2
lim E ( nk=1 {Z (kh) − [Z (k − 1) h]}) − T = 0
n→∞
RT
Hence 0 [dZ (t)]2 = T .
RT
Example 20.3.4. Calculate 0 Z (t) dZ (t)
Partition [0, T ] into [0, h], [h, 2h],...,[(n − 1) h, nh = T ].
Pn−1 Pn
Let In = k=0 Z (kh) {Z [(k + 1) h]−Z (kh)} = k=1 Z [(k − 1) h] {Z (kh)−
Z [(k − 1) h]} ¡ ¢
2
(a + b) − a2 + b2
Use the formula: ab =
2
Let a = Z [(k − 1) h] b = Z (kh) − Z [(k − 1) h]
a
P+n b = Z (kh)
k=1 Z [(k − 1) h] {Z (kh) − Z [(k − 1) h]}
2 2
P [Z (kh)] − Z [(k − 1) h] − {Z (kh) − Z [(k − 1) h]}2
= nk=1
2
1 Pn 2 2 1 Pn
= k=1 {[Z (kh)] − Z [(k − 1) h] }− {Z (kh) − Z [(k − 1) h]}2
2 2 k=1
1 2 1 Pn
= [Z (nh)] − {Z (kh) − Z [(k − 1) h]}2
2 2 k=1
1 2 1 Pn
= [Z (T )] − {Z (kh) − Z [(k − 1) h]}2
2 2 k=1
1 2 1 Pn
→ In = [Z (T )] − {Z (kh) − Z [(k − 1) h]}2
2 2 k=1
Pn
lim k=1 Z [(k − 1) h] {Z (kh) − Z [(k − 1) h]} = n→∞ lim In
n→∞
1 2 1 P n
= [Z (T )] − lim {Z (kh) − Z [(k − 1) h]}2
2 2 n→∞ k=1
Pn
From the previous example, we know that k=1 {Z (kh)−Z [(k − 1) h]}2 ap-
RT RT
proaches 0 [dZ (t)]2 = T in the mean square. So we guess that 0 Z (t) dZ (t) =
1 1
[Z (T )]2 − T
2 2 ∙ µ ¶¸2
1 2 1
Next, we need to prove that E lim In − [Z (T )] − T =0
µ ¶ n→∞ 2 2
1 2 1 1 1 Pn
In − [Z (T )] − T = T − {Z (kh) − Z [(k − 1) h]}2
2 2 2 2 k=1
In − E (In ) µ ¶
1 1 Pn 1 1
= [Z (T )]2 − {Z (kh) − Z [(k − 1) h]} 2
− [Z (T )]2
− T
2 2 k=1 2 2
1¡ Pn 2
¢
= T − k=1 {Z (kh) − Z [(k − 1) h]}
2
From the previous example, we found that
¡ Pn ¢2
lim E T − k=1 {Z (kh) − Z [(k − 1) h]}2 = 0
n→∞
∙ µ ¶¸2
1 1
→ E lim In − [Z (T )]2 − T =0
n→∞ 2 2
172 CHAPTER 20. BROWNIAN MOTION AND ITO’S LEMMA

So we have

Z T
1 2 1
Z (t) dZ (t) = [Z (T )] − T (20.18)
0 2 2
RT
Equation 20.18 is surprising. In the deterministic calculus, we have 0
xdx =
1 2
T .
2
1
Equation 20.18 has an extra term − T . This is why this extra term is
2
needed. Taking expectation of Equation 20.18:
hR i 1 1
T 2
E 0 Z (t) dZ (t) = E [Z (T )] − T
2 2
hR i
T
As to be explained later, E 0 Z (t) dZ (t) = 0. We already know that
hR i 1 1
2 T 2
E [Z (T )] = T . Hence E 0 Z (t) dZ (t) = E [Z (T )] − T = 0. The extra
2 2
1
term − T is needed so the expectations of both sides of Equation 20.18 are
2
equal.

20.4 Properties of the stochastic calculus


RT RT RT
1. Lineality. 0
[c1 g (t) + c2 h (t)] dZ (t) = c1 0
g (t) dZ (t)+c2 0
h (t) dZ (t)

RT £ ¤ ³R ´
T
2. Zero mean property. If 0
E X 2 (t) dt < ∞, then E 0 X (t) dZ (t) =
0

The proof is complex. However, for a simple process g (t) and h (t), Property
#1 holds due to the definition of the stochastic integral. For a simple process
g (t), Property #2 can be easily established.
RT P
0
g (t) dZ (t) = n−1
k=0 g (tk ) [Z (tk+1 ) − Z (tk )]

g (tk ) is constant during each partition interval and independent of Z (tk+1 )−


Z (tk ). Then
E{g (tk ) [Z (tk+1 ) − Z (tk )]} = E [g (tk )] E [Z (tk+1 ) − Z (tk )] = E [g (tk )] 0 =
0
RT RT £ ¤ RT
Let’s apply Property 2 to 0 Z (t) dZ (t). Since 0 E Z 2 (t) dt = 0 tdt =
1 2 hRT i
T < ∞, we have E 0 Z (t) dZ (t) = 0
2
20.5. ITO’S LEMMA 173

20.5 Ito’s lemma


20.5.1 Multiplication rules
dZ dt
dZ dt 0
dt 0 0

The above table means:


[dZ (t)]2 = dt dZ (t) dt = 0 (dt)2 = 0

Example 20.5.1. Explain why [dZ (t)]2 = dt

dZ (t) is a normal random variable with mean 0 and variance dt (see the
footnote of Derivatives Markets Page 652). Hence E [dZ (t)]2 = dt
2
E [dZ (t) − dt] = V ar [dZ (t)] = dt → 0
2 2
Hence dt approach [dZ (t)] in mean square. So [dZ (t)] = dt

Example 20.5.2. Explain why dZ (t) dt = 0

E [dZ (t) dt] = E [dZ (t)] dt = 0 (here dt is treated as a constant)


2 2 2 3
E [dZ (t) dt − 0] = E [dZ (t)] (dt) = (dt) → 0
Hence 0 approaches dZ (t) dt in mean square. dZ (t) dt = 0.
2
Example 20.5.3. Explain why (dt) = 0
2
(dt) doesn’t contain any Brownian motion term dZ. So we need to calculate
(dt)2 according to the deterministic calculus. In the deterministic calculus,
2 2
(dt) → 0 as dt → 0. Hence (dt) = 0.
The textbook Derivatives Markets also gives the following formula:
0
dZ × dZ = ρdt (20.19)
The above formula will be explained later.

20.5.2 Ito’s lemma


In essence, Ito’s lemma is a Taylor series applied to Brownian motion.

Suppose that a stock has an expected instant return α [S (t) , t], dividend
∧ ∧
yield δS (t) , t, and instant volatility σ [S (t) , t] follows geometric Brownian mo-
tion:
µ ¶
∧ ∧ ∧
dS (t) = α − δ dt + σdZ (t) (20.20)

∧ ∧ ∧
Here α, δ, and σ are function of the stock price S (t) and time t.
174 CHAPTER 20. BROWNIAN MOTION AND ITO’S LEMMA

Let C [S (t) , t] represent the value of a call or put option. We want to find
out the change of the option value given there’s a small change of the stock price
and a small change of time. Using Taylor series, we have:

∂C ∂C 1 ∂2C 2 1 ∂2C 2 ∂2C


dC [S (t) , t] = dS + dt+ (dS) + (dt) + dSdt (20.21)
∂S ∂t 2 ∂S 2 2 ∂t2 ∂S∂t
∙µ ¶ ¸2 µ ¶ µ ¶
∧ ∧ ∧ ∧ ∧ 2 ∧ ∧ ∧
2 2
[dS (t)] = α − δ dt + σdZ (t) = α − δ (dt) +2 α − δ σdZ (t) dt+

∧2 2
σ [dZ (t)] µ ¶
∧ ∧ ∧
2
dSdt = α − δ (dt) + σdZ (t) dt
2 2
Using the multiplication rules: (dt) = 0 [dZ (t)] = dt dZ (t) dt = 0
2 ∧2
[dS (t)] = σ dt dSdt = 0
Now we have:

∂C ∂C 1 ∂2C 2 ∂C ∂C 1 ∂ 2 C ∧2
dC [S (t) , t] = dS + dt + (dS) = dS + dt + σ dt
∂S ∂t 2 ∂S 2 ∂S ∂t 2 ∂S 2
(20.22)
Next, apply Equation∙µ 20.20 to¶Equation ??:¸
∂C ∧ ∧ ∧ ∂C 1 ∂ 2 C ∧2
→ dC [S (t) , t] = α − δ dt + σdZ (t) + dt + σ dt
∙µ ¶ ∂S ¸ ∂t 2 ∂S 2
∧ ∧ ∂C 1 ∧2 ∂ 2 C ∂C ∂C ∧
= α−δ + σ + dt + σdZ (t)
∂S 2 ∂S 2 ∂t ∂S
∙µ ¶ ¸
∧ ∧ ∂C 1 ∧2 ∂ 2 C ∂C ∂C ∧
dC [S (t) , t] = α−δ + σ 2
+ dt + σdZ (t) (20.23)
∂S 2 ∂S ∂t ∂S
Equation 20.23 is called the Ito’s lemma.
Tip 20.5.1. Don’t bother memorizing Equation 20.23. Just derive Equation
20.23 on the spot. First, write down the Taylor series Equation 20.22. Next,
apply the multiplication rules. Then you’ll get Equation 20.23.
If S (t) follows a geometric Brownian motion, we have:

• α [S (t) , t] = αS (t)

• δ [S (t) , t] = δS (t)

• σ [S (t) , t] = σS (t)
Equation 20.23 becomes:

∙ ¸
∂C 1 ∂2C ∂C ∂C
dC [S (t) , t] = (α − δ) S + σ2S 2 2 + dt + σSdZ (t) (20.24)
∂S 2 ∂S ∂t ∂S
20.6. GEOMETRIC BROWNIAN MOTION REVISITED 175

Tip 20.5.2. Don’t bother memorizing Equation 20.24. Just derive Equation
20.24 on the spot.
Please note that
∂C ∂2C ∂C
=∆ =Γ =θ (option Greeks)
∂S ∂S 2 ∂t
Then Equation 20.22 becomes:
1 2
dC [S (t) , t] = ∆dS + θdt + Γ (dS) (20.25)
2

Consider a tiny interval dt ≈ h. We have:


dC [S (t) , t] ≈ C [S (t + h) , t + h] − C [S (t) , t]

dS ≈ S (t + h) − S (t)
Equation 20.25 becomes:

1 2
C [S (t + h) , t + h]−C [S (t) , t] ≈ ∆ [S (t + h) − S (t)]+θh+ Γ [S (t + h) − S (t)]
2
(20.26)

Equation 20.26 is just the textbook Equation 13.6 (Derivatives Markets page
426).
If S (t) were deterministic (i.e. if Equation 20.20 didn’t have dZ (t) term),
∂2C
then → 0 and Equation 20.22 would be:
∂S 2
∂C ∂C
dC [S (t) , t] = dS + dt
∂S ∂t

20.6 Geometric Brownian motion revisited

There are two minor concepts under geometric Brownian motion. SOA can
easily write a question on these concepts. So let’s study them.

20.6.1 Relative importance of drift and noise term


Consider a discrete geometric Brownian motion:

X (t + h) − X (t) = αX (t) h + σX (t) Y (t) h
| {z } | {z }
deterministic component random comp onent
One phrase you need to know is called "the ratio of the standard deviation
to the drift" (the drift is actually the deterministic component). The ratio of
the standard deviation to the drift is defined as:

σX (t) h σ
= √ (20.27)
αX (t) h α h
176 CHAPTER 20. BROWNIAN MOTION AND ITO’S LEMMA

20.6.2 Correlated Ito processes


Let W1 (t) and W2 (t) represent two independent Brownian motions. Suppose

Z (t) = W1 (t) (20.28)

0 p
Z (t) = ρW1 (t) + 1 − ρ2 W2 (t) (20.29)

Please note that Z (t) is normally distributed with mean 0 and variance t
because W1 (t) is normally distributed with mean 0 and variance
p t.
You might wonder why Equation 20.29 has constants ρ and 1 − ρ2 . These

0
two constants are needed to make Z (t) normally distributed with variance t.
Because W1 (t) and W2 (t) areptwo independent normal random variables, the
linearhcombination 2
0
i hρW1 (t) + p1 − ρ W2 (t) iis also normally distributed.
E Z (t) = E ρW1 (t) + 1 − ρ2 W2 (t)
hp i
= E [ρW1 (t)] + E 1 − ρ2 W2 (t)
p
= ρE [W1 (t)] + 1 − ρ2 E [W2 (t)]
p
= ρ0 + 1 − ρ2 0 = 0
h 0 i h p i
V ar Z (t) = V ar ρW1 (t) + 1 − ρ2 W2 (t)
hp i
= V ar [ρW1 (t)] + V ar 1 − ρ2 W2 (t)
¡ ¢
= ρ2 V ar [W1 (t)] + 1 − ρ2 V ar [W2 (t)]
¡ ¢
= ρ2 t + 1 − ρ2 t = t
0
h 0
i
The covariance between Z (t) and Z (t) is: Cov Z (t) , Z (t)

Using the
h standard formula:
i h Cov (X, Yi) = E (XY ) −hE (X)iE (Y )
0 0 0
→ Cov Z (t) , Z (t) = E Z (t) Z (t) − E [Z (t)] E Z (t)
h 0
i h 0
i
= E Z (t) Z (t) − 0 × 0 = E Z (t) Z (t)

0
h p i p
Z (t) Z (t) = W1 (t) ρW1 (t) + 1 − ρ2 W2 (t) = ρ [W1 (t)]2 + 1 − ρ2 W1 (t) W2 (t)
h 0
i ³ ´ hp i
E Z (t) Z (t) = E ρ [W1 (t)]2 + E 1 − ρ2 W1 (t) W2 (t)
2
p
= ρE [W1 (t)] + 1 − ρ2 E [W1 (t) W2 (t)]
E [W1 (t)]2 = t
E [W1 (t) W2 (t)] = E [W1 (t)] E [W2 (t)] = 0 × 0 = 0
h 0
i
E Z (t) Z (t) = ρt (20.30)
20.6. GEOMETRIC BROWNIAN MOTION REVISITED 177
h 0
i h 0
i
The textbook says calls Cov Z (t) , Z (t) = E Z (t) Z (t) = ρt the corre-
0
lation between Z (t) and Z (t). However, the term correlation between X and
Y typically means the following:
Cov (X, Y )
ρX,Y =
σX σY
0
If wehuse the typical
i definition, the correlation between Z (t) and Z (t) is:
0
Cov Z (t) , Z (t) ρt
= √ √ =ρ
σ Z(t) σ Z 0 (t) t t
Since Derivatives Markets is the textbook, we have to adopt its definition
0
that the correlation between Z (t) and Z (t) is ρt.
0
dZ (t) and dZ (t) are both normal random variables with mean 0 and vari-
ance dt. Applying Equation 20.30 and replacing t with dt, we have:
h 0
i
E dZ (t) dZ (t) = ρdt (20.31)
0
Finally,
h let’s explain
i why Equation 20.19 dZ × dZ = ρdt holds.
0
E dZ (t) dZ (t) = ρdt
h 0
i2 h 0
i h 0
i2 h 0
i
E dZ (t) dZ (t) − ρdt = V ar dZ (t) dZ (t) = E dZ (t) dZ (t) −E 2 dZ (t) dZ (t)

0
dZ (t) dZ (t) h i
p
= dW1 (t) × d ρW1 (t) + 1 − ρ2 W2 (t)
h p i
= dW1 (t) × ρdW1 (t) + 1 − ρ2 dW2 (t)
p
= ρ [dW1 (t)]2 + 1 − ρ2 dW1 (t) dW2 (t)
h 0
i2
dZ (t) dZ (t)
4 ¡ ¢ 2 2
p 3
= ρ2 [dW1 (t)] + 1 − ρ2 [dW1 (t)] [dW2 (t)] +2ρ 1 − ρ2 [dW1 (t)] dW2 (t)
h 0
i2
E dZ (t) dZ (t)
¡ ¢ ³ ´ p ³ ´
= ρ2 E [dW1 (t)]4 + 1 − ρ2 E [dW1 (t)]2 [dW2 (t)]2 +2ρ 1 − ρ2 E [dW1 (t)]3 dW2 (t)
¡ ¢ p
= ρ2 E [dW1 (t)]4 + 1 − ρ2 E [dW1 (t)]2 E [dW2 (t)]2 +2ρ 1 − ρ2 E [dW1 (t)]3 E [dW2 (t)]
¡ ¢ p
= ρ2 3 (dt)2 + 1 − ρ2 dt × dt + 2ρ 1 − ρ2 0 × 0
2 ¡ ¢
= ρ2 3 (dt) + 1 − ρ2 dt × dt
£ ¡ ¢¤ ¡ ¢
= 3ρ2 + 1 − ρ2 (dt)2 = 2ρ2 + 1 (dt)2
h 0
i
E dZ (t) dZ (t) = ρdt
h 0
i2 h 0
i
E dZ (t) dZ (t) − E 2 dZ (t) dZ (t)
¡ ¢ 2 2 ¡ ¢ 2
= 2ρ2 + 1 (dt) − ρ2 (dt) = ρ2 + 1 (dt)
178 CHAPTER 20. BROWNIAN MOTION AND ITO’S LEMMA

2
(dt) = 0
h 0
i2 h 0
i
→ E dZ (t) dZ (t) − E 2 dZ (t) dZ (t) = 0
h 0
i2 h 0
i
→ E dZ (t) dZ (t) − ρdt = V ar dZ (t) dZ (t) = 0
0
Hence dZ (t) dZ (t) approach ρdt in mean square. Hence
0
dZ × dZ = ρdt (20.32)
Suppose the stock price S (t) follows a geometric Brownian motion:
dS (t)
= αdt + σdZ (t)
S (t)

1. Apply Ito’s lemma to ln S (t) £¡


and derive that
¢ ¤ ln S (t) is a normal random
variable with mean ln S (0) + α − 0.5σ 2 t and variance σ 2 t.
2. Derive that the mean of S (t) is E [S (t)] = S (0) eαt

As explained in my book and my solution manual, you don’t need to mem-


2
orize Ito’s lemma. Just use the Taylor expansion but keep the (dZ) term.
First, use Taylor expansion:
∂ ln S ∂ ln S 1 ∂ 2 ln S 2
d ln S = dS + dt + (dS)
∂S ∂t 2 ∂S 2
∂ ln S ∂ ln S
In the deterministic calculus, you just write d ln S = dS + dt.
∂S ∂t
2 2
However, in the stochastic calculus, we can’t ignore (dZ) since (dZ) = dt.
In this problem, d ln S is a linear function of dZ, so:
h i
2 2 2 2
(dS) = [Sαdt + σdZ] = S 2 α2 (dt) + 2ασdtdZ + σ 2 (dZ)

Using the multiplication


h rule (DM 20.17 a, ib, c), we get:
2 2 2 £ ¤
(dS) = S α (dt) + 2αdtσdZ + σ 2 (dZ) = S 2 α2 × 0 + 2ασ × 0 + σ 2 dt =
2 2

S 2 σ 2 dt
1 ∂ 2 ln S 2
So we have to keep the term (dS) .
2 ∂S 2
Anyway, Taylor expansion gives us:
∂ ln S ∂ ln S 1 ∂ 2 ln S 2 2
d ln S = dS + dt + S σ dt
∂S ∂t 2 ∂S 2
2
∂ ln S 1 ∂ ln S 1 ∂ ln S
Next, = =− 2 =0
∂S S ∂S S ∂t
µ ¶
dS 1 2 1 2 1 2
→ d ln S = − σ dt = αdt + σdZ − σ dt = α − σ dt + σdZ
S 2 2 2
During a tiny time interval [t, t + dt], the change of ln S is d ln S.
d ln S is a normal random variable. This is why. √
First, dZ ∼ N (0, dt) .According to DM 20.4, dZ = Y dt. So dZ is a normal
random variable with mean 0 and variance dt.
20.6. GEOMETRIC BROWNIAN MOTION REVISITED 179

During the fixed interval [t, t + dt], dt is a constant. Since a constant plus a
random variable is also a random variable,
µ ¶ µ ¶
1 2 1 2
d ln S = α − σ dt+σdZ is a normal random variable with mean α − σ dt
2 2
and variance σ2 dt
Now consider the time interval [0, t].
µ ¶
Rt Rt 1 2 Rt
ln S (t) = ln S (0) + 0 d ln S = ln S (0) + 0 α − σ ds + 0 σdZ
2
µ ¶
1 2 Rt Rt ¡ ¢
= ln S (0) + α − σ 0
ds + σ 0 dZ = ln S (0) + α − 0.5σ 2 t + σZ
2

Z is a standard normal random variable with mean 0 and variance 1, that

¡ ¢
is, Z ∼ N 0, σ 2 t . Hence σZ is a normal random variable ¡with mean¢ 0 and
variance σ 2 t. And ln S (t) is normal with mean ln S (0) + α − 0.5σ 2 t and
variance σ 2 t:
£ ¡ ¢ ¤
ln S (t) ∼ N ln S (0) + α − 0.5σ 2 t, σ 2 t

¡ ¢
Next, from ln S (t) = ln S (0) + α − 0.5σ 2 t + σZ, we get:
2
eln S(t) = eln S(0)+(α−0.5σ )t+σZ h i
2 2
→ S (t) = S (0) e(α−0.5σ )t+σZ = S (0) e(α−0.5σ )t eσZ
h 2
i ¡ ¢
→ E [S (t)] = S (0) e(α−0.5σ )t E eσZ

¡ ¢
We can use DM Equation 18.13 to calculate E eσZ . DM Equation ¡ 18.13¢
says that if x is normal with mean m and variance v 2 , that is x ∼ N m, v 2 ,
then

2
E (ex ) = em+0.5v (DM 18.13)
¡ ¢ 2
→ E eσZ = e0+0.5σ t
h 2
i
→ E [S (t)] = S (0) e(α−0.5σ )t e0+0.5σ = S (0) eαt
2

£ ¡ ¢ ¤
Alternative method to calculate E [S (t)]. Since ln S (t) ∼ N S (0) + α − 0.5σ 2 t, σ 2 t ,
using DM 18.13, we have:
2 2
E [S (t)] = eS(0)+(α−0.5σ )t+0.5σ t = S (0) eαt
180 CHAPTER 20. BROWNIAN MOTION AND ITO’S LEMMA

I want you to memorize the following results (these results are used over and
over in Exam MFE):

dZ ∼ N (0, dt) (20.33)

Z ∼ N (0, t) (20.34)

¡ ¢ 2
x ∼ N m, v 2 → E (ex ) = em+0.5v (20.35)

dS (t)
S (t) is geometric Brownian motion → = αdt + σdZ (t) (20.36)
S (t)

dS (t) ¡ ¢
= αdt + σdZ (t) → d ln S (t) = α − 0.5σ 2 dt + σdZ (20.37)
S (t)

dS (t) £ ¡ ¢ ¤
= αdt+σdZ (t) → ln S (t) ∼ N ln S (0) + α − 0.5σ 2 t, σ 2 t (20.38)
S (t)

dS (t) 2
= αdt + σdZ (t) → S (t) = S (0) e(α−0.5σ )t+σZ (20.39)
S (t)

dS (t)
= αdt + σdZ (t) → E [S (t)] = S (0) eαt (20.40)
S (t)

20.7 Sharpe ratio


An asset’s Sharpe ratio is equal to the asset’s risk premium α − r divided by
the asset’s volatility σ:
α−r
SR = (20.41)
σ
If two non-dividend paying assets are perfectly corrected (i.e. they are driven
by the same Brownian motion Z (t)), then their Sharpe ratios are equal. Let’s
derive this.
The price processes of Asset 1 and Asset 2 are:

dS1 = α1 S1 dt + σ1 S1 dZ (20.42)
dS2 = α2 S2 dt + σ2 S2 dZ (20.43)
We can form a riskless portfolio by removing the random Brownian motion
dZ. Rewrite Equation 20.42 and 20.43 as:
20.7. SHARPE RATIO 181

µ ¶
1 α1
dS1 = dt + dZ (20.44)
σ 1 S1 σ1
µ ¶
1 α2
dS2 = dt + dZ (20.45)
σ 2 S2 σ2
1
Suppose at time zero we buy N1 = units of Asset 1 and short sell
σ 1 S1
1
N2 = units of Asset 2. If we hold one unit of Asset 1 at time zero, then
σ 2 S2
after a tiny interval dt, the value of Asset 1 increases by the amount dS1 . If
1
we hold N1 = units of Asset 1 at time zero, then after dt the value of
σ 1 S1 µ ¶
1 1 α1
N1 = units of Asset 1 will increase by dS1 = dt + dZ. Notice
σ 1 S1 σ 1 S1 σ1
that the increase of the value of Asset 1 has a random component dZ, where dZ
is a normal random variable with mean 0 and variance dt.
1
Similarly, if we short sell N2 = units of Asset 2 at time zero, after dt,
σµ2 S2 ¶
1 α2
the value of Asset 2 will increase by dS2 = dt + dZ. The increase of
σ 2 S2 σ2
the value of Asset 2 has a random component dZ, where dZ is a normal random
variable with mean 0 and variance dt.
Suppose at time zero we simultaneously buy N1 units of Asset 1 and short
sell N2 units of Asset 2. Then at dt, we close our position by selling N1 units
of Asset 1 in the open market and buying N2 units of Asset 2 from the open
market.
The cash flow at time zero:
µ ¶
1 1
• We pay N1 S1 = S1 = dollar to buy N1 units of Asset 1.
σ 1 S1 σ1
1
• We receive N2 S2 = dollars for short selling N2 units of Asset 2
σ2
1 1 1 1
• The net cost is − dollars. If − < 0, then we receive
µ ¶ σ1 σ2 σ1 σ2
1 1
− − net cash. To avoid tying up our capital, we go to a bank
σ1 σ2 µ ¶
1 1 1 1 1 1
and borrow − dollars. If − < 0, then we lend − − .
σ1 σ2 σ1 σ2 σ1 σ2
• Our net cash outgo is zero.

Our payoff at time dt

• At time dt, we sell off N1 units of Asset 1 in the open market for the price
1 α1
of S1 + dS1 , receiving N1 (S1 + dS1 ) = N1 S1 + N1 dS1 = + dt + dZ
σ1 σ1
182 CHAPTER 20. BROWNIAN MOTION AND ITO’S LEMMA

• At time dt we buy N2 units of Asset 2 in the open market at the price


1 α2
S2 + dS2 . We pay N2 (S2 + dS2 ) = N2 S2 + N2 dS2 = + dt + dZ
σ2 σ2
• At time dt, we pay back the bank both the principal µ and the¶accrued
1 1
interest. The sum of the principal and the interest is − erdt
σ1 σ2
Our profit at dt is:
µ ¶ µ ¶ µ ¶
1 α1 1 α2 1 1
P rof it = + dt + dZ − + dt + dZ − − erdt (20.46)
σ1 σ1 σ2 σ2 σ1 σ2
µ ¶
1 1 1 1
If − < 0, we’ll lend − − at t = 0. Then at dt, we’ll receive
µ σ 1 σ¶2 σ1 σ2
1 1
− − erdt from the borrower.
σ1 σ2
The Taylor expansion:
r2 2 r3 3
erdt = 1 + rdt + (dt) + (dt) + ... = 1 + rdt
2! 3!
Equation
µ 20.46
¶ can
µ be rewritten ¶ as:
µ ¶
1 α1 1 α2 1 1
+ dt − + dt − − erdt
σµ1 σ 1 ¶ µσ 2 σ2 ¶ µσ 1 σ 2 ¶
1 1 α1 α2 1 1
= − + − dt − − erdt
µ σ1 σ2 ¶ µ σ1 σ2 ¶ µ σ1 σ2 ¶ µ ¶
1 1 α1 α2 1 1 1 1
= − + − dt − − − − rdt
µ σ1 σ2 ¶ σµ1 σ2 ¶ σ1 σ2 σ1 σ2
α1 α2 1 1
= − dt − − rdt
µ σ1 σ2 ¶σ 1 σ 2
α1 − r α2 − r
= − dt
σ1 σ2
= (SR1 − SR2 ) dt
µ ¶
α1 − r α2 − r
P rof it = − dt = (SR1 − SR2 ) dt (20.47)
σ1 σ2

Equation 20.46 and 20.47 don’t have the random term dZ (t), indicating that
the profit is surely made.
Our cash outgo is zero at t = 0, but we’ll have the profit indicated by 20.47
at dt. µ ¶
α1 − r α2 − r
Wealth 0 −→ − dt = (SR1 − SR2 ) dt
σ1 σ2
Time 0 −→ dt

To avoid arbitrage, the profit needs to be zero:


α1 − r α2 − r
→ = SR1 = SR2
σ1 σ2
20.8. RISK NEUTRAL PROCESS 183

20.8 Risk neutral process

This section is difficult because the author tired to put too many complex con-
cepts in this small section. It seems that the author was in a big hurry to finish
this chapter. The author mentioned many concepts (such as martingale, Gir-
sanov’s theorem) but he didn’t really explain them, leaving us hanging in the
air asking why.
The only thing worth studying is how to transform a standard geometric
Brownian motion into a risk neutral process.
The standard geometric Brownian motion is:
dS (t)
= (α − δ) dt + σdZ (t)
S (t)
This is how to transform:
dS (t)
= (α − δ) dt + σdZ (t)
S (t)
= (r − δ) dt + σdZ∙ (t) + (α − δ) dt¸
α−δ
= (r − δ) dt + σ dZ (t) + dt
σ
∼ α−δ
Define dZ (t) = dZ (t) + dt
σ
dS (t) ∼
→ = (α − δ) dt + σdZ (t) = (r − δ) dt + σdZ (t)
S (t)
Just learn this transformation and move on.

20.9 Valuing a claim on S a

In a call or put, the payoff at T is a linear function of the stock price S (T ).


What if the payoff is linear? For example, what if the payoff at T is S (T ) raised
to some power a?

20.9.1 Process followed by S a

Though the textbook uses S a , I’m going to use S A . The reason is explained
later.
Suppose the stock price follows geometric Brownian motion:
dS
= (α − δ) dt + σdZ
S

This is why I use S A . If you use S a , the letter a in S a is very similar to


dS
the α (alpha) in the equation = (α − δ) dt + σdZ. This can easily lead to
S
confusion and mistake. As a matter of fact, when I was deriving the formula for
E (S a ), I couldn’t match the textbook’s formula. It took me a while to figure
out that I accidentally switched a and α.
184 CHAPTER 20. BROWNIAN MOTION AND ITO’S LEMMA

We need to determine the process followed by payoff at T is S A .

∂S A 1 ∂2S A 1
dS A = dS + (dS)2 = AS A−1 dS + A (A − 1) S A−2 (dS)2
∂S 2 ∂S 2 ¡ 2 ¢
2 2
However, (dS) = [(α − δ) dt + σdZ] S 2 = σ 2 dt S 2 = σ 2 S 2 dt

1
→ dS A = AS A−1 dS + A (A − 1) S A−2 σ 2 S 2 dt
2
1
= AS A−1 dS + A (A − 1) S A σ 2 dt
2

dS A dS 1
→ A
=A + A (A − 1) σ 2 dt
S S 2
2
=A
£ (α − δ) dt + AσdZ + 0.5A2(A
¤ − 1) σ dt
= A (α − δ) + 0.5A (A − 1) σ dt + AσdZ

We see that S A follows a geometric brownian motion with drift A (α − δ) +


0.5A (A − 1) σ 2 and risk component AσdZ.
£ ¤
20.9.2 Formula for S A (t) and E S A (t)

Next, we apply Equation 20.37. Replace α with A (α − δ) + 0.5A (A − 1) σ 2 and


σ with Aσ. £ ¤
d ln S A (t) =£ A (α − δ) + 0.5A (A ¤− 1) σ 2 − 0.5A2 σ2 dt + AσdZ
= A (α − δ) dt − 0.5Aσ2 dt + AσdZ

Using Equation£ 20.38, we £get: ¤ ¤


ln S A (t) ∼ N ln S A (0) + A (α − δ) − 0.5Aσ 2 t, A2 σ 2 t

Using Equation 20.39:


2
S A (t) = S A (0) e[A(α−δ)−0.5Aσ ]t+AσZ

Using Equation 20.40 just replacing α with A (α − δ) + 0.5A (A − 1) σ 2


£ ¤ 2
E S A (t) = S A (0) e[A(α−δ)+0.5A(A−1)σ ]t
£ ¤ 2
We can also derive E S A (t) using S A (t) = S A (0) e[A(α−δ)−0.5Aσ ]t+AσZ
£ ¤ ³ 2
´
E S A (t) = E S A (0) e[A(α−δ)−0.5Aσ ]t+AσZ
³ 2
´
= S A (0) E e[A(α−δ)−0.5Aσ ]t+AσZ
³ 2
´
= S A (0) E e[A(α−δ)−0.5Aσ ]t eAσZ
2 ¡ ¢
= S A (0) e[A(α−δ)−0.5Aσ ]t E eAσZ

AσZ is a normal random variable with mean 0 and variance V ar [AσZ (t)] =
A2 σ 2 V ar [Z (t)] = A2 σ2 t. Using Equation 20.35, we get:
20.9. VALUING A CLAIM ON S A 185
¡ ¢ 2 2
E eAσZ = e0.5A σ t
£ ¤ 2 2
→ E S A (t) = S A (0) e[A(α−δ)−0.5Aσ ]t e0.5A σ t = S A (0) e[A(α−δ)+0.5A(A−1)σ ]t
2 2

£ ¤
Alternative method to calculate E S A (t) . Using Equation 20.39, we get:
dS 2
= (α − δ) dt + σdZ → S (t) = S (0) e(α−δ−0.5σ )t+σZ
S
2
→ S A (t) = S A (0)³eA(α−δ−0.5σ )t+AσZ ´
£ ¤ 2
→ E S A (t) = E S A (0) eA(α−δ−0.5σ )t+AσZ
2 ¡ ¢
= S A (0) eA(α−δ−0.5σ )t E eAσZ
2
= S A (0) e[A(α−δ)−0.5Aσ ]t e0.5A σ t
2 2

2
= S A (0) e[A(α−δ)+0.5A(A−1)σ ]t

20.9.3 Expected return of a claim on S A (t)


Consider two geometric brownian motions S (t) and S A (t).
dS
= (α − δ) dt + σdZ
S
where α is the expected return on a claim on S (t) and δ is the continuous
dividend yield earned by S (t)

dS A £ ¤
A
= A (α − δ) + 0.5A (A − 1) σ 2 dt + AσdZ
S

= (γ − δ ∗ ) dt + AσdZ
where γ is the expected return on a claim on S A (t) and δ ∗ is the continuous
dividend yield earned by S A (t). The textbook calls δ ∗ the lease rate.

These two processes have the same risk dZ. Consequently, they have the
same Sharpe ratio:

α−r γ−r
= → γ = r + A (α − r)
σ Aσ
where r is the continuously compounded risk-free interest rate.

20.9.4 Lease rate δ ∗


Since γ − δ ∗ = A 2
£ (α − δ) + 0.5A (A − 1) σ 2 ¤

→ δ = γ − A (α − δ) + 0.5A (A − 1) σ
£ ¤
= r + A (α − r) − A (α − δ) + 0.5A (A − 1) σ 2
= r − A (r − δ) − 0.5A (A − 1) σ 2
So the lease rate for a claim paying S A (t) is δ ∗ = r−A (r − δ)−0.5A (A − 1) σ 2
186 CHAPTER 20. BROWNIAN MOTION AND ITO’S LEMMA

P
£ A ¤
20.9.5 Prepaid forward price F0,T S (T )
P
£ A ¤
F0,T S (T ) is the value at time 0 of a claim paying S A (T ) at T .
Method 1 Use risk-neutral probability

The prepaid forward price is the risk-neutral expected value of the claim
discounted£ Aat the¤ risk-free rate.
£ ¤
P
F0,T S (T ) = e−rT E0∗ S A (T )
£ ¤
E0∗ S A (T ) is the expected value of S A (T ) at time T using the risk-neutral
probability distribution.
We know that the expected value of S A (T ) at time T using the real proba-
£ ¤ 2
bility distribution
£ is E¤ S A (T ) = S A (0) e[A(α−δ)+0.5A(A−1)σ ]T .
To find E0∗ S A (T ) , we just set the expected return α to the risk-free rate
r. In the risk-neutral world, an asset earns the risk-free rate r.
£ ¤ 2
→ E0∗ S A (T ) = S A (0) e[A(r−δ)+0.5A(A−1)σ ]T
£ ¤ £ ¤ 2
→ F P S A (T ) = e−rT E ∗ S A (T ) = e−rT S A (0) e[A(r−δ)+0.5A(A−1)σ ]T
0,T 0

Method
£ A2 ¤ Use−γTthe real
£ probability
¤
P
F0,T S (T ) = e E0 S A (T )
£ ¤ 2
E0 S A (T ) = S A (0) e[A(α−δ)+0.5A(A−1)σ ]T

γ = r + A (α − r)
£ A ¤ 2
P
→ F0,T S (T ) = e−[r+A(α−r)]T S A (0) e[A(α−δ)+0.5A(A−1)σ ]T
2
= e−rT S A (0) e[A(r−δ)+0.5A(A−1)σ ]T

Method 3 Use the lease rate δ ∗


P
DM Equation 5.4 (the concept of tailing): F0,T = S0 e−δT

We need to replace S0 with S A (0) and δ with δ ∗ = r−A (r − δ)−0.5A (A − 1) σ2 .



This means that if we have buy e−δ T unit of the stock at time 0 and reinvest
the dividend, then at T we’ll have exactly S A (T ). This is similar to the concept
that if we have buy e−δT unit of the stock at time 0 and reinvest the dividend,
then at T we’ll have exactly S (T ).
£ A ¤ 2
S (T ) = S A (0) e−δ T = S A (0) e−[r−A(r−δ)−0.5A(A−1)σ ]T

P
→ F0,T
£ ¤ 2
→ F P S A (T ) = e−rT S A (0) e[A(r−δ)+0.5A(A−1)σ ]T
0,T

20.9.6 Specific examples

The textbook keeps mentioning Jensen’s inequality. So let’s first talk about
Jensen’s inequality. Jensen’s inequality is in the appendix C of the textbook.
You can also find information at http://en.wikipedia.org/wiki/Convex_
function
Jensen’s inequality says
20.9. VALUING A CLAIM ON S A 187

1. if f (x) is convex, then for any probability distribution, we have E [f (x)] ≥


f [E (x)]

2. if f (x) is concave, then for any probability distribution, we have E [f (x)] ≤


f [E (x)]

What’s a convex function? What’s a concave function?

If at any point you draw a tangent line, the function f (x) stays above the
tangent line, then f (x) is a convex function.

If at any point you draw a tangent line, the function f (x) stays below the
tangent line, then f (x) is a concave function.

A twice differentiable function of one variable is convex on an interval if and


only if its second derivative is non-negative there.

A twice differentiable function of one variable is concave on an interval if


and only if its second derivative is negative there.

Here’s a simple explanation of Jensen’s inequality.


188 CHAPTER 20. BROWNIAN MOTION AND ITO’S LEMMA

d2 y
Consider y = x2 . Sine = 2 > 0, y = x2 is a convex function. Suppose
dx2
weµtake two points A (1, 1) and¶ B (4, 16). The mid point of the line AB is
1+4 1 + 16
C = 2.5, = 8.5 . The fact that y = x2 is a convex function means
2 2
that y = x2 curves up. Then it follows that the point C must be above the point
¡ ¢ 12 + 42
D 2.5, 2.52 = 6. 25 . From the graph below, we clearly sees that = 8.5
µ ¶2 2
1+4
(which is the height of the point C) is greater than = 6.25 (which
2
is the height of the point D). This is an example where the mean of a convex
function is greater than the function of the average.

µ ¶2
12 + 42 1+4
> , an example of E [f (x)] ≥ f [E (x)].
2 2
20.9. VALUING A CLAIM ON S A 189


Next, let’s consider a concave function
Ãy = x. Consider two points!A (1, 1)
√ √
1+4 1+ 4
and B (4, 2). The mid point of AB is C = 2.5, = 1.5 . Since
2 2
µ ¶
√ 1+4 √
y = x curves down, then it follows that C must be lower than D = 2.5, 2.5 = 1. 58 .
√ √ 2
1+ 4
From the graph below, we clearly sees that = 1.5 (which is the height
r 2
1+4
of the point C) is less than = 1. 58 (which is the height of the point
2
D). This is an example where the mean of a concave function is less than the
function of the average.

√ √ r
1+ 4 1+4
< , an example of E [f (x)] ≤ f [E (x)]
2 2

By now you should have intuitive feel of Jensen’s inequality. Let’s move on.
190 CHAPTER 20. BROWNIAN MOTION AND ITO’S LEMMA

The textbook considers the following examples: A = −1, 0, 1.


If A = 1, the time 0 value of the claim S (T ) at T is:
P
V (0) = F0,T [S (T )] = e−rT S (0) e[(r−δ)]T = S (0) e−δT
This is just DM Equation 5.4.

If A = 0, then the claim is just one dollar: S 0 (T ) = 1


P
→ V (0) = F0,T [1] = e−rT
So the time 0 value of getting $1 at T is e−rT . V (0) is equal $1 discounted
back to time 0 at the risk-free rate.

If A = 2, the time 0 value of the claim S 2 (T ) at T is:


£ 2 ¤ 2
P
V (0) = F0,T S (T ) = e−rT S 2 (0) e[2(r−δ)+σ ]T
As a general rule, the forward price of any asset at T is just the prepaid
forward price accumulating at the risk-rate from time 0 to T :
P
F0,T = F0,T erT

We use the risk-free rate r in the above equation. To get an asset at T , we


P
as a buyer can either pay the seller F0,T at time 0 or pay the seller F0,T at time
T . To avoid the arbitrage, the two payments should differ only in timing. So
P
F0,T = F0,T erT .
£ 2 ¤ £ 2 ¤ 2
→ F0,T S (T ) = F0,T P
S (T ) erT = S 2 (0) e[2(r−δ)+σ ]T
P
F0,T [S (T )] = F0,T [S (T )] erT = S (0) e−δT erT = S (0) e(r−δ)T
£ 2 ¤ 2
→ F0,T S (T ) = S 2 (0) e[2(r−δ)+σ ]T = (F0,T [S (T )])2 eσ T
2

2 £ ¤
Since eσ T ≥ 1, we have F0,T S 2 (T ) ≥ (F0,T [S (T )])2 . This agrees with
Jensen’s inequality. Roughly speaking 1 , F0,T = E (ST ).

d2 2
S 2 is twice differentiable and S = 2 > 0. Hence S 2 is convex.
dS £ ¤ 2
According to Jensen’s inequality, we have E S 2 (T ) ≥ (E [S (T )]) . This
£ 2 ¤ £ 2 ¤ 2 2
leads to F0,T S (T ) = E S (T ) ≥ (F0,T [S (T )]) = (E [S (T )]) .

1
If A = −1, the time 0 value of the claim at T is:
∙ ¸ S (T )
P 1 1 [−(r−δ)+σ2 ]T 1 2
V (0) = F0,T = e−rT e = e−rT (r−δ)T
eσ T
∙ S
¸ (T ) ∙ S (0)
¸ S (0) e
1 P 1 1 2
→ F0,T = F0,T erT = eσ T
S (T ) S (T ) S (0) e(r−δ)T
(r−δ)T
However,
∙ F0,T [S¸ (T )] = S (0) e
1 1 2 1
→ F0,T = eσ T ≥
S (T ) F0,T [S (T )] F0,T [S (T )]
1 Experts don’t agree whether F
0,T = E (ST ). Some say the forward price is the unbiased
estimate of the expected future spot price, that is, F0,T = E (ST ). Others disagree. However,
it’s safe to see that F0,T is very close to E (ST ).
20.9. VALUING A CLAIM ON S A 191

1
Since is concave, according to Jensen’s inequality, we have:
S (T )
∙ ¸ ∙ ¸ µ ∙ ¸¶2 µ ∙ ¸¶2
1 1 1 1
F0,T =E ≥ F0,T = E .
S (T ) S (T ) S (T ) S (T )

d2 −1 2
S −1 is twice differentiable and 2
S = 3 > 0. Hence S 2 is convex.
dS S∙ ¸ µ ∙ ¸¶2
1 1
According to Jensen’s inequality, we have E ≥ E . This
S (T ) S (T )
∙ ¸ ∙ ¸ µ ∙ ¸¶2 µ ∙ ¸¶2
1 1 1 1
leads to F0,T =E ≥ F0,T = E .
S (T ) S (T ) pS (T ) S (T )
If A = 0.5, thehptime 0ivalue of p the claim S (T ) at T is:
2
V (0) = F0,TP
S (T ) = e −rT
S (0)e[0.5(r−δ)+0.5×0.5(0.5−1)σ ]T
p 2
= he−rT S i(0)e[0.5(r−δ)−0.125 σ ]T
p p 2 p
→ F0,T S (T ) = S (0)e[0.5(r−δ)−0.125 σ ]T = S (0)e0.5(r−δ)T
p p
σ2 T 2
= S (0) e(r−δ)T e−0.125h =i F0,T [S (T )]e−0.125 σ T
2 p p
Since e−0.125 σ T ≤ 1, we see that F0,T S (T ) ≤ F0,T [S (T )]
This agrees with Jensen’s inequality.
p d2 √ p
S (T ) is twice differentiable. S = −4S −1.5 < 0. Hence S (T ) is
dS 2
concave. hp i hp i p hp i
→ F0,T S (T ) = E S (T ) ≤ F0,T [S (T )] = E (S (T ))

Example 20.9.1.

The price of a stock follows a geometric Brownian motion:


dS (t)
= (0.1 − 0.04) dt + 0.3dZ
S (t)
The current price of the stock is 10.
The continuously compounded dividend yield is 0.04 per year.
The continuously compounded risk-free rate is 0.06 per year.
The seller and the buyer enter a forward contract. The contract requires the
seller to pay the buyer S 2 (5) five years from now.

Calculate

• the prepaid forward price

• the forward price

• γ

• δ∗
192 CHAPTER 20. BROWNIAN MOTION AND ITO’S LEMMA

• The probability that S 2 (5) ≤ 160.

Solution.
dS (t)
= (0.1 − 0.04) dt + 0.3dZ
S (t)
£ A ¤ 2
P
F0,T S (T ) = e−rT S A (0) e[A(r−δ)+0.5A(A−1)σ ]T
£ 2 ¤ 2
P
F0,5 S (5) = e−0.06×5 102 e(2(0.06−0.04)+0.5×2(2−1)×0.3 )5 = 141. 906 8
£ 2 ¤ P
£ ¤
F0,5 S (5) = F0,5 S 2 (5) e0.06×5 = 141. 906 8e0.06×5 = 191. 554 1
γ = r + A£ (α − r) = 0.06 + 2 (0.1 − 0.06)
¤ = 0.14
δ ∗ = γ − A ¡(α − δ) + 0.5A (A − 1) σ 2 ¢
= 0.14 − 2 (0.1 − 0.04) + 0.5 × 2 (2 − 1) × 0.32
= −0.07

It’s OK for δ ∗£to be negative.


£ ¤ ¤
ln S A (t) ∼ N ln S A (0) + A (α − δ) − 0.5Aσ 2 t, A2 σ 2

2
£ 2
£ 2
¤ 2 2
¤
¡ S (5) ∼ N ln 10 + 2 (0.1 2−¢ 0.04) − 0.5 × 2 × 0.3 5, 2 × 0.3 × 5
ln
2 (0.1 −¡0.04) − 0.5 × 2 × 0.3 5 = 0.15¢
ln 102 + 2 (0.1 − 0.04) − 0.5 × 2 × 0.32 5 = ln 100 + 0.15 = 4. 755 17
£ ¤ £ ¤
P S 2 (5) ≤ 160 = P ln S 2 (5) ≤ ln 160 = Φ (z)
ln 160 − 4. 755 17
z= √ = 0.238 5
22 × 0.32 × 5 £ ¤
Φ (z) = 0.594 3 → P S 2 (5) ≤ 160 = 0.594 3

Example 20.9.2.

The price of a stock follows a geometric Brownian motion:


dS (t)
= (0.12 − 0.05) dt + 0.25dZ
S (t)
The current price of the stock is 20.
The continuously compounded dividend yield is 0.05 per year.
The continuously compounded risk-free rate is 0.07 per year.
The seller and the buyer enter a forward contract. The contract requires the
1
seller to pay the buyer four years from now.
S (4)

Calculate

• the prepaid forward price

• the forward price

• γ
20.9. VALUING A CLAIM ON S A 193

• δ∗
1
• The probability that ≤ 0.03.
S (4)

Solution.
dS (t)
= (0.12 − 0.05) dt + 0.25dZ
S (t)
£ ¤ 2
P
F0,T S A (T ) = e−rT S A (0) e[A(r−δ)+0.5A(A−1)σ ]T
∙ ¸
1 1 2
P
F0,4 = e−0.07×4 e(−1(0.07−0.05)+0.5×(−1)(−1−1)×0.25 )4 = 0.04479
∙ S (4) ¸ ∙ 20 ¸
1 P 1
F0,4 = F0,4 e0.07×4 = 0.04479e0.07×4 = 0.05 926
S (4) S (4)
γ = r + A£ (α − r) = 0.07 − 1 (0.12 − 0.07)
¤ = 0.02
δ ∗ = γ − A ¡(α − δ) + 0.5A (A − 1) σ 2 ¢
= 0.02 − −1 (0.12 − 0.05) + 0.5 × (−1) (−1 − 1) × 0.252 = 0.027 5
£ £ ¤ ¤
ln S A (t) ∼ N ln S A (0) + A (α − δ) − 0.5Aσ 2 t, A2 σ 2
∙ ¸
1 1 £ ¤ 2
ln ∼ N ln + −1 (0.12 − 0.05) − 0.5 × (−1) × 0.252 4, (−1) × 0.252 × 4
¡ S (4) 20 ¢
−1 (0.12 − 0.05) − 0.5 × (−1) × 0.252 4 = −0.155
1 £ ¤ 1
ln + −1 (0.12 − 0.05) − 0.5 × (−1) × 0.252 4 = ln −0.155 = −3. 150 7
20 20
∙ ¸ ∙ ¸
1 1
P ≤ 0.03 = P ln ≤ ln 0.03 = Φ (z)
S (4) S (4)
ln 0.03 − (−3. 150 7)
z=q = −0.711 7
2
(−1) × 0.252 × 4
∙ ¸
1
Φ (z) = 0.238 3 →P ≤ 0.03 = 0.238 3
S (4)

Example 20.9.3.
The price of a stock follows a geometric Brownian motion:
dS (t)
= (0.15 − 0.04) dt + 0.35dZ
S (t)
The current price of the stock is 10.
The continuously compounded dividend yield is 0.04 per year.
The continuously compounded risk-free rate is 0.08 per year.
The seller and the buyer
p enter a forward contract. The contract requires the
seller to pay the buyer S (6) six years from now.

Calculate
194 CHAPTER 20. BROWNIAN MOTION AND ITO’S LEMMA

• the prepaid forward price


• the forward price
• γ
• δ∗
p
• The probability that S (6) ≤ 4.

Solution.
dS (t)
= (0.15 − 0.04) dt + 0.35dZ
S (t)
£ ¤ 2
P
F0,T S A (T ) = e−rT S A (0) e[A(r−δ)+0.5A(A−1)σ ]T
hp i √ 2
P
F0,6 S (6) = e−0.08×6 10e(0.5(0.08−0.04)+0.5×0.5(0.5−1)×0.35 )6 = 2. 012 6
hp i hp i
P
F0,6 S (6) = F0,6 S (6) e0.08×6 = 2. 012 6e0.08×6 = 3. 252 5
γ = r + A£ (α − r) = 0.08 + 0.5 (0.15 −¤0.08) = 0.115
δ ∗ = γ − A (α¡ − δ) + 0.5A (A − 1) σ 2 ¢
= 0.115 − 0.5 (0.15 − 0.04) + 0.5 × 0.5 (0.5 − 1) × 0.352 = 0.075 3
£ £ ¤ ¤
ln S A (t) ∼ N ln S A (0) + A (α − δ) − 0.5Aσ 2 t, A2 σ 2
p £ √ £ ¤ ¤
ln
¡ S (6) ∼ N ln 10 + 0.5 (0.15 −¢0.04) − 0.5 × 0.5 × 0.352 5, 0.52 × 0.352 × 6
0.5 (0.15 − 0.04) − 0.5 × 0.5 × 0.352 6 = 0.146 25
√ £ ¤ √
ln 10 + 0.5 (0.15 − 0.04) − 0.5 × 0.5 × 0.352 5 = ln 10 + 0.146 25 = 1.
297 5
hp i h p i
P S (6) ≤ 4 = P ln S (6) ≤ ln 4 = Φ (z)
ln 4 − 1. 297 5
z=√ = 0.207 1
0.52 × 0.352 × 6 h i
p
Φ (z) = 0.582 0 →P S (6) ≤ 4 = 0.582 0
Chapter 24

Interest rate models

24.1 Market-making and bond pricing

24.1.1 Review of duration and convexity

Duration and convexity are explained in Derivatives Markets Section 7.3. Sec-
tion 7.3 is excluded from the MFE syllabus. However, Derivatives Markets page

781 mentions the duration hedging:

...hedging a bond portfolio based on duration does not result in a perfect


hedge. Recall that the duration of a zero-coupon bond is the bond’s time to
maturity...

Because Chapter 24 mentions duration and Chapter 24 is on the syllabus,


SOA may test your knowledge about duration hedging. So let’s take a quick
review of duration (and convexity).
I attached my Exam FM study guide on duration and convexity to the end
of this pdf file. Please read my explanations.
Next, let’s solve a few problems.

Example 24.1.1.
The yield to maturity of a 5-year zero-coupon bond is 6%. The face amount
of the bond is 100.
Calculate

• the value of the bond


• the Macaulay duration

195
196 CHAPTER 24. INTEREST RATE MODELS

• the modified duration


• the convexity
In addition, calculate the bond value if the yield to maturity
• moves up to 7%
• moves down to 5%.

Solution.

The bond value is P0 = 100 × 1.06−5 = 74. 725 8


The (Macaulay) duration of a zero-coupon bond is its maturity T . So the
(Macaulay) duration is
DMac = T = 5 years.
The modified duration is
DMac 5
Dmod = = = 4. 716 98 years
1 + yield 1.06
The convexity of a zero-coupon bond is:
T (T + 1) 5×6
C= = = 26. 699 893
(1 + y)2 1.062

If the yield to maturity is now 7%, the bond value is:


P1 = 100 × 1.07−5 = 71. 298 6

We can also use duration and convexity to approximate the bond value under
the new yield.
The new bond value µis ¶
∆P
P1 = P0 + ∆P = P0 1 +
P0
∆P 1 2 1 2
= −Dmod ∆y+ C (∆y) = −4. 716 98×0.01+ ×26. 699 893×(0.01) =
P0 2 2
−0.045 8

→ P1 = 74. 725 8 (1 − 0.045 8) = 71. 303 3


This is very close to the correct amount of 71. 298 6.

If the yield to maturity is now 5%, the bond value is:


P1 = 100 × 1.05−5 = 78. 352 6
We can also use duration and convexity to approximate the bond value under
the new yield.
∆P 1 2 1
= −Dmod ∆y + C (∆y) = −4. 716 98 × (−0.01) + × 26. 699 893 ×
P0 2 2
(−0.01)2 = 0.048 5

→ P1 = 74. 725 8 (1 + 0.048 5) = 78. 350 0


This is very close to the correct amount of 78. 352 6.
24.1. MARKET-MAKING AND BOND PRICING 197

Example 24.1.2.

The yield to maturity of a 4-year zero-coupon bond is 8%. The face amount
of the bond is 100.
Calculate

• the value of the bond

• the Macaulay duration

• the modified duration

• the convexity

In addition, calculate the bond value if the yield to maturity

• moves up to 9%

• moves down to 7%.

Solution.

The bond value is P0 = 100 × 1.08−4 = 73. 5030


DMac = T = 4 years.
DMac 4
Dmod = = = 3. 703 7 years
1 + yield 1.08
T (T + 1) 4×5
C= = = 17. 468 8
(1 + y)2 1.072
If the yield to maturity is now 9%, the bond value is:
P1 = 100 × 1.09−4 = 70. 842 5
We can also use duration and convexity to approximate the bond value under
the new yield.
The new bond value µis ¶
∆P
P1 = P0 + ∆P = P0 1 +
P0
∆P 1 2 1 2
= −Dmod ∆y + C (∆y) = −3. 703 7 × 0.01 + × 17. 468 8 × (0.01) =
P0 2 2
−0.036 2
→ P1 = 73. 5030 (1 − 0.036 2) = 70. 842 2
This is very close to the correct amount of 70. 842 5.

If the yield to maturity is now 7%, the bond value is:


P1 = 100 × 1.07−4 = 76. 289 5
We can also use duration and convexity to approximate the bond value under
the new yield.
198 CHAPTER 24. INTEREST RATE MODELS

∆P 1 1
= −Dmod ∆y + C (∆y)2 = −3. 703 7 × (−0.01) + × 17. 468 8 ×
P0 2 2
(−0.01)2 = 0.037 9

→ P1 = 73. 5030 (1 + 0.037 9) = 76. 288 8


This is very close to the correct amount of 76. 289 5.

Example 24.1.3.

Portfolio A consists of one unit of 4-year zero coupon bond.


Portfolio B consists of x units of 1-year zero coupon bond and y units of
8-year zero coupon bond. Portfolio B is used to duration hedge Portfolio A.
Each of the three zero-coupon bonds above has 100 face amount.

The current annual effective interest rate is 5% for all three bonds.

Assume that the yield curve changes by a uniform amount if there’s a change.
Demonstrate why duration-hedging leads to arbitrage under two scenarios:

• the annual effective interest rate moves up to 6% immediately after the


three bonds are issued.

• the annual effective interest rate instantly moves down to 4% immediately


after the three bonds are issued.

Solution.

To duration hedge Portfolio A, we need to satisfy two conditions:

• Portfolio A and B have the same present value

• Portfolio A and B have the same duration

So we set up the following equations:


100 100 100
x+ y=
1.051 1.058 1.054
100 100
x y
1.051 ×1+ 1.058
×8=4
100 100 100 100
x + y x + y
1.051 1.058 1.051 1.058

This is the meaning of the second equation. Portfolio B consists of two


bonds. If a portfolio consists of multiple bonds, then the portfolio’s duration is
just the weighted average of the each bond’s duration, with weight equal to the
present value of each bond.
24.1. MARKET-MAKING AND BOND PRICING 199

µ ¶ µ ¶
100 100 100
The 2nd equation can be simplified as 1
x 1+ 8
y 8 = 4×
1.05 1.05 1.054

100 100 100


1
x+ 8
y=
µ
1.05 ¶1.05 µ 1.05¶4 → x = 0.493 62 , y = 0.520 93
100 100 100
1
x 1+ 8
y 8=4×
1.05 1.05 1.054

So Portfolio B needs to consist of 0.493 62 unit of 1-year zero coupon bond


and 0.520 93 units of 8-year zero coupon bond.

If the interest rate moves up to 6%:


100
Portfolio A is worth: = 79. 2094
1.064

100 100
Portfolio B is worth: × 0.493 62 + × 0.520 93 = 79. 251 7
1.061 1.068
To arbitrage, at t = 0 (when the interest rate is 5%), we buy low and sell
high:

100 100 100


• buy Portfolio B. We pay x+ y= = 82. 270 2
1.051 1.058 1.054
100
• sell Portfolio A. We receive = 82. 270 2
1.054

So our net cost is zero.

Then instantly later at t = 0+ , the interest rate moves up to 6%. Then


Portfolio B (our asset) is worth 79. 251 7; Portfolio A (our liability) is worth 79.
2094.
Our net profit is 79. 251 7 − 79. 2094 = 0.042 3
If the interest rate moves down to 4%:
100
Portfolio A is worth: = 85. 480 4
1.044

100 100
Portfolio B is worth: 1
× 0.493 62 + × 0.520 93 = 85. 527 3
1.04 1.048
To arbitrage, at t = 0 (when the interest rate is 5%), we buy low and sell
high:

100 100 100


• buy Portfolio B. We pay 1
x+ 8
y= = 82. 270 2
1.05 1.05 1.054
100
• sell Portfolio A. We receive = 82. 270 2
1.054
200 CHAPTER 24. INTEREST RATE MODELS

So our net cost is zero.

Then instantly later at t = 0+ , the interest rate moves down to 4%. Then
Portfolio B (our asset) is worth 85. 527 3; Portfolio A (our liability) is worth 85.
480 4.
Our net profit is 85. 527 3 − 85. 480 4 = 0.046 9

So no matter the interest rate moves up to 6% or moves down to 4%, we can


always make free money by buying Portfolio B and selling Portfolio A.
Why is Portfolio B better than Portfolio A? It turns out B has high convexity.

Convexity of A:
TA (TA + 1) 4×5
CA = 2 = = 18. 1406
(1 + y) 1.052

Convexity of B is just the weighted average convexities of the two bonds,


with weights beingµthe present value¶of the bond.µ ¶
1×2 100 8×9 100
× × 0.493 62 + × × 0.520 93
1.052 1.051 1.052 1.058
CB =
100 100
× 0.493 62 + × 0.520 93
1.051 1.058
µ ¶ µ ¶
1×2 100 8×9 100
× × 0.493 62 + × × 0.520 93
1.052 1.051 1.052 1.058
= = 29. 024 9
100
1.054

CB > CA

If the interest rate moves


µ up by
¶ ∆y, then the present value of a portfolio is:
∆P
P1 = P0 + ∆P = P0 1 +
P0

∆P A 1 ∆P B 1
A
A
= −Dmod ∆y + C A (∆y)2 B
= −Dmod ∆y + C B (∆y)2
P0 2 P0B 2
A B 4 B A
However, Dmod = Dmod = = 3. 809 5 C >C
1.05

∆P B ∆P A
→ >
P0B P0A
Since P0 = P0B
A
→ ∆P B > ∆P A
B A
→ P1 > P1 So Portfolio B always worth more than Portfolio A under
a flat yield curve.
For example, if the interest rate moves up to 6%, then
∆P A 1
= −3. 809 5 × 0.01 + × 18. 1406 × 0.012 = −0.037 2
P0A 2
24.1. MARKET-MAKING AND BOND PRICING 201

∆P B 1
= −3. 809 5 × 0.01 + × 29. 024 9 × 0.012 = −0.036 6
P0B 2

100
P1A = (1 − 0.037 2) = 79. 2098
1.054

100
P1B = (1 − 0.036 6) = 79. 259 2
1.054

P1B − P1A = 79. 259 2 − 79. 2098 = 0.049 4


Key point to remember:

1. Two portfolios can have the same present value, the same duration, but
different convexities.

2. Under the assumption of the parallel shift of a flat yield curve, we can
always make free money by buying the high-convexity portfolio and sell
the low-convexity portfolio.

3. The parallel shift of a flat yield curve assumption leads to arbitrage.

Information about three zero-coupon bonds:


Maturity (Yrs) Face
2 100
4 100
7 100

The current interest rate is 7% for all three bonds. Assume a parallel shift
of a flat yield curve. Design an arbitrage strategy.

We need to form 2 portfolios. These two portfolios have the same PV, the
same duration, but different convexity. We can make free money by buying the
high convexity portfolio and selling the low convexity portfolio.
The low convexity portfolio is the 4-year bond (Portfolio A).
The high convexity portfolio consists of x unit of 2-year bond and y unit
of 7-year bond (Portfolio B). This is called a barbell. A barbell bond portfolio
combines short maturities (low duration) with long maturities (high duration)
for a blended, moderate maturity (moderate duration)

Portfolio Maturity (Yrs) PV Duration Convexity Units


100 4×5
A 4 = 76. 29 4 = 17. 468 8 1
1.074 1.072
100 2×3
B 2 = 87. 34 2 = 5. 240 6 x
1.072 1.072
100 7×8
B 7 = 62. 27 7 = 48. 912 6 y
1.077 1.072
202 CHAPTER 24. INTEREST RATE MODELS

100 100 100


2
x+ 7
y=
µ
1.07 ¶1.07 µ 1.07¶4 x = 0.524 1, y = 0.490 0
100 100 100
2
x 2+ 7
y 7=4× 4
1.07 1.07 1.07
At time 0, we buy Portfolio B (which consists of 0.524 1 unit of 2-year bond
and 0.490 0 unit of 7-year bond). Simultaneously, we sell Portfolio A.
The convexity of Portfolio A is: C A = 17. 468 8

The convexity of Portfolio B is:


2×3 100 7×8 100
2
× 2
× 0.524 1 + 2
× 0.490 0 ×
C B = 1.07 1.07 1.07 1.077 = 22. 708 9
100
1.074

For example, if the new interest rate is 7.25% for all bonds with different
maturities, then
100 100
P1B = × 0.524 1 + 0.490 0 × = 75. 584 1
1.07252 1.07257
100
P1A = = 75. 580 9
1.07254
B A
P1 > P1
Our profit is 75. 584 1 − 75. 580 9 = 0.003 2

If the new interest rate is 6.5% for all bonds with different maturities, then
100 100
P1B = × 0.524 1 + 0.490 0 × = 77. 739 6
1.0652 1.0657
100
P1A = = 77. 732 3
1.0654
B A
P1 > P1
Our profit is 77. 739 6 − 77. 732 3 = 0.007 3

Either way, we make money.


By now you should see that the parallel shift of a flat yield curve is a bad
model.

24.1.2 Interest rate is not so simple

We all know what an interest rate is, yet a derivative on interest rate is surpris-
ingly difficult. To get a sense of the difficulty, suppose we want to calculate the
price of a European call on a 1-year zero-coupon bond that pays $100 one year
from now. Here are the inputs:

• The call expires in one year

• The strike price is $100


24.1. MARKET-MAKING AND BOND PRICING 203

• The continuous risks-free rate is r = 6% per year

• The current price of the bond is 100e−0.06 = 94. 18.

• The volatility of the bond return is σ = 30%

Using the Black-Scholes formula, we find:


d1 = 0.15 d2 = −0.15
N (d1 ) = 0.5596 N (d2 ) = 0.4404
→ C = 94. 18 × 0.5596 − 94. 18 × 0.4404 = 11. 23

Everything looks fine. However, after further thinking, you realize that the
call price C should be zero. At T = 1, the bond pays $100. Hence the 100-strike
call value is zero. Why should anyone buy a 100-strike call on an asset worth
$100 at call expiration?
What went wrong? It turns out that we can’t use the Black-Scholes formula
to calculate the price of an interest rate derivative:

• The Black-Scholes option formula assume that the term structure of the
interest rate is flat and deterministic. If you look at DM page 379, you’ll
see that one assumption in the Black-Scholes formula is that the risk-free
rate is known and constant. If the interest rate is known and constant,
there won’t be any need for interest rate derivatives. This is similar to
the idea that if the stock price is known and constant, there won’t be any
need for call or put option.

• The standard deviation of the return σ is a constant. DM 379 also states


that another assumption is that the volatility of continuously compounded
returns is known and constant. This doesn’t hold for a bond. Unlike
a stock, a bond has a finite maturity. At the maturity date, the bond
value is its face amount. Hence the standard deviation of a bond’s return
decreases as the bond approaches its maturity.

The key point. It’s much harder to calculate the price of an option interest
rate because interest rate is not a tradeable underlying asset. For call and put
on stocks, we can buy or short sell stocks to set up the hedge portfolio. However,
we can’t go out and buy a 5% interest rate to hedge an option on interest rate.
Now let’s go to the textbook.

24.1.3 Impossible bond pricing model

A bond is a derivative on interest rate. We normally don’t think this way, but
a bond derives its value from interest rate. If the market interest rate goes up,
the bond value goes down; if the market interest rate goes down, the bond value
goes up.
204 CHAPTER 24. INTEREST RATE MODELS

So our starting point is to set up a stochastic random variable called the


short interest rate. The textbook keeps using the phrase "short interest
rate" or "short rate" without giving a clear definition. Here is the definition:
Definition 24.1.1. A short interest rate or short rate is just the instantaneous
interest rate r (t) over a short (hence the name "short rate") interval [t, t + dt].

First, we assume that the short interest rate r (t) follows the Ito’s process:

dr = α (r) dt + σ (r) dZ (24.1)


Next, the text book explains that we can’t assume r (t) follows a flat yield
curve ("impossible bond pricing model").
A flat yield curve means that the interest rate is independent of time. To
have a flat yield curve, the following two conditions are met:

• The initial interest rate r is a constant regarding time, that is, r (t) = t
• If the interest rate changes, the change is also independent of time (i.e.
parallel shift of the yield curve)

If the continuously compounded interest rate r is constant, then the present


value at time t of $1 to be received at T is:

P (t, T ) = e−r(T −t) (24.2)

For example, if a zero coupon bond pays us $1 at T = 2, the PV of this bond


at time zero is P (0, 2) = e−2r . The PV of this bond at t = 1 is P (1, 2) = e−r .
Next, let’s review the textbook’s proof and find out why Equation 24.1 and
24.2 won’t work together.

Suppose we want to delta hedge a bond. Whereas delta hedging a call means
buying ∆ shares of a stock, delta hedging a bond means buying ∆ units of a
bond with a different maturity date.
Suppose at time t we buy a bond maturing at T2 . "A bond maturing at T2 "
just means that we, the bond holder, will receive $1 at T2 . So the price of this
bond at time t is P (t, T2 ) = e−r(T2 −t) .
To delta hedge this bond, at t, we buy ∆ bonds maturing at T1 (please note
the textbook uses N instead of ∆). The cost is ∆P (t, T1 ) = ∆e−r(T1 −t)
The total cost of buying two bonds is ∆P (t, T1 ) + P (t, T2 ) = ∆e−r(T1 −t) +
−r(T2 −t)
e . To avoid tying up our capital, at time t we borrow ∆e−r(T1 −t) +
−r(T2 −t)
e from a bank to finance the purchase of two bonds. So at time t, our
portfolio is:

• buy 1 bond maturing at T2


• buy ∆ bonds maturing at T1
24.1. MARKET-MAKING AND BOND PRICING 205

• borrow e−r(T2 −t) + ∆e−r(T1 −t) from a bank

Our net position is zero:


£ I = ∆P (t, T1 ) + P (t, T ¤ 2 ) + W = 0 where W = − [∆P (t, T1 ) + P (t, T2 )] =
− ∆e−r(T1 −t) + e−r(T2 −t) . As seen before, if we borrow money, we use a neg-
ative number. If we lend money, we use the a positive number. This is why W
is negative.
In the next instant dt, the interest rate moves up to r + dr. Now change of
our portfolio value is:
dI = ∆ × dP (t, T1 ) + dP (t, T2 ) + dW
Using Ito’s lemma, we have:
2
dP (t, T1 ) = ∂P (t,T
∂t
1)
dt + ∂P (t,T
∂r
1)
dr + 12 ∂ P∂r(t,T
2
1)
(dr)2
∂P (t,T1 ) ∂ −r(T1 −t)
∂t = ∂t e = rP (t, T1 )
∂P (t,T1 ) ∂ −r(T1 −t)
∂r = ∂r e = −P (t, T1 ) (T1 − t)
∂ 2 P (t,T1 )
∂ 2
−r(T1 −t) 2 2
∂r2= ∂r 2e = er(t−T1 ) (T1 − t) = P (t, T1 ) (T1 − t)
2 2 2
(dr) = (αdt + σdZ) = σ2 (dZ) = σ 2 dt
→ dP (t, T1 ) = rP (t, T1 ) dt − P (t, T1 ) (T1 − t) dr + 12 P (t, T1 ) (T1 − t)2 σ 2 dt

Similarly,
2
dP (t, T2 ) = rP (t, T2 ) dt − P (t, T2 ) (T2 − t) dr + 12 P (t, T2 ) (T2 − t) σ 2 dt

However, dW = rW dt. To see why, notice that dW is the interest earned on


W during the interval [t, t + dt]. During [t, t + dt], the continuous interest rate
r can be treated as a discrete interest rate. The interest earned is just
the initial capital × interest rate × length of the interval = rW dt
Now wehhave DM Equation 24.6: i
dI = ∆ rP (t, T1 ) dt − P (t, T1 ) (T1 − t) dr + 12 P (t, T1 ) (T1 − t)2 σ2 dt
h i
2
+ rP (t, T2 ) dt − P (t, T2 ) (T2 − t) dr + 12 P (t, T2 ) (T2 − t) σ 2 dt
+rW dt

We’ll want to have dI = 0. If dI = 0, then the value of our portfolio won’t


change during the same during [t, t + dt]. To make dI = 0, we first choose ∆
such that the dr term is zero.

→ ∆ [−P (t, T1 ) (T1 − t)] + [−P (t, T2 ) (T2 − t)] = 0

→ ∆ = − (T 2 −t)P (t,T2 )
(T1 −t)P (t,T1 )
(T2 −t)P (t,T2 )
This negative delta means that we need to sell (T1 −t)P (t,T1 ) units of bond
maturing at T1 .

By the way, ∆ = − (T2 −t)P (t,T2 )


(T1 −t)P (t,T1 ) is similar to DM Equation 7.13 (Derivatives
Markets page 227):
206 CHAPTER 24. INTEREST RATE MODELS

D1 B1 (y1 ) / (1 + y1 )
N =− (DM 7.13)
D2 B2 (y2 ) / (1 + y2 )
So buying ∆ bonds is really duration-hedging.
Next,
h we want to set the dt term to zero: i h i
2 2
∆ rP (t, T1 ) + 12 P (t, T1 ) (T1 − t) σ 2 + rP (t, T2 ) + 12 P (t, T2 ) (T2 − t) σ 2 +
rW = 0

→ r [∆P (t, T1 ) + P (t, T2 ) + W ]+ 12 ∆P (t, T1 ) (T1 − t)2 σ 2 + 12 P (t, T2 ) (T2 − t)2 σ2 =


0
However,∆P (t, T1 ) + P (t, T2 ) + W = 0
→ 12 ∆P (t, T1 ) (T1 − t)2 σ2 + 12 P (t, T2 ) (T2 − t)2 σ 2 = 0
(T2 −t)P (t,T2 ) 2 2
→ − 12 (T1 −t)P (t,T1 )
P (t, T1 ) (T1 − t) σ 2 + 12 P (t, T2 ) (T2 − t) σ2 = 0
→ − 12 (T2 − t) P (t, T2 ) (T1 − t) σ 2 + 12 P (t, T2 ) (T2 − t)2 σ 2 = 0
→ − 12 (T2 − t) P (t, T2 ) [(T1 − t) − (T2 − t)] σ2 = 0
→ − 12 (T2 − t) P (t, T2 ) (T1 − T2 ) σ 2 = 0
→ T1 = T2 ∆ = −1
∆ = −1 means that sell one bond.
If we buy a bond maturing in T2 years, the only way to hedge the risk is to
sell this bond! So one moment you buy a bond. The next moment you sell it.
Your net position is zero. Of course you are hedged against all risks, but this
hedging is really doing nothing type of hedging.
This tells us that the bond pricing model based on Equation 24.1 and 24.2
are impossible. In our words, Equation 24.1 is OK. But Equation 24.2 is bad.
This confirms that a parallel shift of a flat yield curve is a bad assumption for
pricing a bond.

24.1.4 Equilibrium equation for bonds

Since it’s bad to assume that r is a flat yield curve, we switch our gears and
assume that r isn’t a flat yield curve. Now we just assume that Equation 24.1
holds.
∂P ∂P 1 ∂2P 2
dP (r, t, T ) = dr + dt + (dr)
∂r ∂t 2 ∂t2
2 2 2
(dr) = [α (r) dt + σ (r) dZ] = σ 2 (r) (dZ) = σ 2 (r) dt
∂P ∂P 1 ∂2P 2 ∂P
→ dP (r, t, T ) = dr + dt + 2
σ (r) dt = [α (r) dt + σ (r) dZ] +
∂r ∂t 2 ∂t ∂r
2
∂P 1∂ P 2
dt + σ (r) dt
∂t 2 ∂t2

∙ ¸
∂P 1 ∂2P 2 ∂P ∂P
dP (r, t, T ) = α (r) + σ (r) + dt + σ (r) dZ (24.3)
∂r 2 ∂t2 ∂t ∂r
24.1. MARKET-MAKING AND BOND PRICING 207

Define
∙ ¸
1 ∂P 1 ∂2P 2 ∂P
α (r, t, T ) = α (r) + σ (r) + (24.4)
P (r, t, T ) ∂r 2 ∂t2 ∂t
1 ∂P
q (r, t, T ) = σ (r) (24.5)
P (r, t, T ) ∂r

Now Equation 24.3 becomes:


dP (r, t, T )
= α (r, t, T ) dt + q (r, t, T ) dZ (24.6)
P (r, t, T )

dP (r, t, T )
is the bond’s return. Equation 24.6 says that the bond’s return
P (r, t, T )
is the sum of a drift term α (r, t, T ) dt and a random component q (r, t, T ) dZ.
∂P
Please also note that generally q (r, t, T ) is negative. This is because is
∂r
negative. If r goes up, the bond price goes down; if r goes down, P goes up.
Next, we are going to derive DM Equation 24.16:

α1 (r, t, T1 ) − r α2 (r, t, T2 ) − r
= (24.7)
q1 (r, t, T1 ) q2 (r, t, T2 )

Suppose we have two bonds, Bond #1 and Bond #2 maturing at T1 and T2


respectively. Each bond follows DM Equation 24.6:
dP (r, t, T1 )
= α1 (r, t, T1 ) dt + q1 (r, t, T1 ) dZ
P (r, t, T1 )

dP (r, t, T2 )
= α2 (r, t, T2 ) dt + q2 (r, t, T2 ) dZ
P (r, t, T2 )

At t = 0, we form a self-financing portfolio consisting of

• buying ∆ units of Bond #1


• buying 1 Bond #2
• borrowing ∆P (r, t, T1 ) + P (r, t, T2 ) from a bank at the riskless short rate
r

The value of this portfolio is I


dP (r, t, T1 ) dP (r, t, T2 )
dI = ∆P (r, t, T1 ) + P (r, t, T2 )
P (r, t, T1 ) P (r, t, T2 )
− [∆P (r, t, T1 ) + P (r, t, T2 )] rdt
= ∆P (r, t, T1 ) [α1 (r, t, T1 ) dt + q1 (r, t, T1 ) dZ]
+P (r, t, T2 ) [α2 (r, t, T2 ) dt + q2 (r, t, T2 ) dZ]−[∆P (r, t, T1 ) + P (r, t, T2 )] rdt
= ∆P (r, t, T1 ) [α1 (r, t, T1 ) − r] dt + P (r, t, T2 ) [α2 (r, t, T2 ) − r] dt
208 CHAPTER 24. INTEREST RATE MODELS

+ [∆P (r, t, T1 ) q1 (r, t, T1 ) + P (r, t, T2 ) q2 (r, t, T2 )] dZ


Choose ∆ such that ∆P (r, t, T1 ) q1 (r, t, T1 )+P (r, t, T2 ) q2 (r, t, T2 ) = 0. This
removes the stochastic random term dZ. Now dI is deterministic. Since the
portfolio is self-financing and riskless, it earns zero interest rate. Hence
∆P (r, t, T1 ) [α1 (r, t, T1 ) − r] dt + P (r, t, T2 ) [α2 (r, t, T2 ) − r] dt = 0
P (r, t, T2 ) q2 (r, t, T2 )
→− P (r, t, T1 ) [α1 (r, t, T1 ) − r] dt+P (r, t, T2 ) [α2 (r, t, T2 ) − r] dt =
P (r, t, T1 ) q1 (r, t, T1 )
0
q2 (r, t, T2 )
→− [α1 (r, t, T1 ) − r] dt + [α2 (r, t, T2 ) − r] dt = 0
q1 (r, t, T1 )
α1 (r, t, T1 ) − r α2 (r, t, T2 ) − r
→ =
q1 (r, t, T1 ) q2 (r, t, T2 )

Define the following term as Sharp ratio:

α (r, t, T ) − r
= φ (r, t) (24.8)
q (r, t, T )

Apply Equation
∙ 24.4 and 24.6 to 24.8: ¸
1 ∂P 1 ∂2P 2 ∂P
α (r) + σ (r) + −r
P (r, t, T ) ∂r 2 ∂t2 ∂t
= φ (r, t)
1 ∂P
σ (r)
P (r, t, T ) ∂r
∙ ¸
∂P 1 ∂2P 2 ∂P ∂P
→ α (r) + σ (r) + − rP = φ (r, t) σ (r)
∂r 2 ∂t2 ∂t ∂r
1 2 ∂2P ∂P ∂P
σ (r) 2 + [α (r) − σ (r) φ (r, t)] + − rP = 0 (24.9)
2 ∂r ∂r ∂t
Any interest-dependent securities (not just zero coupon bonds) must satisfy
Equation 24.9.

To solve the bond price P (r, t, T ), we need to use Equation 24.9 together
with the following boundary condition:

P (r, T, T ) = 1 (24.10)

The solution to Equation 24.9 and 24.10 is:

à " Z #!
T
P (r, t, T ) = Et∗ exp − r (s) ds = Et∗ (exp [−R (t, T )]) (24.11)
t

where E ∗ means that the expectation is based on the risk neutral probability
and Z T
R (t, T ) = r (s) ds (24.12)
t
24.2. EQUILIBRIUM SHORT-RATE BOND PRICE MODELS 209

Equation 24.11 is the general formula for a bond price.

24.1.5 Delta-Gamma approximation for bonds


The key formula is:
1 ∗
E (dP ) = rP (24.13)
dt
Don’t worry about how to prove Equation 24.13. Just memorize its meaning.
Equation 24.13 says that under the risk neutral distribution, the bond is priced
to earn a risk-free rate.

24.2 Equilibrium short-rate bond price models


24.2.1 Arithmetic Brownian motion (i.e. Merton model)

A simple model is to assume that the short rate r (t) follows arithmetic Brownian
motion:

dr (t) = αdt + σdZ (24.14)

In addition, we assume φ (r, t) = φ


Advantage:

• The model is simple.

Disadvantages:

• r (t) can go negative. Since r (t) is normally distributed with mean r0 + αt


and variance σ 2 t, r (t) can be negative. A negative interest rate won’t
make any sense.
• r (t) is not mean reverting.
• V ar [r (t)] = σ 2 t. This is undesirable because the volatility of the interest
should depend on the interest rate. If the interest rate is high, then the
volatility is high.

We can derive the bond price under the Merton model. Under this model,
Equation 24.9 now becomes:
1 2 ∂2P ∂P ∂P
2 σ ∂r2 + [α − σφ] ∂r + ∂t − rP = 0
We guess the solution is P (r, t, T ) = A (T − t) e−B(T −t)r(t) where A (T − t)
and B (T − t) are two functions of T − t.
∂P ∂2P ∂P 0
= −ABe−Br 2
= AB 2 e−Br = rAB e−Br − A0 e−Br
∂r ∂r 0
∂t
→ 12 σ 2 AB 2 e−Br − [α − σφ] ABe−Br + rAB e−Br − A0 e−Br − rAe−Br = 0
210 CHAPTER 24. INTEREST RATE MODELS

1 2 ³ 0
´
σ AB 2 e−Br − [α − σ] ABe−Br − A0 e−Br = rA e−Br − B e−Br (24.15)
2

Equation 24.15 should hold for any r. The only way to make it work for any
r is:
1 2 2 0
2 σ³ AB −´[α − σφ] AB − A = 0
0
A 1−B =0
Using the boundary condition is P (r, T, T ) = 1, we get A (0) e−B(0)r = 1.
For this equation
³ to hold
´ for any r, we need to have A (0) = 1 and B (0) = 0.
0
Hence from A 1 − B = 0, we get:
0
B =1 →B =T −t
From 12 σ 2 AB 2 − [α − σφ] AB − A0 = 0. we get:
A0
= 12 σ 2 B 2 − [α − σφ] B = 12 σ 2 (T − t)2 − [α − σφ] (T − t)
A
→ d ln A (T − t) = 12 σ 2 (T − t)2 − (α − σφ) (T − t)
3 1 2
→ ln A (T − t) = 16 σ 2 (T − t) − (α − σφ) (T − t) + C where C is a con-
2
stant. ∙ ¸
1 2 3 1 2
→ A = A (0) exp 6 σ (T − t) − (α − σφ) (T − t)
∙ 2 ¸
1 2 3 1 2
= exp 6 σ (T − t) − (α − σφ) (T − t)
2
Finally, we have:∙ ¸
1 2 3 1 2
P (r, t, T ) = exp 6 σ (T − t) − (α − σφ) (T − t) e−(T −t)r(t)
2
You don’t need to memorize the above formula. Just understand how to
derive the formula in case SOA or CAS gives you a tough problem.

24.2.2 Rendleman-Bartter model


This model assume that r (t) follows a geometric Brownian motion:

dr (t)
= αdt + σdZ (24.16)
r (t)
Advantage:

• r (t) can’t be negative


• V ar [r (t)] = r2 (t) σ2 t. The variance increases if r (t) increases. This is
desirable because the volatility of the interest is high if the interest rate
is high.
Disadvantage:
• not mean-reverting
24.2. EQUILIBRIUM SHORT-RATE BOND PRICE MODELS 211

24.2.3 Vasicek model


According to Wikipedia, Vasicek’s model was the first one to capture mean re-
version, an essential characteristic of the interest rate that sets it apart from
other financial prices. Stock prices can rise indefinitely. However, interest rates
cannot. Excessively high interest rate would hamper economic activity, prompt-
ing a decrease in interest rates. Similarly, interest rates can not decrease indef-
initely. If interest rates are too low, few people are willing to lend their money.
This tends to push up the interest rate. As a result, interest rates move in a
limited range, showing a tendency to revert to a long run value.
The model is:

dr (t) = α (b − r) dt + σdZ (24.17)


2
The variance of r (t) is σ t
Advantage:

• mean reverting

Disadvantage:

• can produce a negative interest rate

• V ar [r (t)] = σ 2 t. So the variance doesn’t increase if r (t) increases.

We can also derive the bond price P (t, T ) under the Vasicek model. How-
ever, the derivation is far more complex than in the Merton model. Here is the
outline of the derivation.
Under the Vasicek model, Equation 24.9 becomes:
1 2 ∂2P ∂P ∂P
2 σ ∂r2 + [α (b − r) − σφ] ∂r + ∂t − rP = 0
subject to the boundary condition P (r, T, T ) = 1
Once again, we guess the solution is P (r, t, T ) = A (T − t) e−B(T −t)r(t)
∂P ∂2P ∂P 0
→ = −ABe−Br = AB 2 e−Br = rAB e−Br − A0 e−Br
∂r ∂r2 ∂t0
→ 12 σ 2 AB 2 e−Br −[α (b − r) − σφ] ABe−Br +rAB e−Br −A0 e−Br −rAe−Br =
0
0
→ 12 σ 2 AB 2 − [α (b − r) − σφ] AB + rAB − A³0 − rA = 0 ´
0
→ 12 σ 2 AB 2 − (αb − σφ) AB + σφAB − A0 = A − AB − aAB r
For the above equation to hold for any r, we need to have:
0
A − AB − αAB = 0
1 2 2 0
2 σ AB − (αb − σφ) AB + σφAB − A = 0

0 0
A − AB − αAB = 0 → 1 − B − αB = 0
The boundary condition is B (0) = 0
1 − e−α(T −t)
This gives us: B =
α
212 CHAPTER 24. INTEREST RATE MODELS

Equation 12 σ 2 AB 2 − (αb − σφ) AB + σφAB − A0 = 0 is hard to solve.


Someone solved this equation for us. The result is DM Equation 24.26. You
don’t need to memorize the solution for A or B. Just memorize P (r, t, T ) =
A (T − t) e−B(T −t)r(t) .
Example 24.2.1. May 2007 SOA MFE #13
Let P (r, t, T ) denote the price at time t of $1 to be paid with certainty at
time T , t ≤ T , if the short rate at time t is equal to r. For a Vasicek model
you are given:
P (0.04, 0, 2) = 0.9445
P (0.05, 1, 3) = 0.9321
P (r∗ , 2, 4) = 0.8960
Calculate r∗ .
The price of any bond must satisfy Equation 24.9. We guess that the solution
to Equation 24.9 is P (r, t, T ) = A (T − t) e−B(T −t)r(t)
P (0.04, 0, 2) = 0.9445 → A (2) e−B(2)0.04 = 0.9445
P (0.05, 1, 3) = 0.9321 → A (2) e−B(2)0.05 = 0.9321
−B(2)0.04
A (2) e 0.9445 0.9445
→ −B(2)0.05
= eB(2)0.01 =
A (2) e 0.9321 0.9321
µ ¶ 1 µ ¶100
0.9445 0.01 0.9445
→ eB(2) = = = 3. 749 26
0.9321 0.9321
£ ¤0.04 ¡ ¢
→ A (2) = 0.9445eB(2)0.04 = 0.9445 eB(2) = 0.9445 3. 749 260.04 =
0.995 77
P (r∗ , 2, 4) = 0.8960
∗ £ ¤−r∗
→ A (2) e−B(2)r = A (2) eB(2) = 0.8960
¡ ∗ ¢
→ 0.995 77 3. 749 26−r = 0.8960
¡ ∗¢ 0.8960
→ 3. 749 26−r = = 0.899 81
0.995 77
→ −r∗ ln 3. 749 26 = ln 0.899 81
ln 0.899 81
→ r∗ = − = 0.07988 4 = 0.08
ln 3.

749 26
Please note r can be solved without using any specifics of the Vasicek model.
As a matter of fact, this problem can be rewritten as:
Let P (r, t, T ) denote the price at time t of $1 to be paid with certainty at
time T , t ≤ T , if the short rate at time t is equal to r. You are given:
P (0.04, 0, 2) = 0.9445
P (0.05, 1, 3) = 0.9321
P (r∗ , 2, 4) = 0.8960
Calculate r∗ .

24.2.4 CIR model


The model assumes that the short interest rate r (t) follows the stochastic dif-
ferential equation: p
dr (t) = α (b − r) dt + σ r (t)dZ (24.18)
24.3. BOND OPTIONS, CAPS, AND THE BLACK MODEL 213

In CIR model, V ar [r (t)] = σ 2 r (t) t.


The drift factor α (b − r) in the CIR model is the same as the drift factor
in the Vasicek model. It ensures mean reversion of the interest rate towards
the long run value b, with speed of adjustment governed by the strictly positive
parameter a. p
The standard deviation factor, σ r (t), corrects the main drawback of Va-
sicek’s model, ensuring that the interest rate cannot become negative. Thus,
at low values of the interest rate, the standard deviation becomes close to zero,
canceling the effect of the random shock on the interest rate. Consequently,
when the interest rate gets close to zero, its evolution becomes dominated by
the drift factor, which pushes the rate upwards.
Advantage:

• Mean-reverting
• Not allow a negative interest rate
• Variance of V ar [r (t)] = σ2 r (t) t. The higher the r (t), the higher the
V ar [r (t)]. This is a desirable feature.

To solve the bond price under CIR model, again we guess the solution is
P (r, t, T ) = A (T − t) e−B(T −t)r(t) . The solution is listed in DM page 788. You
don’t need to memorize the solution. Just memorize P (r, t, T ) = A (T − t) e−B(T −t)r(t) .

24.3 Bond options, caps, and the Black model


24.3.1 Black formula
Notations:

• Pt (T, T + s). This is the price agreed upon at time t, which will be paid at
T in order to receive $1 at T + s. Simply put, Pt (T, T + s) is the present
value of $1 discounted from T + s to T using the interest rate available
at t. For example, at time zero we know that the annual interest rate
from t = 1 to t = 3 is 10% compounded continuously. Then P0 (1, 1 + 2)
is just PV of $1 discounted from t = 3 to t = 1 using 10% continuously
compounded interest rate. So P0 (1, 1 + 2) = e−0.1(2) = 0.818 73
• PT (T, T + s). This is PV $1 discounted from T + s to T . Generally, we
simplify the symbol PT (T, T + s) as P (T, T + s)

Consider a call option with strike price K, expiring at time T , on a zero-


coupon bond paying $1 at time T + s. If you buy this call option, then at T you
have the right to buy a bond maturing at time T + s for the guaranteed price
K. Since time T cost of a bond maturing in T + s is PV of $1 discounted from
T + s to T , buying a bond maturing at time T + s for the guaranteed price K
really means "give up K and receive PV of $1 discounted from T + s to T ." The
PV of $1 discounted from T + s to T is PT (T, T + s). So the call payoff is
214 CHAPTER 24. INTEREST RATE MODELS

Call P ayof f = max [0, PT (T, T + s) − K] (DM 24.30)


Next, the textbook has a difficult formula:

P (t, T + s)
Ft,T [P (T, T + s)] = (DM 24.31)
P (t, T )
To understand the meaning of DM 24.31, use an example. Suppose t = 0,
T = 1, and s = 2. In addition, assume that the interest rate is always 10%
compounded continuously.

• Ft,T [P (T, T + s)] = F0,1 [P (1, 3)] is the price agreed upon at t = 0 to be
paid at T = 1 in order to receive $1 at T + s = 3. In other words, if you
pay F0,1 [P (1, 3)] at T = 1, you should receive PV of $1 discounted from
T + s = 3 to T = 1. To avoid arbitrage, F0,1 [P (1, 3)] = e−0.1(2) = e−0.02

• P (t, T + s) = P (0, 3) is PV of $1 discounted from T + s = 3 to time zero.


P (0, 3) = e−0.1(3) = e−0.03

• P (t, T ) = P (0, 1) is PV of $1 discounted from T = 1 to time zero.


P (0, 1) = e−0.1
−0.03
Clearly we see that e−0.02 = ee−0.1 .
Next, the textbook gives us the price of the call on a bond. Consider a call
option is written at time zero with strike price K, expiring at time T , on a
zero-coupon bond paying $1 at time T + s. If you buy this call option at time
zero, then at T you have the right to pay K and receive the PV of $1 discounted
from T + s to T .

To find the call price, we’ll use DM Equation 12.5:


P P
C = F0,T (S) N (d1 ) − F0,T (K) N (d2 )
P (S)
F0,T
ln +0.5σ 2 T √
F P (K)
0,T
d1 = √
σ T
d2 = d1 − σ T

This is how I memorize DM Equation 12.5:


C =Time zero cost of what you get at T ×N (d1 ) −Time zero cost of what
you give³at T ×N (d2 ) ´ √
Tim e zero cost of what you get at T 2
d1 = ln Time zero cost of what you give at T + 0.5σ T /σ T


d2 = d1 − σ T

Let’s apply DM Equation 12.5 to call on bond.

• What we get at T is P (T, T + s), PV of $1 discounted from T + s to T


24.3. BOND OPTIONS, CAPS, AND THE BLACK MODEL 215

• Time zero cost of P (T, T + s) is P (0, T + s), PV of $1 discounted from


T + s to time 0
• What we give at T is K
• Time zero cost of what we give at T is KP (0, T ), PV of K discounted
from T to time 0

C =Time zero cost of what you get at T ×N (d1 ) −Time zero cost of what

you give³at T ×N (d2 ) ´ √


Time zero cost of what you get at T 2
d1 = ln Time zero cost of what you give at T + 0.5σ T /σ T

d2 = d1 − σ T

This gives us the price of the call on a bond:

C = P (0, T + s) N (d1 ) − P (0, T ) KN (d2 )

ln P (0,T +s) 2
P (0,T ) +0.5σ T

d1 = √
σ T
d2 = d1 − σ T

We can also calculate the price of the put on the bond:


P = P (0, T ) KN (−d2 ) − P (0, T + s) N (−d1 )

Please note that DM Equation 24.32 uses the following formula:


C = P (0, T ) [F N (d1 ) − KN (d2 )]
F 2
ln K +0.5σ T
d1 = √
σ T

This is how to derive these formulas. Notice, F = F0,T [P (T, T + s)] =


P (0,T +s)
P (0,T )
→ C = P (0, T + s) N (d1 ) − P (0, T ) KN (d2 )
= F P (0, T ) N (d1 ) − P (0, T ) KN (d2 )
= P (0, T ) [F N (d1 ) − KN (d2 )]
Example 24.3.1. SOA May 2007 MFE #7
You are given the following information:

Bond maturity (years) 1 2


Zero-coupon bond price 0.9434 0.8817
A European call option, that expires in 1 year, gives you the right to purchase
a 1-year bond for 0.9259.
The bond forward price is lognormally distributed with volatility σ = 0.05.
Using the Black formula, calculate the price of the call option.
216 CHAPTER 24. INTEREST RATE MODELS

Solution.
If you buy this option, then at T = 1, you can pay K = 0.9259 and buy a
1-year bond. This 1-year bond will give you $1 at time T + s = 2. .

• The value of this bond at T = 1 is PV of $1 discounted from T + s = 2 to


T = 1.Time zero cost of PV of $1 discounted from T + s = 2 to T = 1 is
just PV of $1 discounted from T + s = 2 to t = 0. The cost is 0.8817
• Time zero cost of the strike price K at T = 1 is just PV of K discounted
from T = 1 to time zero. So P V (K) = K × 0.9434 = 0.9259 × 0.9434

C =Time zero cost of what you get at T ×N (d1 ) −Time zero cost of what
you give³at T ×N (d2 ) ´ √
Tim e zero cost of what you get at T 2
d1 = ln Time zero cost of what you give at T + 0.5σ T /σ T

0.8817 2
ln 0.9259×0.9434 +0.5×0.05 ×1
→ d1 = √ = 0.212 0
√ 0.05 1 √
→ d2 = d1 − σ T = 0.212 0 − 0.05 1 = 0.162 0

N (d1 ) = 0.583 9 N (d2 ) = 0.564 3


C = 0.8817 × 0.583 9 − 0.9259 × 0.9434 × 0.564 3 = 0.021 9

We can also calculate the put price.


P =Time zero cost of what you give at T ×N (−d2 ) −Time zero cost of what
you get at T ×N (−d1 )
P = 0.9259 × 0.9434 × (1 − 0.564 3) − 0.8817 × (1 − 0.583 9) = 0.013 7

If you want to use the formula C = P (0, T ) [F N (d1 ) − KN (d2 )], this is
how:

• P (0, T ) = P (0, 1) is PV of $1 discounted from T = 1 to time zero. So


P (0, 1) = 0.9434
• F = F0,T [P (T, T + s)] = F0,1 [P (1, 2)] is the forward price of 1-year bond.
This is the price agreed up at time zero, paid at T = 1 in order to receive
$1 at T + s = 2. Using Equation DM 24/31, we have F0,1 [P (1, 2)] =
P (0,2) 0.8817
P (0,1) = 0.9434

0.8817
ln F
+0.5σ 2 T ln 0.9434+0.5×0.052 ×1
d1 = K √
σ T
= 0.9259 √
0.05 1
= 0.212 0

d2 = 0.212 0 − 0.05 1 = 0.162 0
N (d1 ) = 0.583 9 N (d2 ) = 0.564 3

C = P (0, T ) [F N (d1 ) − KN (d2 )]


24.4. BINOMIAL INTEREST RATE MODEL 217
¡ 0.8817 ¢
= 0.9434 0.9434 × 0.583 9 − 0.9259 × 0.564 3 = 0.0220

P = P (0,¡T ) [KN (d2 ) − F N (d1 )] ¢


0.8817
= 0.9434 0.9259 × (1 − 0.564 3) − 0.9434 × (1 − 0.583 9)
= 0.013 7

24.3.2 Interest rate caplet

Notation

• Rt (T, T + s). The (not annualized) interest rate pre-agreed upon at time
t where t ≤ T that applies to the future time interval [T, T + s].

• RT (T, T + s). The (not annualized) interest rate agreed upon at time T
that applies to the time interval [T, T + s].

• Caplet. A caplet gives the buyer the right to buy the time-T market
interest rate RT (T, T + s) by paying a fixed strike interest rate KR . If
KR ≥ RT (T, T + s), the caplet expires worthless. The payoff of the caplet
at T + s is max [0, RT (T, T + s) − KR ]. The payoff of the caplet at T is
max [0, RT (T, T + s) − KR ]
1 + RT (T, T + s)

To calculate the price of the caplet, we first modify


∙ the payoff: ¸
max [0, RT (T, T + s) − KR ] RT (T, T + s) − KR
= (1 + KR ) max 0, =
1 + R∙T (T, T + s) ¸ (1 + RT (T, T∙ + s)) (1 + KR ) ¸
1 1 1
(1 + KR ) max 0, − = (1 + KR ) max 0, − PT (T, T + s)
∙ 1 + KR 1 + RT¸(T, T + s) 1 + KR
1
max 0, − PT (T, T + s) is the payoff of a put on a bond. This put
1 + KR
gives the buyer the right, at T , to sell a bond that matures at T + s for a
1
guaranteed price 1+K R
. Let P represent the price of this put. Hence the price
of the caplet is (1 + KR ) P .

24.4 Binomial interest rate model


Binomial interest rate tree is really simple. Unfortunately, the author of the
textbook uses too many math symbols and formulas, making this section hard
to read. What I will do here is to walk you through a few examples. If you
understand these examples, you are fine.

Example 24.4.1. (DM Example 24.3)

The following is the 3-period interest rate tree (DM Figure 24.3)
218 CHAPTER 24. INTEREST RATE MODELS

t=0 t=1 t=2


0.18
0.14
0.10
0.10
0.10
0.06
0.02

Make sure you understand the above table. The 10% interest rate at t = 0
applies to the interval [t = 0, t = 1]. The 14% and 6% interest rates apply to
the interval [t = 1, t = 2]. The interest rates 0.18, 0.10, and 0.02 at t = 2 apply
to the interval [t = 2, t = 3].
The price of the 1-year bond is just the PV of $1 discounted from t = 1 to
t = 0. Hence P (0, 1) = e−0.1

The price of the 2-year bond is just the PV of $1 discounted from t = 2 to


t = 0. What’s tricky is that we have two interest rates during [t = 1, t = 2].

• If the path is u, then $1 at t = 2 travels back to t = 0 through 2 interest


rates: 0.14, and 0.1. The PV of $1 discounted from t = 2 to t = 1 is
e−0.14 . The PV of e−0.14 discounted from t = 1 to t = 0 is e−0.14 e−0.11 =
e−(0.14+0.1) . So the 2-year bond is worth e−(0.14+0.1) at t = 0

• If the path is d, then $1 at t = 2 travels back to t = 0 through 2 interest


rates: 0.06 and 0.1. The 2-year bond is worth e−(0.06+0.1) at t = 0

Since the risk-neutral probability of up or down is 50%, then the 2-year bond
is worth the following at t = 0:
P (0, 2) = 0.5e−(0.14+0.1) + 0.5e−(0.06+0.1) = 0.819 4
The term 0.5e−(0.14+0.1) + 0.5e−(0.06+0.1)
h S can be irewritten as:
2
0.5e−(0.14+0.1) + 0.5e−(0.06+0.1) = E ∗ e− i=0 ri h with h = 1

The price of the 3-year bond is just the PV of $1 discounted from t = 3 to


t = 0.

• If the path is uu, then $1 at t = 3 travels back to t = 0 through 3 interest


rates: 0.18, 0.14, and 0.1. The PV of $1 discounted from t = 3 to t = 0 is
e−(0.18+0.14+0.1) . So the 3-year bond is worth e−(0.18+0.14+0.1) at t = 0

• If the path is ud, then $1 at t = 3 travels back to t = 0 through 3 interest


rates: 0.1, 0.14, and 0.1. The 3-year bond is worth e−(0.1+0.14+0.1) at t = 0

• If the path is du, then $1 at t = 3 travels back to t = 0 through 3 interest


rates: 0.1, 0.06, and 0.1. The 3-year bond is worth e−(0.1+0.06+0.1) at t = 0
24.4. BINOMIAL INTEREST RATE MODEL 219

• If the path is dd, then $1 at t = 3 travels back to t = 0 through 3 interest


rates: 0.02, 0.06, and 0.1. The 3-year bond is worth e−(0.02+0.06+0.1) at
t=0

Since the risk-neutral probability for each is 0.52 = 0.25, the price of a 3-year
bond is the following at t = 0:
¡ −(0.18+0.14+0.1) ¢
P (0, 3) = 0.25 e + e−(0.1+0.14+0.1) + e−(0.1+0.06+0.1) + e−(0.02+0.06+0.1) =
0.743 8

¡ ¢
The term 0.25 e−(0.18+0.14+0.1) + e−(0.1+0.14+0.1) + e−(0.1+0.06+0.1) + e−(0.02+0.06+0.1)
can be rewritten as:
¡ −(0.18+0.14+0.1) ¢
0.25
h e i + e−(0.1+0.14+0.1) + e−(0.1+0.06+0.1) + e−(0.02+0.06+0.1) =
S3
E ∗ e− i=0 ri h

The general formula is DM 24.44:


h Sn i
P (0, n) = E ∗ e− i=0 ri h (DM 24.31)

Option pricing example

Example 24.4.2. (SOA May 2007 #9)

You use a binomial interest rate model to evaluate a 7.5% interest rate cap
on a $100 three-year loan. You are given:
(i) The interest rates for the binomial tree are as follows:

• r0 = 6%

• ru = 7.704%

• rd = 4.673%

• ruu = 9.892%

• rud = rdu = 6.000%

• rdd = 3.639%

(ii) All interest rates are annual effective rates.


(iii) The risk-neutral probability that the annual effective interest rate moves
up or down is 0.5.
(iv) The loan interest payments are made annually.
Using the binomial interest rate model, calculate the value of this interest
rate cap.
220 CHAPTER 24. INTEREST RATE MODELS

t=0 t=1 t=2


9.892%
7.704%
6.000%
6%
6.000%
4.673%
3.639%

Once again, the interest rate at t in the above table applies to the period
[t, t + 1]. For example, the 6% rate applies to [0, 1] (i.e. Year 1).
First, let’s understand what’s an interest rate cap. Imagine you borrowed
$100 from a bank. Your interest accrued on the loan each year is not based
on a fixed interest rate (such as 8%), but is based on the then market interest
rate. For example, if the market interest rate is 6% in Year 1, then your interest
payment at the end of Year 1 is 100 × 0.06 = 6.0.
As a borrower, you are worried that the market interest may go up. For
example, if the Year 1 interest rate is 20%, then your interest payment at the
end of Year 1 is 100 × 0.2 = 20.
How can you reduce your risk? One thing you can do is to buy an interest
cap. Suppose you buy an interest rate cap of 7.5%. This is what happens:
• If the market interest rate is at or below 7.5%, then the cap doesn’t kick
in. So you get nothing from the cap
• if the market interest rate is above 7.5%, then the party who sold you
the cap will pay you the excess of the market interest rate over the cap
rate. For example, if the market interest rate in Year 1 is 10%, then the
seller of the cap will pay you 100 (0.1 − 0.075) = 2. 5 at the end of Year
1. You still own the bank 100 × 0.1 = 10 at the end of Year 1, but you
pay 100 × 0.075 = 7. 5 out of your own pocket. The cap seller pays you
100 (0.1 − 0.075) = 2. 5. So together you get 7. 5 + 2.5 = 10. You mail a
$10 check to the bank.
In summary, if you buy an interest capped of 7.5%, then the interest on
your loan is capped at 7.5% regardless of the market interest rate (because any
excess of the market rate over the cap 7.5% is paid by the cap seller).
Now you know what an interest cap is, let’s solve this problem.

The first year market rate 6% is below the cap rate. At the end of Year 1,
you get nothing from the cap

If the 2nd year market rate is 7.704%, then at the end of year 2 the cap
seller pays you 100 (0.07704 − 0.075) = 0.204 . Notice that 0.204 occurs at
t = 2. To help keep track of payments, we’ll discount this payment to t = 1.
0.204
The discounted value is at t = 1.
1 + 0.07704
24.5. BLACK-DERMAN-TOY MODEL 221

If the 2nd year market rate is 4.673%, then the payment at the end of year
2 is zero.

Let’s calculate the cap payoff in Year 3. Of the 4 interest rates during
[t = 2, t = 3] , only when the market interest rate is 9.892% do we get a payoff
of 100 (0.09892 − 0.075) = 2. 392. This payment occurs at t = 3. The PV of
2. 392
this payment at t = 2 is .
1 + 0.09892

Now we can draw a payoff table:

t=0 t=1 t=2


100 (0.09892 − 0.075)
9.892% Payoff:
1 + 0.09892
100 (0.07704 − 0.075)
7.704% Payoff:
1 + 0.07704
6.000%
6%
6.000%
4.673%
3.639%

100 (0.07704 − 0.075)


The PV of payoff = 0.189 4 1 at t = 1 discounted to
1 + 0.07704
100 (0.07704 − 0.075)
t = 0 is . The risk neutral probability of reaching u
(1 + 0.09892) (1 + 0.06)
node is 0.5.
100 (0.09892 − 0.075)
The PV of payoff = 2. 176 68 at t = 2 discounted to
1 + 0.09892
100 (0.09892 − 0.075)
t = 0 is . The risk neutral probability of
(1 + 0.09892) (1 + 0.07704) (1 + 0.06)
reaching uu node is 0.52 = 0.25

Hence risk neutral based expected present value of the cap payoff is:
100 (0.07704 − 0.075) 100 (0.09892 − 0.075)
×0.5+ ×0.52 =
(1 + 0.07704) (1 + 0.06) (1 + 0.09892) (1 + 0.07704) (1 + 0.06)
0.565 99 = 0.57

So the price of the cap is $0.57

24.5 Black-Derman-Toy model


The BDT model is a procedure to produce a discrete binomial interest rate
tree that matches the observed term structure of interest rates. Once again,
to quickly explain the essence of the BDT model, I’m walk you through an
example. If you understand this example, you are ready for the exam.
222 CHAPTER 24. INTEREST RATE MODELS

Suppose we gathered the following data from the market (DM Table 24.2):
Maturity n (Yrs) YTM Bond Price Volatility in Yr 1 on (n − 1)-Yr bond
1 10% 0.9091
2 11% 0.8116 10%
3 12% 0.7118 15%
4 12.5% 0.6243 14%

We want to produce an interest rate tree that will match the above table.
What should we do?
First, let’s go through some terms.
YTM (yield to maturity) is the discrete annual effective interest rate earned
by a bond. The formula is
P (0, T ) = (1 + Y T M )−T
For example, a 2-year bond in the table is worth 0.8116. This means that
PV of $1 discounted from t = 2 to t = 0 is 0.8116. To find YTM, we solve the
following equation:
−2
0.8116 = (1 + i) → i = 0.11
So the YTM for this 2-year bond is 11%

The volatility in Yr 1 on (n − 1) bond is the standard deviation of the natural


log of the YTM on an (n − 1)-year bond issued at t = 1 and maturity at time
n. For example, 10% volatility in the table means that standard deviation of
the natural log of the YTM on a 1-year bond issued at t = 1 maturing in t = 2
is 10%. This concept will be clear to you later.

Next, next use the BDT model to build an interest rate tree. We’ll first find
the interest rate ru and rd , the two possible interest rates for Year 2 (i.e. for
the time interval [t = 1, t = 2]).
t=0 t=h=1
ru
r0 = 10%
rd

Once again, rt is the interest for the interval [t, t + 1]. For example, r0 = 10%
is the interest rate for Year 1. ru and rd are the two interest rates for Year 2
(i.e. from t = 1 to t = 2).
We assume the risk neutral probability of up and down is 0.5. We want to
find ru and rd to satisfy the condition that standard deviation of the natural
log of the YTM on a 1-year bond issued at t = 1 maturing in t = 2 is 10%.
The price of a 1-year bond issued at t = 1 maturing in t = 2 is
1 1
P (1, 2) = or
1 + ru 1 + rd
The YTM is:
1
(1 + Y T Mu )−1 = → Y T Mu = ru
1 + ru
24.5. BLACK-DERMAN-TOY MODEL 223

−1 1
or (1 + Y T Md ) = → Y T Md = rd
1 + rd

We know that the standard deviation of ln Y T Mu = ln ru and ln Y T Md =


ln rd is 10%.

The mean of the log of the YTM on a 1-year bond issued at t = 1 maturing
in t = 2 rate is:
E (ln r1 ) = 0.5 ln ru + 0.5 ln rd = 0.5 (ln ru + ln rd )

2
We use the variance formula V ar (X) = E [X − E (X)] to calculate the
variance of ln Y T Mu = ln ru and ln Y T Md = ln rd :
2 2
0.5 [ln ru − E (ln r1 )] + 0.5 [ln rd − E (ln r1 )]
= 0.5 (ln ru − 0.5 ln ru − 0.5 ln rd )2 + 0.5 (ln rd − 0.5 ln ru − 0.5 ln rd )2
= 0.5 (0.5 ln ru − 0.5 ln rd )2 + 0.5 (0.5 ln rd − 0.5 ln ru )2
2 2
= 0.5 (0.5 ln ru − 0.5 ln rd ) + 0.5 (0.5 ln ru − 0.5 ln rd )
µ ¶2
ru
= (0.5 ln ru − 0.5 ln rd )2 = 0.5 ln
rd

The standard deviation of ln Y T Mu = ln ru and ln Y T Md = ln rd is σ 1 =


10%.
ru ru
→ 0.5 ln = σ1 ln = 2σ 1 ru = rd e2σ1 = rd e0.2
rd rd

Now we see that matching volatility requires ru = rd e2σ . This relationship


holds for every node.

Next, we want to reproduce the 2-year bond price of 0.8116.


If the Year 2 interest rate is ru , then the PV of $1 discounted from t = 2 to
1
t = 0 is
(1 + ru ) (1 + r0 )
If the Year 2 interest rate is rd , then the PV of $1 discounted from t = 2 to
1
t = 0 is
(1 + rd ) (1 + r0 )
The risk-neutral probability of up and down is 0.5. The expected 2-year
bond price is:
1 1
0.5 × + 0.5 ×
(1 + ru ) (1 + r0 ) (1 + rd ) (1 + r0 )

To match the observed price P (0, 2) = 0.8116, we have:


1 1
0.5 × + 0.5 × = P (0, 2)
(1 + ru ) (1 + r0 ) (1 + rd ) (1 + r0 )

To sum up, we have two equations:


224 CHAPTER 24. INTEREST RATE MODELS

⎨ ru = rd e2σ1
1 1
⎩ 0.5 × + 0.5 × = P (0, 2)
(1 + ru ) (1 + r0 ) (1 + rd ) (1 + r0 )

Or

⎨ ru = rd e0.2
1 1
⎩ 0.5 × + 0.5 × = 0.8116
(1 + ru ) (1 + 0.1) (1 + rd ) (1 + 0.1)

1 1
→ + = 2 × 0.8116 (1 + 0.1)
1 + rd 1 + rd e0.2
→ rd = 0.108 265 = 10.83%

→ ru = 0.108 265e0.2 = 0.132 2 = 13.22%

We can use the same procedure to calculate ruu , rud , rdu , rdd . To simplify
our model, we arbitrary set rud = rdu (i.e. we assume the tree is recombining).
t=0 t=h=1 t = 2h = 2
ruu = rdd e4σ2
ru = 13.22%
r0 = 10% rud = rdu = rdd e2σ2
rd = 10.83%
rdd

We want to choose rdd such that the standard deviation of the natural log
of the YTM on a 2-year bond issued at t = 1 and maturing at t = 3 is 15%.

First, we calculate P (1, 3), the price of a 2-year bond issued at t = 1 and
maturing at t = 3.
If the 2nd year interest rate is ru = 13.22%, then the 3rd year rate is either
ruu or rud with equal risk-neutral probability of 0.5. Then the expected PV of
$1 discounted from t = 3 to t = 1 is
1 1
P (1, 3, ru ) = 0.5 × + 0.5 ×
µ (1 + ru ) (1 + ruu ) ¶ (1 + ru ) (1 + rud )
0.5 1 1
= +
1.1322 1 + rdd e4σ2 1 + rdd e2σ2
The YTM can solved as follows:
−2 −0.5
P (1, 3, ru ) = (1 + i) → i = P (1, 3, ru ) −1
If the 2nd year interest rate is rd = 10.83%, then the 3rd year rate is either
rdu or rdd with equal risk-neutral probability of 0.5. Then the expected PV of
$1 discounted from t = 3 to t = 1 is
1 1
P (1, 3, rd ) = 0.5 × + 0.5 ×
µ (1 + rd ) (1 + r¶
du ) (1 + rd ) (1 + rdd )
0.5 1 1
= +
1.1083 1 + rdd e2σ2 1 + rdd
24.5. BLACK-DERMAN-TOY MODEL 225

The YTM can solved as follows:


−2 −0.5
P (1, 3, rd ) = (1 + i) → i = P (1, 3, rd ) −1
−0.5
P (1, 3, ru ) −1
→ 0.15 = 0.5 ln −0.5
P (1, 3, rd ) −1
∙ µ ¶¸−0.5
0.5 1 1
+ −1
1.1322 1 + rdd e4σ2 1 + rdd e2σ2
= 0.5 ln ∙ µ ¶¸−0.5
0.5 1 1
+ −1
1.1083 1 + rdd e2σ2 1 + rdd

By the way, please note that σ 2 is not 10%.

We should also match the 3-year bond price P (0, 3) = 0.7118


The PV of $1 at t = 3 discounted to t = 0 is

1
• if the path is 0 → u → uu
(1 + r0 ) (1 + ru ) (1 + ruu )
1
• if the path is 0 → u → ud
(1 + r0 ) (1 + ru ) (1 + rud )
1
• if the path is 0 → d → du
(1 + r0 ) (1 + rd ) (1 + rdu )
1
• if the path is 0 → d → dd
(1 + r0 ) (1 + rd ) (1 + rdd )

Each path has a risk neutral probability of 0.25. Now we have:

0.25 0.25
+
(1 + r0 ) (1 + ru ) (1 + ruu ) (1 + r0 ) (1 + ru ) (1 + rud )
0.25 0.25
+ + = P (0, 3)
(1 + r0 ) (1 + rd ) (1 + rdu ) (1 + r0 ) (1 + rd ) (1 + rdd )

0.25 0.25
+
(1.1) (1.1322) (1 + rdd e4σ ) (1.1) (1.1322) (1 + rdd e2σ )
0.25 0.25
+ 2σ
+ = 0.7118
(1.1) (1.1083) (1 + rdd e ) (1.1) (1.1083) (1 + rdd )

Now our equations are:


∙ µ ¶¸−0.5
0.5 1 1
+ −1
1.1322 1 + rdd e4σ2 1 + rdd e2σ2
0.15 = 0.5 ln ∙ µ ¶¸−0.5
0.5 1 1
+ −1
1.1083 1 + rdd e2σ2 1 + rdd
226 CHAPTER 24. INTEREST RATE MODELS

0.25 0.25
+
(1.1) (1.1322) (1 + rdd e4σ2 ) (1.1) (1.1322) (1 + rdd e2σ2 )
0.25 0.25
+ + = 0.7118
(1.1) (1.1083) (1 + rdd e2σ2 ) (1.1) (1.1083) (1 + rdd )

These equations are hard to solve manually. Special software is needed to


solve them.

You can verify that the solutions are: rdd = 0.0925 σ 2 = 0.195 0
µ µ ¶¶−0.5
0.5 1 1
+ −1
1.1322 1 + 0.0925e4×0.1950 1 + 0.0925e2×0.1950
0.5 ln µ µ ¶¶−0.5 = 0.149 97 =
0.5 1 1
+ −1
1.1083 1 + 0.0925e2×0.1950 1 + 0.0925
0.15
0.25 0.25
+
(1.1) (1.1322) (1 + 0.0925e4×0.1950 ) (1.1) (1.1322) (1 + 0.0925e2×0.1950 )
0.25 0.25
+ +
(1.1) (1.1083) (1 + 0.0925e2×0.1950 ) (1.1) (1.1083) (1 + 0.0925)
= 0.711 755 = 0.7118

Then the year 3 interest rates are:


rdd = 0.0925
rdu = rud = 0.0925e2×0.1950 = 0.136 6
ruu = 0.0925e4×0.1950 = 0.201 8

We can use the same logic and calculate the Year 4 interest rates. However,
I’m not going to do the calculation because the calculation is overly intensive.

Make sure you can reproduce the Yr 2 and Yr 3 rates. Yr 2 rates can be
easily reproduced. SOA or CAS can ask you to calculate the Year 2 rates using
the BDT model. Yr 3 rates are harder. A full calculation of Year 3 rates by
hand is difficult. However, SOA or CAS can give you some partial information
on Year 3 rates and ask you to calculate the rest.
Solution to Derivatives Markets: SOA Exam
MFE and CAS Exam 3F

Yufeng Guo

June 1, 2008
www.myactuaryexam.com c
°Yufeng Guo ii
Contents

Introduction vii

9 Parity and other option relationships 1

10 Binomial option pricing I 23

11 Binomial option pricing II 91

12 Black-Scholes formula 107

13 Market making and delta hedging 125

14 Exotic options: I 149

20 Brownian motion and Ito’s lemma 161

24 Interest rate models 167

iii
CONTENTS CONTENTS

www.myactuaryexam.com c
°Yufeng Guo iv
Preface

This is Guo’s solution to Derivatives Markets (2nd edition ISBN 0-321-28030-X)


for SOA MFE or CAS Exam 3F. Unlike the official solution manual published
by Addison-Wesley, this solution manual provides solutions to both the even-
numbered and odd-numbered problems for the chapters that are on the SOA
Exam MFE and CAS Exam 3F syllabus. Problems that are out of the scope of
the SOA Exam MFE and CAS Exam 3F syllabus are excluded.

Please report any errors to yufeng_guo@msn.com.

This book is the exclusive property of Yufeng Guo. Redistribution of this


book in any form is prohibited.

v
PREFACE PREFACE

www.myactuaryexam.com c
°Yufeng Guo vi
Introduction

Recommendations on using this solution manual:

1. Obviously, you’ll need to buy Derivatives Markets (2nd edition) to see the
problems.
2. Make sure you download the textbook errata from http://www.kellogg.
northwestern.edu/faculty/mcdonald/htm/typos2e_01.html

vii
CHAPTER 8. INTRODUCTION

www.myactuaryexam.com c
°Yufeng Guo viii
Chapter 9

Parity and other option


relationships

Problem 9.1.
S0 = 32 T = 6/12 = 0.5 K = 35
C = 2.27 r = 0.04 δ = 0.06

C + P V (K) = P + S0 e−δT
2.27 + 35e−0.04(0.5) = P + 32e−0.06(0.5) P = 5. 522 7

Problem 9.2.
S0 = 32 T = 6/12 = 0.5 K = 30
C = 4.29 P = 2.64 r = 0.04
C + P V (K) = P + S0 − P V (Div)
4.29 + 30e−0.04(0.5) = 2.64 + 32 − P V (Div)
P V (Div) = 0.944

Problem 9.3.
S0 = 800 r = 0.05 δ=0
T =1 K = 815 C = 75 P = 45
a. Buy stock+ sell call+buy put=buy P V (K)
C + P V (K) = P + S0
→ P V (K = 815) = S0 + −C + |{z} P = 800 + (−75) + 45 = 770
|{z} |{z}
buy stock sell call buy put

So the position is equivalent to depositing 770 in a savings account (or buy-


ing a bond with present value equal to 770) and receiving 815 one year later.
770eR = 815 R = 0.056 8

1
CHAPTER 9. PARITY AND OTHER OPTION RELATIONSHIPS

So we earn 5.68%.

b. Buying a stock, selling a call, and buying a put is the same as depositing
P V (K) in the savings account. As a result, we should just earn the risk free
interest rate r = 0.05. However, we actually earn R = 0.056 8 > r. To arbitrage,
we "borrow low and earn high." We borrow 770 from a bank at 5%. We use the
borrowed 770 to finance buying a stock, selling a call, and buying a put. Notice
that the net cost of buying a stock, selling a call, and buying a put is 770.
One year later, we receive 770eR = 815. We pay the bank 770e0.05 = 809.
48. Our profit is 815 − 809. 48 = 5. 52 per transaction.
If we do n such transactions, we’ll earn 5. 52n profit.

Alternative answer: we can burrow at 5% (continuously compounding) and


lend at 5.6 8% (continuously compounding), earning a risk free 0.68%. So if
we borrow $1 at time zero, our risk free profit at time one is e0.0568 − e0.05 =
0.00717 3; if we borrow $770 at time zero, our risk free profit at time one is
0.00717 3×770 = 5. 52. If we borrow n dollars at time zero, we’ll earn 0.00717 3n
dollars at time one.

c. To avoid arbitrage, we need to have:


P V (K = 815) = S0 + |{z} −C + |{z} P = 815e−0.05 = 775. 25
|{z}
buy stock sell call buy put

→ C − P = S0 − P V (K) = 800 − 775. 25 = 24. 75

d. C − P = S0 − P V (K) = 800 − Ke−rT = 800 − Ke−0.05


If K = 780 C − P = 800 − 780e−0.05 = 58. 041
If K = 800 C − P = 800 − 800e−0.05 = 39. 016
If K = 820 C − P = 800 − 820e−0.05 = 19. 992
If K = 840 C − P = 800 − 840e−0.05 = 0.967

Problem 9.4.
To solve this type of problems, just use the standard put-call parity.
To avoid calculation errors, clearly identify the underlying asset.
The underlying asset is €1. We want to find the dollar cost of a put option
on this underlying.
The typical put-call parity:
C + P V (K) = P + S0 e−δT
C, K, P , and S0 should all be expressed in dollars. S0 is the current (dollar
price) of the underlying. So S0 = $0.95.

C = $0.0571 K = $0.93
δ is the internal growth rate of the underlying asset (i.e. €1). Hence δ = 0.04
Since K is expressed in dollars, P V (K) needs to be calculated using the
dollar risk free interest r = 0.06.
0.0571 + 0.93e−0.06(1) = P + 0.95e−0.04(1) P = $0.02 02

www.myactuaryexam.com c
°Yufeng Guo 2
CHAPTER 9. PARITY AND OTHER OPTION RELATIONSHIPS

Problem 9.5.
As I explained in my study guide, don’t bother memorizing the following
complex formula: µ ¶
1 1
C$ (x0 , K, T ) = x0 KPf , ,T
x0 K
Just use my approach to solve this type of problems.
Convert information to symbols:
1
The exchange rate is 95 yen per euro. Y 95 =€1 or Y 1 =€
95

Yen-denominated put on 1 euro with strike price Y100 has a premium Y8.763
→ (€1 → Y 100)0 =Y8.763

What’s the strike price of a euro-denominated call on 1 yen? €K → 1Y

Calculate the price of a euro-denominated call on 1 yen with strike price €K


(€K → 1Y )0 = €?

1
€1 → Y 100 → € →Y1
100
1
The strike price of the corresponding euro-denominated yen call is K =€ =€0.01
µ ¶ 100
1 1 1
€ →Y1 = × (€1 → Y 100)0 = (Y 8.763)
100 0 100 100
1
Since Y 1 =€ , we have:
95
µ ¶
1 1 1
(Y 8.763) = (8.763) € =€9. 224 2 × 10−4
100µ 100
¶ 95
1
→ € → Y 1 =€9. 224 2 × 10−4
100 0
1
So the price of a euro-denominated call on 1 yen with strike price K =€
100
is €9. 224 2 × 10−4

www.myactuaryexam.com c
°Yufeng Guo 3
CHAPTER 9. PARITY AND OTHER OPTION RELATIONSHIPS

Problem 9.6.
The underlying asset is €1. The standard put-call parity is:
C + P V (K) = P + S0 e−δT
C, K, P , and S0 should all be expressed in dollars. S0 is the current (dollar
price) of the underlying.
δ is the internal growth rate of the underlying asset (i.e. €1).
We’ll solve Part b first.

b. 0.0404 + 0.9e−0.05(0.5) = 0.0141 + S0 e−0.035(0.5) S0 = $0.920 04


So the current price of the underlying (i.e. €1) is S0 = $0.920 04. In other
words, the currency exchange rate is $0.920 04 =€1

a. According to the textbook Equation 5.7, the forward price is:


F0,T = S0 e−δT erT = 0.920 04e−0.035(0.5) e0.05(0.5) = $0.926 97

Problem 9.7.
The underlying asset is one yen.
a. C + Ke−rT = P + S0 e−δT
0.0006 + 0.009e−0.05(1) = P + 0.009e−0.01(1)
0.0006 + 0.008561 = P + 0.008 91 P = $0.00025

b. There are two puts out there. One is the synthetically created put using
the formula:

P = C + Ke−rT − S0 e−δT
The other is the put in the market selling for $0.0004.

To arbitrage, build a put a low cost and sell it at a high price. At t = 0, we:

• Sell the expensive put for $0.0004


• Build a cheap put for $0.00025. To build a put, we buy a call, deposit
Ke−rT in a savings account, and sell e−δT unit of Yen.

T =1 T =1
t=0 ST < 0.009 ST ≥ 0.009
Sell expensive put 0.0004 ST − 0.009 0
Buy call −0.0006 0 ST − 0.009
Deposit Ke−rT in savings −0.009e−0.05(1) 0.009 0.009
Short sell e−δT unit of Yen 0.009e−0.01(1) ST ST
Total $0.00015 0 0

0.0004 − 0.0006 − 0.009e−0.05(1) + 0.009e−0.01(1) = $0.00015

www.myactuaryexam.com c
°Yufeng Guo 4
CHAPTER 9. PARITY AND OTHER OPTION RELATIONSHIPS

At t = 0, we receive $0.00015 yet we don’t incur any liabilities at T = 1 (so


we receive $0.00015 free money at t = 0).

c. At-the-money means K = S0 (i.e. the strike price is equal to the current


exchange rate).
Dollar-denominated at-the-money yen call sells for $0.0006. To translate this
into symbols, notice that under the call option, the call holder can give $0.009
and get Y 1.

"Give $0.009 and get Y 1" is represented by ($0.009 → Y 1). This option’s
premium at time zero is $0.0006. Hence we have:
($0.009 → Y 1)0 = $0.0006
We are asked to find the yen denominated at the money call for $1. Here
the call holder can give c yen and get $1. "Give c yen and get $1" is represented
by (Y c → $1). This option’s premium at time zero is (Y c → $1)0 .
First, we need to calculate c, the strike price of the yen denominated dollar
1
call. Since at time zero $0.009 = Y 1, we have $1 = Y . So the at-the-
0.009
1
money yen denominated call on $1 is c = . Our task is to find this option’s
µ ¶ 0.009
1
premium: Y → $1 =?
0.009 0
We’ll find the premium for Y 1 →$0.009, the option of "give 1 yen and get
$0.009." Once we find this premium, we’ll scale it and find the premium of "give
1
yen and get $1."
0.009
We’ll use the general put-call parity:
(AT → BT )0 + P V (AT ) = (BT → AT )0 + P V (BT )

($0.009 → Y 1)0 + P V ($0.009) = (Y 1 → $0.009)0 + P V (Y 1)

P V ($0.009) = $0.009e−0.05(1)
Since we are discounting $0.009 at T = 1 to time zero, we use the dollar
interest rate 5%.

P V (Y 1) = $0.009e−0.01(1)
If we discount Y1 from T = 1 to time zero, we get e−0.01(1) yen, which is
equal to $0.009e−0.01(1) .

So we have:
$0.0006+$0.009e−0.05 = (Y 1 → $0.009)0 + $0.009e−0.01(1)

(Y
µ 1 → $0.009)0 ¶ = $2. 506 16 × 10−4
1 1 2. 506 16 × 10−4
Y 1 → $1 = (Y 1 → $0.009)0 = $ = $2.
0.009 0 0.009 0.009
−2
2. 784 62 × 10
784 62 × 10−2 = Y = Y 3. 094
0.009

www.myactuaryexam.com c
°Yufeng Guo 5
CHAPTER 9. PARITY AND OTHER OPTION RELATIONSHIPS

So the yen denominated at the money call for $1 is worth $2. 784 62 × 10−2
or Y 3. 094.

We are also asked to identify the relationship between the yen denominated
at the money call for $1 and the dollar-denominated yen put. The relationship
is that we use the premium of the latter option to calculate the premium of the
former option.
Next, we calculate the premium for the yen denominated at-the-money put
for $1:
µ ¶
1 1
$→Y = ($0.009 → Y 1)0
0.009 0 0.009
1
= × $0.0006 = $ 0.0 666 7
0.009
1
= Y 0.0 666 7 × = Y 7. 407 8
0.009

So the yen denominated at-the-money put for $1 is worth $ 0.0 666 7 or Y


7. 407 8.
I recommend that you use my solution approach, which is less prone to errors
than using complex notations and formulas in the textbook.

Problem 9.8.

The textbook Equations 9.13 and 9.14 are violated.


This is how to arbitrage on the calls. We have two otherwise identical
calls, one with $50 strike price and the other $55. The $50 strike call is more
valuable than the $55 strike call, but the former is selling less than the latter.
To arbitrage, buy low and sell high.
We use T to represent the common exercise date. This definition works
whether the two options are American or European. If the two options are
American, we’ll find arbitrage opportunities where the two American options
are exercised simultaneously. If the two options are European, T is the common
expiration date.

The payoff is:


T T T
Transaction t=0 ST < 50 50 ≤ ST < 55 ST ≥ 55
Buy 50 strike call −9 0 ST − 50 ST − 50
Sell 55 strike call 10 0 0 − (ST − 55)
Total 1 0 ST − 50 ≥ 0 5

At t = 0, we receive $1 free money.


At T , we get non-negative cash flows (so we may get some free money, but
we certainly don’t owe anybody anything at T ). This is clearly an arbitrage.

www.myactuaryexam.com c
°Yufeng Guo 6
CHAPTER 9. PARITY AND OTHER OPTION RELATIONSHIPS

This is how to arbitrage on the two puts. We have two otherwise identical
puts, one with $50 strike price and the other $55. The $55 strike put is more
valuable than the $50 strike put, but the former is selling for less than the latter.
To arbitrage, buy low and sell high.
The payoff is:

T T T
Transaction t = 0 ST < 50 50 ≤ ST < 55 ST ≥ 55
Buy 55 strike put −6 55 − ST 55 − ST 0
Sell 50 strike put 7 − (50 − ST ) 0 0
Total 1 5 55 − ST > 0 0
At t = 0, we receive $1 free money.
At T , we get non-negative cash flows (so we may get some free money, but
we certainly don’t owe anybody anything at T ). This is clearly an arbitrage.

Problem 9.9.

The textbook Equation 9.15 and 9.16 are violated.


We use T to represent the common exercise date. This definition works
whether the two options are American or European. If the two options are
American, we’ll find arbitrage opportunities where the two American options
are exercised simultaneously. If the two options are European, T is the common
expiration date.
This is how to arbitrage on the calls. We have two otherwise identical calls,
one with $50 strike price and the other $55. The premium difference between
these two options should not exceed the strike difference 15 − 10 = 5. In other
words, the 50-strike call should sell no more than 10 + 5. However, the 50-strike
call is currently selling for 16 in the market. To arbitrage, buy low (the 55-strike
call) and sell high (the 50-strike call).

The $50 strike call is more valuable than the $55 strike call, but the former
is selling less than the latter.
The payoff is:
T T T
Transaction t = 0 ST < 50 50 ≤ ST < 55 ST ≥ 55
Buy 55 strike call −10 0 0 ST − 55
Sell 50 strike call 16 0 − (ST − 50) − (ST − 50)
Total 6 0 − (ST − 50) ≥ −5 −5

So we receive $6 at t = 0. Then at T , our maximum liability is $5. So make


at least $1 free money.

This is how to arbitrage on the puts. We have two otherwise identical calls,
one with $50 strike price and the other $55. The premium difference between
these two options should not exceed the strike difference 15 − 10 = 5. In other

www.myactuaryexam.com c
°Yufeng Guo 7
CHAPTER 9. PARITY AND OTHER OPTION RELATIONSHIPS

words, the 55-strike put should sell no more than 7 + 5 = 12. However, the
55-strike put is currently selling for 14 in the market. To arbitrage, buy low
(the 50-strike put) and sell high (the 55-strike put).
The payoff is:
T T T
Transaction t = 0 ST < 50 50 ≤ ST < 55 ST ≥ 55
Buy 50 strike put −14 50 − ST 0 0
Sell 55 strike put 7 − (55 − ST ) − (55 − ST ) 0
Total 7 −5 − (55 − ST ) < −5 0

So we receive $7 at t = 0. Then at T , our maximum liability is $5. So make


at least $2 free money.

Problem 9.10.

Suppose there are 3 options otherwise identical but with different strike price
K1 < K2 < K3 where K2 = λK1 + (1 − λ) K2 and 0 < λ < 1.
Then the price of the middle strike price K2 must not exceed the price of a
diversified portfolio consisting of λ units of K1 -strike option and (1 − λ) units
of K2 -strike option:

C [λK1 + (1 − λ) K3 ] ≤ λC (K1 ) + (1 − λ) C (K3 )


P [λK1 + (1 − λ) K3 ] ≤ λP (K1 ) + (1 − λ) P (K3 )

The above conditions are called the convexity of the option price with respect
to the strike price. They are equivalent to the textbook Equation 9.17 and 9.18.
If the above conditions are violated, arbitrage opportunities exist.

We are given the following 3 calls:


Strike K1 = 50 K2 = 55 K3 = 60
Call premium 18 14 9.50

λ50 + (1 − λ) 60 = 55
→ λ = 0.5 0.5 (50) + 0.5 (60) = 55

Let’s check:
C [0.5 (50) + 0.5 (60)] = C (55) = 14

0.5C (50) + 0.5C (60) = 0.5 (18) + 0.5 (9.50) = 13. 75


C [0.5 (50) + 0.5 (60)] > 0.5C (50) + 0.5C (60)
So arbitrage opportunities exist. To arbitrage, we buy low and sell high.
The cheap asset is the diversified portfolio consisting of λ units of K1 -strike
option and (1 − λ) units of K3 -strike option. In this problem, the diversified
portfolio consists of half a 50-strike call and half a 60-strike call.
The expensive asset is the 55-strike call.

www.myactuaryexam.com c
°Yufeng Guo 8
CHAPTER 9. PARITY AND OTHER OPTION RELATIONSHIPS

Since we can’t buy half a call option, we’ll buy 2 units of the portfolio (i.e.
buy one 50-strike call and one 60-strike call). Simultaneously,we sell two 55-
strike call options.
We use T to represent the common exercise date. This definition works
whether the options are American or European. If the options are American,
we’ll find arbitrage opportunities where the American options are exercised si-
multaneously. If the options are European, T is the common expiration date.
The payoff is:

T T T T
Transaction t=0 ST < 50 50 ≤ ST < 55 55 ≤ ST < 60 ST ≥ 60
buy two portfolios
buy a 50-strike call −18 0 ST − 50 ST − 50 ST − 50
buy a 60-strike call −9.5 0 0 0 ST − 60
Portfolio total −27. 5 0 ST − 50 ST − 50 2ST − 110

Sell two 55-strike calls 2 (14) = 28 0 0 −2 (ST − 55) −2 (ST − 55)


Total 0.5 0 ST − 50 ≥ 0 60 − ST > 0 0

−27. 5 + 28 = 0.5
ST − 50 − 2 (ST − 55) = 60 − ST
2ST − 110 − 2 (ST − 55) = 0
So we get $0.5 at t = 0, yet we have non negative cash flows at the expiration
date T . This is arbitrage.

The above strategy of buying λ units of K1 -strike call, buying (1 − λ) units


of K3 -strike call, and selling one unit of K2 -strike call is called the butterfly
spread.

We are given the following 3 puts:


Strike K1 = 50 K2 = 55 K3 = 60
Put premium 7 10.75 14.45

λ50 + (1 − λ) 60 = 55
→ λ = 0.5 0.5 (50) + 0.5 (60) = 55

Let’s check:
P [0.5 (50) + 0.5 (60)] = P (55) = 10.75

0.5P (50) + 0.5P (60) = 0.5 (7) + 0.5 (14.45) = 10. 725
P [0.5 (50) + 0.5 (60)] > .5P (50) + 0.5P (60)
So arbitrage opportunities exist. To arbitrage, we buy low and sell high.
The cheap asset is the diversified portfolio consisting of λ units of K1 -strike
put and (1 − λ) units of K3 -strike put. In this problem, the diversified portfolio
consists of half a 50-strike put and half a 60-strike put.
The expensive asset is the 55-strike put.

www.myactuaryexam.com c
°Yufeng Guo 9
CHAPTER 9. PARITY AND OTHER OPTION RELATIONSHIPS

Since we can’t buy half a option, we’ll buy 2 units of the portfolio (i.e. buy
one 50-strike put and one 60-strike put). Simultaneously,we sell two 55-strike
put options.
The payoff is:

T T T T
Transaction t=0 ST < 50 50 ≤ ST < 55 55 ≤ ST < 60 ST ≥ 60
buy two portfolios
buy a 50-strike put −7 50 − ST 0 0 0
buy a 60-strike put −14.45 60 − ST 60 − ST 60 − ST 0
Portfolio total −21. 45 110 − 2ST 60 − ST 60 − ST 0

Sell two 55-strike puts 2 (10.75) −2 (55 − ST ) −2 (55 − ST ) 0 0


Total 0.05 0 ST − 50 ≥ 0 60 − ST > 0 0
−21. 45 + 2 (10.75) = 0.05
50 − ST + 60 − ST = 110 − 2ST

−21. 45 + 2 (10.75) = 0.05


110 − 2ST − 2 (55 − ST ) = 0
60 − ST − 2 (55 − ST ) = ST − 50
So we get $0.05 at t = 0, yet we have non negative cash flows at the expiration
date T . This is arbitrage.
The above strategy of buying λ units of K1 -strike put, buying (1 − λ) units
of K3 -strike put, and selling one unit of K2 -strike put is also called the butterfly
spread.

Problem 9.11.

This is similar to Problem 9.10.


We are given the following 3 calls:
Strike K1 = 80 K2 = 100 K3 = 105
Call premium 22 9 5

80λ + 105 (1 − λ) = 100


→ λ = 0.2 0.2 (80) + 0.8 (105) = 100
C [0.2 (80) + 0.8 (105)] = C (100) = 9

0.2C (80) + 0.8C (105) = 0.2 (22) + 0.8 (5) = 8. 4


C [0.2 (80) + 0.8 (105)] > 0.2C (80) + 0.8C (105)
So arbitrage opportunities exist. To arbitrage, we buy low and sell high.
The cheap asset is the diversified portfolio consisting of λ units of K1 -strike
option and (1 − λ) units of K3 -strike option. In this problem, the diversified
portfolio consists of 0.2 unit of 80-strike call and 0.8 unit of 105-strike call.
The expensive asset is the 100-strike call.

www.myactuaryexam.com c
°Yufeng Guo 10
CHAPTER 9. PARITY AND OTHER OPTION RELATIONSHIPS

Since we can’t buy a fraction of a call option, we’ll buy 10 units of the port-
folio (i.e. buy two 80-strike calls and eight 105-strike calls). Simultaneously,we
sell ten 100-strike call options.
We use T to represent the common exercise date. This definition works
whether the options are American or European. If the options are American,
we’ll find arbitrage opportunities where the American options are exercised si-
multaneously. If the options are European, T is the common expiration date.
The payoff is:

T T
Transaction t=0 ST < 80 80 ≤ ST < 100
buy ten portfolios
buy two 80-strike calls −2 (22) 0 2 (ST − 80)
buy eight 105-strike calls −8 (5) 0 0
Portfolio total −84 0 2 (ST − 80)

Sell ten 100-strike calls 10 (9) 0 0


Total 6 0 2 (ST − 80) ≥ 0

T T
Transaction t=0 100 ≤ ST < 105 ST ≥ 105
buy ten portfolios
buy two 80-strike calls −2 (22) 2 (ST − 80) 2 (ST − 80)
buy eight 105-strike calls −8 (5) 0 8 (ST − 105)
Portfolio total −84 2 (ST − 80) 10ST − 1000

Sell ten 100-strike calls 10 (9) −10 (ST − 100) −10 (ST − 100)
Total 6 8 (105 − ST ) > 0 0
−2 (22) − 8 (5) = −44 − 40 = −84

−84 + 10 (9) = −84 + 90 = 6


2 (ST − 80) + 8 (ST − 105) = 10ST − 1000
2 (ST − 80) − 10 (ST − 100) = 840 − 8ST = 8 (105 − ST )
10ST − 1000 − 10 (ST − 100) = 0
So we receive $6 at t = 0, yet we don’t incur any negative cash flows at
expiration T . So we make at least $6 free money.

We are given the following 3 put:


Strike K1 = 80 K2 = 100 K3 = 105
Put premium 4 21 24.8

80λ + 105 (1 − λ) = 100


→ λ = 0.2 0.2 (80) + 0.8 (105) = 100
P [0.2 (80) + 0.8 (105)] = P (100) = 21

0.2P (80) + 0.8P (105) = 0.2 (4) + 0.8 (24.8) = 20. 64

www.myactuaryexam.com c
°Yufeng Guo 11
CHAPTER 9. PARITY AND OTHER OPTION RELATIONSHIPS

P [0.2 (80) + 0.8 (105)] > 0.2P (80) + 0.8P (105)


So arbitrage opportunities exist. To arbitrage, we buy low and sell high.
The cheap asset is the diversified portfolio consisting of λ units of K1 -strike
option and (1 − λ) units of K3 -strike option. In this problem, the diversified
portfolio consists of 0.2 unit of 80-strike put and 0.8 unit of 105-strike put.
The expensive asset is the 100-strike put.
Since we can’t buy half a fraction of an option, we’ll buy 10 units of the port-
folio (i.e. buy two 80-strike puts and eight 105-strike puts). Simultaneously,we
sell ten 100-strike put options.
The payoff is:

T T
Transaction t=0 ST < 80 80 ≤ ST < 100
buy ten portfolios
buy two 80-strike puts −2 (4) 2 (80 − ST ) 0
buy eight 105-strike puts −8 (24.8) 8 (105 − ST ) 8 (105 − ST )
Portfolio total −84 1000 − 10ST 8 (105 − ST )

Sell ten 100-strike puts 10 (21) −10 (100 − ST ) −10 (100 − ST )


Total 3. 6 0 2 (ST − 80) ≥ 0

T T
Transaction t=0 100 ≤ ST < 105 ST ≥ 105
buy ten portfolios
buy two 80-strike puts −2 (4) 0 0
buy eight 105-strike puts −8 (24.8) 8 (105 − ST ) 0
Portfolio total −84 8 (105 − ST ) 0

Sell ten 100-strike puts 10 (21) 0 0


Total 3. 6 8 (105 − ST ) > 0 0

−2 (4) − 8 (24.8) = −206. 4


2 (80 − ST ) + 8 (105 − ST ) = 1000 − 10ST
−206. 4 + 10 (21) = 3. 6
1000 − 10ST − 10 (100 − ST ) = 0
8 (105 − ST ) − 10 (100 − ST ) = 2 (ST − 80)

We receive $3. 6 at t = 0, but we don’t incur any negative cash flows at T .


So we make at least $3. 6 free money.

Problem 9.12.

www.myactuaryexam.com c
°Yufeng Guo 12
CHAPTER 9. PARITY AND OTHER OPTION RELATIONSHIPS

For two European options differing only in strike price, the following condi-
tions must be met to avoid arbitrage (see my study guide for explanation):
0 ≤ CEur (K1 , T ) − CEur (K2 , T ) ≤ P V (K2 − K1 ) if K1 < K2
0 ≤ PEur (K2 , T ) − PEur (K1 , T ) ≤ P V (K2 − K1 ) if K1 < K2

a.
Strike K1 = 90 K2 = 95
Call premium 10 4

C (K1 ) − C (K2 ) = 10 − 4 = 6
K2 − K1 = 95 − 90 = 5
C (K1 ) − C (K2 ) > K2 − K1 ≥ P V (K2 − K1 )
Arbitrage opportunities exist.
To arbitrage, we buy low and sell high. The cheap call is the 95-strike call;
the expensive call is the 90-strike call.
We use T to represent the common exercise date. This definition works
whether the two options are American or European. If the two options are
American, we’ll find arbitrage opportunities if two American options are ex-
ercised simultaneously. If the two options are European, T is the common
expiration date.
The payoff is:
T T T
Transaction t = 0 ST < 90 90 ≤ ST < 95 ST ≥ 95
Buy 95 strike call −4 0 0 ST − 95
Sell 90 strike call 10 0 − (ST − 90) − (ST − 90)
Total 6 0 − (ST − 90) ≥ −5 −5

We receive $6 at t = 0, yet we our max liability at T is −5. So we’ll make


at least $1 free money.

b.
T =2 r = 0.1
Strike K1 = 90 K2 = 95
Call premium 10 5.25

C (K1 ) − C (K2 ) = 10 − 5.25 = 4. 75


K2 − K1 = 95 − 90 = 5
P V (K2 − K1 ) = 5e−0.1(2) = 4. 094
C (K1 ) − C (K2 ) > P V (K2 − K1 )
Arbitrage opportunities exist.

Once again, we buy low and sell high. The cheap call is the 95-strike call;
the expensive call is the 90-strike call.
The payoff is:

www.myactuaryexam.com c
°Yufeng Guo 13
CHAPTER 9. PARITY AND OTHER OPTION RELATIONSHIPS

T T T
Transaction t=0 ST < 90 90 ≤ ST < 95 ST ≥ 95
Buy 95 strike call −5.25 0 0 ST − 95
Sell 90 strike call 10 0 − (ST − 90) − (ST − 90)
Deposit 4. 75 in savings −4. 75 4. 75e0.1(2) 4. 75e0.1(2) 4. 75e0.1(2)
Total 0 5. 80 95. 80 − ST > 0 0.80

4. 75e0.1(2) = 5. 80
− (ST − 90) + 4. 75e0.1(2) = 95. 80 − ST
ST − 95 − (ST − 90) + 4. 75e0.1(2) = 0.80
Our initial cost is zero. However, our payoff is always non-negative. So we
never lose money. This is clearly an arbitrage.
It’s important that the two calls are European options. If they are American,
they can be exercised at different dates. Hence the following non-arbitrage
conditions work only for European options:
0 ≤ CEur (K1 , T ) − CEur (K2 , T ) ≤ P V (K2 − K1 ) if K1 < K2
0 ≤ PEur (K2 , T ) − PEur (K1 , T ) ≤ P V (K2 − K1 ) if K1 < K2

c.
We are given the following 3 calls:
Strike K1 = 90 K2 = 100 K3 = 105
Call premium 15 10 6

1
λ90 + (1 − λ) 105 = 100 λ=
3
1 2
→ (90) + (105) = 100
∙3 3 ¸
1 2
C (90) + (105) = C (100) = 10
3 3

1 2 1 2
C (90) + C (105) = (15) + (6) = 9
3∙ 3 ¸ 3 3
1 2 1 2
C (90) + (105) > C (90) + C (105)
3 3 3 3
Hence arbitrage opportunities exist. To arbitrage, we buy low and sell high.
1
The cheap asset is the diversified portfolio consisting of unit of 90-strike
3
2
call and unit of 105-strike call.
3
The expensive asset is the 100-strike call.
Since we can’t buy a partial option, we’ll buy 3 units of the portfolio (i.e.
buy one 90-strike call and two 105-strike calls). Simultaneously,we sell three
100-strike calls.

The payoff at expiration T :

www.myactuaryexam.com c
°Yufeng Guo 14
CHAPTER 9. PARITY AND OTHER OPTION RELATIONSHIPS

T T T T
Transaction t=0 ST < 90 90 ≤ ST < 100 100 ≤ ST < 105 ST ≥ 105
buy 3 portfolios
buy one 90-strike call −15 0 ST − 90 ST − 90 ST − 90
buy two 105-strike calls 2 (−6) 0 0 0 2 (ST − 105)
Portfolios total −27 0 ST − 90 ST − 90 3ST − 300

Sell three 100-strike calls 3 (10) 0 0 −3 (ST − 100) −3 (ST − 100)


Total 3 0 ST − 90 ≥ 0 2 (105 − ST ) > 0 0
−15 + 2 (−6) = −27
ST − 90 + 2 (ST − 105) = 3ST − 300
−27 + 3 (10) = 3
ST − 90 − 3 (ST − 100) = 210 − 2ST = 2 (105 − ST )
3ST − 300 − 3 (ST − 100) = 0

So we receive $3 at t = 0, but we incur no negative payoff at T . So we’ll


make at least $3 free money.

Problem 9.13.

a. If the stock pays dividend, then early exercise of an American call option
may be optimal.
Suppose the stock pays dividend at tD .
Time 0 ... ... tD ... ... T

Pros and cons for exercising the call early at tD .

• +. If you exercise the call immediately before tD , you’ll receive dividend


and earn interest during [tD , T ]

• −. You’ll pay the strike price K at tD , losing interest you could have
earned during [tD , T ]. If the interest rate, however, is zero, you won’t lose
any interest.

• −. You throw away the remaining call option during [tD , T ]. Had you
waited, you would have had the call option during [tD , T ]

If the accumulated value of the dividend exceeds the value of the remaining
call option, then it’s optimal to exercise the stock at tD .
As explained in my study guide, it’s never optimal to exercise an American
put early if the interest rate is zero.

Problem 9.14.

www.myactuaryexam.com c
°Yufeng Guo 15
CHAPTER 9. PARITY AND OTHER OPTION RELATIONSHIPS

a. The only reason that early exercise might be optimal is that the underlying
asset pays a dividend. If the underlying asset doesn’t pay dividend, then it’s
never optimal to exercise an American call early. Since Apple doesn’t pay
dividend, it’s never optimal to exercise early.

b. The only reason to exercise an American put early is to earn interest on


the strike price. The strike price in this example is one share of AOL stock.
Since AOL stocks won’t pay any dividends, there’s no benefit for owning an
AOL stock early. Thus it’s never optimal to exercise the put.
If the Apple stock price goes to zero and will always stay zero, then there’s
no benefit for delaying exercising the put; there’s no benefit for exercising the
put early either (since AOL stocks won’t pay dividend). Exercising the put
early and exercising the put at maturity have the same value.
If, however, the Apple stock price goes to zero now but may go up in the
future, then it’s never optimal to exercise the put early. If you don’t exercise
early, you leave the door open that in the future the Apple stock price may
exceed the AOL stock price, in which case you just let your put expire worthless.
If the Apple stock price won’t exceed the AOL stock price, you can always
exercise the put and exchange one Apple stock for one AOL stock. There’s no
hurry to exercise the put early.

c. If Apple is expected to pay dividend, then it might be optimal to exercise


the American call early and exchange one AOL stock for one Apple stock.
However, as long as the AOL stock won’t pay any dividend, it’s never optimal
to exercise the American put early to exchange one Apple stock for one AOL
stock.

Problem 9.15.
This is an example where the strike price grows over time.

If the strike price grows over time, the longer-lived option is at least as
valuable as the shorter lived option. Refer to Derivatives Markets Page 298.
We have two European calls:
Call #1 K1 = 100e0.05(1.5) = 107. 788 T1 = 1.5 C1 = 11.50
Call #2 K2 = 100e0.05 = 105. 127 T2 = 1 C2 = 11.924

The longer-lived call is cheaper than the shorter-lived call, leading to arbi-
trage opportunities. To arbitrage, we buy low (Call #1) and sell high (Call
#2).
The payoff at expiration T1 = 1.5 if ST2 < 100e0.05 = 105. 127

T1 T1
Transaction t=0 T2 ST1 < 100e0.05(1.5) ST1 ≥ 100e0.05(1.5)
Sell Call #2 11.924 0 0 0
buy Call #1 −11.50 0 ST1 − 100e0.05(1.5)
Total 0.424 0 ST1 − 100e0.05(1.5) ≥ 0

www.myactuaryexam.com c
°Yufeng Guo 16
CHAPTER 9. PARITY AND OTHER OPTION RELATIONSHIPS

We receive $0.424 at t = 0, yet our payoff at T1 is always non-negative. This


is clearly an arbitrage.

The payoff at expiration T1 = 1.5 if ST2 ≥ 100e0.05 = 105. 127

T1 T1
Transaction t=0 T2 ST1 < 100e0.05(1.5) ST1 ≥ 100e0.05(1.5)
Sell Call #2 11.924 100e0.05 − ST2 100e0.05(1.5) − ST1 100e0.05(1.5) − ST1
buy Call #1 −11.50 0 ST1 − 100e0.05(1.5)
Total 0.424 100e0.05(1.5) − ST1 < 0 0

If ST2 ≥ 100e0.05 , then payoff of the sold Call #2 at T2 is 100e0.05 − ST2 .


From T2 to T1 ,
¡ ¢ ¡ ¢
• 100e0.05 grows into 100e0.05 e0.05(T1 −T2 ) = 100e0.05 e0.05(0.5) = 100e0.05(1.5)
• ST2 becomes ST1 (i.e. the stock price changes from ST2 to ST1 )
We receive $0.424 at t = 0, yet our payoff at T1 can be negative. This is not
an arbitrage.

So as long as ST2 < 100e0.05 = 105. 127 , there’ll be arbitrage opportunities.

Problem 9.16.
Suppose we do the following at t = 0:
1. Pay C a to buy a call
2. Lend P V (K) = Ke−rL at rL
3. Sell a put, receiving P b
4. Short sell one stock, receiving S0b

The net cost is P b + S0b − (C a + Ke−rL ).


The payoff at T is:
If ST < K If ST ≥ K
Transactions t=0
Buy a call −C a 0 ST − K
Lend Ke−rL at rL −KerL K K
Sell a put Pb ST − K 0
Short sell one stock S0b −ST −ST
Total P b + S0b − (C a + Ke−rL ) 0 0

The payoff is always zero. To avoid arbitrage, we need to have


P b + S0b − (C a + Ke−rL ) ≤ 0

Similarly, we can do the following at t = 0:

www.myactuaryexam.com c
°Yufeng Guo 17
CHAPTER 9. PARITY AND OTHER OPTION RELATIONSHIPS

1. Sell a call, receiving C b


2. Borrow P V (K) = Ke−rB at rB
3. Buy a put, paying P a
4. Buy one stock, paying S0a

¡ ¢ ¡ ¢
The net cost is C b + Ke−rB − P b + S0b .
The payoff at T is:
If ST < K If ST ≥ K
Transactions t=0
Sell a call Cb 0 K − ST
Borrow Ke−rB at rB Ke−rB −K −K
Buy a put −P a K− ST 0
a
Buy one stock −S
¡ 0b ¢ ¡ ¢ ST ST
Total C + Ke−rB − P b + S0b 0 0

The
¡ b payoff−ris ¢always
¡ zero. ¢To avoid arbitrage, we need to have
C + Ke B − P b + S0b ≤ 0

Problem 9.17.
a. According to the put-call parity, the payoff of the following position is
always zero:

1. Buy the call


2. Sell the put
3. Short the stock
4. Lend the present value of the strike price plus dividend

The existence of the bid-ask spread and the borrowing-lending rate difference
doesn’t change the zero payoff of the above position. The above position always
has a zero payoff whether there’s a bid-ask spread or a difference between the
borrowing rate and the lending rate.
If there is no transaction cost such as a bid-ask spread, the initial gain of
the above position is zero. However, if there is a bid-ask spread, then to avoid
arbitrage, the initial gain of the above position should be zero or negative.
¡Theb initial
¢ gain of the position is:
P + S0b − [C a + P VrL (K) + P VrL (Div)]
There’s
¡ b no
¢ arbitrage if
P + S0b − [C a + P VrL (K) + P VrL (Div)] ≤ 0

In this problem, we are given

www.myactuaryexam.com c
°Yufeng Guo 18
CHAPTER 9. PARITY AND OTHER OPTION RELATIONSHIPS

• rL = 0.019

• rB = 0.02

• S0b = 84.85. We are told to ignore the transaction cost. In addition, we


are given that the current stock price is 84.85. So S0b = 84.85.

• The dividend is 0.18 on November 8, 2004.

To find the expiration date, you need to know this detail. Puts and calls
are called equity options at the Chicago Board of Exchange (CBOE). In CBOE,
the expiration date of an equity option is the Saturday immediately following
the third Friday of the expiration month. (To verify this, go to www.cboe.com.
Click on "Products" and read "Production Specifications.")
If the expiration month is November, 2004, the third Friday is November 19.
Then the expiration date is November 20.
11/20/2004 − 10/15/2004 36
T = = = 0.09863
365 365
If the expiration month is January, 2005, the third Friday is January 21.
Then the expiration date is January 22, 2005.
1/22/2005 − 10/15/2004
T =
365
Calculate the days between 1/22/2005 and10/15/2004 isn’t easy. Fortu-

nately,we can use a calculator. BA II Plus and BA II Plus Professional have


"Date" Worksheet. When using Date Worksheet, use the ACT mode. ACT
mode calculates the actual days between two dates. If you use the 360 day
mode, you are assuming that there are 360 days between two dates.
When using the date worksheet, set DT1 (i.e. Date 1) as October 10, 2004
by entering 10.1504; set DT2 (i.e. Date 2) as January 22, 2004 by entering
1.2204. The calculator should tell you that DBD=99 (i.e. the days between two
days is 99 days).
1/22/2005 − 10/15/2004 99
So T = = = 0.271 23
365 365

If you have trouble using the date worksheet, refer to the guidebook of BA
II Plus or BA II Plus Professional.
11/8/2004 − 10/15/2004 24
The dividend day is tD = = = 0.06 575
365 365

P VrL (Div) = 0.18e−0.06575(0.019) = 0.18


P VrL (K) = Ke−0.019T
¡ b ¢
P + S0b − [C a + P VrL (K) + P VrL (Div)]
¡ ¢
= P b + 84.85 − C a + Ke−0.019T + 0.18

www.myactuaryexam.com c
°Yufeng Guo 19
CHAPTER 9. PARITY AND OTHER OPTION RELATIONSHIPS

¡ b ¢
K T Ca Pb P + S0b − [C a
¡ + P VrL (K) + P VrL (Div)] ¢
75 0.0986 10.3 0.2 0.2 + 84.85 − ¡10.3 + 75e−0.019×0.0986 + 0.18 ¢ = −0.29
80 0.0986 5.6 0.6 0.6 + 84.85 − ¡5.6 + 80e−0.019×0.0986 + 0.18¢ = −0.18
85 0.0986 2.1 2.1 2.1 + 84.85 − ¡2.1 + 85e−0.019×0.0986 + 0.18 ¢= −0.17
90 0.0986 0.35 5.5 5.5 + 84.85 − ¡0.35 + 90e−0.019×0.0986 + 0.18 ¢ = −1. 15
−0.019×0.271 2
75 0.271 2 10.9 0.7 0.7 + 84.85 − 10.9
¡ + 75e−0.019×0.271 2 + 0.18¢ = −0.14
80 0.271 2 6.7 1.45 1.45 + 84.85 −¡ 6.7 + 80e + 0.18¢ = −0.17
85 0.271 2 3.4 3.1 3.1 + 84.85 − ¡3.4 + 85e−0.019×0.271 2 + 0.18 ¢= −0.19
90 0.271 2 1.35 6.1 6.1 + 84.85 − 1.35 + 90e−0.019×0.271 2 + 0.18 = −0.12

b. According to the put-call parity, the payoff of the following position is


always zero:

1. Sell the call

2. Borrow the present value of the strike price plus dividend

3. Buy the put

4. Buy one stock

If there is transaction cost such as the bid-ask spread, then to avoid arbitrage,
the initial gain of the above position is zero. However, if there is a bid-ask spread,
the initial gain of the above position can be zero or negative.
The initial gain of the position is:
C b + P VrB (K) + P VrB (Div) − (P a + S0a )
There’s no arbitrage if
C b + P VrB (K) + P VrB (Div) − (P a + S0a ) ≤ 0
P VrB (Div) = 0.18e−0.06575(0.02) = 0.18
P VrL (K) = Ke−0.02T
C b +P VrB (K)+P VrB (Div)−(P a + S0a ) = C b +Ke−0.02T +0.18−(P a + 84.85)

K T Cb Pa C b + P VrB (K) + P VrB (Div) − (P a + S0a )


75 0.0986 9.9 0.25 9.9 + 75e−0.02×0.0986 + 0.18 − (0.25 + 84.85) = −0.17
80 0.0986 5.3 0.7 5.3 + 80e−0.02×0.0986 + 0.18 − (0.7 + 84.85) = −0.23
85 0.0986 1.9 2.3 1.9 + 85e−0.02×0.0986 + 0.18 − (2.3 + 84.85) = −0.24
90 0.0986 0.35 5.8 0.35 + 90e−0.02×0.0986 + 0.18 − (5.8 + 84.85) = −0.30
75 0.271 2 10.5 0.8 10.5 + 75e−0.02×0.271 2 + 0.18 − (0.8 + 84.85) = −0.38
80 0.271 2 6.5 1.6 6.5 + 80e−0.02×0.271 2 + 0.18 − (1.6 + 84.85) = −0.20
85 0.271 2 3.2 3.3 3.2 + 85e−0.02×0.271 2 + 0.18 − (3.3 + 84.85) = −0.23
90 0.271 2 1.2 6.3 1.2 + 90e−0.02×0.271 2 + 0.18 − (6.3 + 84.85) = −0.26

Problem 9.18.

Suppose there are 3 options otherwise identical but with different strike price
K1 < K2 < K3 where K2 = λK1 + (1 − λ) K2 and 0 < λ < 1.

www.myactuaryexam.com c
°Yufeng Guo 20
CHAPTER 9. PARITY AND OTHER OPTION RELATIONSHIPS

Then the price of the middle strike price K2 must not exceed the price of a
diversified portfolio consisting of λ units of K1 -strike option and (1 − λ) units
of K2 -strike option:

C [λK1 + (1 − λ) K3 ] ≤ λC (K1 ) + (1 − λ) C (K3 )


P [λK1 + (1 − λ) K3 ] ≤ λP (K1 ) + (1 − λ) P (K3 )

The above conditions are called the convexity of the option price with respect
to the strike price. They are equivalent to the textbook Equation 9.17 and 9.18.
If the above conditions are violated, arbitrage opportunities exist.

K T Cb Ca
80 0.271 2 6.5 6.7
85 0.271 2 3.2 3.4
90 0.271 2 1.2 1.35

85 = λ (80) + (1 − λ) (90) → λ = 0.5


a.
If we buy a 80-strike call, buy a 90-strike call, sell two 85-strike calls

• A 80-strike call and a 90-strike call form a diversified portfolio of calls,


which is always as good as two 85-strike calls

• So the cost of buying a 80-strike call and a 90-strike call can never be less
than the revenue of selling two 85-strike calls

What we pay if we buy a 80-strike call and a 90-strike call: 6.7 + 1.35 = 8.
05
What we get if we sell two 85-strike calls: 3.2 × 2 = 6. 4
8. 05 > 6. 4
So the convexity condition is met.
I recommend that you don’t bother memorizing textbook Equation 9.17 and
9.18.

b. If we sell a 80-strike call, sell a 90-strike call, buy two 85-strike calls

• A 80-strike call and a 90-strike call form a diversified portfolio of calls,


which is always as good as two 85-strike calls

• So the revenue of selling a 80-strike call and a 90-strike call should never
be less than the cost of buying two 85-strike calls.

What we get if we sell a 80-strike call and a 90-strike call: 6.5 + 1.2 = 7. 7
What we pay if we buy two 85-strike calls: 3.4 × 2 = 6. 8
7. 7 > 6.8

www.myactuaryexam.com c
°Yufeng Guo 21
CHAPTER 9. PARITY AND OTHER OPTION RELATIONSHIPS

So the convexity condition is met.

c. To avoid arbitrage, the following two conditions must be met:


C [λK1 + (1 − λ) K3 ] ≤ λC (K1 ) + (1 − λ) C (K3 )
P [λK1 + (1 − λ) K3 ] ≤ λP (K1 ) + (1 − λ) P (K3 )
These conditions must be met no matter whether you are a market-maker
or anyone else buying or selling options, no matter whether you pay a bid-ask
spread or not.

www.myactuaryexam.com c
°Yufeng Guo 22
Chapter 10

Binomial option pricing I

Problem 10.1.

The stock price today is S = 100. The stock at T is either

• Su = uS = 1.3 × 100 = 130

• Sd = dS = 0.8 × 100 = 80

a. For a call, the payoff at T is

• Vu = max (0, Su − K) = max (0, 130 − 105) = 25

• Vd = max (0, Sd − K) = max (0, 80 − 105) = 0

We hold a replicating portfolio (4, B) at t = 0. This portfolio will have


value Vu if the stock goes up to Su or Vd if the stock goes down to Sd . We set
up the following equations:
½ ½
4Su + BerT = Vu 4130 + Be0.08(0.5) = 25
rT →
4Sd + Be = Vd 480 + Be0.08(0.5) = 0
→ B = −38. 431 6 4 = 0.5
So the option premium is:
V = 4S + B = 0.5 × 100 + (−38. 431 6) = 11. 568 4

b. For a put, the payoff at T is either

• Vu = max (0, K − Su ) = max (0, 105 − 130) = 0

• Vd = max (0, Sd − K) = max (0, 105 − 80) = 25

23
CHAPTER 10. BINOMIAL OPTION PRICING I

We hold a replicating portfolio (4, B) at t = 0. This portfolio will have


value Vu if the stock goes up to Su or Vd if the stock goes down to Sd . We set
up the following equations:
½ ½
4Su + BerT = Vu 4130 + Be0.08(0.5) = 0
rT
4Sd + Be = Vd 480 + Be0.08(0.5) = 25
B = 62. 451 3 4 = −0.5
So the option premium is:
V = 4S + B = −0.5 × 100 + 62. 451 3 = 12. 451 3

Problem 10.2.

The stock price today is S = 100.The stock at T

• Su = uS = 1.3 × 100 = 130

• Sd = dS = 0.8 × 100 = 80

a. Payoff at T is either

• Vu = max (0, Su − K) = max (0, 130 − 95) = 35

• Vd = max (0, Sd − K) = max (0, 80 − 95) = 0

We hold a replicating portfolio (4, B) at t = 0. This portfolio will have


value Vu if the stock goes up to Su or Vd if the stock goes down to Sd . We set
up the following equations:
½ ½
4Su + BerT = Vu 4130 + Be0.08(0.5) = 35
rT
4Sd + Be = Vd 480 + Be0.08(0.5) = 0
B = −53. 804 2 4 = 0.7

So the option premium is:


V = 4S + B = 0.7 × 100 + (−53. 804 2) = 16. 195 8

b. There are two calls out there. One can be synthetically built by buying
0.7 share of a stock and borrowing $53. 804 2 from a bank. The other is in the
market selling for $17. To arbitrage, we buy low and sell high.

• Buy low. Buy 0.7 share of a stock and borrow $53. 804 2 from a bank. Our
initial cash outgo is 0.7 × 100 + (−53. 804 2) = 16. 195 8. This grows to
either (0.7) 130−53. 804 2e0.08(0.5) = 35 if the stock goes up or (0.7) 80−53.
804 2e0.08(0.5) = 0 if the stock goes down at T .

• Sell high. We sell a call for 17, receiving $17 at time zero. We pay either
max (0, 130 − 95) = 35 if the stock goes up or max (0, 80 − 95) = 0 if the
stock goes down at T .

www.myactuaryexam.com c
°Yufeng Guo 24
CHAPTER 10. BINOMIAL OPTION PRICING I

The net cash flow at T is zero. The net cash inflow at time zero is 17 −
16. 195 8 = 0.804 2. So we receive $0.804 2 at time zero without incurring any
liability at T ; we have made $0.804 2 free money. Millions of investors will copy
this arbitraging strategy, which will bid down the call price from 17 to the fair
price of 16. 195 8.

c. There are two calls out there. One can be synthetically built by buying
0.7 share of a stock and borrowing $16. 195 8 from a bank. The other is in the
market selling for $15.5. To arbitrage, we buy low and sell high.

• Sell high. Short sell 0.7 share of a stock and deposit $53. 804 2 in a bank.
Our initial cash inflow is 0.7 × 100 + (−53. 804 2) = 16. 195 8. At T , we
need to buy back 0.7 share of a stock from the market to close our short
position. In addition, our bank account grows into 53. 804 2e0.08(0.5) . So
our cash inflow at T is either (−0.7) 130 + 53. 804 2e0.08(0.5) = −35 if the
stock goes up or (−0.7) 80 + 53. 804 2e0.08(0.5) = 0 if the stock goes down.
• Buy low. We buy a call for 15.5 at time zero, paying $15.5. We get either
max (0, 130 − 95) = 35 if the stock goes up or max (0, 80 − 95) = 0 if the
stock goes down at T .

The net cash flow at T is zero. The initial cash inflow is 16. 195 8 − 15.5 =
0.695 8. So we receive $0.695 8 at time zero without incurring any liability
at T ; we have made $0.695 8 free money. Millions of investors will copy this
arbitraging strategy, which will bid up the call price from 15.5 to the fair price
of 16. 195 8.

Problem 10.3.
The stock price today is S = 100. The stock at T is

• Su = uS = 1.3 × 100 = 130


• Sd = dS = 0.8 × 100 = 80

a. Payoff at T is

• Vu = max (0, K − Su ) = max (0, 95 − 130) = 0


• Vd = max (0, K − Sd ) = max (0, 95 − 80) = 15

We hold a replicating portfolio (4, B) at t = 0. This portfolio will have


value Vu if the stock goes up to Su or Vd if the stock goes down to Sd . We set
up the
½ following rT equations: ½
4Su + Be = Vu 4130 + Be0.08(0.5) = 0
4Sd + BerT = Vd 480 + Be0.08(0.5) = 15
B = 37. 470 9 4 = −0.3

www.myactuaryexam.com c
°Yufeng Guo 25
CHAPTER 10. BINOMIAL OPTION PRICING I

So the option premium is:


V = 4S + B = −0.3 × 100 + 37. 470 9 = 7. 470 9

b. There are two puts out there. One can be synthetically built by short
selling 0.3 share of a stock and depositing $37. 470 9 in a bank. The other is in
the market selling for $8. To arbitrage, we buy low and sell high.

• Buy low. Short sell 0.3 share of a stock and deposit $37. 470 9 in a bank.
Our initial cash inflow is 0.3 × 100 − 37. 470 9 = −7. 470 9. At T , we need
to buy back 0.3 share of a stock to close our short position. In addition,
our initial deposit grows to 37. 470 9e0.08(0.5) = 39. If the stock goes up,
our cash outgo at T is (−0.3) 130 + 37. 470 9e0.08(0.5) = 0; if the stock goes
down to 80 at T , our cash outgo is (−0.3) 80 + 37. 470 9e0.08(0.5) = 15.

• Sell high. We sell a put for 8, receiving $8 at time zero. At T , we pay


either max (0, 95 − 130) = 0 if the stock goes up or max (0, 95 − 80) = 15
if the stock goes down.

The net cash flow of "buy low, sell high" is zero at T . The net cash inflow at
time zero is 8 − 7. 470 9 = 0.529 1. So we receive $0.529 1 at time zero without
incurring any liability at T ; we have made $0.529 1 free money. Millions of
investors will copy this arbitraging strategy, which will bid down the put price
from 8 to the fair price of 7. 470 9.

c. There are two puts out there. One can be synthetically built by short
selling 0.3 share of a stock and depositing $37. 470 9 in a bank. The other is in
the market selling for $6. To arbitrage, we buy low and sell high.

• Buy low. We buy a put for $6 at time zero. At T we receive either


max (0, 95 − 130) = 0 if the stock goes up or max (0, 95 − 80) = 15 if the
stock goes down.

• Sell high. At time zero we buy 0.3 share of a stock and borrow $37. 470 9
in a bank. Our initial cash inflow is −0.3 × 100 + 37. 470 9 = 7. 470 9. At
T , we sell 0.3 share of a stock, receiving 0.3 × 130 = 39 if the stock goes
up or receiving 0.3 × 80 = 24 if the stock goes down. In addition, we need
to pay the bank 37. 470 9e0.08(0.5) = 39. So our net cash inflow at T is
39 − 39 = 0 if the stock goes up or 24 − 39 = −15 if the stock goes down.

The net cash flow of "buy low, sell high" is zero at T . The net cash inflow at
time zero is 7. 470 9 − 6 = 1. 470 9. So we receive $1. 470 9 at time zero without
incurring any liability at T ; we have made $1. 470 9 free money. Millions of
investors will copy this arbitraging strategy, which will bid up the put price
from 6 to the fair price of 7. 470 9.

Problem 10.4.

www.myactuaryexam.com c
°Yufeng Guo 26
CHAPTER 10. BINOMIAL OPTION PRICING I

The problem doesn’t say the option is European or American. Let’s assume
the option is European.
T =1 n=2 →period length h = T /2 = 0.5
Period 0 1 2 ¡ ¢
Suu = 100 1.32 = 169
Su = 100 (1.3) = 130 Vuu = 169 − 95 = 74
Vu =?
(∆u , Bu )
Sud = 100 (1.3) (0.8) = 104
S = 100 Vud = 104 − 95 = 9
V =?
(∆, B) =?
Sd = 100 (0.8) = 80 ¡ ¢
Vd =? Sdd = 100 0.82 = 64
(∆d , Bd ) Vdd = 0

We start from right to left.


Step 1 Calculate (∆u , Bu ), the replicating portfolio at Node u which will
produce
½ the payoff Vud and Vdd . ½
4u Suu + Bu erh = Vuu 4u 169 + Bu e0.08(0.5) = 74
rh →
4u Sud + Bu e = Vud 4u 104 + Bu e0.08(0.5) = 9
4u = 1 Bu = −91. 2750

The premium at Node u is:


Vu = 4u Su + Bu = 1 (130) − 91. 2750 = 38. 725

Step 2 Calculate(∆d , Bd ), the replicating portfolio at Node d which will


produce
½ the payoff Vud and Vdd . ½
4d Sud + Bd erh = Vud 4d 104 + Bd e0.08(0.5) = 9
rh →
4d Sdd + Bd e = Vdd 4d 64 + Bd e0.08(0.5) = 0
4d = 0.225 , Bd = −13. 835 4

The premium at Node d is:


Vd = 4d Sd + Bd = 0.225 (80) − 13. 835 4 = 4. 164 6

Step 3 Calculate(∆, B), the replicating portfolio at time zero, which will
produce
½ the payoff Vu and Vd . ½
4Su + BerT = Vu 4130 + Be0.08(0.5) = 38. 725
rT →
4Sd + Be = Vd 480 + Be0.08(0.5) = 4. 164 6
4 = 0.691 2 B = −49. 127 1

The premium at time zero is:


V = 4S + B = 0.691 2 (100) − 49. 127 1 = 19. 992 9

The final diagram is:

www.myactuaryexam.com c
°Yufeng Guo 27
CHAPTER 10. BINOMIAL OPTION PRICING I

Period 0 1 2 ¡ ¢
Suu = 100 1.32 = 169
Su = 100 (1.3) = 130 Vuu = 169 − 95 = 74
Vu = 38. 725
∆u = 1
Bu = −$91. 2750
S = 100 Sud = 100 (1.3) (0.8) = 104
V = 19. 992 9 Vud = 104 − 95 = 9
∆ = 0.691 2
B = −$49. 127 1
Sd = 100 (0.8) = 80 ¡ ¢
Vd = 4. 164 6 Sdd = 100 0.82 = 64
∆d = 0.225 Vdd = 0
Bd = −$13. 835 4

Problem 10.5.

This question asks us to redo the previous problem by setting the initial
stock price to 80, 90, 110, 120, and 130.

• S = 80.
Period 0 1 2 ¡ ¢
Suu = 80 1.32 = 135. 2
Su = 80 (1.3) = 104 Vuu = 135. 2 − 95 = 40. 2
Vu =?
(∆u , Bu ) =?
Sud = 80 (1.3) (0.8) = 83. 2
S = 80 Vud = 0
V =?
(∆, B) =?
Sd = 80 (0.8) = 64 ¡ ¢
Vd =? Sdd = 80 0.82 = 51. 2
(∆d , Bd ) =? Vdd = 0
½
4u 135. 2 + Bu e0.08(0.5) = 40. 2
4u = 0.773 08 Bu = −61. 7980
4u 83. 2 + Bu e0.08(0.5) = 0
Vu = 4u Su + Bu = 0.773 08 (104) − 61. 7980 = 18. 602 32
½
4d 83. 2 + Bd e0.08(0.5) = 0
4d = 0.0 Bd = 0
4d 51. 2 + Bd e0.08(0.5) = 0
Vd = 4d Sd + Bd = 0 (64) − 0 = 0
½
4104 + Be0.08(0.5) = 18. 602 32
4 = 0.465 058 B = −28. 596 7
464 + Be0.08(0.5) = 0

www.myactuaryexam.com c
°Yufeng Guo 28
CHAPTER 10. BINOMIAL OPTION PRICING I

V = 4S + B = 0.465 058 (80) − 28. 596 7 = 8. 607 94

The final diagram is:


Period 0 1 2 ¡ ¢
Suu = 80 1.32 = 135. 2
Su = 80 (1.3) = 104 Vuu = 135. 2 − 95 = 40. 2
Vu = 18. 602 32
∆u = 0.773 08
Bu = −61. 798
Sud = 80 (1.3) (0.8) = 83. 2
S = 80 Vud = 0
V = 8. 607 94
∆ = 0.465 058
B = −28. 596 7
Sd = 80 (0.8) = 64 ¡ ¢
Vd = 0 Sdd = 80 0.82 = 51. 2
∆d = 0 Vdd = 0
Bd = 0

www.myactuaryexam.com c
°Yufeng Guo 29
CHAPTER 10. BINOMIAL OPTION PRICING I

• S = 90.
Period 0 1 2 ¡ ¢
Suu = 90 1.32 = 152. 1
Su = 90 (1.3) = 117 Vuu = 152. 1 − 95 = 57. 1
Vu =?
4u =?
Bu =?
S = 90 Sud = 90 (1.3) (0.8) = 93. 6
V =? Vud = 0
∆ =?
B =?
Sd = 90 (0.8) = 72 ¡ ¢
Vd =? Sdd = 90 0.82 = 57. 6
∆d =? Vdd = 0
Bd =?
½
4u 152. 1 + Bu e0.08(0.5) = 57. 1
4u = 0.976 068 Bu = −87. 777 7
4u 93. 6 + Bu e0.08(0.5) = 0
Vu = 4u Su + Bu = 0.976 068 (117) − 87. 777 7 = 26. 422 26
½
4d 93. 6 + Bd e0.08(0.5) = 0
4d = 0.0 Bd = 0
4d 57. 6 + Bd e0.08(0.5) = 0
Vd = 4d Sd + Bd = 0 (72) − 0 = 0
½
4117 + Be0.08(0.5) = 26. 422 26
4 = 0.587 161 B = −40. 617 97
472 + Be0.08(0.5) = 0
V = 4S + B = 0.587 161 (90) − 40. 617 97 = 12. 226 52

The final diagram is:


Period 0 1 2 ¡ ¢
Suu = 90 1.32 = 152. 1
Su = 90 (1.3) = 117 Vuu = 152. 1 − 95 = 57. 1
Vu = 26. 422 26
4u = 0.976 068
Bu = −87. 777 7
S = 90 Sud = 90 (1.3) (0.8) = 93. 6
V = 12. 226 52 Vud = 0
4 = 0.587 161
B = −40. 617 97
Sd = 90 (0.8) = 72 ¡ ¢
Vd = 0 Sdd = 90 0.82 = 57. 6
∆d = 0 Vdd = 0
Bd = 0 0

www.myactuaryexam.com c
°Yufeng Guo 30
CHAPTER 10. BINOMIAL OPTION PRICING I

• S = 110.
Period 0 1 ¡2 ¢
Suu = 110 1.32 = 185. 9
Su = 110 (1.3) = 143 Vuu = 185. 9 − 95 = 90. 9
Vu =?
4u =?
Bu =?
S = 110 Sud = 110 (1.3) (0.8) = 114. 4
V =? Vud = 114. 4 − 95 = 19. 4
∆ =?
B =?
Sd = 110 (0.8) = 88 ¡ ¢
Vd =? Sdd = 110 0.82 = 70. 4
∆d =? Vdd = 0
Bd =?
½
4u 185. 9 + Bu e0.08(0.5) = 90. 9
4u = 1 Bu = −91. 2750
4u 114. 4 + Bu e0.08(0.5) = 19. 4
Vu = 4u Su + Bu = 1 (143) − 91. 2750 = 51. 725
½
4d 114. 4 + Bd e0.08(0.5) = 19. 4
4d = 0.440 909 Bd = −29. 822 9
4d 70. 4 + Bd e0.08(0.5) = 0
Vd = 4d Sd + Bd = 0.440 909 (88) − 29. 822 9 = 8. 977 092
½
4143 + Be0.08(0.5) = 51. 725
4 = 0.777 235 B = −57. 089 7
488 + Be0.08(0.5) = 8. 977 092
V = 4S + B = 0.777 235 (110) − 57. 089 7 = 28. 406 15

The final diagram is:


Period 0 1 2 ¡ ¢
Suu = 110 1.32 = 185. 9
Su = 110 (1.3) = 143 Vuu = 185. 9 − 95 = 90. 9
Vu = 51. 725
4u = 1
Bu = −91. 2750
S = 110 Sud = 110 (1.3) (0.8) = 114. 4
28. 406 15 Vud = 114. 4 − 95 = 19. 4
4 = 0.777 235
B = −57. 089 7
Sd = 110 (0.8) = 88 ¡ ¢
Vd = 8. 977 092 Sdd = 110 0.82 = 70. 4
∆d = 0.440 909 Vdd = 0
Bd = −29. 822 9

www.myactuaryexam.com c
°Yufeng Guo 31
CHAPTER 10. BINOMIAL OPTION PRICING I

• S = 120.
Period 0 1 2 ¡ ¢
Suu = 120 1.32 = 202. 8
Su = 120 (1.3) = 156 Vuu = 202. 8 − 95 = 107. 8
Vu =?
4u =?
Bu =?
S = 120 Sud = 120 (1.3) (0.8) = 124. 8
V =? Vud = 124. 8 − 95 = 29. 8
∆ =?
B =?
Sd = 120 (0.8) = 96 ¡ ¢
Vd =? Sdd = 120 0.82 = 76. 8
∆d =? Vdd = 0
Bd =?
½
4u 202. 8 + Bu e0.08(0.5) = 107. 8
4u = 1 Bu = −91. 275
4u 124. 8 + Bu e0.08(0.5) = 29. 8
Vu = 4u Su + Bu = 1 (156) − 91. 275 = 64. 725
½
4d 124. 8 + Bd e0.08(0.5) = 29. 8
4d = 0.620 833 Bd = −45. 810 44
4d 76. 8 + Bd e0.08(0.5) = 0
Vd = 4d Sd + Bd = 0.620 833 (96) − 45. 810 44 = 13. 789 528
½
4156 + Be0.08(0.5) = 64. 725
4 = 0.848 925 B = −65. 052 4
496 + Be0.08(0.5) = 13. 789 528
V = 4S + B = 0.848 925 (120) − 65. 052 4 = 36. 818 6

The final diagram is:


Period 0 1 ¡2 ¢
Suu = 120 1.32 = 202. 8
Su = 120 (1.3) = 156 Vuu = 202. 8 − 95 = 107. 8
Vu = 64. 725
4u = 1
Bu = −91. 275
S = 120 Sud = 120 (1.3) (0.8) = 124. 8
V = 36. 818 6 Vud = 124. 8 − 95 = 29. 8
4 = 0.848 925
B = −65. 052 4
Sd = 120 (0.8) = 96 ¡ ¢
Vd = 13. 789 528 Sdd = 120 0.82 = 76. 8
4d = 0.620 833 Vdd = 0
Bd = −45. 810 44

www.myactuaryexam.com c
°Yufeng Guo 32
CHAPTER 10. BINOMIAL OPTION PRICING I

• S = 130.
Period 0 1 2 ¡ ¢
Suu = 130 1.32 = 219. 7
Su = 130 (1.3) = 169 Vuu = 219. 7 − 95 = 124. 7
Vu =?
4u =?
Bu =?
S = 130 Sud = 130 (1.3) (0.8) = 135. 2
V =? Vud = 135. 2 − 95 = 40. 2
∆ =?
B =?
Sd = 130 (0.8) = 104 ¡ ¢
Vd =? Sdd = 130 0.82 = 83. 2
∆d =? Vdd = 0
Bd =?
½
4u 219. 7 + Bu e0.08(0.5) = 124. 7
4u = 1 Bu = −91. 275
4u 135. 2 + Bu e0.08(0.5) = 40. 2
Vu = 4u Su + Bu = 1 (169) − 91. 275 = 77. 725
½
4d 135. 2 + Bd e0.08(0.5) = 40. 2
4d = 0.773 077 Bd = −61. 7980
4d 83. 2 + Bd e0.08(0.5) = 0
Vd = 4d Sd + Bd = 0.773 077 (104) − 61. 7980 = 18. 602
½
4169 + Be0.08(0.5) = 77. 725
4 = 0.909 585 B = −73. 015
4104 + Be0.08(0.5) = 18. 602
V = 4S + B = 0.909 585 (130) − 73. 015 = 45. 231
The final diagram is:
Period 0 1 2 ¡ ¢
Suu = 130 1.32 = 219. 7
Su = 130 (1.3) = 169 Vuu = 219. 7 − 95 = 124. 7
Vu = 77. 725
4u = 1
Bu = −91. 275
S = 130 Sud = 130 (1.3) (0.8) = 135. 2
V = 45. 231 Vud = 135. 2 − 95 = 40. 2
4 = 0.909 585
B = −73. 015
Sd = 130 (0.8) = 104 ¡ ¢
Vd = 18. 602 Sdd = 130 0.82 = 83. 2
4d = 0.773 077 Vdd = 0
Bd = −61. 7980

Notice that the delta for a call is always positive. A positive delta means
buying stocks. If you sell a call, the risk you face is that the stock price may go

www.myactuaryexam.com c
°Yufeng Guo 33
CHAPTER 10. BINOMIAL OPTION PRICING I

up to infinity, at which case the call holder will pay only the strike price to buy
a priceless stock from you. To hedge your risk, you need to already own some
stocks at t = 0. If indeed the future stock price goes up at T , your stock will
also go up in value, offsetting your loss in the call.
Intuitively, the higher the initial stock price, everything else equal, the higher
the stock price in the future, the higher the payoff of the call. Hence to hedge
the risk, the market maker needs to buy more shares of stocks at t = 0. So the
higher the initial stock price, the higher the initial delta, the more stocks the
market maker needs to buy at t = 0.

www.myactuaryexam.com c
°Yufeng Guo 34
CHAPTER 10. BINOMIAL OPTION PRICING I

Problem 10.6.
Period 0 1 2 ¡ ¢
Suu = 100 1.32 = 169
Su = 100 (1.3) = 130 Vuu = 0
Vu =?
4u =?
Bu =?
S = 100 Sud = 100 (1.3) (0.8) = 104
V =? Vud = 0
∆ =?
B =?
Sd = 100 (0.8) = 80 ¡ ¢
Vd =? Sdd = 100 0.82 = 64
∆d =? Vdd = 95 − 64 = 31
Bd =?
½
4u 169 + Bu e0.08(0.5) = 0
4u = 0 Bu = 0
4u 104 + Bu e0.08(0.5) = 0
Vu = 4u Su + Bu = 0 (130) + 0 = 0
½
4d 104 + Bd e0.08(0.5) = 0
4d = −0.775 Bd = 77. 439 6
4d 64 + Bd e0.08(0.5) = 31
Vd = 4d Sd + Bd = −0.775 (80) + 77. 439 6 = 15. 439 6
½
4130 + Be0.08(0.5) = 0
4 = −0.308 792 B = 38. 568 932
480 + Be0.08(0.5) = 15. 439 6
V = 4S + B = −0.308 792 (100) + 38. 568 932 = 7. 689 732
The final diagram is:
Period 0 1 2 ¡ ¢
Suu = 100 1.32 = 169
Su = 100 (1.3) = 130 Vuu = 0
Vu = 0
4u = 0
Bu = 0
S = 100 Sud = 100 (1.3) (0.8) = 104
V = 7. 689 732 Vud = 0
4 = −0.308 792
B = 38. 568 932
Sd = 100 (0.8) = 80 ¡ ¢
Vd = 15. 439 6 Sdd = 100 0.82 = 64
4d = −0.775 Vdd = 95 − 64 = 31
Bd = 77. 439 6

www.myactuaryexam.com c
°Yufeng Guo 35
CHAPTER 10. BINOMIAL OPTION PRICING I

Problem 10.7.

• S = 80
Period 0 1 2 ¡ ¢
Suu = 80 1.32 = 135. 2
Su = 80 (1.3) = 104 Vuu = 0
Vu =?
4u =?
Bu =?
S = 80 Sud = 80 (1.3) (0.8) = 83. 2
V =? Vud = 95 − 83. 2 = 11. 8
∆ =?
B =?
Sd = 80 (0.8) = 64 ¡ ¢
Vd =? Sdd = 80 0.82 = 51. 2
∆d =? Vdd = 95 − 51. 2 = 43. 8
Bd =?
½
4u 135. 2 + Bu e0.08(0.5) = 0
4u = −0.226 923 Bu = 29. 477 02
4u 83. 2 + Bu e0.08(0.5) = 11. 8
Vu = 4u Su + Bu = −0.226 923 (104) + 29. 477 02 = 5. 877
½
4d 83. 2 + Bd e0.08(0.5) = 11. 8
4d = −1 Bd = 91. 275
4d 51. 2 + Bd e0.08(0.5) = 43. 8
Vd = 4d Sd + Bd = −1 (64) + 91. 275 = 27. 275
½
4104 + Be0.08(0.5) = 5. 877
4 = −0.534 95 B = 59. 1
464 + Be0.08(0.5) = 27. 275
V = 4S + B = −0.534 95 (80) + 59. 1 = 16. 304
The final diagram is:
Period 0 1 2 ¡ ¢
Suu = 80 1.32 = 135. 2
Su = 80 (1.3) = 104 Vuu = 0
Vu = 5. 877
4u = −0.226 923
Bu = 29. 477 02
S = 80 Sud = 80 (1.3) (0.8) = 83. 2
V = 16. 304 Vud = 95 − 83. 2 = 11. 8
4 = −0.534 95
B = 59. 1
Sd = 80 (0.8) = 64 ¡ ¢
Vd = 27. 275 Sdd = 80 0.82 = 51. 2
4d = −1 Vdd = 95 − 51. 2 = 43. 8
Bd = 91. 275

www.myactuaryexam.com c
°Yufeng Guo 36
CHAPTER 10. BINOMIAL OPTION PRICING I

• S = 90
Period 0 1 2 ¡ ¢
Suu = 90 1.32 = 152. 1
Su = 90 (1.3) = 117 Vuu = 0
Vu =?
4u =?
Bu =?
S = 90 Sud = 90 (1.3) (0.8) = 93. 6
V =? Vud = 95 − 93. 6 = 1. 4
∆ =?
B =?
Sd = 90 (0.8) = 72 ¡ ¢
Vd =? Sdd = 90 0.82 = 57. 6
∆d =? Vdd = 95 − 57. 6 = 37. 4
Bd =?
½
4u 152. 1 + Bu e0.08(0.5) = 0
4u = −0.02 393 Bu = 3. 497 27
4u 93. 6 + Bu e0.08(0.5) = 1. 4
Vu = 4u Su + Bu = −0.02 393 (117) + 3. 497 27 = 0.697 5
½
4d 93. 6 + Bd e0.08(0.5) = 1. 4
4d = −1 Bd = 91. 275
4d 57. 6 + Bd e0.08(0.5) = 37. 4
Vd = 4d Sd + Bd = −1 (72) + 91. 275 = 19. 275
½
4117 + Be0.08(0.5) = 0.697 5
4 = −0.412 833 B = 47. 077 722
472 + Be0.08(0.5) = 19. 275
V = 4S + B = −0.412 833 (90) + 47. 077 722 = 9. 923
The final diagram is:
Period 0 1 2 ¡ ¢
Suu = 90 1.32 = 152. 1
Su = 90 (1.3) = 117 Vuu = 0
Vu = 0.697 5
4u = −0.02 393
Bu = 3. 497 27
S = 90 Sud = 90 (1.3) (0.8) = 93. 6
V = 9. 923 Vud = 95 − 93. 6 = 1. 4
4 = −0.412 833
B = 47. 077 722
Sd = 90 (0.8) = 72 ¡ ¢
Vd = 19. 275 Sdd = 90 0.82 = 57. 6
4d = −1 Vdd = 95 − 57. 6 = 37. 4
Bd = 91. 275

www.myactuaryexam.com c
°Yufeng Guo 37
CHAPTER 10. BINOMIAL OPTION PRICING I

• S = 110
Period 0 1 2 ¡ ¢
Suu = 110 1.32 = 185. 9
Su = 110 (1.3) = 143 Vuu = 0
Vu =?
4u =?
Bu =?
S = 110 Sud = 110 (1.3) (0.8) = 114. 4
V =? Vud = 0
∆ =?
B =?
Sd = 110 (0.8) = 88 ¡ ¢
Vd =? Sdd = 110 0.82 = 70. 4
∆d =? Vdd = 95 − 70. 4 = 24. 6
Bd =?
½
4u 185. 9 + Bu e0.08(0.5) = 0
4u = 0 Bu = 0
4u 114. 4 + Bu e0.08(0.5) = 0
Vu = 4u Su + Bu = 0 (143) + 0 = 0
½
4d 114. 4 + Bd e0.08(0.5) = 0
4d = −0.559 091 Bd = 61. 452 1
4d 70. 4 + Bd e0.08(0.5) = 24. 6
Vd = 4d Sd + Bd = −0.559 091 (88) + 61. 452 1 = 12. 252
½
4143 + Be0.08(0.5) = 0
4 = −0.222 76 B = 30. 606 14
488 + Be0.08(0.5) = 12. 252
V = 4S + B = −0.222 76 (110) + 30. 606 14 = 6. 103
The final diagram is:
Period 0 1 2 ¡ ¢
Suu = 110 1.32 = 185. 9
Su = 110 (1.3) = 143 Vuu = 0
Vu = 0
4u = 0
Bu = 0
S = 110 Sud = 110 (1.3) (0.8) = 114. 4
V = 6. 103 Vud = 0
4 = −0.222 76
B = 30. 606 14
Sd = 110 (0.8) = 88 ¡ ¢
Vd = 12. 252 Sdd = 110 0.82 = 70. 4
4d = −0.559 091 Vdd = 95 − 70. 4 = 24. 6
Bd = 61. 452 1

www.myactuaryexam.com c
°Yufeng Guo 38
CHAPTER 10. BINOMIAL OPTION PRICING I

• S = 120
Period 0 1 2 ¡ ¢
Suu = 120 1.32 = 202. 8
Su = 120 (1.3) = 156 Vuu = 0
Vu =?
4u =?
Bu =?
S = 120 Sud = 120 (1.3) (0.8) = 124. 8
V =? Vud = 0
∆ =?
B =?
Sd = 120 (0.8) = 96 ¡ ¢
Vd =? Sdd = 120 0.82 = 76. 8
∆d =? Vdd = 95 − 76. 8 = 18. 2
Bd =?
½
4u 202. 8 + Bu e0.08(0.5) = 0
4u = 0 Bu = 0
4u 124. 8 + Bu e0.08(0.5) = 0
Vu = 4u Su + Bu = 0 (156) + 0 = 0
½
4d 124. 8 + Bd e0.08(0.5) = 0
4d = −0.379 167 Bd = 45. 464 56
4d 76. 8 + Bd e0.08(0.5) = 18. 2
Vd = 4d Sd + Bd = −0.379 167 (96) + 45. 464 56 = 9. 065
½
4156 + Be0.08(0.5) = 0
4 = −0.151 083 B = 22. 644 85
496 + Be0.08(0.5) = 9. 065
V = 4S + B = −0.151 083 (120) + 22. 644 85 = 4. 514 89

The final diagram is:


Period 0 1 2 ¡ ¢
Suu = 120 1.32 = 202. 8
Su = 120 (1.3) = 156 Vuu = 0
Vu = 0
4u = 0
Bu = 0
S = 120 Sud = 120 (1.3) (0.8) = 124. 8
V = 4. 514 89 Vud = 0
4 = −0.151 083
B = 22. 644 85
Sd = 120 (0.8) = 96 ¡ ¢
Vd = 9. 065 Sdd = 120 0.82 = 76. 8
4d = −0.379 167 Vdd = 95 − 76. 8 = 18. 2
Bd = 45. 464 56

www.myactuaryexam.com c
°Yufeng Guo 39
CHAPTER 10. BINOMIAL OPTION PRICING I

• S = 130
Period 0 1 2 ¡ ¢
Suu = 130 1.32 = 219. 7
Su = 130 (1.3) = 169 Vuu = 0
Vu =?
4u =?
Bu =?
S = 130 Sud = 130 (1.3) (0.8) = 135. 2
V =? Vud = 0
∆ =?
B =?
Sd = 130 (0.8) = 104 ¡ ¢
Vd =? Sdd = 130 0.82 = 83. 2
∆d =? Vdd = 95 − 83. 2 = 11. 8
Bd =?
½
4u 219. 7 + Bu e0.08(0.5) = 0
4u = 0 Bu = 0
4u 135. 2 + Bu e0.08(0.5) = 0
Vu = 4u Su + Bu = 0 (169) + 0 = 0
½
4d 135. 2 + Bd e0.08(0.5) = 0
4d = −0.226 923 Bd = 29. 477 0
4d 83. 2 + Bd e0.08(0.5) = 11. 8
Vd = 4d Sd + Bd = −0.226 923 (104) + 29. 477 0 = 5. 877
½
4169 + Be0.08(0.5) = 0
4 = −0.09 041 5 B = 14. 681 05
4104 + Be0.08(0.5) = 5. 877
V = 4S + B = −0.09 041 5 (130) + 14. 681 05 = 2. 927 1
The final diagram is:
Period 0 1 2 ¡ ¢
Suu = 130 1.32 = 219. 7
Su = 130 (1.3) = 169 Vuu = 0
Vu = 0
4u = 0
Bu = 0
S = 130 Sud = 130 (1.3) (0.8) = 135. 2
V = 2. 927 1 Vud = 0
4 = −0.09 041 5
B = 14. 681 05
Sd = 130 (0.8) = 104 ¡ ¢
Vd = 5. 877 Sdd = 130 0.82 = 83. 2
4d = −0.226 923 Vdd = 95 − 83. 2 = 11. 8
Bd = 29. 477 0

www.myactuaryexam.com c
°Yufeng Guo 40
CHAPTER 10. BINOMIAL OPTION PRICING I

Notice that the delta for a put is always negative. A negative delta means
shorting sell stocks. If you sell a put, the risk you face is that the stock price
may go down to zero, at which case the put holder sells you a worthless stock for
the strike price. To hedge your risk, you need to short sell some stocks at t = 0.
If indeed the future stock price goes down at T , you can buy back stocks in
the market at low price to close your short position on stocks, earning a profit.
Your profit from the short sale can offset your loss in the put.
Intuitively, the higher the initial stock price, everything else equal, the higher
the stock price in the future, the lower the payoff of the put. Hence to hedge
the risk, the market maker needs to short sell fewer shares of stocks at t = 0. So
the higher the initial stock price, the lower the absolute value of the delta for a
put, the fewer of the stocks that the market maker needs to short sell initially.

Problem 10.8.
Period 0 1 2 ¡ ¢
Suu = 100 1.32 = 169
Su = 100 (1.3) = 130 Vuu = 0
EVu = 0
Vu =?
4u =?
Bu =?
S = 100 Sud = 100 (1.3) (0.8) = 104
EV = 0 Vud = 0
V =?
∆ =?
B =?
Sd = 100 (0.8) = 80
EVd = 95 − 80 = 15 ¡ ¢
Vd =? Sdd = 100 0.82 = 64
∆d =? Vdd = 95 − 64 = 31
Bd =?
½
4u 169 + Bu e0.08(0.5) = 0
4u = 0 Bu = 0
4u 104 + Bu e0.08(0.5) = 0
Vu = max (4u Su + Bu , EVu ) = max [0 (130) + 0, 0] = 0

This is the logic behind the calculation of Vu . The American put can be
exercised at the end of Period 1 and end of Period 2.

• If exercised at the end of Period 1, the put is worth EVu = max (0, 95 − 130) =
0 at the end of Period 1.
• If exercised at Period 2, the put is worth 4u Su + Bu = 0 (130) + 0 = 0 at
the end of Period 1.

www.myactuaryexam.com c
°Yufeng Guo 41
CHAPTER 10. BINOMIAL OPTION PRICING I

We compare the two values and take the greater.


½
4d 104 + Bd e0.08(0.5) = 0
4d = −0.775 Bd = 77. 439 6
4d 64 + Bd e0.08(0.5) = 31
Vd = max (4d Sd + Bd , EVd ) = max [−0.775 (80) + 77. 439 6, 15] = 15. 439 6
½
4130 + Be0.08(0.5) = 0
4 = −0.308 792 B = 38. 568 932
480 + Be0.08(0.5) = 15. 439 6
V = max (4S + B, EV ) = max [−0.308 792 (100) + 38. 568 932 , 0] = 7. 689 732

The final diagram is:


Period 0 1 2 ¡ ¢
Suu = 100 1.32 = 169
Su = 100 (1.3) = 130 Vuu = 0
EVu = 0
Vu = 0
4u = 0
Bu = 0
S = 100 Sud = 100 (1.3) (0.8) = 104
EV = 0 Vud = 0
V = 7. 689 7
∆ = −0.308 792
B = 38. 568 932
Sd = 100 (0.8) = 80
EVd = 95 − 80 = 15 ¡ ¢
Vd = 15. 439 6 Sdd = 100 0.82 = 64
4d = −0.775 Vdd = 95 − 64 = 31
Bd = 77. 439 6

Problem 10.9.
a.
time 0 T
Su = 100 (1.2) = 120
Vu = 120 − 50 = 70
S = 100
V =?
∆ =?, B =?
Sd = 100 (1.05) = 105
Vd = 105 − 50 = 55

Here d = 1.05 > 1. Is there anything wrong with this? No necessarily. The
non-arbitrage requirement is textbook Equation 10.4: u > e(r−δ)h > d
Let’s check. e(r−δ)h = e(0.07696−0)1 = 1. 08
u = 1.2 > e(r−δ)h > d = 1.05

www.myactuaryexam.com c
°Yufeng Guo 42
CHAPTER 10. BINOMIAL OPTION PRICING I

So it’s OK to have d > 1 as long as u > e(r−δ)h > d is met.


½
4120 + Be0.07696(1) = 70
4=1 B = −46. 296 3
4105 + Be0.07696(1) = 55
V = 4S + B = 1 (100) − 46. 296 3 = 53. 703 7

The final diagram is:


time 0 T
Su = 100 (1.2) = 120
Vu = 120 − 50 = 70
S = 100
V = 53. 703 7
4 = 1, B = −46. 296 3
Sd = 100 (1.05) = 105
Vd = 105 − 50 = 55

b. Now the stock price has a bigger increase and a bigger decrease. We
are not clear whether the call premium will increase or decrease. We have to
calculate the premium.
time 0 T
Su = 100 (1.4) = 140
Vu = 140 − 50 = 90
S = 100
V =?
∆ =?, B =?
Sd = 100 (0.6) = 60
Vd = 60 − 50 = 10
½
4140 + Be0.07696(1) = 90
4=1 B = −46. 296 3
460 + Be0.07696(1) = 10
V = 4S + B = 1 (100) − 46. 296 3 = 53. 703 7

The call premium is the same. What’s going on?


It turns out that as long Su > Sd > K, the call premium is fixed regardless
of how you choose u and d.
time 0 T
Su = uS > K
Vu = uS − K
S
V =?
∆ =?, B =?
Sd = dS > K
Vd = dS − K
½
4uS + Berh = uS − K
4dS + Berh = dS − K

www.myactuaryexam.com c
°Yufeng Guo 43
CHAPTER 10. BINOMIAL OPTION PRICING I

→ 4 = 1 B = −Ke−rh C = 4S + B = S − Ke−rh = 100 −


−0.07696(1)
50e = 53. 703 7

We can explain this using the put-call parity. If Su > Sd > K, the put is
never exercised.
C + P V (K) = P + S
Since P = 0, we have C = S −P V (K) = S −Ke−rh = 100−50e−0.07696(1) =
53. 703 7

c. I think the question wants us to compare b and c, not a and c.


Compared with b, now u is the same but d gets smaller. In fact, now at
Node d, the call expires worthless.

time 0 T
Su = 100 (1.4) = 140
Vu = 140 − 50 = 90
S = 100
V =?
∆ =?, B =?
Sd = 100 (0.4) = 40
Vd = 0

We might think the call option should be worth less since it expires worthless
at Node
½ d.
4140 + Be0.07696(1) = 90
4 = 0.9 B = −33. 333
440 + Be0.07696(1) = 0

V = 4S + B = 0.9 (100) − 33. 333 = 56. 667 > 53. 703 7


Why did the call premium goes up to 56. 667? Does this lead to arbitrage?
First, I’m going to show you this is not an arbitrage. Now we have two call
options:
Call b Call c
time 0 T time 0 T
Su = 100 (1.4) = 140 Su = 100 (1.4) = 140
Vu = 140 − 50 = 90 Vu = 140 − 50 = 90
S = 100 S = 100
V = 53. 703 7 V = 56. 667
4 = 1, B = −46. 296 3 4 = 0.9, B = −33. 333
Sd = 100 (0.6) = 60 Sd = 100 (0.4) = 40
Vd = 60 − 50 = 10 Vd = 0

What if we buy Call b and simultaneously sell Call c? Does this lead to
arbitrage?
The answer is No because these two calls are on two different stocks. If
these two options have the same underlying asset, then we’ll make free money

www.myactuaryexam.com c
°Yufeng Guo 44
CHAPTER 10. BINOMIAL OPTION PRICING I

by "buy low and sell high." However, these two calls are on two different stocks;
if the stock is the same, there won’t be two different values of d0 s.
Call b is on a stock whose up price is 140 and down price is 60. Call c is
on the stock whose up price is 140 and down price is 40. Clearly, the stock
under Call c is more volatile than the stock under Call b. Hence Call c is more
valuable than Call b.
We can also explain why Call b is more valuable using the put-call parity.
C + P V (K) = P + S
For b, the put is never exercised. Hence P = 0 and C = S − P V (K).
For c, the put is exercised Node d. Hence P > 0 and C = P + S − P V (K).
So Call c exceeds Call b.

Problem 10.10.

h = T /n = 1/3 √

u = e(r−δ)h+σ h
= e(0.08−0)1/3+0.3 1/3 = 1. 221 246
√ √
d = e(r−δ)h−σ h
= e(0.08−0)1/3−0.3 1/3 = 0.863 693

e(r−δ)h − d e(0.08−0)1/3 − 0.863 693


πu = = = 0.456 806
u−d 1. 221 246 − 0.863 693

π d = 1 − πu = 1 − 0.456 806 = 0.543 194

a. Calculate the call premium


Period 2 3 ¡ ¢
¡ ¢ Suuu = 100 1. 221 2463 = 182. 141 7
Suu = 100 1. 221 2462 = 149. 144 2 Vuuu = 182. 141 7 − 95 = 87. 141 7
EVuu = 149. 144 2 − 95 = 54. 144 2 ¡ ¢
Vuu =? Suud = 100 1. 221 2462 (0.863 693) = 128. 814 8
Vuud = 128. 814 8 − 95 = 33. 814 8
Sud = 100 (1. 221 246) (0.863 693) = 105. 478 2
EVud = 105. 478 2 − 95 = 10. 478 2
Vud =? ¡ ¢
¡ ¢ Sudd = 100 (1. 221 246) 0.863 6932 = 91. 100 8
Sdd = 100 0.863 6932 = 74. 596 6 Vudd = 0
EVdd = 0 ¡ ¢
Vdd =? Sddd = 100 0.863 6933 = 64. 428 5
Vddd = 0

Let R represent the roll-back value.


R
Vuu = e−rh (π u Vuuu + π d Vuud ) = e−(0.08)1/3 (0.456 806 × 87. 141 7 + 0.543 194 × 33. 814 8) =
56. 644 02 ¡ R ¢
Vuu = max Vuu , EVuu = max (56. 644 02, 54. 144 2) = 56. 644 02

www.myactuaryexam.com c
°Yufeng Guo 45
CHAPTER 10. BINOMIAL OPTION PRICING I

R
Vud = e−rh (πu Vuud + π d Vudd ) = e−(0.08)1/3 (0.456 806 × 33. 814 8 + 0.543 194 × 0) =
15. 040 33 ¡ R ¢
Vud = max Vud , EVud = max (15. 040 33, 10. 478 2) = 15. 040 33

R
Vdd = e−rh (π u Vudd + πd Vddd ) = e−(0.08)1/3 (0.456 806 × 0 + 0.543 194 × 0) =
0 ¡ R ¢
Vdd = max Vdd , EVdd = max (0, 0) = 0

Now we have:
Period 1 2 ¡ ¢
Suu = 100 1. 221 2462 = 149. 144 2
EVuu = 149. 144 2 − 95 = 54. 144 2
Su = 100 (1. 221 246) = 122. 124 6 Vuu = 56. 644 02
EVu = 122. 124 6 − 95 = 27. 124 6
Vu =? Sud = 100 (1. 221 246) (0.863 693) = 105. 478 2
EVud = 105. 478 2 − 95 = 10. 478 2
Vud = 15. 040 33
Sd = 100 (0.863 693) = 86. 369 3 ¡ ¢
EVd = 0 Sdd = 100 0.863 6932 = 74. 596 6
Vd =? EVdd = 0
Vdd = 0

VuR = e−rh (πu Vuu + π d Vud ) = e−(0.08)1/3 (0.456 806 × 56. 644 02 + 0.543 194 × 15. 040 33) =
33. 149 27 ¡ ¢
Vu = max VuR , EVu = max (33. 149 27, 27. 124 6) = 33. 149 27

VdR = e−rh (πu Vud + πd Vdd ) = e−(0.08)1/3 (0.456 806 × 15. 040 33 + 0.543 194 × 0) =
6. 689 72 ¡ ¢
Vd = max VdR , EVd = max (6. 689 72, 0) = 6. 689 72

Now we have:
Period 0 1
Su = 100 (1. 221 246) = 122. 124 6
EVu = 122. 124 6 − 95 = 27. 124 6
Vu = 33. 149 27
S = 100
EV = 100 − 95 = 5
V =? Sd = 100 (0.863 693) = 86. 369 3
EVd = 0
Vd = 6. 689 72

V R = e−rh (πu Vu + πd Vd ) = e−(0.08)1/3 (0.456 806 × 33. 149 27 + 0.543 194 × 6. 689 72) =
18. 282 51 ¡ ¢
V = max V R , EV = max (18. 282 51, 5) = 18. 282 51

www.myactuaryexam.com c
°Yufeng Guo 46
CHAPTER 10. BINOMIAL OPTION PRICING I

From the above calculation you can see that the exercise value is never
greater than the roll back value. Hence the American call option is never exer-
cised early; the American call and European call have the same value of $18.
282 51.
Generally, if a stock doesn’t pay dividend, then an American call and an
otherwise identical European call on this stock are worth the same.

c. Calculate the American put premium.

Period 2 3 ¡ ¢
¡ ¢ Suuu = 100 1. 221 2463 = 182. 141 7
Suu = 100 1. 221 2462 = 149. 144 2 Vuuu = 0
EVuu = 0 ¡ ¢
Vuu =? Suud = 100 1. 221 2462 (0.863 693) = 128. 814 8
Vuud = 0
Sud = 100 (1. 221 246) (0.863 693) = 105. 478 2
EVud = 0
Vud =? ¡ ¢
¡ ¢ Sudd = 100 (1. 221 246) 0.863 6932 = 91. 100 8
Sdd = 100 0.863 6932 = 74. 596 6 Vudd = 95 − 91. 100 8 = 3. 899 2
EVdd = 95 − 74. 596 6 = 20. 403 4 ¡ ¢
Vdd =? Sddd = 100 0.863 6933 = 64. 428 5
Vddd = 95 − 64. 428 5 = 30. 571 5

R
Vuu = e−rh (π u Vuuu + π d Vuud ) = e−(0.08)1/3 (0.456 806 × 0 + 0.543 194 × 0) =
0
¡ R ¢
Vuu = max Vuu , EVuu = max (0, 0) = 0

R
Vud = e−rh (π u Vuud + πd Vudd ) = e−(0.08)1/3 (0.456 806 × 0 + 0.543 194 × 3. 899 2) =
2. 062 29
¡ R ¢
Vud = max Vud , EVud = max (2. 062 29, 0) = 2. 062 29

R
Vdd = e−rh (πu Vudd + πd Vddd ) = e−(0.08)1/3 (0.456 806 × 3. 899 2 + 0.543 194 × 30. 571 5) =
17. 903 58
¡ R ¢
Vdd = max Vdd , EVdd = max (17. 903 58, 20. 403 4) = 20. 403 4
At the dd node, the exercise value is greater than the roll-back value. The
put is exercised at dd. Now we have:

www.myactuaryexam.com c
°Yufeng Guo 47
CHAPTER 10. BINOMIAL OPTION PRICING I

Period 1 2 ¡ ¢
Suu = 100 1. 221 2462 = 149. 144 2
EVuu = 0
Su = 100 (1. 221 246) = 122. 124 6 Vuu = 0
EVu = 0
Vu =? Sud = 100 (1. 221 246) (0.863 693) = 105. 478 2
EVud = 0
Vud = 2. 062 29
Sd = 100 (0.863 693) = 86. 369 3 ¡ ¢
EVd = 95 − 86. 369 3 = 8. 630 7 Sdd = 100 0.863 6932 = 74. 596 6
Vd =? EVdd = 95 − 74. 596 6 = 20. 403 4
Vdd = 20. 403 4

VuR = e−rh (πu Vuu + π d Vud ) = e−(0.08)1/3 (0.456 806 × 0 + 0.543 194 × 2. 062 29) =
1. 090 75
¡ ¢
Vu = max VuR , EVu = max (1. 090 75, 0) = 1. 090 75

VdR = e−rh (πu Vud + πd Vdd ) = e−(0.08)1/3 (0.456 806 × 2. 062 29 + 0.543 194 × 20. 403 4) =
11. 708 64
¡ ¢
Vd = max VdR , EVd = max (11. 708 64, 8. 630 7) = 11. 708 64

Now we have:
Period 0 1
Su = 100 (1. 221 246) = 122. 124 6
EVu = 0
Vu = 1. 090 75
S = 100
EV = 0
V =? Sd = 100 (0.863 693) = 86. 369 3
EVd = 95 − 86. 369 3 = 8. 630 7
Vd = 11. 708 64
V R = e−rh (πu Vu + πd Vd ) = e−(0.08)1/3 (0.456 806 × 1. 090 75 + 0.543 194 × 11. 708 64) =
6. 677 85
¡ ¢
V = max V R , EV = max (6. 677 85, 0) = 6. 677 85

b. Calculate the European put premium. Verify that the put-call parity
holds.
Since the option is European, we just calculate the roll-back value.

www.myactuaryexam.com c
°Yufeng Guo 48
CHAPTER 10. BINOMIAL OPTION PRICING I

Period 2 3
Vuuu = 0
Vuu =?
Vuud = 0
Vud =?
Vudd = 3. 899 2
Vdd =?
Vddd = 30. 571 5
−rh
Vuu = e (π u Vuuu + π d Vuud ) = e−(0.08)1/3 (0.456 806 × 0 + 0.543 194 × 0) =
0
Vud = e−rh (π u Vuud + πd Vudd ) = e−(0.08)1/3 (0.456 806 × 0 + 0.543 194 × 3. 899 2) =
2. 062 29
Vdd = e−rh (πu Vudd + πd Vddd ) = e−(0.08)1/3 (0.456 806 × 3. 899 2 + 0.543 194 × 30. 571 5) =
17. 903 58

Now we have:
Period 1 2
Vuu = 0
Vu =?
Vud = 2. 062 29
Vd =?
Vdd = 17. 903 58

Vu = e−rh (πu Vuu + π d Vud ) = e−(0.08)1/3 (0.456 806 × 0 + 0.543 194 × 2. 062 29) =
1. 090 75
Vd = e−rh (π u Vud + πd Vdd ) = e−(0.08)1/3 (0.456 806 × 2. 062 29 + 0.543 194 × 17. 903 58) =
10. 386 48

Now we have:
Period 0 1
Vu = 1. 090 75
V =?
Vd = 10. 386 48
−rh
V =e (π u Vu + π d Vd ) = e−(0.08)1/3 (0.456 806 × 1. 090 75 + 0.543 194 × 10. 386 48) =
5. 978 6
Check whether the put-call parity-holds:
C + Ke−rT = 18. 282 51 + 95e−(0.08)1 = 105. 978 6
P + S = 5. 978 6 + 100 = 105. 978 6
→ C + Ke−rT = P + S

Problem 10.11.

h = T /n = 1/3 √

u = e(r−δ)h+σ h
= e(0.08−0.08)1/3+0.3 1/3
= 1. 189 11

www.myactuaryexam.com c
°Yufeng Guo 49
CHAPTER 10. BINOMIAL OPTION PRICING I

√ √
d = e(r−δ)h−σ h
= e(0.08−0.08)1/3−0.3 1/3 = 0.840 965

e(r−δ)h − d e(0.08−0.08)1/3 − 0.840 965


πu = = = 0.456 807
u−d 1. 189 11 − 0.840 965

πd = 1 − π u = 1 − 0.456 807 = 0.543 193

a.

• Calculate the price of the American call option

Period 2 3 ¡ ¢
¡ ¢ Suuu = 100 1. 189 113 = 168. 138 1
Suu = 100 1. 189 112 = 141. 398 3 Vuuu = 168. 138 1 − 95 = 73. 138 1
EVuu = 141. 398 3 − 95 = 46. 398 3 ¡ ¢
Vuu =? Suud = 100 1. 189 112 (0.840 965) = 118. 9110
Vuud = 118. 9110 − 95 = 23. 911
Sud = 100 (1. 189 11) (0.840 965) = 100
EVud = 100 − 95 = 5
Vud =? ¡ ¢
¡ ¢ Sudd = 100 (1. 189 11) 0.840 9652 = 84. 096 5
Sdd = 100 0.840 9652 = 70. 722 2 Vudd = 0
EVdd = 0 ¡ ¢
Vdd =? Sddd = 100 0.840 9653 = 59. 474 9
Vddd = 0

R
Vuu = e−rh (π u Vuuu + π d Vuud ) = e−(0.08)1/3 (0.456 807 × 73. 138 1 + 0.543 193 × 23. 911) =
45. 177 3
¡ R ¢
Vuu = max Vuu , EVuu = max (45. 177 3, 46. 398 3) = 46. 398 3
The call is early exercised at the uu node.

R
Vud = e−rh (πu Vuud + π d Vudd ) = e−(0.08)1/3 (0.456 807 × 23. 911 + 0.543 193 × 0) =
10. 635 3
¡ R ¢
Vud = max Vud , EVud = max (10. 635 3, 5) = 10. 635 3

R
Vdd = e−rh (π u Vudd + πd Vddd ) = e−(0.08)1/3 (0.456 807 × 0 + 0.543 193 × 0) =
0
¡ R ¢
Vdd = max Vdd , EVdd = max (0, 0) = 0

www.myactuaryexam.com c
°Yufeng Guo 50
CHAPTER 10. BINOMIAL OPTION PRICING I

Period 1 2
Vuu = 46. 398 3
Su = 100 (1. 189 11) = 118. 911
EVu = 118. 911 − 95 = 23. 911
Vu =?
Vud = 10. 635 3
Sd = 100 (0.840 965) = 84. 096 5
EVd = 0
Vd =?
Vdd = 0

VuR = e−rh (π u Vuu + π d Vud ) = e−(0.08)1/3 (0.456 807 × 46. 398 3 + 0.543 193 × 10. 635 3) =
26. 262 3 ¡ ¢
Vu = max VuR , EVu = max (26. 262 3, 23. 911) = 26. 262 3

VdR = e−rh (π u Vud + π d Vdd ) = e−(0.08)1/3 (0.456 807 × 10. 635 3 + 0.543 193 × 0) =
4. 730 4 ¡ ¢
Vd = max VdR , EVd = max (4. 730 4, 0) = 4. 730 4

Period 0 1
Vu = 26. 262 3
S = 100
EV = 100 − 95 = 5
Vu =?
Vd = 4. 730 4

V R = e−rh (π u Vu + π d Vd ) = e−(0.08)1/3 (0.456 807 × 26. 262 3 + 0.543 193 × 4. 730 4) =


14. 183 0 ¡ ¢
V = max V R , EV = max (14. 183 0, 5) = 14. 183 0

• Calculate the price of the European call option


Period 0 1 2 3 ¡ ¢
Suuu = 100 1. 189 113 = 168. 138 1
Vuuu = 168. 138 1 − 95 = 73. 138 1
Vuu ¡ ¢
Suud = 100 1. 189 112 (0.840 965) = 118. 9110
Vu Vuud = 118. 9110 − 95 = 23. 911

V Vud ¡ ¢
Sudd = 100 (1. 189 11) 0.840 9652 = 84. 096 5
Vd Vudd = 0
Vdd ¡ ¢
Sddd = 100 0.840 9653 = 59. 474 9
Vddd = 0

www.myactuaryexam.com c
°Yufeng Guo 51
CHAPTER 10. BINOMIAL OPTION PRICING I

Vuu = e−rh (π u Vuuu + π d Vuud ) = e−(0.08)1/3 (0.456 807 × 73. 138 1 + 0.543 193 × 23. 911) =
45. 177 3
Vud = e−rh (πu Vuud + π d Vudd ) = e−(0.08)1/3 (0.456 807 × 23. 911 + 0.543 193 × 0) =
10. 635 3
Vdd = e−rh (π u Vudd + πd Vddd ) = e−(0.08)1/3 (0.456 807 × 0 + 0.543 193 × 0) =
0

Vu = e−rh (πu Vuu + π d Vud ) = e−(0.08)1/3 (0.456 807 × 45. 177 3 + 0.543 193 × 10. 635 3) =
25. 719 3
Vd = e−rh (π u Vud + π d Vdd ) = e−(0.08)1/3 (0.456 807 × 10. 635 3 + 0.543 193 × 0) =
4. 730 4

V = e−rh (πu Vu + πd Vd ) = e−(0.08)1/3 (0.456 807 × 25. 719 3 + 0.543 193 × 4. 730 4) =
13. 941 5
Since the American call is early exercised at Node uu, the price of the Amer-
ican call 14. 183 0 is greater than the price of the European call 13. 941 5.

c. Calculate the American put premium.

Period 2 3 ¡ ¢
¡ ¢ Suuu = 100 1. 189 113 = 168. 138 1
Suu = 100 1. 189 112 = 141. 398 3 Vuuu = 0
EVuu = 0 ¡ ¢
Vuu =? Suud = 100 1. 189 112 (0.840 965) = 118. 9110
Vuud = 0
Sud = 100 (1. 189 11) (0.840 965) = 100
EVud = 0
Vud =? ¡ ¢
¡ ¢ Sudd = 100 (1. 189 11) 0.840 9652 = 84. 096 5
Sdd = 100 0.840 9652 = 70. 722 2 Vudd = 95 − 84. 096 5 = 10. 903 5
EVdd = 95 − 70. 722 2 = 24. 277 8 ¡ ¢
Vdd =? Sddd = 100 0.840 9653 = 59. 474 9
Vddd = 95 − 59. 474 9 = 35. 525 1
R −rh −(0.08)1/3
Vuu =e (π u Vuuu + π d Vuud ) = e (0.456 807 × 0 + 0.543 193 × 0) =
0 ¡ R ¢
Vuu = max Vuu , EVuu = max (0, 0) = 0

R
Vud = e−rh (πu Vuud + π d Vudd ) = e−(0.08)1/3 (0.456 807 × 0 + 0.543 193 × 10. 903 5) =
5. 766 9 ¡ R ¢
Vud = max Vud , EVud = max (5. 766 9, 0) = 5. 766 9

R
Vdd = e−rh (π u Vudd + πd Vddd ) = e−(0.08)1/3 (0.456 807 × 10. 903 5 + 0.543 193 × 35. 525 1) =
23. 638 9 ¡ R ¢
Vdd = max Vdd , EVdd = max (23. 638 9, 24. 277 8) = 24. 277 8
The American put is early exercised at Node dd.

www.myactuaryexam.com c
°Yufeng Guo 52
CHAPTER 10. BINOMIAL OPTION PRICING I

Period 1 2
Vuu = 0
Su = 100 (1. 189 11) = 118. 911
EVu = 0
Vu =?
Vud = 5. 766 9
Sd = 100 (0.840 965) = 84. 096 5
EVd = 95 − 84. 096 5 = 10. 903 5
Vd =?
Vdd = 24. 277 8

VuR = e−rh (π u Vuu + π d Vud ) = e−(0.08)1/3 (0.456 807 × 0 + 0.543 193 × 5. 766 9) =
3. 050 1 ¡ ¢
Vu = max VuR , EVu = max (3. 050 1, 0) = 3. 050 1

VdR = e−rh (π u Vud + π d Vdd ) = e−(0.08)1/3 (0.456 807 × 5. 766 9 + 0.543 193 × 24. 277 8) =
15. 405 6 ¡ ¢
Vd = max VdR , EVd = max (15. 405 6, 10. 903 5) = 15. 405 6

Period 0 1
Vu = 3. 050 1
S = 100
EV = 0
Vu =?
Vd = 15. 405 6

V R = e−rh (π u Vu + π d Vd ) = e−(0.08)1/3 (0.456 807 × 3. 050 1 + 0.543 193 × 15. 405 6) =


9. 504 7 ¡ ¢
V = max V R , EV = max (9. 504 7, 0) = 9. 504 7

b. Calculate the European put premium. Verify that the put-call parity
holds.
Period 0 1 2 3 ¡ ¢
Suuu = 100 1. 189 113 = 168. 138 1
Vuuu = 0
Vuu ¡ ¢
Vu Suud = 100 1. 189 112 (0.840 965) = 118. 9110
Vuud = 0
V
Vud ¡ ¢
Sudd = 100 (1. 189 11) 0.840 9652 = 84. 096 5
Vd Vudd = 95 − 84. 096 5 = 10. 903 5
Vdd ¡ ¢
Sddd = 100 0.840 9653 = 59. 474 9
Vddd = 95 − 59. 474 9 = 35. 525 1

www.myactuaryexam.com c
°Yufeng Guo 53
CHAPTER 10. BINOMIAL OPTION PRICING I

Vuu = e−rh (π u Vuuu + π d Vuud ) = e−(0.08)1/3 (0.456 807 × 0 + 0.543 193 × 0) =


0
Vud = e−rh (πu Vuud + π d Vudd ) = e−(0.08)1/3 (0.456 807 × 0 + 0.543 193 × 10. 903 5) =
5. 766 9
Vdd = e−rh (π u Vudd + πd Vddd ) = e−(0.08)1/3 (0.456 807 × 10. 903 5 + 0.543 193 × 35. 525 1) =
23. 638 9

Vu = e−rh (πu Vuu + π d Vud ) = e−(0.08)1/3 (0.456 807 × 0 + 0.543 193 × 5. 766 9) =
3. 050 1
Vd = e−rh (π u Vud + π d Vdd ) = e−(0.08)1/3 (0.456 807 × 5. 766 9 + 0.543 193 × 23. 638 9) =
15. 067 6

V = e−rh (πu Vu + πd Vd ) = e−(0.08)1/3 (0.456 807 × 3. 050 1 + 0.543 193 × 15. 067 6) =
9. 325 9

So the European put is worth 9. 325 9.


Verify the put-call parity:
C + Ke−rT = 13. 941 5 + 95e−(0.08)1 = 101. 637 55
P + Se−δT = 9. 325 9 + 100e−(0.08)1 = 101. 637 53
Ignoring rounding difference, we get C + Ke−rT = P + Se−δT

The put-call parity holds.

Problem 10.12.
a. h = T /n =√0.5/2 = 0.25 √
u = e(r−δ)h+σ h = e(0.08)0.25+0.3 0.25 = 1. 185 305
√ √
d = e(r−δ)h−σ h
= e(0.08)0.25−0.30.25
= 0.878 095
e(r−δ)h − d e(0.08)0.25 − 0.878 095
πu = = = 0.462 57
u−d 1. 185 305 − 0.878 095
πd = 1 − π u = 1 − 0.462 57 = 0.537 43

b. Since the stock doesn’t pay dividend, the American call and an otherwise
identical European call have the same value.
Period 0 1 2 ¡ ¢
Suu = 40 1. 185 3052 = 56. 197 92
Vuu = 56. 197 92 − 40 = 16. 197 92
Vu =?
Sud = 40 (1. 185 305) (0.878 095) = 41. 632 4
V =? Vud = 41. 632 4 − 40 = 1. 632 4
¡ ¢
Vd =? Sdd = 40 0.878 0952 = 30. 842 03
Vdd = 0

www.myactuaryexam.com c
°Yufeng Guo 54
CHAPTER 10. BINOMIAL OPTION PRICING I

Vu = e−rh (πu Vuu + π d Vud ) = e−(0.08)0.25 (0.462 57 × 16. 197 92 + 0.537 43 × 1. 632 4) =
8. 204 2
Vd = e−rh (π u Vud + πd Vdd ) = e−(0.08)0.25 (0.462 57 × 1. 632 4 + 0.537 43 × 0) =
0.740 1
V = e−rh (π u Vu + π d Vd ) = e−(0.08)0.25 (0.462 57 × 8. 204 2 + 0.537 43 × 0.740 1) =
4. 109 7

So both the American call and the European call are worth 4. 109 7.

We also calculate the European call price using the following shortcut:
Node Payoff Risk Neutral Prob
uu 16. 197 92 π 2u = 0.462 572
ud 1. 632 4 2π u πd = 2 × 0.462 57 × 0.537 43
dd 0 π 2d = 0.537 432
P
V = e−rT ¡payof f × RiskN eutralP rob ¢
= e−0.08(0.5) 16. 197 92 × 0.462 572 + 1. 632 4 × 2 × 0.462 57 × 0.537 43 + 0 × 0.537 432
= 4. 109 8
Please note that the above shortcut works only for European options.

c. Even if the stock doesn’t pay dividend, it may be still optimal to exercise
an American put early.

• Calculate the premium of the American put

Period 0 1 2 ¡ ¢
Suu = 40 1. 185 3052 = 56. 197 92
Su = 40 (1. 185 305) = 47. 412 2 Vuu = 0
EVu = 0
Vu =?
S = 40 Sud = 40 (1. 185 305) (0.878 095) = 41. 632 4
EV = 0 Vud = 0
V =?
Sd = 40 (0.878 095) = 35. 123 8
EVd = 40 − 35. 123 8 = 4. 876 2 ¡ ¢
Vd =? Sdd = 40 0.878 0952 = 30. 842 03
Vdd = 40 − 30. 842 03 = 9. 157 97

VuR = e−rh (π u Vuu + πd Vud ) = e−(0.08)1/3 (0.456 807 × 0 + 0.543 193 × 0) =

0 ¡ ¢
Vu = max VuR , EVu = max (0, 0) = 0

VdR = e−rh (π u Vud + π d Vdd ) = e−(0.08)1/3 (0.456 807 × 0 + 0.543 193 × 9. 157 97) =
4. 843 6 ¡ ¢
Vd = max VdR , EVd = max (4. 843 6, 4. 876 2) = 4. 876 2

www.myactuaryexam.com c
°Yufeng Guo 55
CHAPTER 10. BINOMIAL OPTION PRICING I

So the American put is early exercised at Node d.

V R = e−rh (πu Vu + πd Vd ) = e−(0.08)1/3 (0.456 807 × 0 + 0.543 193 × 4. 876 2) =


2. 579 0 ¡ ¢
V = max V R , EV = max (2. 579 0, 0) = 2. 579 0

The American put is worth 2. 579 0.

• Calculate the premium of the European put

Period 0 1 2 ¡ ¢
Suu = 40 1. 185 3052 = 56. 197 92
Vuu = 0
Vu
Sud = 40 (1. 185 305) (0.878 095) = 41. 632 4
V Vud = 0
¡ ¢
Vd Sdd = 40 0.878 0952 = 30. 842 03
Vdd = 40 − 30. 842 03 = 9. 157 97

Vu = e−rh (πu Vuu + π d Vud ) = e−(0.08)1/3 (0.456 807 × 0 + 0.543 193 × 0) = 0

Vd = e−rh (π u Vud + π d Vdd ) = e−(0.08)1/3 (0.456 807 × 0 + 0.543 193 × 9. 157 97) =
4. 843 6

V = e−rh (πu Vu + πd Vd ) = e−(0.08)1/3 (0.456 807 × 0 + 0.543 193 × 4. 843 6) =


2. 561 8

The European put is worth 2. 561 8.

Problem 10.13.

a. From the previous problem, we know:


Period 0 1 2 ¡ ¢
Suu = 40 1. 185 3052 = 56. 197 92
Su = 40 (1. 185 305) = 47. 412 2 Vuu = 56. 197 92 − 40 = 16. 197 92
Vu = 8. 204 2
S = 40 (∆u , Bu ) Sud = 40 (1. 185 305) (0.878 095) = 41. 632 4
V = 4. 109 7 Vud = 41. 632 4 − 40 = 1. 632 4
(∆, B)
Sd = 40 (0.878 095) = 35. 123 8 ¡ ¢
Vd = 0.740 1 Sdd = 40 0.878 0952 = 30. 842 03
(∆d , Bd ) Vdd = 0

First, let find the replicating portfolio.

www.myactuaryexam.com c
°Yufeng Guo 56
CHAPTER 10. BINOMIAL OPTION PRICING I

• The replicating portfolio at t = 0:


½
4Su + Berh = Vu
4Sd + Berh = Vd
½
447. 412 2 + Be0.08(0.25) = 8. 204 2
4 = 0.607 41 B = −20. 186 7
435. 123 8 + Be0.08(0.25) = 0.740 1

• The replicating portfolio Node u:


½
4u Suu + Bu erh = Vuu
4u Sud + Berh = Vud
½
4u 56. 197 92 + Bu e0.08(0.25) = 16. 197 92
4u = 1 Bu = −39.
4u 41. 632 4 + Bu e0.08(0.25) = 1. 632 4
207 9

• The replicating portfolio Node d:


½
4d Sud + Bd erh = Vud
4d Sdd + Berh = Vdd
½
4d 41. 632 4 + Bd e0.08(0.25) = 1. 632 4
4d = 0.151 28 Bd = −4.
4d 30. 842 03 + Bd e0.08(0.25) = 0
573 5
At t = 0, we

• sell the over-priced call for $5

• build a synthetic call by buying 4 = 0.607 41 share of the stock and


borrowing $20. 186 7 from a bank. Cost: 4S + B = 0.607 41 (40) − 20.
186 7 = 4. 109 7.

So we receive 5 − 4. 109 7 = 0.890 3 at t = 0.

b.

• Suppose the call in the market at time h is fairly priced

To liquidate our position at t = h (i.e. the end of Period 1) yet hedge our
sold call, we’ll do the following:

1. Sell our 4 = 0.607 41 share of the stock, receiving 4Sh

2. Pay the bank Berh = 20. 186 7e0.08(0.25) = 20. 594 5

www.myactuaryexam.com c
°Yufeng Guo 57
CHAPTER 10. BINOMIAL OPTION PRICING I

3. Buy the call from the open market for the fair price of Vh , which is either
Vu = 8. 204 2 if the stock price goes up or Vd = 0.740 1 if the stock price
goes down.

Suppose the stock price goes up to Su and we liquidate our position at Node
u. We’ll

1. Sell our 4 = 0.607 41 share of the stock, receiving 4Su = 0.607 41 (47. 412 2) =
$28. 798 6

2. Pay the bank Berh = 20. 186 7e0.08(0.25) = $20. 594 5

3. Buy the call from the open market for the fair price of Vu = $8. 204 2

The net receipt is: 28. 798 6 − 20. 594 5 − 8. 204 2 = −0.000 1 = $0 (if we
ignore rounding errors)
So we receive $0.890 3 free money at t = 0 without incurring any liability at
time h.

Suppose the stock price goes up to Sd and we liquidate our position at Node
u. We’ll

1. Sell our 4 = 0.607 41 share of the stock, receiving 4Sd = 0.607 41 (35. 123 8) =
$21. 334 5

2. Pay the bank Berh = 20. 186 7e0.08(0.25) = $20. 594 5

3. Buy the call from the open market for the fair price of Vd = $0.740 1

The net receipt is: 21. 334 5 − 20. 594 5 − 0.740 1 = −0.000 1 = 0 (if we ignore
rounding errors)
So we receive $0.890 3 free money at t = 0 without incurring any liability at
time h.

• Suppose the call in the market at h is over priced

We’ll not liquidate our position at h. Instead, we rebalance our replicating


portfolio at t = h and liquidate our position at T = 2h. We’ll consider two
situation.

Situation #1 — the stock price goes up to Su = 40 (1. 185 305) = 47. 412 2
at time h
We’ll change our replicating portfolio from (∆, B) = (0.607 41, B = −$20. 186 7)
to (∆u , Bu ) = (1, −$39. 207 9) at h. We need to buy 1 − 0.607 41 = 0.392 59
share of the stock. The cost is 0.392 59 (47. 412 2) = 18. 613 6. We’ll borrow 18.
613 6 from the bank to pay for this.

www.myactuaryexam.com c
°Yufeng Guo 58
CHAPTER 10. BINOMIAL OPTION PRICING I

So at time h, our total debt to the bank is: 18. 613 4 + 20. 186 7e0.08(0.25) =
39. 207 9. The number of stocks we have is 1. Now at time h, our replicating
portfolio is exactly (∆u , Bu ) = (1, −$39. 207 9).
Then at T = ¡ 2h ¢
If Suu = 40 1. 185 3052 = 56. 197 92

1. Our written call is exercised against us. We need to pay the call holder
Vuu = 56. 197 92 − 40 = 16. 197 92.
2. We sell ∆u stocks in the market and pay off our loan from the bank. Our
net cash receipt is ∆u Suu + Bu erh = 1 × 56. 197 92 − 39. 207 9e0.08(0.25) =
16. 197 97.
3. Our net cash flow is zero (ignoring rounding).

If Sud = 40 (1. 185 305) (0.878 095) = 41. 632 4

1. Our written call is exercised against us. We need to pay the call holder
Vud = 41. 632 4 − 40 = 1. 632 4
2. We sell ∆u stocks in the market and pay off our loan from the bank. Our
net cash receipt is ∆u Sud + Bu erh = 1 × 41. 632 4 − 39. 207 9e0.08(0.25) =
1. 632 4
3. Our net cash flow is zero.

So we receive $0.890 3 free money at t = 0 without incurring any liability at


T.
Situation #2 — the stock price goes down to Sd = 40 (0.878 095) = 35.
123 8 at time h
We’ll change our replicating portfolio from (∆, B) = (0.607 41, B = −$20. 186 7)
to (∆d , Bd ) = (0.151 28, −$4. 573 5) at h. We need to sell 0.607 4 − 0.151 28 =
0.456 12 share of the stock, receiving 0.456 12 (35. 123 8) = $16. 020 7. We im-
mediately send a check of $16. 020 7 to the bank to partially pay our debt. Now
our remaining debt to the bank is
20. 186 7e0.08(0.25) − 16. 020 7 = 4. 573 8 = 4. 573 5 (ignore rounding)
Now at time h, our replicating portfolio is exactly (∆d , Bd ) = (0.151 28, −$4. 573 5)
Then at T = 2h
If Sdu = 40 (1. 185 305) (0.878 095) = 41. 632 4

1. Our written call is exercised against us. We need to pay the call holder
Vdu = 41. 632 4 − 40 = 1. 632 4
2. We sell ∆d stocks in the market and pay off our loan from the bank. Our
net cash receipt is ∆d Sdu +Bd erh = 0.151 28×41. 632 4−4. 573 5e0.08(0.25) =
1. 632 3
3. Our net cash flow is zero (ignoring rounding).

www.myactuaryexam.com c
°Yufeng Guo 59
CHAPTER 10. BINOMIAL OPTION PRICING I

¡ ¢
If Sdd = 40 0.878 0952 = 30. 842 03

1. Our written call expires worthless.

2. We sell ∆d stocks in the market and pay off our loan from the bank. Our
net cash receipt is ∆d Sdd +Bd erh = 0.151 28×30. 842 03−4. 573 5e0.08(0.25) ≈
0

3. Our net cash flow is zero.

So we receive $0.890 3 free money at t = 0 without incurring any liability at


T.

c. If the call option is under priced at h, we just sell off our replicating


portfolio (i.e. sell ∆ share of the stock and pay back our loan to the bank) and
purchase a call from the market. Because the value of our replicating portfolio
at h represents the fair price of a call and the call in the market is lower than
the fair price, the money we get from selling our replicating portfolio exceeds
the call price in the market. So we’ll make some profit.
Now we are standing at time h. The call we buy at time h has life from time
h to T = 2h. The call we initially sold at time zero has life from h to T = 2h.
These two calls exactly offset each other so our liability at T is zero.

Problem 10.14.

We can use the general binomial tree formula by setting S = 0.92 and
δ = r€ = 0.03.
h = T /n = 0.75/3 = 0.25

u = 1.2 d = 0.9
e(r−δ)h − d e(0.04−0.03)0.25 − 0.9
πu = = = 0.341 677
u−d 1.2 − 0.9

πd = 1 − π u = 1 − 0.341 677 = 0.658 323

We’ll calculate part b first.


b. Calculate the American call premium.

www.myactuaryexam.com c
°Yufeng Guo 60
CHAPTER 10. BINOMIAL OPTION PRICING I

Period 2 3 ¡ ¢
¡ ¢ Suuu = 0.92 1. 23 = 1. 589 76
Suu = 0.92 1. 22 = 1. 324 8 Vuuu = 1. 589 76 − 0.85 = 0.739 76
EVuu = 1. 324 8 − 0.85 = 0.474 8 ¡ ¢
Vuu =? Suud = 0.92 1. 22 (0.9) = 1. 192 32
Vuud = 1. 192 32 − 0.85 = 0.342 32
Sud = 0.92 (1. 2) (0.9) = 0.993 6
EVud = 0.993 6 − 0.85 = 0.143 6
Vud =? ¡ ¢
¡ ¢ Sudd = 0.92 (1. 2) 0.92 = 0.894 24
Sdd = 0.92 0.92 = 0.745 2 Vudd = 0.894 24 − 0.85 = 0.044 24
EVdd = 0 ¡ ¢
Vdd =? Sddd = 0.92 0.93 = 0.670 68
Vddd = 0

R
Vuu = e−rh (π u Vuuu + π d Vuud ) = e−(0.04)0.25 (0.341 677 × 0.739 76 + 0.658 323 × 0.342 32) =
0.473 359 ¡ R ¢
Vuu = max Vuu , EVuu = max (0.473 359, 0.474 8) = 0.474 8
So the American call is early exercised at the uu node.
R
Vud = e−rh (π u Vuud + πd Vudd ) = e−(0.04)0.25 (0.341 677 × 0.342 32 + 0.658 323 × 0.044 24) =
0.144 633
¡ R ¢
Vud = max Vud , EVud = max (0.144 633, 0.143 6) = 0.144 633
R
Vdd = e−rh (πu Vudd + πd Vddd ) = e−(0.04)0.25 (0.341 677 × 0.044 24 + 0.658 323 × 0) =
0.01 496 5 ¡ R ¢
Vdd = max Vdd , EVdd = max (0.01 496 5, 0) = 0.014 965

Period 1 2
Vuu = 0.474 8
Su = 0.92 (1. 2) = 1. 104
EVu = 1. 104 − 0.85 = 0.254
Vu =?
Vud = 0.144 633
Sd = 0.92 (0.9) = 0.828
EVd = 0
Vd =?
Vdd = 0.01 496 5

VuR = e−rh (π u Vuu + π d Vud ) = e−(0.04)0.25 (0.341 677 × 0.474 8 + 0.658 323 × 0.144 633) =
0.254 882 ¡ ¢
Vu = max VuR , EVu = max (0.254 882 , 0.254) = 0.254 882
VdR = e−rh (π u Vud + π d Vdd ) = e−(0.04)0.25 (0.341 677 × 0.144 633 + 0.658 323 × 0.01 496 5) =
0.05 8680
¡ ¢
Vd = max VdR , EVd = max (0.05 8680 , 0) = 0.058 68

www.myactuaryexam.com c
°Yufeng Guo 61
CHAPTER 10. BINOMIAL OPTION PRICING I

Period 0 1
Vu = 0.254 882
S = 0.92
EV = 0.92 − 0.85 = 0.07
V =?
Vd = 0.058 68

V R = e−rh (πu Vu + πd Vu ) = e−(0.04)0.25 (0.341 677 × 0.254 882 + 0.658 323 × 0.058 68) =
0.124 467 ¡ ¢
V = max V R , EV = max (0.124 467 , 0.07) = 0.124 467

So the American call premium is 0.124 467 .

a. Calculate the European call premium.


Period 0 1 2 3 ¡ ¢
Suuu = 0.92 1. 23 = 1. 589 76
Vuuu = 1. 589 76 − 0.85 = 0.739 76
Vuu ¡ ¢
Suud = 0.92 1. 22 (0.9) = 1. 192 32
Vu Vuud = 1. 192 32 − 0.85 = 0.342 32

V Vud ¡ ¢
Sudd = 0.92 (1. 2) 0.92 = 0.894 24
Vd Vudd = 0.894 24 − 0.85 = 0.044 24
Vdd ¡ ¢
Sddd = 0.92 0.93 = 0.670 68
Vddd = 0

Vuu = e−rh (π u Vuuu + π d Vuud ) = e−(0.04)0.25 (0.341 677 × 0.739 76 + 0.658 323 × 0.342 32) =
0.473 359
Vud = e−rh (πu Vuud + π d Vudd ) = e−(0.04)0.25 (0.341 677 × 0.342 32 + 0.658 323 × 0.044 24) =
0.144 633
Vdd = e−rh (π u Vudd + πd Vddd ) = e−(0.04)0.25 (0.341 677 × 0.044 24 + 0.658 323 × 0) =
0.01 496 5

Vu = e−rh (πu Vuu + π d Vud ) = e−(0.04)0.25 (0.341 677 × 0.473 359 + 0.658 323 × 0.144 633) =
0.254 394
VdR = e−rh (πu Vud + πd Vdd ) = e−(0.04)0.25 (0.341 677 × 0.144 633 + 0.658 323 × 0.01 496 5) =
0.05 8680

V = e−rh (πu Vu + πd Vu ) = e−(0.04)0.25 (0.341 677 × 0.254 394 + 0.658 323 × 0.058 68) =
0.124 302

So the European call premium is 0.124 302 .

www.myactuaryexam.com c
°Yufeng Guo 62
CHAPTER 10. BINOMIAL OPTION PRICING I

Problem 10.15.

We can use the general binomial tree formula by setting S = 0.92 and
δ = r€ = 0.03.
h = T /n = 0.75/3 = 0.25

u = 1.2 d = 0.9
e(r−δ)h − d e(0.04−0.03)0.25 − 0.9
πu = = = 0.341 677
u−d 1.2 − 0.9

π d = 1 − πu = 1 − 0.341 677 = 0.658 323

b. Calculate the American put premium.


Period 2 3 ¡ ¢
¡ 2¢ Suuu = 0.92 1. 23 = 1. 589 76
Suu = 0.92 1. 2 = 1. 324 8 Vuuu = 0
EVuu = 0 ¡ ¢
Vuu =? Suud = 0.92 1. 22 (0.9) = 1. 192 32
Vuud = 0
Sud = 0.92 (1. 2) (0.9) = 0.993 6
EVud = 1 − 0.993 6 = 0.006 4
Vud =? ¡ 2¢
¡ ¢ S udd = 0.92 (1. 2) 0.9 = 0.894 24
Sdd = 0.92 0.92 = 0.745 2 Vudd = 1 − 0.894 24 = 0.105 76
EVdd = 1 − 0.745 2 = 0.254 8 ¡ ¢
Vdd =? Sddd = 0.92 0.93 = 0.670 68
Vddd = 1 − 0.670 68 = 0.329 32

R
Vuu = e−rh (π u Vuuu + π d Vuud ) = e−(0.04)0.25 (0.341 677 × 0 + 0.658 323 × 0) =
0
¡ R ¢
Vuu = max Vuu , EVuu = max (0, 0) = 0
R
Vud = e−rh (π u Vuud + πd Vudd ) = e−(0.04)0.25 (0.341 677 × 0 + 0.658 323 × 0.105 76) =
0.06893 1

¡ R ¢
Vud = max Vud , EVud = max (0.06893 1, 0.006 4) = 0.068 931
R
Vdd = e−rh (πu Vudd + πd Vddd ) = e−(0.04)0.25 (0.341 677 × 0.105 76 + 0.658 323 × 0.329 32) =
0.250 418
¡ R ¢
Vdd = max Vdd , EVdd = max (0.250 418, 0.254 8) = 0.254 8
The American put is early exercised at the dd node.

www.myactuaryexam.com c
°Yufeng Guo 63
CHAPTER 10. BINOMIAL OPTION PRICING I

Period 1 2
Vuu = 0
Su = 0.92 (1. 2) = 1. 104
EVu = 0
Vu =?
Vud = 0.068 931
Sd = 0.92 (0.9) = 0.828
EVd = 1 − 0.828 = 0.172
Vd =?
Vdd = 0.254 8

VuR = e−rh (πu Vuu + π d Vud ) = e−(0.04)0.25 (0.341 677 × 0 + 0.658 323 × 0.068 931 ) =
0.04 492 7 ¡ ¢
Vu = max VuR , EVu = max (0.04 492 7 , 0) = 0.04 492 7
VdR = e−rh (πu Vud + πd Vdd ) = e−(0.04)0.25 (0.341 677 × 0.068 931 + 0.658 323 × 0.254 8) =
0.189 389
¡ ¢
Vd = max VdR , EVd = max (0.189 389 , 0.172) = 0.189 389

Period 0 1
Vu = 0.04 492 7
S = 0.92
EV = 1 − 0.92 = 0.08
V =?
Vd = 0.189 389

V R = e−rh (πu Vu + πd Vu ) = e−(0.04)0.25 (0.341 677 × 0.04 492 7 + 0.658 323 × 0.189 389) =
0.138 636 ¡ ¢
V = max V R , EV = max (0.138 636 , 0.08) = 0.138 636

So the American put premium is 0.138 636 .

a. Calculate the European put premium.


Period 0 1 2 3 ¡ ¢
Suuu = 0.92 1. 23 = 1. 589 76
Vuuu = 0
Vuu ¡ ¢
Vu Suud = 0.92 1. 22 (0.9) = 1. 192 32
Vuud = 0
V Vud ¡ ¢
Sudd = 0.92 (1. 2) 0.92 = 0.894 24
Vd Vudd = 1 − 0.894 24 = 0.105 76
Vdd ¡ ¢
Sddd = 0.92 0.93 = 0.670 68
Vddd = 1 − 0.670 68 = 0.329 32

www.myactuaryexam.com c
°Yufeng Guo 64
CHAPTER 10. BINOMIAL OPTION PRICING I

Vuu = e−rh (π u Vuuu + π d Vuud ) = e−(0.04)0.25 (0.341 677 × 0 + 0.658 323 × 0) =

0
Vud = e−rh (π u Vuud + πd Vudd ) = e−(0.04)0.25 (0.341 677 × 0 + 0.658 323 × 0.105 76) =
0.06893 1
Vdd = e−rh (πu Vudd + πd Vddd ) = e−(0.04)0.25 (0.341 677 × 0.105 76 + 0.658 323 × 0.329 32) =
0.250 418

Vu = e−rh (πu Vuu + π d Vud ) = e−(0.04)0.25 (0.341 677 × 0 + 0.658 323 × 0.068 931 ) =
0.04 492 7
Vd = e−rh (π u Vud + πd Vdd ) = e−(0.04)0.25 (0.341 677 × 0.068 931 + 0.658 323 × 0.250 418) =
0.186 533

V = e−rh (π u Vu + π d Vu ) = e−(0.04)0.25 (0.341 677 × 0.04 492 7 + 0.658 323 × 0.186 533) =
0.136 775

So the European put premium is 0.136 775 .

Problem 10.16.
This problem looks scary, but it’s actually simple if you know what’s the
underlying asset.
The underlying asset is $1. The call option gives the call holder the right to
purchase $1 with a guaranteed price (i.e. strike price) of 120 Yen. So the strike
price and the option price are expressed in Yen.
To simplify the problem, we can treat the underlying asset $1 as a stock and
translate the original problem into the following:

• Now we are living in Japan and are interested in options on a stock.


• The current stock price is S = 120 Yen.
• The continuously compounded risk free interest rate is rf = 0.01 (remem-
ber we are living in Japan).
• The stock’s volatility is σ = 0.1.
• The stock’s continuous dividend yield is δ = 0.05; the dollar interest rate
is like the dividend yield.
• T =1
• h = T /3 = 1/3

Now we can use the standard binomial formula.


√ √
u = e(r−δ)h+σ h = e(0.01−0.05)1/3+0.1 1/3 = 1. 045 402

www.myactuaryexam.com c
°Yufeng Guo 65
CHAPTER 10. BINOMIAL OPTION PRICING I

√ √
d = e(r−δ)h−σ h
= e(0.01−0.05)1/3−0.1 1/3 = 0.931 398
e(r−δ)h − d e(0.01−0.05)1/3 − 0.931 398
πu = = = 0.485 57
u−d 1. 045 402 − 0.931 398
πd = 1 − π u = 1 − 0.485 57 = 0.514 43

a. Calculate the price of the American call option


Period 2 3 ¡ ¢
¡ ¢ Suuu = 120 1. 045 4023 = 137. 098 0
Suu = 120 1. 045 4022 = 131. 143 8 Vuuu = 137. 098 0 − 120 = 17. 098
EVuu = 131. 143 8 − 120 = 11. 143 8 ¡ ¢
Vuu =? Suud = 120 1. 045 4022 (0.931 398) = 122. 147 1
Vuud = 122. 147 1 − 120 = 2. 147 1
Sud = 120 (1. 045 402) (0.931 398) = 116. 842 2
EVud = 0
Vud =? ¡ ¢
¡ ¢ Sudd = 120 (1. 045 402) 0.931 3982 = 108. 826 6
Sdd = 120 0.931 3982 = 104. 100 3 Vudd = 0
EVdd = 0 ¡ ¢
Vdd =? Sddd = 120 0.931 3983 = 96. 958 8
Vddd = 0
R
Vuu = e−rh (π u Vuuu + π d Vuud ) = e−(0.01)1/3 (0.485 57 × 17. 098 + 0.514 43 × 2. 147 1) =
9. 375 5 ¡ R ¢
Vuu = max Vuu , EVuu = max (9. 375 5, 11. 143 8) = 11. 143 8
So the American call is early exercised at the uu node.
R
Vud = e−rh (πu Vuud + π d Vudd ) = e−(0.01)1/3 (0.485 57 × 2. 147 1 + 0.514 43 × 0) =
1. 039 1
¡ R ¢
Vud = max Vud , EVud = max (1. 039 1, 0) = 1. 039 1
R
Vdd = e−rh (πu Vudd + π d Vddd ) = e−(0.01)1/3 (0.485 57 × 0 + 0.514 43 × 0) =
0 ¡ R ¢
Vdd = max Vdd , EVdd = max (0, 0) = 0

Period 1 2
Vuu = 11. 143 8
Su = 120 (1. 045 402) = 125. 448 2
EVu = 125. 448 2 − 120 = 5. 448 2
Vu =?
Vud = 1. 039 1
Sd = 120 (0.931 398) = 111. 767 8
EVd = 0
Vd =?
Vdd = 0

www.myactuaryexam.com c
°Yufeng Guo 66
CHAPTER 10. BINOMIAL OPTION PRICING I

VuR = e−rh (π u Vuu + π d Vud ) = e−(0.01)1/3 (0.485 57 × 11. 143 8 + 0.514 43 × 1. 039 1) =
5. 925 9 ¡ ¢
Vu = max VuR , EVu = max (5. 925 9 , 5. 448 2) = 5. 925 9

VdR = e−rh (π u Vud + π d Vdd ) = e−(0.01)1/3 (0.485 57 × 1. 039 1 + 0.514 43 × 0) =


0.502 9
¡ ¢
Vd = max VdR , EVd = max (0.502 9 , 0) = 0.502 9

Period 0 1
Vu = 5. 925 9
S = 120
EV = 0
V =?
Vd = 0.502 9

V R = e−rh (π u Vu + π d Vu ) = e−(0.01)1/3 (0.485 57 × 5. 925 9 + 0.514 43 × 0.502 9) =


3. 125 7 ¡ ¢
V = max V R , EV = max (3. 125 7 , 0) = 3. 125 7

So the American call premium is 3. 125 7 Yen.

Calculate the price of the European call


Period 0 1 2 3 ¡ ¢
Suuu = 120 1. 045 4023 = 137. 098 0
Vuuu = 137. 098 0 − 120 = 17. 098
Vuu ¡ ¢
Vu Suud = 120 1. 045 4022 (0.931 398) = 122. 147 1
Vuud = 122. 147 1 − 120 = 2. 147 1
V Vud ¡ ¢
Sudd = 120 (1. 045 402) 0.931 3982 = 108. 826 6
Vd Vudd = 0
Vdd ¡ ¢
Sddd = 120 0.931 3983 = 96. 958 8
Vddd = 0

Vuu = e−rh (π u Vuuu + π d Vuud ) = e−(0.01)1/3 (0.485 57 × 17. 098 + 0.514 43 × 2. 147 1) =
9. 375 5
Vud = e−rh (π u Vuud + πd Vudd ) = e−(0.01)1/3 (0.485 57 × 2. 147 1 + 0.514 43 × 0) =
1. 039 1
Vdd = e−rh (π u Vudd + π d Vddd ) = e−(0.01)1/3 (0.485 57 × 0 + 0.514 43 × 0) =
0

Vu = e−rh (πu Vuu + π d Vud ) = e−(0.01)1/3 (0.485 57 × 9. 375 5 + 0.514 43 × 1. 039 1) =


5. 070 1

www.myactuaryexam.com c
°Yufeng Guo 67
CHAPTER 10. BINOMIAL OPTION PRICING I

Vd = e−rh (π u Vud + π d Vdd ) = e−(0.01)1/3 (0.485 57 × 1. 039 1 + 0.514 43 × 0) =


0.502 9

V = e−rh (πu Vu + πd Vu ) = e−(0.01)1/3 (0.485 57 × 5. 070 1 + 0.514 43 × 0.502 9) =


2. 711 5

So the European call premium is 2. 711 5 Yen.

b. Calculate the price of the American put


Period 2 3 ¡ ¢
¡ ¢ Suuu = 120 1. 045 4023 = 137. 098 0
Suu = 120 1. 045 4022 = 131. 143 8 Vuuu = 0
EVuu = 0 ¡ ¢
Vuu =? Suud = 120 1. 045 4022 (0.931 398) = 122. 147 1
Vuud = 0
Sud = 120 (1. 045 402) (0.931 398) = 116. 842 2
EVud = 120 − 116. 842 2 = 3. 157 8
Vud =? ¡ ¢
¡ ¢ Sudd = 120 (1. 045 402) 0.931 3982 = 108. 826 6
Sdd = 120 0.931 3982 = 104. 100 3 Vudd = 120 − 108. 826 6 = 11. 173 4
EVdd = 120 − 104. 100 3 = 15. 899 7 ¡ ¢
Vdd =? Sddd = 120 0.931 3983 = 96. 958 8
Vddd = 120 − 96. 958 8 = 23. 041 2
R
Vuu = e−rh (π u Vuuu + π d Vuud ) = e−(0.01)1/3 (0.485 57 × 0 + 0.514 43 × 0) =
0 ¡ R ¢
Vuu = max Vuu , EVuu = max (0, 0) = 0
R
Vud = e−rh (πu Vuud + π d Vudd ) = e−(0.01)1/3 (0.485 57 × 0 + 0.514 43 × 11. 173 4) =
5. 728 8
¡ R ¢
Vud = max Vud , EVud = max (3. 157 8, 5. 728 8) = 5. 728 8
R
Vdd = e−rh (π u Vudd + πd Vddd ) = e−(0.01)1/3 (0.485 57 × 11. 173 4 + 0.514 43 × 23. 041 2) =
17. 221 1 ¡ R ¢
Vdd = max Vdd , EVdd = max (17. 221 1, 15. 899 7) = 17. 221 1

Period 1 2
Vuu = 0
Su = 120 (1. 045 402) = 125. 448 2
EVu = 0
Vu =?
Vud = 5. 728 8
Sd = 120 (0.931 398) = 111. 767 8
EVd = 120 − 111. 767 8 = 8. 232 2
Vd =?
Vdd = 17. 221 1

www.myactuaryexam.com c
°Yufeng Guo 68
CHAPTER 10. BINOMIAL OPTION PRICING I

VuR = e−rh (π u Vuu + π d Vud ) = e−(0.01)1/3 (0.485 57 × 0 + 0.514 43 × 5. 728 8) =


2. 937 3 ¡ ¢
Vu = max VuR , EVu = max (2. 937 3 , 0) = 2. 937 3

VdR = e−rh (π u Vud + π d Vdd ) = e−(0.01)1/3 (0.485 57 × 5. 728 8 + 0.514 43 × 17. 221 1) =
11. 602 0
¡ ¢
Vd = max VdR , EVd = max (11. 602 0 , 8. 232 2) = 11. 602

Period 0 1
Vu = 2. 937 3
S = 120
EV = 0
V =?
Vd = 11. 602

V R = e−rh (π u Vu + π d Vu ) = e−(0.01)1/3 (0.485 57 × 2. 937 3 + 0.514 43 × 11. 602) =


7. 370 1 ¡ ¢
V = max V R , EV = max (7. 370 1 , 0) = 7. 370 1

Since the exercise value is never greater than the roll-back value, the Amer-
ican put is not early exercised. Hence the American put and the European put
have the same value.
Both the American put and European put are worth 7. 370 1 Yen.

c. Since the underlying asset generates dividend (the Yen interest rate is
like the dividend rate), it may be optimal to early exercise an American call.
However, even if the asset generates dividend, it may not be optimal to early
exercise an American put.

Problem 10.17.

Method 1 Apply the stock binomial formula to futures


As explained in my study guide, we can apply the stock binomial formula
to options on futures if we do the following three things:

• Set the dividend yield equal to the risk free rate (i.e. δ = r)

Cu − Cd Cu − Cd
• 4= instead of 4 = e−rh
Su − Sd Su − Sd
µ ¶
1−d u−1 Su Cd − Sd Cu
• B = V = e−rh Cu + Cd instead of B = e−rh .
u−d u−d Su − Sd

www.myactuaryexam.com c
°Yufeng Guo 69
CHAPTER 10. BINOMIAL OPTION PRICING I

h = 1 year long.
√ √ √
u = e(r−δ)h+σ√ h = eσ h√= e0.1 1 = 1. 105 170 9
d = e(r−δ)h−σ h = e−0.1 1 = 0.904 837 4
e(r−δ)h − d 1−d 1 − 0.904 837 4
πu = = = = 0.475 021
u−d u−d 1. 105 170 9 − 0.904 837 4
πd = 1 − π u = 1 − 0.475 021 = 0.524 979

Period 0 1
Su = 300 (1. 105 170 9) = 331. 551 27
Vu = 331. 551 27 − 290 = 41. 551 27
S = 300
EV = 300 − 290 = 10
V =?
(∆, B) =?
Sd = 300 (0.904 837 4) = 271. 451 22
Vd = 0

V R = e−rh (πu Vu + πd Vu ) = e−(0.06)1 (0.475 021 × 41. 551 27 + 0.524 979 × 0) =


18. 588 29 ¡ ¢
V = max V R , EV = max (18. 588 29 , 10) = 18. 588 29
So the call premium is $18. 588 29 at t = 0.
Cu − Cd 41. 551 27 − 0
4= = = 0.691 368
Su − Sd 331. 551 27 − 271. 451 22
B = V = 18. 588 29

So to form a replicating portfolio at time zero, we buy 0.691 368 unit of


futures contract (i.e. enter 0.691 368 unit of futures contract as a buyer) and
invest $18. 588 29 in a savings account.

Method 2
Just as√in Method

1, we calculate
u = eσ √h
= e0.1 1 √= 1. 105 170 9
d = e−σ h = e−0.1 1 = 0.904 837 4

Next, we build a futures price tree.


Period 0 1
u
F1,1 = 300 (1. 105 170 9) = 331. 551 27
Vu = 331. 551 27 − 290 = 41. 551 27
F0,1 = 300
EV = 300 − 290 = 10
V =?
(∆, B) =?
d
F1,1 = 300 (0.904 837 4) = 271. 451 22
Vd = 0

www.myactuaryexam.com c
°Yufeng Guo 70
CHAPTER 10. BINOMIAL OPTION PRICING I

If we buy ∆ futures contract at t = 0, our gain in these futures contracts at


T is:
¡ u ¢
• ∆ F1,1 − F0,1 = ∆F0,1 (u − 1) in the u node
¡ d ¢
• ∆ F1,1 − F0,1 = ∆F0,1 (d − 1) in the d node

If we put $B in a savings account, we’ll get BerT at T .


We want our replicating portfolio and the call option have the same payoff
at T½: ¡ ¢ ½
u
∆ ¡F1,1 − F0,1 ¢ + BerT = Vu ∆F0,1 (u − 1) + BerT = Vu

d rT
∆ F1,1 − F0,1 + Be = Vd ∆F0,1 (d − 1) + BerT = Vd

½
∆300 (1. 105 170 9 − 1) + Be0.06(1) = 41. 551 27
∆300 (0.904 837 4 − 1) + Be0.06(1) = 0
∆ = 0.691 368 B = 18. 588 29

Since it costs nothing to enter a future contract, the call premium is equal
to B:
V = B = 18. 588 29

The statement "Replicating a call option always entails borrowing to buy


the underlying asset" is true for calls on stocks. However, it’s not true for calls
on futures; it costs nothing to enter a futures contract (so you don’t need to
borrow money to finance your transaction on futures). If you write a call on
futures, you need to enter ∆ futures contracts and simultaneously deposit the
call premium into a savings account. This way, at call expiration date T , the
combined payoff of your ∆ futures contracts and your deposit in the savings
account will replicate the payoff of a call on futures.

Problem 10.18.

We’ll apply the stock binomial formula to futures. We need to do the fol-
lowing three things:

• Set the dividend yield equal to the risk free rate (i.e. δ = r)

Cu − Cd Cu − Cd
• 4= instead of 4 = e−rh
Su − Sd Su − Sd
µ ¶
1−d u−1 Su Cd − Sd Cu
• B = V = e−rh Cu + Cd instead of B = e−rh .
u−d u−d Su − Sd

www.myactuaryexam.com c
°Yufeng Guo 71
CHAPTER 10. BINOMIAL OPTION PRICING I

h = 1/3

√ √ √
= e0.3 1/3 = 1. 189 110
u = e(r−δ)h+σ√ h = eσ h√
d = e(r−δ)h−σ h = e−0.3 1/3 = 0.840 965
Notice that ud = 1
e(r−δ)h − d 1−d 1 − 0.840 965
πu = = = = 0.456 807
u−d u−d 1. 189 110 − 0.840 965

πd = 1 − π u = 1 − 0.456 807 = 0.543 193

• calculate the American call premium

Period 2 3
Suuu = 1000u3 = 1681. 380 8
Suu = 1000u2 = 1413. 982 6 Vuuu = 1681. 380 8 − 1000 = 681. 380 8
EVuu = 1413. 982 6 − 1000 = 413. 982 6
Vuu =? Suud = 1000u2 d = 1000u = 1189. 1100
Vuud = 1189. 11 − 1000 = 189. 11
Sud = 1000ud = 1000
EVud = 0
Vud =?
Sudd = 1000ud2 = 1000d = 840. 965
2
Sdd = 1000d = 707. 222 1 Vudd = 0
EVdd = 0 ¡ ¢
Vdd =? Sddd = 1000d3 = 1000 0.840 965 3 = 594. 749
Vddd = 0

R
Vuu = e−rh (π u Vuuu + π d Vuud ) = e−(0.05)1/3 (0.456 807 × 681. 380 8 + 0.543 193 × 189. 11) =
407. 140 2 ¡ R ¢
Vuu = max Vuu , EVuu = max (407. 140 2, 413. 982 6) = 413. 982 6

The American call is early exercised at Node uu.


R
Vud = e−rh (πu Vuud + π d Vudd ) = e−(0.05)1/3 (0.456 807 × 189. 11 + 0.543 193 × 0) =
84. 958 9

¡ R ¢
Vud = max Vud , EVud = max (84. 958 9, 0) = 84. 958 9

R
Vdd = e−rh (π u Vudd + πd Vddd ) = e−(0.05)1/3 (0.456 807 × 0 + 0.543 193 × 0) =
0 ¡ R ¢
Vdd = max Vdd , EVdd = max (0, 0) = 0

www.myactuaryexam.com c
°Yufeng Guo 72
CHAPTER 10. BINOMIAL OPTION PRICING I

Period 1 2
Vuu = 413. 982 6
Su = 1000 (1. 189 11) = 1189. 11
EVu = 1189. 11 − 1000 = 189. 11
Vu =?
Vud = 84. 958 9
Sd = 1000 (0.840 965 ) = 840. 965
EVd = 0
Vd =?
Vdd = 0

VuR = e−rh (π u Vuu + π d Vud ) = e−(0.05)1/3 (0.456 807 × 413. 982 6 + 0.543 193 × 84. 958 9) =
231. 370 7 ¡ ¢
Vu = max VuR , EVu = max (231. 370 7 , 189. 11) = 231. 370 7

VdR = e−rh (π u Vud + π d Vdd ) = e−(0.05)1/3 (0.456 807 × 84. 958 9 + 0.543 193 × 0) =
38. 168 4
¡ ¢
Vd = max VdR , EVd = max (38. 168 4 , 0) = 38. 168 4

Period 0 1
Vu = 231. 370 7
S = 1000
EV = 0
V =?
Vd = 38. 168 4

V R = e−rh (π u Vu + π d Vu ) = e−(0.05)1/3 (0.456 807 × 231. 370 7 + 0.543 193 × 38. 168 4) =
124. 334 9 ¡ ¢
V = max V R , EV = max (124. 334 9 , 0) = 124. 334 9
So the American call premium is 124. 334 9.

• Calculate the European call premium


Period 0 1 2 3
Suuu = 1000u3 = 1681. 380 8
Vuuu = 1681. 380 8 − 1000 = 681. 380 8
Vuu
Suud = 1000u2 d = 1000u = 1189. 1100
Vu Vuud = 1189. 11 − 1000 = 189. 11
V Vud

Vd Sudd = 1000ud2 = 1000d = 840. 965


Vudd = 0 ¡ ¢
Vdd Sddd = 1000d3 = 1000 0.840 965 3 = 594. 749
Vddd = 0

www.myactuaryexam.com c
°Yufeng Guo 73
CHAPTER 10. BINOMIAL OPTION PRICING I

Vuu = e−rh (π u Vuuu + π d Vuud ) = e−(0.05)1/3 (0.456 807 × 681. 380 8 + 0.543 193 × 189. 11) =
407. 140 2
Vud = e−rh (πu Vuud + π d Vudd ) = e−(0.05)1/3 (0.456 807 × 189. 11 + 0.543 193 × 0) =
84. 958 9
Vdd = e−rh (π u Vudd + πd Vddd ) = 0

Vu = e−rh (πu Vuu + π d Vud ) = e−(0.05)1/3 (0.456 807 × 407. 140 2 + 0.543 193 × 84. 958 9) =
228. 296 7
Vd = e−rh (π u Vud + π d Vdd ) = e−(0.05)1/3 (0.456 807 × 84. 958 9 + 0.543 193 × 0) =
38. 168 4

V = e−rh (πu Vu + πd Vu ) = e−(0.05)1/3 (0.456 807 × 228. 296 7 + 0.543 193 × 38. 168 4) =
122. 953 9

So the European call premium is $122. 953 9.

• Time zero replicating portfolio for the European call

B = V = $122. 953 9
Vu − Vd 228. 296 7 − 38. 168 4
4= = = 0.546 118
Su − Sd 1000 (1. 189 11) − 1000 (0.840 965 )

• calculate the American put premium

Period 2 3
Suuu = 1000u3 = 1681. 380 8
Suu = 1000u2 = 1413. 982 6 Vuuu = 0
EVuu = 0
Vuu =? Suud = 1000u2 d = 1000u = 1189. 1100
Vuud = 0
Sud = 1000ud = 1000
EVud = 0
Vud =?
Sudd = 1000ud2 = 1000d = 840. 965
2
Sdd = 1000d = 707. 222 1 Vudd = 1000 − 840. 965 = 159. 035
EVdd = 1000 − 707. 222 1 = 292. 777 9 ¡ ¢
Vdd =? Sddd = 1000d3 = 1000 0.840 965 3 = 594. 749
Vddd = 1000 − 594. 749 = 405. 251

R
Vuu = e−rh (π u Vuuu + π d Vuud ) = e−(0.05)1/3 (0.456 807 × 0 + 0.543 193 × 0) =
0 ¡ R ¢
Vuu = max Vuu , EVuu = max (0, 0) = 0
R
Vud = e−rh (πu Vuud + π d Vudd ) = e−(0.05)1/3 (0.456 807 × 0 + 0.543 193 × 159. 035) =
84. 958 9

www.myactuaryexam.com c
°Yufeng Guo 74
CHAPTER 10. BINOMIAL OPTION PRICING I

¡ R ¢
Vud = max Vud , EVud = max (84. 958 9, 0) = 84. 958 9

R
Vdd = e−rh (πu Vudd + πd Vddd ) = e−(0.05)1/3 (0.456 807 × 159. 035 + 0.543 193 × 405. 251) =
287. 938 62 ¡ R ¢
Vdd = max Vdd , EVdd = max (287. 938 62, 292. 777 9) = 292. 777 9
The American put is early exercised at the node dd.

Period 1 2
Vuu = 0
Su = 1000 (1. 189 11) = 1189. 11
EVu = 0
Vu =?
Vud = 84. 958 9
Sd = 1000 (0.840 965 ) = 840. 965
EVd = 1000 − 840. 965 = 159. 035
Vd =?
Vdd = 292. 777 9

VuR = e−rh (π u Vuu + π d Vud ) = e−(0.05)1/3 (0.456 807 × 0 + 0.543 193 × 84. 958 9) =
45. 386 3 ¡ ¢
Vu = max VuR , EVu = max (45. 386 3 , 0) = 45. 386 3

VdR = e−rh (π u Vud + π d Vdd ) = e−(0.05)1/3 (0.456 807 × 84. 958 9 + 0.543 193 × 292. 777 9) =
194. 574 6

¡ ¢
Vd = max VdR , EVd = max (194. 574 6 , 159. 035) = 194. 574 6

Period 0 1
Vu = 45. 386 3
S = 1000
EV = 0
V =?
Vd = 194. 574 6

V R = e−rh (π u Vu + π d Vu ) = e−(0.05)1/3 (0.456 807 × 45. 386 3 + 0.543 193 × 194. 574 6) =
124. 334 7 ¡ ¢
V = max V R , EV = max (124. 334 7 , 0) = 124. 334 7
So the American put premium is 124. 334 7.

• Calculate the European put premium.

www.myactuaryexam.com c
°Yufeng Guo 75
CHAPTER 10. BINOMIAL OPTION PRICING I

Period 0 1 2 3
Suuu = 1000u3 = 1681. 380 8
Vuuu = 0
Vuu
Suud = 1000u2 d = 1000u = 1189. 1100
Vu Vuud = 0

V Vud
Sudd = 1000ud2 = 1000d = 840. 965
Vd Vudd = 1000 − 840. 965 = 159. 035
Vdd ¡ ¢
Sddd = 1000d3 = 1000 0.840 965 3 = 594. 749
Vddd = 1000 − 594. 749 = 405. 251

Vuu = e−rh (π u Vuuu + π d Vuud ) = e−(0.05)1/3 (0.456 807 × 0 + 0.543 193 × 0) =


0
Vud = e−rh (πu Vuud + π d Vudd ) = e−(0.05)1/3 (0.456 807 × 0 + 0.543 193 × 159. 035) =
84. 958 9

Vdd = e−rh (π u Vudd + πd Vddd ) = e−(0.05)1/3 (0.456 807 × 159. 035 + 0.543 193 × 405. 251) =
287. 938 62
Vu = e−rh (πu Vuu + π d Vud ) = e−(0.05)1/3 (0.456 807 × 0 + 0.543 193 × 84. 958 9) =
45. 386 3
Vd = e−rh (π u Vud + π d Vdd ) = e−(0.05)1/3 (0.456 807 × 84. 958 9 + 0.543 193 × 287. 938 62) =
191. 989 4
V = e−rh (πu Vu + πd Vu ) = e−(0.05)1/3 (0.456 807 × 45. 386 3 + 0.543 193 × 191. 989 4) =
122. 9537

So the European call premium is $122. 9537.


• Time zero replicating portfolio for the European call
B = V = $122. 9537
Vu − Vd 45. 386 3 − 191. 989 4
4= = = −0.421 1
Su − Sd 1000 (1. 189 11) − 1000 (0.840 965 )
So at t = 0 we need to enter 0.421 1 futures contract as a seller and deposit
122. 9537 in a savings account.
• Why the European call and a European put have the same premium
The standard put-call parity is:
C + P V (K) = P + S
S is the price of the underlying asset at time zero. For the futures contract,
S is the present value of the forward price:

S = P V (F0,T )
In this problem, the forward price and the strike price are equal (i.e. K =
F0,T ). Hence P V (K) = P V (F0,T ) = S. This gives us C = P .

www.myactuaryexam.com c
°Yufeng Guo 76
CHAPTER 10. BINOMIAL OPTION PRICING I

Problem 10.19.

For options on√ a stock index, you can



use the standard binomial formula.
(r−δ)h+σ h (0.05−0.03)1+0.3 1
u=e √ =e √ = 1. 377 128
d = e(r−δ)h−σ h = e(0.05−0.03)1−0.3 1 = 0.755 784

e(r−δ)h − d e(0.05−0.03)1 − 0.755 784


πu = = = 0.425 557
u−d 1. 377 128 − 0.755 784

π d = 1 − πu = 1 − 0.425 557 = 0.574 443

a. Calculate the price of the European call option


Period 0 1 2 3 ¡ ¢
Suuu = 100 1. 377 1283 = 261. 169 8
Vuuu = 261. 169 8 − 95 = 166. 169 8
Vuu ¡ ¢
Vu Suud = 100 1. 377 1282 (0.755 784) = 143. 333 0
Vuud = 143. 333 0 − 95 = 48. 3330
V Vud ¡ ¢
Sudd = 100 (1. 377 128) 0.755 7842 = 78. 662 9
Vd Vudd = 0
Vdd ¡ ¢
Sddd = 100 0.755 7843 = 43. 171 1
Vddd = 0

Vuu = e−rh (π u Vuuu + π d Vuud ) = e−(0.05)1 (0.425 557 × 166. 169 8 + 0.574 443 × 48. 333) =
93. 676 4
Vud = e−rh (π u Vuud + πd Vudd ) = e−(0.05)1 (0.425 557 × 48. 3330 + 0.574 443 × 0) =
19. 565 3
Vdd = e−rh (πu Vudd + πd Vddd ) = 0

Vu = e−rh (πu Vuu + π d Vud ) = e−(0.05)1 (0.425 557 × 93. 676 4 + 0.574 443 × 19. 565 3) =
48. 611 4
Vd = e−rh (π u Vud + πd Vdd ) = e−(0.05)1 (0.425 557 × 19. 565 3 + 0.574 443 × 0) =
7. 920 1

V = e−rh (π u Vu + π d Vu ) = e−(0.05)1 (0.425 557 × 48. 611 4 + 0.574 443 × 7. 920 1) =


24. 005 8

So the European call premium is $24. 005 8.

b. Calculate the European put premium

www.myactuaryexam.com c
°Yufeng Guo 77
CHAPTER 10. BINOMIAL OPTION PRICING I

Period 0 1 2 3 ¡ ¢
Suuu = 100 1. 377 1283 = 261. 169 8
Vuuu = 0
Vuu ¡ ¢
Vu Suud = 100 1. 377 1282 (0.755 784) = 143. 333 0
Vuud = 0
V Vud ¡ ¢
Sudd = 100 (1. 377 128) 0.755 7842 = 78. 662 9
Vd Vudd = 95 − 78. 662 9 = 16. 337 1
Vdd ¡ ¢
Sddd = 100 0.755 7843 = 43. 171 1
Vddd = 95 − 43. 171 1 = 51. 828 9

Vuu = e−rh (π u Vuuu + π d Vuud ) = e−(0.05)1 (0.425 557 × 166. 169 8 + 0.574 443 × 48. 333) =
0
Vud = e−rh (πu Vuud + π d Vudd ) = e−(0.05)1 (0.425 557 × 0 + 0.574 443 × 16. 337 1) =
8. 927 0
Vdd = e−rh (π u Vudd + πd Vddd ) = e−(0.05)1 (0.425 557 × 16. 337 1 + 0.574 443 × 51. 828 9) =
34. 934 0

Vu = e−rh (πu Vuu + π d Vud ) = e−(0.05)1 (0.425 557 × 0 + 0.574 443 × 8. 927 0) =
4. 8780
Vd = e−rh (π u Vud + π d Vdd ) = e−(0.05)1 (0.425 557 × 8. 927 0 + 0.574 443 × 34. 934 0) =
22. 702 6

V = e−rh (πu Vu + πd Vu ) = e−(0.05)1 (0.425 557 × 4. 8780 + 0.574 443 × 22. 702 6) =
14. 379 9

So the European put premium is $14. 379 9.

c. If we switch S and K and switch r and δ and recalculate the option price,
what happens?
After the switch, we have:
• S = 95
• K = 100
• r = 3%
• δ = 5%
√ √
u = e(r−δ)h+σ√ h = e(0.03−0.05)1+0.3√1 = 1. 323 13
d = e(r−δ)h−σ h = e(0.03−0.05)1−0.3 1 = 0.726 149
e(r−δ)h − d e(0.03−0.05)1 − 0.726 149
πu = = = 0.425 557
u−d 1. 323 13 − 0.726 149
πd = 1 − π u = 1 − 0.425 557 = 0.574 443

www.myactuaryexam.com c
°Yufeng Guo 78
CHAPTER 10. BINOMIAL OPTION PRICING I

• Calculate the European call premium

Period 0 1 2 3 ¡ ¢
Suuu = 95 1. 323 133 = 220. 0550
Vuuu = 220. 0550 − 100 = 120. 055
Vuu ¡ ¢
Suud = 95 1. 323 132 (0.726 149) = 120. 768 7
Vu Vuud = 120. 768 7 − 100 = 20. 768 7
V Vud ¡ ¢
Sudd = 95 (1. 323 13) 0.726 1492 = 66. 279 3
Vd Vudd = 0
Vdd ¡ ¢
Sddd = 95 0.726 1493 = 36. 374 8
Vddd = 0

Vuu = e−rh (π u Vuuu + π d Vuud ) = e−(0.03)1 (0.425 557 × 120. 055 + 0.574 443 × 20. 768 7) =
61. 158 1
Vud = e−rh (π u Vuud + πd Vudd ) = e−(0.03)1 (0.425 557 × 20. 768 7 + 0.574 443 × 0) =
8. 577 1
Vdd = e−rh (π u Vudd + πd Vddd ) = e−(0.03)1 (0.425 557 × 0 + 0.574 443 × 0) =
0

Vu = e−rh (πu Vuu + π d Vud ) = e−(0.03)1 (0.425 557 × 61. 158 1 + 0.574 443 × 8. 577 1) =
30. 038 5
Vd = e−rh (π u Vud + πd Vdd ) = e−(0.03)1 (0.425 557 × 8. 577 1 + 0.574 443 × 0) =
3. 542 2

V = e−rh (π u Vu + π d Vd ) = e−(0.03)1 (0.425 557 × 30. 038 5 + 0.574 443 × 3. 542 2) =


14. 3799
The call premium after the switch is equal to the put premium before the
switch.
• Calculate the European put premium

Period 0 1 2 3 ¡ ¢
Suuu = 95 1. 323 133 = 220. 0550
Vuuu = 0
Vuu ¡ ¢
Suud = 95 1. 323 132 (0.726 149) = 120. 768 7
Vu Vuud = 0
V Vud ¡ ¢
Sudd = 95 (1. 323 13) 0.726 1492 = 66. 279 3
Vd Vudd = 100 − 66. 279 3 = 33. 720 7
Vdd ¡ ¢
Sddd = 95 0.726 1493 = 36. 374 8
Vddd = 100 − 36. 374 8 = 63. 625 2

www.myactuaryexam.com c
°Yufeng Guo 79
CHAPTER 10. BINOMIAL OPTION PRICING I

Vuu = e−rh (π u Vuuu + π d Vuud ) = e−(0.03)1 (0.425 557 × 0 + 0.574 443 × 0) =


0
Vud = e−rh (πu Vuud + π d Vudd ) = e−(0.03)1 (0.425 557 × 0 + 0.574 443 × 33. 720 7) =
18. 798 1
Vdd = e−rh (π u Vudd + πd Vddd ) = e−(0.03)1 (0.425 557 × 33. 720 7 + 0.574 443 × 63. 625 2) =
49. 394 8

Vu = e−rh (πu Vuu + π d Vud ) = e−(0.03)1 (0.425 557 × 0 + 0.574 443 × 18. 798 1) =
10. 479 3
Vd = e−rh (π u Vud + π d Vdd ) = e−(0.03)1 (0.425 557 × 18. 798 1 + 0.574 443 × 49. 394 8) =
35. 299 1

V = e−rh (πu Vu + πd Vd ) = e−(0.03)1 (0.425 557 × 10. 479 3 + 0.574 443 × 35. 299 1) =
24. 005 8

The put premium after the switch is equal to the call premium before the
switch.
By the way, you can also use the textbook’s spreadsheet "optbasics2" to
calculate the European option premium and verify

• the European call price is $14. 379 9 (which is the European put price
before the switch)
• the European put price is $24. 005 8 (which is the European call price
before the switch)

What a coincidence, you might wonder. Why? This can be explained using
the Black-Scholes option formulas:
The price of a European call option is:δ

C = Se−δT N (d1 ) − Ke−rT N (d2 ) (Textbook 12.1)

The price of a European put option is:

P = Ke−rT N (−d2 ) − Se−δT N (−d1 ) (Textbook 12.3)

µ ¶
S 1 2 Se−δT 1 Se−δT
ln + r−δ+ σ T ln + σ2T ln
K 2 −rT
Ke √ 2 Ke −rT 1 √
d1 = √ = = √ + σ T
σ T σ T σ T 2
(Textbook 12.2a)


d2 = d1 − σ T (Textbook 12.2b)

Before the switch:

www.myactuaryexam.com c
°Yufeng Guo 80
CHAPTER 10. BINOMIAL OPTION PRICING I

C = Se−δT N (d1 ) − Ke−rT N (d2 )


P = Ke−rT N (−d2 ) − Se−δT N (−d1 )

Se−δT
ln −rT 1 √
d1 = Ke
√ + σ T
σ T 2
Se−δT Se−δT
√ ln −rT 1 √ √ ln −rT 1 √
d2 = d1 − σ T = Ke
√ + σ T −σ T = Ke
√ − σ T
σ T 2 σ T 2
⎛ ⎞ ⎛ ⎞
Se−δT Se−δT
⎜ ln Ke−rT 1 √ ⎟ ⎜ ln Ke−rT 1 √ ⎟
→ C = Se−δT N ⎜⎝ σ√T + σ T⎟ ⎠ −Ke−rT N ⎜⎝ √ − σ T⎟ ⎠
2 σ T 2
⎛ ⎞ ⎛ ⎞
Se−δT Se−δT
⎜ ln Ke−rT 1 √ ⎟ ⎜ ln −rT 1 √ ⎟
→ P = Ke−rT N ⎜ − √ + σ T ⎟−Se−δT N ⎜− Ke √ − σ T⎟
⎝ σ T 2 ⎠ ⎝ σ T 2 ⎠

After£ we switch S and K and switch


¤r←r→δ
and δ:
C = Se−δT N (d1 ) − Ke−rT N (d2 ) K←→S = Ke−rT N (d1 ) − Se−δT N (d2 )
£ ¤r←→δ
P = Ke−rT N (−d2 ) − Se−δT N (−d1 ) K←→S = Se−δT N (−d2 )−Ke−rT N (−d1 )
⎡ ⎤r←→δ
Se−δT Ke−rT
⎢ ln Ke−rT 1 √ ⎥ ln
Se −δT 1 √
d1 = ⎢
⎣ σ √T + σ T⎥
⎦ = √ + σ T
2 σ T 2
K←→S
Ke−rT Se−δT
ln −δT 1 √ ln −rT 1 √
→ −d1 = − Se √ − σ T = Ke
√ − σ T
σ T 2 σ T 2
Ke−rT Ke−rT
√ ln −δT 1 √ √ ln −δT 1 √
d2 = d1 − σ T = Se
√ + σ T −σ T = Se
√ − σ T
σ T 2 σ T 2
Ke−rT Ke−rT Se−δT
ln 1 √ − ln √ ln
→ −d2 = − Se √
−δT
+ σ T = Se−δT + 1 σ T =
√ Ke

−rT
+
σ T 2 σ T 2 σ T
1 √
σ T
2
⎛ ⎞ ⎛ ⎞
Ke−rT Ke−rT
ln
⎜ Se−δT 1 √ ⎟ ln
⎜ Se−δT 1 √ ⎟
→ C = Ke−rT N ⎜ ⎝ σ √T + σ T⎟ ⎠ −Se−δT N ⎜ ⎝ √ − σ T⎟ ⎠
2 σ T 2
⎛ ⎞ ⎛ ⎞
Se−δT Se−δT
⎜ ln Ke−rT 1 √ ⎟ ⎜ ln Ke−rT 1 √ ⎟
→ P = Se−δT N ⎜
⎝ σ √T + σ T⎟
⎠ −Ke−rT N ⎜
⎝ √ − σ T⎟

2 σ T 2

www.myactuaryexam.com c
°Yufeng Guo 81
CHAPTER 10. BINOMIAL OPTION PRICING I

You can see that the call premium after the switch is equal the put pre-
mium before the switch; the put premium after the switch is equal the call
premium before the switch. This conclusion applies to both European options
and American options. It’s more complex to prove this is true for American
option. However, we are not going to worry about the proof.

Intuitively, you can think that the by switching S and K and switching r and
δ, we are switching the strike asset and the underlying asset. So after-the-switch
put is like a before-the-switch call; after-the-switch call is like a before-the-switch
put.

By the way, the payoff of the before-switch call is not equal to the payoff of
the after-switch put (however the premiums are the same):

Period 3 European
¡ call ¢payoff before switch Period 3 European
¡ call
¢ payoff after switch
Suuu = 100 1. 377 1283 = 261. 169 8 Suuu = 95 1. 323 133 = 220. 0550
Vuuu = 261. 169 8 − 95 = 166. 169 8 Vuuu = 0
¡ ¢ ¡ ¢
Suud = 100 1. 377 1282 (0.755 784) = 143. 333 0 Suud = 95 1. 323 132 (0.726 149) = 120. 768 7
Vuud = 143. 333 0 − 95 = 48. 3330 Vuud = 0
¡ ¢ ¡ ¢
Sudd = 100 (1. 377 128) 0.755 7842 = 78. 662 9 Sudd = 95 (1. 323 13) 0.726 1492 = 66. 279 3
Vudd = 0 Vudd = 100 − 66. 279 3 = 33. 720 7
¡ ¢ ¡ ¢
Sddd = 100 0.755 7843 = 43. 171 1 Sddd = 95 0.726 1493 = 36. 374 8
Vddd = 0 Vddd = 100 − 36. 374 8 = 63. 625 2

the payoff of the before-switch put is not equal to the payoff of the after-
switch call (however the premiums are the same):
Period 3 European put payoff before switch Period 3 European
¡ call
¢ payoff after switch
Suuu = 1000u3 = 1681. 380 8 Suuu = 95 1. 323 133 = 220. 0550
Vuuu = 0 Vuuu = 220. 0550 − 100 = 120. 055
¡ ¢
Suud = 1000u2 d = 1000u = 1189. 1100 Suud = 95 1. 323 132 (0.726 149) = 120. 768 7
Vuud = 0 Vuud = 120. 768 7 − 100 = 20. 768 7
¡ ¢
Sudd = 1000ud2 = 1000d = 840. 965 Sudd = 95 (1. 323 13) 0.726 1492 = 66. 279 3
Vudd = 1000 − 840. 965 = 159. 035 Vudd = 0
¡ ¢ ¡ ¢
Sddd = 1000d3 = 1000 0.840 965 3 = 594. 749 Sddd = 95 0.726 1493 = 36. 374 8
Vddd = 1000 − 594. 749 = 405. 251 Vddd = 0

Problem 10.20.

www.myactuaryexam.com c
°Yufeng Guo 82
CHAPTER 10. BINOMIAL OPTION PRICING I

For options on√ a stock index, you can



use the standard binomial formula.
u = e(r−δ)h+σ√ h = e(0.05−0.03)1+0.3√ 1 = 1. 377 128
d = e(r−δ)h−σ h = e(0.05−0.03)1−0.3 1 = 0.755 784

e(r−δ)h − d e(0.05−0.03)1 − 0.755 784


πu = = = 0.425 557
u−d 1. 377 128 − 0.755 784

π d = 1 − πu = 1 − 0.425 557 = 0.574 443

a. Calculate the price of the American call option


Period 2 3 ¡ ¢
¡ ¢ Suuu = 100 1. 377 1283 = 261. 169 8
Suu = 100 1. 377 1282 = 189. 648 2 Vuuu = 261. 169 8 − 95 = 166. 169 8
EVuu = 189. 648 2 − 95 = 94. 648 2
Vuu =? ¡ ¢
Suud = 100 1. 377 1282 (0.755 784) = 143. 333 0
Sud = 100 (1. 377 128) (0.755 784) = 104. 081 1 Vuud = 143. 333 0 − 95 = 48. 3330
EVud = 104. 081 1 − 95 = 9. 081 1
Vud =? ¡ ¢
¡ ¢ Sudd = 100 (1. 377 128) 0.755 7842 = 78. 662 9
Sudd = 100 0.755 7842 = 57. 120 9 Vudd = 0
EVdd = 0 ¡ ¢
Vdd =? Sddd = 100 0.755 7843 = 43. 171 1
Vddd = 0

R
Vuu = e−rh (π u Vuuu + π d Vuud ) = e−(0.05)1 (0.425 557 × 166. 169 8 + 0.574 443 × 48. 333) =
93. 676 4 ¡ R ¢
Vuu = max Vuu , EVuu = max (93. 676 4, 94. 648 2) = 94. 648 2
R
Vud = e−rh (π u Vuud + πd Vudd ) = e−(0.05)1 (0.425 557 × 48. 3330 + 0.574 443 × 0) =
19. 565 3 ¡ R ¢
Vud = max Vud , EVud = max (19. 565 3, 9. 081 1) = 19. 565 3
R
Vdd = e−rh ¡(πu Vudd + ¢πd Vddd ) = 0
R
Vdd = max Vdd , EVdd = 0

Period 1 2
Vuu = 94. 648 2
Su = 100 (1. 377 128) = 137. 712 8
EVu = 137. 712 8 − 95 = 42. 712 8
Vu =?
Vud = 19. 565 3
Sd = 100 (0.755 784) = 75. 578 4
EVd = 0
Vd =?
Vdd = 0

www.myactuaryexam.com c
°Yufeng Guo 83
CHAPTER 10. BINOMIAL OPTION PRICING I

VuR = e−rh (πu Vuu + π d Vud ) = e−(0.05)1 (0.425 557 × 94. 648 2 + 0.574 443 × 19. 565 3) =
49. 004 8 ¡ ¢
Vu = max VuR , EVu = 49. 004 8
VdR = e−rh (πu Vud + πd Vdd ) = e−(0.05)1 (0.425 557 × 19. 565 3 + 0.574 443 × 0) =
7. 920 1 ¡ ¢
Vd = max VdR , EVd = 7. 920 1

Period 0 1
Vu = 49. 004 8
S = 100
EV = 100 − 95 = 5
V =?
Vd = 7. 920 1

V R = e−rh (πu Vu + πd Vu ) = e−(0.05)1 (0.425 557 × 49. 004 8 + 0.574 443 × 7. 920 1) =
24. 165 0 ¡ ¢
V = max V R , EV = 24. 165 0

So the American call premium is $24. 165 0.

b. Calculate the American put premium


Period 2 3 ¡ ¢
¡ ¢ Suuu = 100 1. 377 1283 = 261. 169 8
Suu = 100 1. 377 1282 = 189. 648 2 Vuuu = 0
EVuu = 0
Vuu =? ¡ ¢
Suud = 100 1. 377 1282 (0.755 784) = 143. 333 0
Sud = 100 (1. 377 128) (0.755 784) = 104. 081 1 Vuud = 0
EVud = 0
Vud =? ¡ ¢
¡ ¢ Sudd = 100 (1. 377 128) 0.755 7842 = 78. 662 9
Sudd = 100 0.755 7842 = 57. 120 9 Vudd = 95 − 78. 662 9 = 16. 337 1
EVdd = 95 − 57. 120 9 = 37. 879 1 ¡ ¢
Vdd =? Sddd = 100 0.755 7843 = 43. 171 1
Vddd = 95 − 43. 171 1 = 51. 828 9

R
Vuu = e−rh (π u Vuuu + π d Vuud ) = e−(0.05)1 (0.425 557 × 0 + 0.574 443 × 0) =
0 ¡ R ¢
Vuu = max Vuu , EVuu = 0
R
Vud = e−rh (πu Vuud + π d Vudd ) = e−(0.05)1 (0.425 557 × 0 + 0.574 443 × 16. 337 1) =
8. 927 0 ¡ R ¢
Vud = max Vud , EVud = 8. 927 0
R
Vdd = e−rh (π u Vudd + πd Vddd ) = e−(0.05)1 (0.425 557 × 16. 337 1 + 0.574 443 × 51. 828 9) =
34. 934 0 ¡ R ¢
Vud = max Vdd , EVdd = 37. 879 1

www.myactuaryexam.com c
°Yufeng Guo 84
CHAPTER 10. BINOMIAL OPTION PRICING I

Period 1 2
Vuu = 0
Su = 100 (1. 377 128) = 137. 712 8
EVu = 0
Vu =?
Vud = 8. 927 0
Sd = 100 (0.755 784) = 75. 578 4
EVd = 85 − 75. 578 4 = 9. 421 6
Vd =?
Vdd = 37. 879 1

VuR = e−rh (π u Vuu + π d Vud ) = e−(0.05)1 (0.425 557 × 0 + 0.574 443 × 8. 927 0) =
4. 8780 ¡ ¢
Vu = max VuR , EVu = 4. 8780
VdR = e−rh (π u Vud + π d Vdd ) = e−(0.05)1 (0.425 557 × 8. 927 0 + 0.574 443 × 37. 879 1) =
24. 311 8 ¡ ¢
Vd = max VdR , EVd = 24. 311 8

Period 0 1
Vu = 4. 8780
S = 100
EV = 0
V =?
Vd = 24. 311 8

V R = e−rh (π u Vu + π d Vu ) = e−(0.05)1 (0.425 557 × 4. 8780 + 0.574 443 × 24. 311 8) =


15. 259 3 ¡ ¢
V = max V R , EV = 15. 259 3

So the American put premium is $15. 259 3

c. If we switch S and K and switch r and δ and recalculate the option price,
what happens?
After the switch, we have:

• S = 95

• K = 100

• r = 3%

• δ = 5%

√ √
u = e(r−δ)h+σ√ h = e(0.03−0.05)1+0.3√ 1 = 1. 323 13
d = e(r−δ)h−σ h = e(0.03−0.05)1−0.3 1 = 0.726 149

www.myactuaryexam.com c
°Yufeng Guo 85
CHAPTER 10. BINOMIAL OPTION PRICING I

e(r−δ)h − d e(0.03−0.05)1 − 0.726 149


πu = = = 0.425 557
u−d 1. 323 13 − 0.726 149

πd = 1 − π u = 1 − 0.425 557 = 0.574 443

Calculate the price of the American call option


Period 2 3 ¡ ¢
¡ ¢ Suuu = 95 1. 323 133 = 220. 0550
Suu = 95 1. 323 132 = 166. 313 9 Vuuu = 220. 0550 − 100 = 120. 055
EVuu = 166. 313 9 − 100 = 66. 313 9
Vuu =? ¡ ¢
Suud = 95 1. 323 132 (0.726 149) = 120. 768 7
Sud = 95 (1. 323 13) (0.726 149) = 91. 275 0 Vuud = 120. 768 7 − 100 = 20. 768 7
EVud = 0
Vud =? ¡ ¢
¡ ¢ Sudd = 95 (1. 323 13) 0.726 1492 = 66. 279 3
Sudd = 95 0.726 1492 = 50. 092 8 Vudd = 0
EVdd = 0 ¡ ¢
Vdd =? Sddd = 95 0.726 1493 = 36. 374 8
Vddd = 0

R
Vuu = e−rh (π u Vuuu + π d Vuud ) = e−(0.03)1 (0.425 557 × 120. 055 + 0.574 443 × 20. 768 7) =
61. 158 1 ¡ R ¢
Vuu = max Vuu , EVuu = 66. 313 9
R
Vud = e−rh (πu Vuud + π d Vudd ) = e−(0.03)1 (0.425 557 × 20. 768 7 + 0.574 443 × 0) =
8. 577 1 ¡ R ¢
Vud = max Vud , EVud = 8. 577 1
R
Vdd = e−rh ¡(π u Vudd + ¢πd Vddd ) = 0
R
Vdd = max Vdd , EVdd = 0

Period 1 2
Vuu = 66. 313 9
Su = 95 (1. 323 13) = 125. 697 4
EVu = 125. 697 4 − 100 = 25. 697 4
Vu =?
Vud = 8. 577 1
Sd = 95 (0.726 149) = 68. 984 2
EVd = 0
Vd =?
Vdd = 0

VuR = e−rh (πu Vuu + π d Vud ) = e−(0.03)1 (0.425 557 × 66. 313 9 + 0.574 443 × 8. 577 1) =
32. 167 7 ¡ ¢
Vu = max VuR , EVu = 32. 167 7

www.myactuaryexam.com c
°Yufeng Guo 86
CHAPTER 10. BINOMIAL OPTION PRICING I

VdR = e−rh (π u Vud + π d Vdd ) = e−(0.03)1 (0.425 557 × 8. 577 1 + 0.574 443 × 0) =
3. 542 2 ¡ ¢
Vd = max VdR , EVd = 3. 542 2

Period 0 1
Vu = 32. 167 7
S = 95
EV = 0
V =?
Vd = 3. 542 2

V R = e−rh (π u Vu + π d Vu ) = e−(0.03)1 (0.425 557 × 32. 167 7 + 0.574 443 × 3. 542 2) =


15. 2593 ¡ ¢
V = max V R , EV = 15. 2593

So the American call premium is $15. 2593, which is equal to the American
put premium before the switch.

Calculate the American put premium


Period 2 3 ¡ ¢
¡ ¢ Suuu = 95 1. 323 133 = 220. 0550
Suu = 95 1. 323 132 = 166. 313 9 Vuuu = 0
EVuu = 0
Vuu =? ¡ ¢
Suud = 95 1. 323 132 (0.726 149) = 120. 768 7
Sud = 95 (1. 323 13) (0.726 149) = 91. 275 0 Vuud = 0
EVud = 100 − 91. 275 0 = 8. 725
Vud =? ¡ ¢
¡ ¢ Sudd = 95 (1. 323 13) 0.726 1492 = 66. 279 3
Sudd = 95 0.726 1492 = 50. 092 8 Vudd = 100 − 66. 279 3 = 33. 720 7
EVdd = 100 − 50. 092 8 = 49. 907 2 ¡ ¢
Vdd =? Sddd = 95 0.726 1493 = 36. 374 8
Vddd = 100 − 36. 374 8 = 63. 625 2

R
Vuu = e−rh (πu Vuuu + πd Vuud ) = e−(0.03)1 (0.425 557 × 0 + 0.574 443 × 0) =
0 ¡ R ¢
Vuu = max Vuu , EVuu = 0
R
Vud = e−rh (π u Vuud + πd Vudd ) = e−(0.03)1 (0.425 557 × 0 + 0.574 443 × 33. 720 7) =
18. 798 1 ¡ R ¢
Vud = max Vud , EVud = 18. 798 1
R
Vdd = e−rh (πu Vudd + πd Vddd ) = e−(0.03)1 (0.425 557 × 33. 720 7 + 0.574 443 × 63. 625 2) =
49. 394 8 ¡ R ¢
Vud = max Vdd , EVdd = 49. 907 2

www.myactuaryexam.com c
°Yufeng Guo 87
CHAPTER 10. BINOMIAL OPTION PRICING I

Period 1 2
Vuu = 0
Su = 95 (1. 323 13) = 125. 697 4
EVu = 0
Vu =?
Vud = 18. 798 1
Sd = 95 (0.726 149) = 68. 984 2
EVd = 100 − 68. 984 2 = 31. 015 8
Vd =?
Vdd = 49. 907 2

VuR = e−rh (πu Vuu + π d Vud ) = e−(0.03)1 (0.425 557 × 0 + 0.574 443 × 18. 798 1) =
10. 479 3 ¡ ¢
Vu = max VuR , EVu = 10. 479 3
VdR = e−rh (πu Vud + πd Vdd ) = e−(0.03)1 (0.425 557 × 18. 798 1 + 0.574 443 × 49. 907 2) =
35. 584 8 ¡ ¢
Vd = max VdR , EVd = 35. 584 8

Period 0 1
Vu = 10. 479 3
S = 95
EV = 100 − 95 = 5
V =?
Vd = 35. 584 8

V R = e−rh (πu Vu + πd Vu ) = e−(0.03)1 (0.425 557 × 10. 479 3 + 0.574 443 × 35. 584 8) =
24. 165 04 ¡ ¢
V = max V R , EV = 24. 165 0

So the American put premium is $24. 165 0, which is equal to the American
call premium before the switch.

Problem 10.21.
Suppose u < e(r−δ)h . Since d < u, we have d < u < e(r−δ)h . This means
that the savings account is always better than the stock. So at t = 0, we short
sell e−δh share of stock and investment the short sale proceeds Se−δh into the
savings account. Then at time h, we close our short position by buying one
stock from the market. The stock price at time h is either uS or dS.
t=0 t = h, u mode t = h, d mode
short e−δh stock Se−δh −uS −dS
deposit Se−δh in savings −Se−δh Se¡(r−δ)h ¢ Se¡−δh ¢
−δh
Total 0 S e − u > 0 S e−δh − d > 0

So initial cost is zero yet we have positive payoff at time h. This is an


arbitrage opportunity.

www.myactuaryexam.com c
°Yufeng Guo 88
CHAPTER 10. BINOMIAL OPTION PRICING I

Suppose d > e(r−δ)h . Since d < u, we have u > d > e(r−δ)h . This means that
the investing in stocks is always better off than investing in a savings account.
So at t = 0, we buy e−δh share of stock and borrow money from a bank to
finance the purchase.
t=0 t = h, u mode t = h, d mode
buy e−δh stock −Se−δh uS dS
borrow Se−δh in savings Se−δh −Se
¡
(r−δ)h
¢ −Se
¡
−δh
¢
−δh
Total 0 S u−e > 0 S d − e−δh > 0
So initial cost is zero yet we have positive payoff at time h. This is an
arbitrage opportunity.

www.myactuaryexam.com c
°Yufeng Guo 89
CHAPTER 10. BINOMIAL OPTION PRICING I

www.myactuaryexam.com c
°Yufeng Guo 90
Chapter 11

Binomial option pricing II

Problem 11.1.
√ √
u = e(r−δ)h+σ√ h = e(0−0.08)1+0.3√1 = 1. 246 077
d = e(r−δ)h−σ h = e(0−0.08)1−0.3 1 = 0.683 861
e(r−δ)h − d e(0−0.08)1 − 0.683 861
πu = = = 0.425 56
u−d 1. 246 077 − 0.683 861
π d = 1 − πu = 1 − 0.425 56 = 0.574 44

• K = 70
t=0 t=1
Su = 100 (1. 246 077) = 124. 607 7
Vu = 124. 607 7 − 70 = 54. 607 7
S = 100
EV = 100 − 70 = 30
V Sd = 100 (0.683 861 ) = 68. 386 1
Vd = 0

V R = e−rh (π u Vu + π d Vu ) = e−0(1) (0.425 56 × 54. 607 7 + 68. 386 1 × 0) =


23. 24 ¡ ¢
V = max V R , EV = max (23. 24, 30) = 30

• K = 80
t=0 t=1
Su = 100 (1. 246 077) = 124. 607 7
Vu = 124. 607 7 − 80 = 44. 607 7
S = 100
EV = 100 − 80 = 20
V Sd = 100 (0.683 861 ) = 68. 386 1
Vd = 0

91
CHAPTER 11. BINOMIAL OPTION PRICING II

V R = e−rh (π u Vu + π d Vu ) = e−0(1) (0.425 56 × 44. 607 7 + 68. 386 1 × 0) =


18. 98 ¡ ¢
V = max V R , EV = max (18. 98, 20) = 20

• K = 90
t=0 t=1
Su = 100 (1. 246 077) = 124. 607 7
Vu = 124. 607 7 − 90 = 34. 607 7
S = 100
EV = 100 − 90 = 10
V Sd = 100 (0.683 861 ) = 68. 386 1
Vd = 0

V R = e−rh (π u Vu + π d Vu ) = e−0(1) (0.425 56 × 34. 607 7 + 68. 386 1 × 0) =


14. 73 ¡ ¢
V = max V R , EV = max (14. 73, 10) = 14. 73

• K = 100
t=0 t=1
Su = 100 (1. 246 077) = 124. 607 7
Vu = 124. 607 7 − 100 = 24. 607 7
S = 100
EV = 100 − 100 = 0
V Sd = 100 (0.683 861 ) = 68. 386 1
Vd = 0

V R = e−rh (π u Vu + π d Vu ) = e−0(1) (0.425 56 × 24. 607 7 + 68. 386 1 × 0) =


10. 47 ¡ ¢
V = max V R , EV = max (10. 47, 0) = 10. 47

a. Early exercise occurs at t = 0 with K = 70, 80

b. The European options satisfy the put-call parity:


CEur + Ke−rT = PEur + Se−δT
CEur = PEur + Se−δT − Ke−rT = PEur + 100e−0.08 − K = PEur − K + 92.
311 6
Since K = 70, 80, 90, 100 ≤ 100, we have
EV = max (0, S − K) = max (0, 100 − K) = 100 − K
To have EV > CEur , we need to have:
100 − K > PEur − K + 92. 311 6
→ PEur < 100 − 100e−0.08 = 7. 69
To early exercise the American call at t = 0, the European put premium
needs to be less than 7. 69.

www.myactuaryexam.com c
°Yufeng Guo 92
CHAPTER 11. BINOMIAL OPTION PRICING II

Clearly, the smaller the European put premium is, the more likely the early
exercise of the American call is optimal at t = 0.

Everything else equal, the higher the strike price, the higher the price of a
European put. As a result, it can be optimal to early exercise the American
call with K = 70, 80, while it’s not optimal to early exercise the American with
K = 90, 100.
Use the European put premium using the Black-Scholes option formula (the
textbook Equation 12.3):

P = Ke−rT N (−d2 ) − Se−δT N (−d1 )

We find the following put price (S = 100, T = 1, r = 0, δ = 0.08, σ = 0.3):


K PEur
70 $2.3087
80 $5.2537
90 $9.7517
100 $15.7113

Clearly, the condition PEur < 7. 69 is met when K = 70, 80 and violated
when K = 90, 100.

By the way, the price calculated using the Black-Scholes option formula won’t
match the price calculated under the binomial option formula under h = 1.
This is because the Black-Scholes option formula is the binomial option pricing
method where n → ∞ and h = T /n → 0 .
c.To early exercise the American call, we need to have PEur < 7. 69. This
condition is met if K = 70, 80 and violated when K = 90, 100.

Problem 11.2.
Now r = 0.08 instead of r = 0. This increases the cost of early exercising
an American call. By early exercising an American call, you lost interest on
the strike asset K. We expect that it’s still not optimal to early exercise the
American call at K = 90, 100 (it’s not optimal to early exercise the American
call at these strike prices even when r = 0, let alone when r = 0.08). However,
we are not clear whether it’s optimal to early exercise the American call when
K = 70, 80. We have to check.

√ √
u = e(r−δ)h+σ√ h = e(0.08−0.08)1+0.3√ 1 = 1. 349 859
d = e(r−δ)h−σ h = e(0.08−0.08)1−0.3 1 = 0.740 818
e(r−δ)h − d e(0.08−0.08)1 − 0.740 818
πu = = = 0.425 558
u−d 1. 349 859 − 0.740 818
π d = 1 − πu = 1 − 0.425 558 = 0.574 442

www.myactuaryexam.com c
°Yufeng Guo 93
CHAPTER 11. BINOMIAL OPTION PRICING II

• K = 70

t=0 t=1
Su = 100 (1. 349 859) = 134. 985 9
Vu = 134. 985 9 − 70 = 64. 985 9
S = 100
EV = 100 − 70 = 30
V Sd = 100 (0.740 818 ) = 74. 081 8
Vd = 74. 081 8 − 70 = 4. 081 8

V R = e−rh (πu Vu + πd Vu ) = e−0.08(1) (0.425 558 × 64. 985 9 + 0.574 442 × 4. 081 8) =
27. 69 ¡ ¢
V = max V R , EV = max (27. 69, 25) = 27. 69

• K = 80

t=0 t=1
Su = 100 (1. 349 859) = 134. 985 9
Vu = 134. 985 9 − 80 = 54. 985 9
S = 100
EV = 100 − 80 = 20
V Sd = 100 (0.740 818 ) = 74. 081 8
Vd = 0

V R = e−rh (πu Vu + πd Vu ) = e−0.08(1) (0.425 558 × 54. 985 9 + 0.574 442 × 0) =


21. 60 ¡ ¢
V = max V R , EV = max (21. 60, 20) = 21. 60

• K = 90

t=0 t=1
Su = 100 (1. 349 859) = 134. 985 9
Vu = 134. 985 9 − 90 = 44. 985 9
S = 100
EV = 100 − 90 = 10
V Sd = 100 (0.740 818 ) = 74. 081 8
Vd = 0

V R = e−rh (πu Vu + πd Vu ) = e−0.08(1) (0.425 558 × 44. 985 9 + 0.574 442 × 0) =


17. 67 ¡ ¢
V = max V R , EV = max (17. 67, 10) = 17. 67

• K = 100

www.myactuaryexam.com c
°Yufeng Guo 94
CHAPTER 11. BINOMIAL OPTION PRICING II

t=0 t=1
Su = 100 (1. 349 859) = 134. 985 9
Vu = 134. 985 9 − 100 = 34. 985 9
S = 100
EV = 0
V Sd = 100 (0.740 818 ) = 74. 081 8
Vd = 0

V R = e−rh (π u Vu + π d Vu ) = e−0.08(1) (0.425 558 × 34. 985 9 + 0.574 442 × 0) =


13. 74 ¡ ¢
V = max V R , EV = max (13. 74, 0) = 13. 74

a. Early exercise occurs at t = 0 with K = 70. Now the risk free rate is no
longer zero and we’ll lose interest on the strike price by exercising the American
call early. The higher the strike price, the more interest we lose. In contrast,
the previous problem has r = 0 and we don’t lose any interest by early exercise.
It makes sense that if r is not zero (i.e. positive) then fewer strike prices will
lead to optimal exercise than if r = 0.

b, c. The European options satisfy the put-call parity:


CEur + Ke−rT = PEur + Se−δT
CEur = PEur + Se−δT − Ke−rT = PEur + 100e−0.08 − Ke−0.08
Since K = 70, 80, 90, 100 ≤ 100, we have
EV = max (0, S − K) = max (0, 100 − K) = 100 − K
To have EV > CEur , we need to have:
100 − K > PEur + 100e−0.08 − Ke ¡
−0.08
¢
−0.08
→ PEur < 100 − 100e − K 1 − e−0.08 = 7. 69 − 0.0769K

Using the Black-Scholes option pricing formula, we find the following put
price (S = 100, T = 1, r = 0.08, δ = 0, σ = 0.3):

K PEur 7. 69 − 0.0769K
70 0.7752 7. 69 − 0.0769 (70) = 2. 307
80 2.0904 7. 69 − 0.0769 (80) = 1. 538
90 4.4524 7. 69 − 0.0769 (90) = 0.769
100 8.0229 7. 69 − 0.0769 (100) = 0

Among the 4 strike prices, only K = 70 satisfies the condition PEur < 7.
69 − 0.0769K. Hence of the 4 strike prices given, only K = 70 leads to optimal
early exercise.

Problem 11.3.
If δ = 0, then the stock doesn’t pay any dividend. It’s never optimal to early
exercise an American call on a non-dividend paying stock. So early exercise will
never occur.

www.myactuaryexam.com c
°Yufeng Guo 95
CHAPTER 11. BINOMIAL OPTION PRICING II

Problem 11.4.
√ √
u = e(r−δ)h+σ√ h = e(0.08−0)1+0.3√1 = 1. 462 285
d = e(r−δ)h−σ h = e(0.08−0)1−0.3 1 = 0.802 519
e(r−δ)h − d e(0.08−0)1 − 0.802 519
πu = = = 0.425 557
u−d 1. 462 285 − 0.802 519
πd = 1 − π u = 1 − 0.425 557 = 0.574 443

• K = 100
t=0 t=1
Su = 100 (1. 462 285) = 146. 228 5
Vu = 0
S = 100
EV = 0
V Sd = 100 (0.802 519 ) = 80. 251 9
Vd = 100 − 80. 251 9 = 19. 748 1

V R = e−rh (πu Vu + πd Vu ) = e−0.08(1) (0.425 557 × 0 + 0.574 443 × 19. 748 1) =


10. 4720 ¡ ¢
V = max V R , EV = max (10. 4720, 0) = 10. 4720

• K = 110
t=0 t=1
Su = 100 (1. 462 285) = 146. 228 5
Vu = 0
S = 100
EV = 110 − 100 = 10
V Sd = 100 (0.802 519 ) = 80. 251 9
Vd = 110 − 80. 251 9 = 29. 748 1

V R = e−rh (πu Vu + πd Vu ) = e−0.08(1) (0.425 557 × 0 + 0.574 443 × 29. 748 1) =


15. 774 8 ¡ ¢
V = max V R , EV = max (15. 774 8, 10) = 15. 774 8

• K = 120
t=0 t=1
Su = 100 (1. 462 285) = 146. 228 5
Vu = 0
S = 100
EV = 120 − 100 = 20
V Sd = 100 (0.802 519 ) = 80. 251 9
Vd = 120 − 80. 251 9 = 39. 748 1

www.myactuaryexam.com c
°Yufeng Guo 96
CHAPTER 11. BINOMIAL OPTION PRICING II

V R = e−rh (π u Vu + π d Vu ) = e−0.08(1) (0.425 557 × 0 + 0.574 443 × 39. 748 1) =


21. 077 5 ¡ ¢
V = max V R , EV = max (21. 077 5, 20) = 21. 077 5

• K = 130
t=0 t=1
Su = 100 (1. 462 285) = 146. 228 5
Vu = 0
S = 100
EV = 130 − 100 = 30
V Sd = 100 (0.802 519 ) = 80. 251 9
Vd = 130 − 80. 251 9 = 49. 748 1

V R = e−rh (π u Vu + π d Vu ) = e−0.08(1) (0.425 557 × 0 + 0.574 443 × 49. 748 1) =


26. 380 3 ¡ ¢
V = max V R , EV = max (26. 380 3, 30) = 30

a. Early exercise occurs at t = 0 with K = 130.

b, c. The European options satisfy the put-call parity:


CEur + Ke−rT = PEur + Se−δT
PEur = CEur + Ke−rT − Se−δT = CEur + Ke−0.08(1) − 100e−0(1) = CEur +
−0.08
Ke − 100

The early exercise value is


EV = max (0, K − S)
Since K = 100, 100, 120, 130 ≥ 100, we have
EV = max (0, K − S) = K − S = K − 100
To have EV > CEur , we need to have:
−0.08
K − 100 > CEur
¡ + Ke ¢ − 100
−0.08
→ CEur < K 1 − e = 0.07 69K

Using the Black-Scholes option pricing formula, we find the following put
price (S = 100, T = 1, r = 0.08, δ = 0, σ = 0.3):

K CEur 0.07 69K


100 15.7113 0.07 69 (100) = 7. 69
110 11.2596 0.07 69 (110) = 8. 459
120 7.8966 0.07 69 (120) = 9. 228
130 5.4394 0.07 69 (130) = 9. 997

Among the 4 strike prices, only K = 130 satisfies the condition CEur < 0.07
69K. Hence of the 4 strike prices given, only K = 130 leads to optimal early
exercise.

www.myactuaryexam.com c
°Yufeng Guo 97
CHAPTER 11. BINOMIAL OPTION PRICING II

The main reason for early exercising an American put is to earn interest on
the strike price. Hence higher the strike price, everything else equal, the more
likely an American put may be early exercised.

Problem 11.5.

Now δ = 0.08 as opposed to δ = 0 in the previous problem. This increases


the cost of early exercising an American put. If you early exercise an American
put, you can earn interest on the strike asset, but you also lose the opportunity
of earning dividend on the stock.
With δ = 0.08, we expect that it’s still not optimal to early exercise the
American put when K = 100, 110, 120. Because it’s not optimal to early exercise
the American put with these strike prices even when δ = 0, let alone δ = 0.08.
However, we are not clear whether it’s optimal to early exercise the American
put when K = 130. We have to check.
The put-call parity is:
CEur + Ke−rT = PEur + Se−δT
CEur + Ke−0.08 = PEur + 100e−0.08
→ PEur = CEur + Ke−0.08 − 100e−0.08
EV = max (0, K − S) = K − S = K − 100
Early exercise if:
K − 100 > CEur + Ke−0.08 − 100e−0.08
→ CEur < K − Ke−0.08 − 100 + 100e−0.08 = 0.07 69K − 7. 69

Using the Black-Scholes option pricing formula, we find the following put
price (S = 100, T = 1, r = 0.08, δ = 0.08, σ = 0.3):

K CEur 0.07 69K 0.07 69K − 7. 69


100 15.6584 0.07 69 (100) − 7. 69 = 0
110 11.2165 0.07 69 (110) − 7. 69 = 0.769
120 7.8627 0.07 69 (120) − 7. 69 = 1. 538
130 5.4136 0.07 69 (130) − 7. 69 = 2. 307
The condition CEur < 0.07 69K − 7. 69 is met only when K = 130. So it’s
optimal to early exercise the American put only when K = 130.

Problem 11.6.

The only reason to early exercise an American put is to earn interest on the
strike asset. If r = 0 we’ll never earn any interest on the strike price. So it’s
never optimal to early exercise an American put if r = 0.
We can also use the put-call parity to verify that it’s never optimal to early
exercise the American put when r = 0.
CEur + Ke−rT = PEur + Se−δT
CEur + K = PEur + 100e−0.08

www.myactuaryexam.com c
°Yufeng Guo 98
CHAPTER 11. BINOMIAL OPTION PRICING II

→ PEur = CEur + K − 100e−0.08


EV = max (0, K − S) = K − S = K − 100
Early exercise if:
K − 100 > CEur + K − 100e−0.08
→ CEur < −100 + 100e−0.08 = −7. 688
Since CEur ≥ 0, it’s never optimal to early exercise the American put.

Problem 11.7.
Skip. This is a spreadsheet problem. Most likely SOA and CAS won’t ask
you to build a 10 period binomial model in the exam.

Problem 11.8.
Skip. This is a spreadsheet problem. Most likely SOA and CAS won’t ask
you to build a 10 period binomial model in the exam.

Problem 11.9.
Skip. This is a spreadsheet problem. Most likely SOA and CAS won’t ask
you to build a 10 period binomial model in the exam.

Problem 11.10.
Skip. This is a spreadsheet problem. Most likely SOA and CAS won’t ask
you to build a 10 period binomial model in the exam.

Problem 11.11.
Skip. This is a spreadsheet problem. Most likely SOA and CAS won’t ask
you to build a 10 period binomial model in the exam.

Problem 11.12.
Skip. This is a spreadsheet problem. Most likely SOA and CAS won’t ask
you to build a 10 period binomial model in the exam.

Problem 11.13.
Skip. This is a spreadsheet problem. Most likely SOA and CAS won’t ask
you to build a 50 period binomial model in the exam.

Problem 11.14.

www.myactuaryexam.com c
°Yufeng Guo 99
CHAPTER 11. BINOMIAL OPTION PRICING II

The textbook Section 10 Equation 10.7 says that the undiscounted risk-
neutral stock price is the forward price:
time t time t + h
Su
S
Sd

πu Su + πd Sd = Se(r−δ)h = Ft,t+h (Textbook Equation 10.7)

We are reusing the data in the textbook Figure 11.4 except that we set
S = 100. We have:

S = 100 σ = 0.3 T =1 h = T /n = 1/3


r = 0.08 δ=0

Then: √ √
u = e(r−δ)h+σ√ h = e(0.08−0)1/3+0.3√1/3 = 1. 221 246
d = e(r−δ)h−σ h = e(0.08−0)1/3−0.3 1/3 = 0.863 693
e(r−δ)h − d e(0.08−0)1/3 − 0.863 693
πu = = = 0.456 806
u−d 1. 221 246 − 0.863 693
πd = 1 − π u = 1 − 0.456 806 = 0.543 194

The stock price tree is:


Period 0 1
S u = 100 (1. 221 246) = 122. 124 6
S = 100
S d = 100 (0.863 693) = 86. 369 3

Period 2 3 ¡ ¢
¡ ¢ S uuu = 100 1. 221 2462 = 182. 141 7
S uu = 100 1. 221 2462 = 149. 144 2 ¡ ¢
S uud = 100 1. 221 2462 (0.863 693) = 128. 814 8
S ud = 100 (1. 221 246) (0.863 693) = 105. 478 2 ¡ ¢
¡ ¢ S udd = 100 (1. 221 246) 0.863 6932 = 91. 100 8
S dd = 100 0.863 6932 = 74. 596 6 ¡ ¢
S ddd = 100 0.863 6933 = 64. 428 5

a.The forward price is calculated using the textbook Equation 5.7:


F0,T = S0 e(r−δ)T

The 4-month forward price is:


F0,1/3 = 100e(0.08−0)1/3 = 102. 702 5

The 8-month forward price is:


F0,2/3 = 100e(0.08−0)2/3 = 105. 478 1

www.myactuaryexam.com c
°Yufeng Guo 100
CHAPTER 11. BINOMIAL OPTION PRICING II

The 1-year forward price is:


F0,1 = 100e(0.08−0)1 = 108. 328 7

b. √ √
u = e(r−δ)h+σ√ h = e(0.08−0)1/3+0.3√1/3 = 1. 221 246
d = e(r−δ)h−σ h = e(0.08−0)1/3−0.3 1/3 = 0.863 693
e(r−δ)h − d e(0.08−0)1/3 − 0.863 693
πu = = = 0.456 806
u−d 1. 221 246 − 0.863 693
π d = 1 − πu = 1 − 0.456 806 = 0.543 194

t = 1/3 Stock Price Risk neutral prob


S u = 100 (1. 221 246) = 122. 124 6 πu
S d = 100 (0.863 693) = 86. 369 3 πd
Total 1
The undiscounted risk-neutral expected stock price at t = 1/3 is:
π u Su + π d Sd = 0.456 806 (122. 124 6) + 0.543 194 (86. 369 3) = 102. 702 53 =
F0,1/3

t = 2/3 Stock ¡ Price ¢ Risk neutral prob


S uu = 100 1. 221 2462 = 149. 144 2 π 2u
ud
S = 100 ¡(1. 221 246)¢(0.863 693) = 105. 478 2 2π u πd
S dd = 100 0.863 6932 = 74. 596 6 π 2d
Total 1
The undiscounted risk-neutral expected stock price at t = 2/3 is:
π 2u Suu +π 2d Sdd +2π u πd S ud = 0.456 8062 (149. 144 2)+2 (0.456 806) (0.543 194) (105. 478 2)+
0.543 1942 (74. 596 6) = 105. 478 1 = F0,2/3

t = 1 Stock ¡Price ¢ Risk neutral prob


S uuu = 100 ¡ 1. 221 2462¢ = 182. 141 7 π 3u
S uud = 100 1. 221 2462¡ (0.863 693) ¢ = 128. 814 8 3π 2u πd
S udd = 100 (1.
¡ 221 246) ¢0.863 6932
= 91. 100 8 3π u π2d
S ddd = 100 0.863 6933 = 64. 428 5 π 3d
Total 1

The undiscounted risk-neutral expected stock price at t = 2/3 is:


π 3u Suuu + 3π 2u πd S uud + 3π u¡π2d S udd + π¢3d S ddd ¡ ¢
3
= 0.456
¡ 806 (182. ¢ 141 7)+3 0.456 8062 (0.543 194) 128. 814 8+3 (0.456 806) 0.543 1942 91.
100 8 + 0.543 1943 64. 428 5 = 108. 328 7 = F0,1

Problem 11.15.
Skip. This is a spreadsheet problem. Most likely SOA and CAS won’t ask
you to build a 50 period binomial model in the exam.

Problem 11.16.

www.myactuaryexam.com c
°Yufeng Guo 101
CHAPTER 11. BINOMIAL OPTION PRICING II

Skip. This is a spreadsheet problem. Most likely SOA and CAS won’t ask
you to build a 8 period binomial model in the exam.

Problem 11.17.
Skip. This is a spreadsheet problem. Most likely SOA and CAS won’t ask
you to build a 8 period binomial model in the exam.

Problem 11.18.
Skip. If you understand the volatility calculation examples in my study
guide, you are fine.

Problem 11.19.
Skip. If you understand the volatility calculation examples in my study
guide, you are fine.

Problem 11.20.
This is a labor intensive problem. However, it’s a good practice problem
for using the Schroder method. The solution is similar to the textbook Figure
11.11.

First, we’ll build a prepaid forward price tree.

St
σF = σS × P
Ft,T
P
Ft,T = St − P Vt (D) = 50 − 4e−0.08(0.25) = 46. 079 21
St 50
σ F = σ S × P = 0.3 × = 0.325 5265
Ft,T 46. 079 2
√ √
u = erh+σF √ h = e0.08(0.25)+0.325 5265 √0.25 = 1. 200 53
d = erh−σF h = e0.08(0.25)−0.325 5265 0.25 = 0.866 959

Prepaid forward price tree:

Time 0 0.25 0.5 0.75 1


95.71879
79.73044
66.41269 69.12305
55.31947 57.57710
46.07921 47.95973 49.91701
39.94880 41.57914
34.63398 36.04742
30.02625
26.03154

www.myactuaryexam.com c
°Yufeng Guo 102
CHAPTER 11. BINOMIAL OPTION PRICING II

For
³ example,
´u the prepaid forward price at t = 0.25 is calculated as follows:
P P
Ft+h,T = Ft,T u = 46. 079 21 × 1. 200 53 = 55. 319 47
³ ´d
P P
Ft+h,T = Ft,T d = 46. 079 21 × 0.866 959 = 39. 948 8

The prepaid
³ ´uu forward price at t = 0.5 is calculated as follows:
P P
Ft+2h,T = Ft,T u2 = 46. 079 21 × 1. 200 532 = 66. 412 7
³ ´ud
P P
Ft+2h,T = Ft,T ud = 46. 079 21 × 1. 200 53 × 0.866 959 = 47. 959 73
³ ´dd
P P 2
Ft+2h,T = Ft,T d = 46. 079 21 × 0.866 9592 = 34. 633 98
(My numbers are calculated using Excel so you might not be able to fully
match mine.)
Next, convert the prepaid forward price tree into a stock price tree. The
one-to-one mapping between the prepaid forward
½ price and the stock price is
P P De−r(TD −t−∆t) if TD ≥ t + ∆t
St+∆t = Ft+∆t,T +P V (Div) = Ft+∆t,T +
0 if TD < t + ∆t

The PV of the dividend at each interval is:


Time 0 0.25 0.5 0.75 1
Dividend time 0.25
Dividend amount $4
P V (Div) 3. 920 79 4 0 0 0
P Vt=0 (Div) = 4e−0.08(0.25−0) = 3. 920 79
We discount $4 from t = 0.25 to time zero

P Vt=0.25 (Div) = 4e−0.08(0.25−0.25) = 4


We discount $4 from t = 0.25 to t = 0.25

For t > 0.25, we have P Vt (Div) = 0


Next, we add the P V (Div) to the prepaid forward price:
Time 0 0.25 0.5 0.75 1
95.71879
79.73044
66.41269 69.12305
55.31947 + 4 57.57710
46.07921 + 3. 920 79 47.95973 49.91701
39.94880 + 4 41.57914
34.63398 36.04742
30.02625
26.03154

Now the stock price tree is:

www.myactuaryexam.com c
°Yufeng Guo 103
CHAPTER 11. BINOMIAL OPTION PRICING II

Time 0 0.25 0.5 0.75 1


95.718794
79.730440
66.412694 69.123048
59.319474 57.577104
50 47.959733 49.917007
43.948797 41.579138
34.633981 36.047421
30.026253
26.031540

The risk neutral probabilities are:


erh − d e0.08(0.25) − 0.866 959
πu = = = 0.459 399
u−d 1. 200 53 − 0.866 959
πd = 1 − 0.459 399 = 0.540 601

From this point on, we can just use the standard binomial tree formula.
First, we calculate the European call premium. We start from right to left,
calculating the roll-back value.
Time 0 0.25 0.5 0.75 1
50.71879
35.62150
23.17717 24.12305
14.27212 13.46816
8.43381 7.23801 4.91701
3.78761 2.21414
0.99703 0
0
0

In the above table, the final column is the call payoff. For example,
50.71879 = max (0, 95.718794 − 45) = 50. 718 794

The first 4 columns are the roll-back values. For example,


35.62150 = e−0.08(0.25) (50.71879 × 0.459 399 + 24.12305 × 0.540 601) = 35.
621 504
8.43381 = e−0.08(0.25) (14.27212 × 0.459 399 + 3.78761 × 0.540 601) = 8. 433 809
The European call premium is 8.43381.

To calculate the American call premium, we still work from right to left.
However, we’ll need to compare the roll back value and the exercise value and
take the greater of the two.

The American call premium is calculated as follows:

www.myactuaryexam.com c
°Yufeng Guo 104
CHAPTER 11. BINOMIAL OPTION PRICING II

Time 0 0.25 0.5 0.75 1


50.71879
35.62150
23.17717 24.12305
max (14.27212, 14.31947) 13.46816
8.45513 7.23801 4.91701
3.78761 2.21414
0.99703 0
0
0

It’s optimal to exercise the American call at the upper node at t = 0.25. The
roll back value is 14.27212. The exercise value is 59.319474 − 45 = 14. 319 474,
which is greater than the roll back value. The premium of 8.45513 is calculated
as follows:
The roll back value is: e−0.08(0.25) (14. 319 474 × 0.459 399 + 3.78761 × 0.540 601) =
8. 455 132 8
The early exercise value at t = 0 is 50 − 45 = 5.
We take the greater of the two. So the American call premium is 8.45513.

By the way, if you bother to calculate the European put and the American
put premium with strike price K = 45, here are the results:
The prepaid forward price tree and the stock price tree won’t change whether
the option is a call or put.

The European put premium:


Time 0 0.25 0.5 0.75 1
0
0
0 0
1.33205 0
3.89484 2.51380 0
6.21822 4.74394
9.59857 8.95258
14.08269
18.96846

So the European put premium is 3.89484.


Check the put-call parity: C + P V (K) = P + S − P V (Div)
C + P V (K) = 8.43381 + 45e−0.08(1) = 49. 974 05
P + S − P V (Div) = 3.89484 + 50 − 3. 920 79 = 49. 974 05
So C + P V (K) = P + S − P V (Div) holds.

The American put premium:

www.myactuaryexam.com c
°Yufeng Guo 105
CHAPTER 11. BINOMIAL OPTION PRICING II

Time 0 0.25 0.5 0.75 1


0
0
0 0
1.33205 0
4.11033 2.51380 0
6.62489 4.74394
10.36602 8.95258
14.97375
18.96846

The two bold numbers indicate that early exercise is optimal.


The American put premium is 4.11033.

www.myactuaryexam.com c
°Yufeng Guo 106
Chapter 12

Black-Scholes formula

Problem 12.1.
Skip this spreadsheet problem.

Problem 12.2.
Skip this problem but remember the following key point. As n gets bigger,
the price calculated using the discrete binomial tree method approach the price
calculated using the Black-Scholes formula.

Problem 12.3.
a. r = 8% δ=0
T European call price
1 7.8966
10 56.2377
100 99.9631
1, 000 100
1, 0000 100
1, 0000 100

As T → ∞, the European call premium approaches the current stock price.

b. r = 8% δ = 0.1%
T European call price
1 7.8542
10 55.3733
100 90.4471
1, 000 36.7879
1, 0000 0.0045
10, 0000 0

107
CHAPTER 12. BLACK-SCHOLES FORMULA

Dividend reduces the value of the stock. Over a long period of time, the
value of the underlying stock is reduced to zero; the value of the call is reduced
to zero.

Problem 12.4.

a. r = 0% δ = 8%
T European call price
1 18.6705
10 10.1571
100 0.0034
1, 000 0.0000

Dividend reduces the value of the stock. Over a long period of time, the
value of the underlying stock is reduced to zero; the value of the call is reduced
to zero.

b. r = 0.1% δ = 8%
T European call price
1 18.7281
10 10.2878
100 0.0036
1, 000 0.0000

Dividend reduces the value of the stock, but you can earn interest on the
strike asset with r = 0.1% (vs. r = 0 in the previous problem).
Consequently, the call is more valuable when r = 0.1% than when r = 0.
However, even with r = 0.1%, over a long period of time, the value of the
underlying stock is reduced to zero due to the dividend paid out; the call value
approaches zero.

Problem 12.5.

a. We need to find the 90-strike yen-denominated euro put.

• The underlying asset is 1 euro.

• The strike asset is expressed in yen.

• The current price of the underlying is 95 yen. S = 95

• The strike asset is 90 yen. K = 90

• The strike asset 90 yen earns 1.5%. So the risk free rate is r = 1.5%
(always remember that r is the earning rate of the strike asset)

www.myactuaryexam.com c
°Yufeng Guo 108
CHAPTER 12. BLACK-SCHOLES FORMULA

• The underlying (i.e. 1 euro) earns 3.5%. So the dividend rate is δ = 3.5%
(always remember that δ is the earning rate of the underlying asset)
• T = 0.5
µ ¶ µ ¶
S 1 95 1
ln + r − δ + σ2 T ln + 0.015 − 0.035 + × 0.12 0.5
K 2 90 2
d1 = √ = √ =
σ T 0.1 0.5
0.658 560

N (d1 ) = NormalDist (0.658 560) = 0.744 910 8


N (−d1 ) = 1√− N (d1 ) = 1 − 0.744√910 8 = 0.255 089
d2 = d1 − σ T = 0.658 560 − 0.1 0.5 = 0.587 849
N (d2 ) = 0.721 683
N (−d2 ) = 1 − N (d2 ) = 1 − 0.721 683 = 0.278 317

P = −Se−δT N (−d1 ) + Ke−rT N (−d2 )


= −95e−0.035(0.5) 0.255 089 + 90e−0.015(0.5) 0.278 317 = 1. 048 3 (yen)
1
b. Now we need to find the price of a strike euro-denominated yen call
90
with 6 months to expiration.
• The underlying asset is 1 yen. The earning rate of the underlying asset is
1.5%. So δ = 1.5%
1 1
• The current price of the underlying asset is euro. So S =
95 95
1 1
• The strike asset is euro. So K = . The earning rate of the strike
90 90
asset is 3.5%. So r = 3.5%
• Volatility is still 10%
C = Se−δT N (d1 ) − Ke−rT N (d2 )
µ ¶ µ ¶
S 1 2 1/95 1 2
ln + r−δ+ σ T ln + 0.035 − 0.015 + × 0.1 0.5
K 2 1/90 2
d1 = √ = √ =
σ T 0.1 0.5
−0.587 849
N (d1 ) = 0.278 316 8
√ √
d2 = d1 − σ T = −0.587 849 − 0.1 0.5 = −0.658 559 7
N (d2 ) = 0.255 089
1 −0.015(0.5) 1
C = Se−δT N (d1 )−Ke−rT N (d2 ) = e 0.278 316 8− e−0.035(0.5)
95 90
0.255 089 = 0.0001 226 1 (euro)

c. A 90-strike yen-denominated euro put is worth 1. 048 3 (yen). This put is


"give 1 euro and get 90 yen."

www.myactuaryexam.com c
°Yufeng Guo 109
CHAPTER 12. BLACK-SCHOLES FORMULA

• The privilege of "give 1 euro and get 90 yen" at T = 0.5 is worth 1. 048 3
(yen) at t = 0 (statement a)

• This can be expressed at (1€ → 90Y ) = 1. 048 3Y

1
A strike euro-denominated yen call is worth 0.0001 226 1 (euro). This
90
1
call is "give euro and get 1 yen."
90

1
• The privilege of "give euro and get 1 yen" at T = 0.5 is worth 0.0001
90
226 1 (euro) at t = 0 (statement b)
µ ¶
1
• This can be expressed at € → 1Y = 0.0001 226 1€
90

Statement a and b are essentially the same statement. Using Statement a,


we can derive Statement b; using Statement b, we can derive Statement a.
First, we derive Statement b from Statement a.
(1€ → 90Yµ ) = 1. 048¶3Y (statement a)
1 1 1. 048 3
⇒ € → 1Y = (1€ → 90Y ) = Y
90 90 90
The above equation means this: If you can sell 1€ for a guaranteed price
1
90Y , then you must be able to sell € for a guaranteed price of 1Y . This
90
should make intuitive sense.
1. 048 3 1
We convert Y into euros. Since at t = 0, 1€=95Y or 1Y = €,
90 95
then
1. 048 3 1. 048 3 1
Y = × €= 0.0001226 1€
90 µ 90 ¶ 95
1 1 1. 048 3
⇒ € → 1Y = (1€ → 90Y ) = Y = 0.0001226 1€
90 90 90
This is exactly Statement b

Next,
µ we derive
¶ Statement b from Statement a.
1
€ → 1Y = 0.0001 226 1€ (statement b)
90 µ ¶
1
⇒ (1€ → 90Y ) = 90 € → 1Y = 90 (0.0001 226 1€) = 90 (0.0001 226 1) 95Y =
90
1. 048 3Y
This is exactly Statement a.

Problem 12.6.

www.myactuaryexam.com c
°Yufeng Guo 110
CHAPTER 12. BLACK-SCHOLES FORMULA

a.
C = Se−δT N (d1 ) − Ke−rT N (d2 )
µ ¶ µ ¶
S 1 100 1
ln + r − δ + σ2 T ln + 0.06 − 0 + × 0.42 1
K 2 105 2
d1 = √ = √ = 0.228 024 589 ≈
σ T 0.4 1
0.228 025
N (d1 ) = 0.590 186 6
√ √
d2 = d1 − σ T = 0.228 025 − 0.4 1 = −0.171 975
N (d2 ) = 0.431 729
C = Se−δT N (d1 ) − Ke−rT N (d2 ) = 100e−0(1) 0.590 186 6 − 105e−0.06(1)
0.431 729 = 16. 327 019

b.
F0,T = Se(r−δ)T = 100e(0.06−0)1 = 106. 183 65

c
According to the textbook Equation 12.7, if the underlying asset is futures
instead of stocks, we can use the general Black-Scholes formula except

• set F = S (replace the stock price with futures price)


• set δ = r
µ ¶
S 1 F 1 2 100e(0.06−0)1 1
ln + r − δ + σ2 T ln + σ T ln + × 0.42 (1)
K 2 K √2 105
d1 = √ = = √ 2 =
µ σ T ¶ σ T 0.4 1
100 1
ln + 0.06 − 0 + × 0.42 1
105 2
√ = 0.228 024 589 ≈ 0.228 025
0.4 1
N (d1 ) = 0.590 186 6
√ √
d2 = d1 − σ T = 0.228 025 − 0.4 1 = −0.171 975
N (d2 ) = 0.431 729
C = Se−rT N (d1 ) − Ke−rT N (d2 ) = 100e−0(1) 0.590 186 6 − 105e−0.06(1)
0.431 729 = 16. 327 019

The call premium in c is equal to the call premium in a. Why? We can


prove this mathematically. Replace S with F = Se(r−δ)T and set δ = r , we
get: µ ¶ µ ¶
Se(r−δ)T 1 S 1
ln + r − r + σ2 T ln + r − δ + σ2 T
K 2 K 2
d1 = √ = √
√ σ T σ T
d2 = d1 − σ T
£ ¤
C = Se(r−δ)T e−rT N (d1 ) − Ke−rT N (d2 ) = Se−δT N (d1 ) − Ke−rT N (d2 )

www.myactuaryexam.com c
°Yufeng Guo 111
CHAPTER 12. BLACK-SCHOLES FORMULA

This is the
£ same call
¤ formula when ¡the underlying
¢ asset is a stock.
P = − Se(r−δ)T e−rT N (−d1 ) + Ke−rT N (−d2 ) = −Se−δT N (−d1 ) +
Ke−rT N (−d2 )
This is the same put formula when the underlying asset is a stock.

We can also prove that the futures option premium is equal to the underlying
stock option premium intuitively. On the maturity date T , the futures price is
equal to the stock price. So if we stand at T , the payoff of a futures option and
the payoff of an otherwise identical stock option are identical. Consequently, the
premium of a futures option is equal to the premium of an otherwise identical
stock option.

Problem 12.7.

a. µ ¶ µ ¶
S 1 100 1
ln + r − δ + σ2 T ln + 0.08 − 0.03 + × 0.32 0.75
K 2 95 2
d1 = √ = √ =
σ T 0.3 0.75
0.471 669 √ √
d2 = d1 − σ T = 0.471 669 − 0.3 0.75 = 0.211 861
N (d1 ) = 0.681 418
N (d2 ) = 0.583 892
C = Se−δT N (d1 ) − Ke−rT N (d2 ) = 100e−0.03(0.75) 0.681 418 − 95e−0.08(0.75)
0.583 892 = 14. 386 295

b. µ ¶ µ ¶
S 1 100e−0.03(0.75) 1
ln + r − δ + σ2 T ln + 0 − 0 + × 0.32
0.75
K 2 95e−0.08(0.75) 2
d1 = √ = √ =
σ T 0.3 0.75
0.471 669100 √ √
d2 = d1 − σ T = 0.471 669 − 0.3 0.75 = 0.211 861

N (d1 ) = 0.681 418


N (d2 ) = 0.583 892
C = Se−δT N (d1 ) − Ke−rT N (d2 ) = 100e−0.03(0.75) 0.681 418 − 95e−0.08(0.75)
0.583 892 = 14. 386 295

The call premium is b is the same as the call premium in a. This is because
the call and
¡ put¢premium ¡formula ¢can be rewritten as:
C = Se−δT N (d1 ) − Ke−rT N (d2 )
¡ ¢ ¡ ¢
P = − Se−δT N (−d1 ) + Ke−rT N (−d2 )
µ ¶ µ ¶
S 1 Se−δT 1 2
ln + r − δ + σ2 T ln + 0 − 0 + σ T
K 2 Ke−rT 2
d1 = √ = √
√ σ T σ T
d2 = d1 − σ T

www.myactuaryexam.com c
°Yufeng Guo 112
CHAPTER 12. BLACK-SCHOLES FORMULA

From the above equations, you see that instead of using r and δ, we can
replace S with Se−δT , replace K with Ke−rT , and set r = δ = 0. This will also
give us the correct option premium.

Problem 12.8.
a. F0,T =0.75 = Se(r−δ)T = 100e(0.08−0.03)0.75 = 103. 821 20

b. To find the futures option premium, we replace S with F0,T and replace
δ with r in the standard
µ Black-Scholes
¶ formula: µ ¶
S 1 103. 821 20 1
ln + r − δ + σ2 T ln + 0.08 − 0.08 + × 0.32 0.75
K 2 95 2
d1 = √ = √ =
σ T 0.3 0.75
0.471 669 √ √
d2 = d1 − σ T = 0.471 669 − 0.3 0.75 = 0.211 861
N (d1 ) = 0.681 418
N (d2 ) = 0.583 892
C = Se−δT N (d1 )−Ke−rT N (d2 ) = 103. 821 20e−0.08(0.75) 0.681 418−95e−0.08(0.75)
0.583 892 = 14. 386 295

Here is another method. Since future option premium is equal to the stock
option premium, we can just calculate the stock option premium. Actually, the
stock call option premium is calculated in 12.7 a.

c. 12.7 a and 12.8 b have the same premium.

Problem 12.9.
a.When a stock pays discrete dividend, we’ll use the textbook Equation 12.5.
P
C = F0,T N (d1 ) − P V (K) N (d2 )
P
F0,T = S0 − P V (Div) = 50 − P V (Div)
P V (Div) = 2e−0.08(1/360) = 1. 999 555 6 = 2
P
F0,T (S) = 50 − 2 = 48

P
F0,T 1 48 1
ln + σ2 T ln + × 0.32 × 0.5
P V (K) 2 −0.08(0.5)
d1 = √ = 40e √ 2 = 1. 154 1
√ σ T √ 0.3 0.5
d2 = d1 − σ T = 1. 154 1 − 0.3 0.5 = 0.9420

N (d1 ) = 0.875 77
N (d2 ) = 0.826 90

C = 48 (0.875 77) − 40e−0.08(0.5) (0.826 90) = 10. 258

So the European call premium is 10. 258

www.myactuaryexam.com c
°Yufeng Guo 113
CHAPTER 12. BLACK-SCHOLES FORMULA

If it’s ever optimal to early exercise an American call, the best time to early
exercise is immediately before the dividend payment. Since the dividend is
paid tomorrow, the best time to early exercise is today. The exercise value is
EV = 50 − 40 = 10, which is less than the European call premium 10. 258. So
it’s not optimal to early exercise the American call.
Some of you might think that it’s optimal to early exercise the American
call. If we early exercise today, we get a stock, which will pay us a dividend 2.
So the total exercise value is 12.
This reasoning is flawed. If you exercise the call today and take ownership
of a stock, you’ll get $2 dividend tomorrow. However, after the dividend is paid,
the price of your stock is reduced by the amount of the dividend $2 (so you also
lose $2). So your exercise value is 10, not 12. And it’s not optimal to early
exercise the American call.

P
b. C = F0,T N (d1 ) − P V (K) N (d2 )
P
F0,T = S0 − P V (Div) = 50 − P V (Div)
P V (Div) = 2e−0.08(1/360) = 1. 999 555 6 = 2
P
F0,T (S) = 50 − 2 = 48

P
F0,T 1 48 1
ln + σ2T ln + × 0.32 × 0.5
P V (K) 2 40e−0.08(0.5) 2
d1 = √ = √ = 1. 154 1
√σ T √ 0.3 0.5
d2 = d1 − σ T = 1. 154 1 − 0.3 0.5 = 0.9420

N (d1 ) = 0.875 77
N (d2 ) = 0.826 90

C = 48 (0.875 77) − 40e−0.08(0.5) (0.826 90) = 10. 258

The exercise value is EV = 60 − 40 = 20 > C


Hence it’s optimal to early exercise the American put at t = 0.

c. It’s optimal to early exercise the American call at t = 0 if the exercise


value is greater than the European call premium. It’s not optimal to exercise
the American call at t = 0 if the exercise value is equal to or less than the he
European call premium. However, keep in mind that if the stock doesn’t pay
dividend, then it’s never optimal to early exercise an American call.

Problem 12.10.
The statement means that the absolute value |θ (t) | = | ∂V
∂t | reaches its max-
imum value when t → T . In other words, the closer to the expiration date, the
higher the |θ (t) |. This statement is not correct. I don’t know of any intuitive
way to explain why this statement is not correct. That’s why the textbook asks
you to test this statement using a spreadsheet.

www.myactuaryexam.com c
°Yufeng Guo 114
CHAPTER 12. BLACK-SCHOLES FORMULA

If you want to test the statement using a spreadsheet, you can use the
spreadsheet titled "optbasic2." Then you can make up a case and validate the
statement.
However, for the purpose of passing the exam, you can ignore this problem.

Problem 12.11.
a. If you look at the Appendix 12.A (which is excluded from both CAS and
SOA exam), you can see that Vega is the derivative of an option price regarding
volatility:
V ega = ∂V
∂σ
The formula V ega = V (σ+ )−V
2
(σ− )
is an approximation. For this approx-
imation to work, needs to be small. Since Appendix 12.A is excluded from
both CAS and SOA exam, you can ignore this part.

b. If you want to solve this problem, you can set up some test cases and
compare the approximated Vega with the actual Vega (using the spreadsheet).
For the purpose of passing the exam, you can ignore this part.

Problem 12.12.
Since Appendix 12.A is excluded from both CAS and SOA exam, you can
ignore this problem.

Problem 12.13.
Let’s not worry about drawing a diagram and focus on how to calculate the
profit. I’ll do some sample calculations.

Let’s calculate the profit after 6 months (i.e. at expiration) assuming the
stock price after 6 months is $60.
Using the Black-Scholes formula, we can find:

• The 40-strike call premium is 4.1553. This call premium is calculated


using the Black-Scholes formula by setting S = 40, K = 40, T = 0.5, r =
0.08, σ = 0.3, δ = 0
• The 45-strike call premium is 2.1304. This call premium is calculated
using the Black-Scholes formula by setting S = 40, K = 45, T = 0.5, r =
0.08, σ = 0.3, δ = 0

• The net cost of the bull spread at t = 0 is 4.1553 − 2.1304 = 2. 024 9


• Its future value at expiration is 2. 024 9e0.08(0.5) = 2. 107 5

The stock price at expiration is 60.

www.myactuaryexam.com c
°Yufeng Guo 115
CHAPTER 12. BLACK-SCHOLES FORMULA

• The 40-strike call payoff is 60 − 40 = 20

• The 45-strike call payoff is − (60 − 45) = −15

• The total payoff is 20 − 15 = 5

So our profit is: −2. 107 5 + 5 = 2. 892 5


Let’s calculate the profit after 3 months assuming the stock price after 3
months is $60.
Using the Black-Scholes formula, we can find:

• The 40-strike call premium is 4.1553

• The 45-strike call premium is 2.1304

• The net cost of the bull spread at t = 0 is 4.1553 − 2.1304 = 2. 024 9

• Its future value after 3 months is 2. 024 9e0.08(0.25) = 2. 065 8

3 months later (i.e. at t = 0.25), we close our position. Right now, our
purchased 40-strike call and sold 45-strike call both have 3 months to expiration.
To close our position (i.e. to cancel out our position), at t = 0.25, we sell a 40-
strike call and buy a 45-strike call. After this, our net position is zero. At
t = 0.25, the stock price is 60.

• At t = 0.25, the 40-strike call with 3 months to expiration is worth 20.7969


(so we receive 20.7969). This call premium is calculated using the Black-
Scholes formula by setting S = 60, K = 40 T = 0.25, r = 0.08, σ = 0.3, δ =
0.

• At t = 0.25, the 45-strike call with 3 months to expiration is worth 15.9480


(so we pay 15.9480). This call premium is calculated using the Black-
Scholes formula by setting S = 60, K = 45 T = 0.25, r = 0.08, σ =
0.3, δ = 0

• The net receipt is 20.7969 − 15.9480 = 4. 848 9

So our profit is: −2. 065 8 + 4. 848 9 = 2. 783 1


Let’s calculate the profit after 1 day assuming the stock price after 1day is
$60.
Using the Black-Scholes formula, we can find:

• The 40-strike call premium is 4.1553

• The 45-strike call premium is 2.1304

• The net cost of the bull spread at t = 0 is 4.1553 − 2.1304 = 2. 024 9

• Its future value after 1 day is 2. 024 9e0.08(1/365) = 2. 025 34

www.myactuaryexam.com c
°Yufeng Guo 116
CHAPTER 12. BLACK-SCHOLES FORMULA

1 day later (i.e. at t = 364/365), we close our position. Right now, our
purchased 40-strike call and sold 45-strike call both have 364 days to expiration.
To close our position (i.e. to cancel out our position), at t = 364/365 we sell a
40-strike call and buy a 45-strike call. After this, our net position is zero. At
t = 364/365, the stock price is 60.

• At t = 364/365, the 40-strike call with 364 days to expiration is worth


23.3775 (so we receive 23.3775). This call premium is calculated using
the Black-Scholes formula by setting S = 60, K = 40 T = 364/365, r =
0.08, σ = 0.3, δ = 0.

• At t = 364/365, the 45-strike call with 364 days to expiration is worth


19.2391 (so we pay 19.2391). This call premium is calculated using the
Black-Scholes formula by setting S = 60, K = 45 T = 364/365, r =
0.08, σ = 0.3, δ = 0

• The net receipt is 23.3775 − 19.2391 = 4. 138 4

So our profit is: −2. 038 44 + 4. 138 4 = 2. 099 96

Problem 12.14.

Appendix 12.A is excluded from both CAS and SOA exam. So SOA and
CAS can’t ask you to calculate option Greeks using formulas in Appendix 12.A.
(SOA and CAS can ask you to calculate delta ∆ using the textbook formula
10.1).
However, SOA and CAS can ask you to calculate the Greeks for a portfolio
using the formula presented in Page 389. Page 389 is on the syllabus.
So you need to learn how to calculate portfolio’s Greeks using the following
formula (in the textbook Page 389):
Pn
Greekoption = i=1 ω i Greeki

a. S = 40 Pn
ω1 Greek1 ω2 Greek2 Greekoption = i=1 ω i Greeki
bought 40K call sold 45K call Portfolio Price and Greeks
Price 1 4.1553 1 −2.1304 1 (4.1553) + 1 (−2.1304) = 2. 024 9
Delta 1 0.6159 1 −0.3972 1 (0.6159) + 1 (−0.3972) = 0.218 7
Gamma 1 0.0450 1 −0.0454 1 (0.0450) + 1 (−0.0454) = −0.000 4
Vega 1 0.1080 1 −0.1091 1 (0.1080) + 1 (−0.1091) = −0.001 1
Theta 1 −0.0134 1 0.0120 1 (−0.0134) + 1 (0.0120) = −0.001 4
Rho 1 0.1024 1 −0.0688 1 (0.1024) + 1 (−0.0688) = 0.033 6

Column 3 and 5 are calculated using the "optbasic2" spreadsheet. You


don’t need to worry about these two columns. YouP just need to focus on the
n
last column and practice the formula Greekoption = i=1 ω i Greeki .

www.myactuaryexam.com c
°Yufeng Guo 117
CHAPTER 12. BLACK-SCHOLES FORMULA

b. S = 45 P
ω1 Greek1 ω2 Greek2 Greekoption = ni=1 ω i Greeki
bought 40K call sold 45K call Portfolio Price and Greeks
Price 1 7.7342 1 −4.6747 1 (7.7342) + 1 (−4.6747) = 3. 059 5
Delta 1 0.8023 1 −0.6159 1 (0.8023) + 1 (−0.6159) = 0.186 4
Gamma 1 0.0291 1 −0.0400 1 (0.0291) + 1 (−0.0400) = −0.010 9
Vega 1 0.0885 1 −0.1216 1 (0.0885) + 1 (−0.1216) = −0.033 1
Theta 1 −0.0135 1 0.0150 1 (−0.0135) + 1 (0.0150) = 0.001 5
Rho 1 0.1418 1 −0.1152 1 (0.1418) + 1 (−0.1152) = 0.026 6

c. Ignore this part. Not sure what this problem wants to accomplish.

Problem 12.15.
a. S = 40 P
ω1 Greek1 ω2 Greek2 Greekoption = ni=1 ω i Greeki
bought 40K call sold 45K call Portfolio Price and Greeks
Price 1 2.5868 1 −5.3659 1 (2.5868) + 1 (−5.3659) = −2. 779 1
Delta 1 −0.3841 1 0.6028 1 (−0.3841) + 1 (0.6028) = 0.218 7
Gamma 1 0.0450 1 −0.0454 1 (0.0450) + 1 (−0.0454) = −0.000 4
Vega 1 0.1080 1 −0.1091 1 (0.1080) + 1 (−0.1091) = −0.001 1
Theta 1 −0.0049 1 0.0025 1 (−0.0049) + 1 (0.0025) = −0.002 4
Rho 1 −0.0898 1 0.1474 1 (−0.0898) + 1 (0.1474) = 0.057 6

b. S = 45 Pn
ω1 Greek1 ω2 Greek2 Greekoption = i=1 ω i Greeki
bought 40K call sold 45K call Portfolio Price and Greeks
Price 1 1.1658 1 −2.9102 1 (1.1658) + 1 (−2.9102) = −1. 744 4
Delta 1 −0.1977 1 0.3841 1 (−0.1977) + 1 (0.3841) = 0.186 4
Gamma 1 0.0291 1 −0.0400 1 (0.0291) + 1 (−0.0400) = −0.010 9
Vega 1 0.0885 1 −0.1216 1 (0.0885) + 1 (−0.1216) = −0.033 1
Theta 1 −0.0051 1 0.0056 1 (−0.0051) + 1 (0.0056) = 0.000 5
Rho 1 −0.0503 1 0.1010 1 (−0.0503) + 1 (0.1010) = 0.050 7

c. Ignore

Problem 12.16.
There’s no easy way to solve this problem manually. This type of problems
shouldn’t show up in the exam.

Problem 12.17.
There’s no easy way to solve this problem manually. This type of problems
shouldn’t show up in the exam.

www.myactuaryexam.com c
°Yufeng Guo 118
CHAPTER 12. BLACK-SCHOLES FORMULA

Problem 12.18.

S = 50 K = 60 r = 0.06 σ = 0.4 δ = 0.03

a. Solve for h.
1 2
σ h (h − 1) + (r − δ) h − r = 0
2
1
× 0.42 h (h − 1) + (0.06 − 0.03) h − 0.06 = 0
2
h = −0.608 182 5 h = 1. 233 182 5
Use the bigger h for call and the smaller h for put.
∗ hcall 1. 233 182 5
HCall = K= × 60 = 317. 309 19
hcall − 1 1. 233 182 5 − 1
µ ¶hcall µ ¶1. 233 182 5
∗ S 50
Cperpetual = (HCall − K) ∗ = (317. 309 19 − 60) =
HCall 317. 309 19
26. 351 83

The call should be exercised when the stock price reaches 317. 309 19. This
price is called the barrier.
The call premium is 26. 351 83.

b. Now δ = 0.04 (instead of δ = 0.03). Everything else is the same as in a.


Higher dividend yield means that the stock price will decrease more quickly.
Recall that the stock price drops by the dividend amount immediately after
the dividend payment time. We expect that under δ = 0.04 the optimal stock
price (i.e. barrier) is lower than the barrier when δ = 0.03. In addition, we
expect that the option price under δ = 0.04 is lower than the option price under
δ = 0.03.
Solve for h.
1 2
σ h (h − 1) + (r − δ) h − r = 0
2
1
× 0.42 h (h − 1) + (0.06 − 0.04) h − 0.06 = 0
2
h = −0.568 729 3 h = 1. 318 729 3
Use the bigger h for call and the smaller h for put.
∗ hcall 1. 318 729 3
HCall = K= × 60 = 248. 247 52
hcall − 1 1. 318 729 3 − 1
µ ¶hcall µ ¶1. 318 729 3
∗ S 50
Cperpetual = (HCall − K) ∗ = (248. 247 52 − 60) =
HCall 248. 247 52
22. 751 3

The call should be exercised when the stock price reaches 248. 247 52.
The call premium is 22. 751 28

c.Now r = 0.07 (instead of r = 0.06). Everything else is the same as in a.

www.myactuaryexam.com c
°Yufeng Guo 119
CHAPTER 12. BLACK-SCHOLES FORMULA

When the risk free interest rate goes up, stocks are expected to generate high
returns. Consequently, the value of a stock goes up; the call option premium
goes up (while the put option premium goes down). We expect that both the
barrier and the call premium go up when r goes up from 0.06 to 0.07.
Solve for h.
1 2
σ h (h − 1) + (r − δ) h − r = 0
2
1
× 0.42 h (h − 1) + (0.07 − 0.03) h − 0.07 = 0
2
h = 1. 218 245 8 h = −0.718 245 8
Use the bigger h for call and the smaller h for put.
∗ hcall 1. 218 245 8
HCall = K= × 60 = 334. 919 4
hcall − 1 1. 218 245 8 − 1
µ ¶hcall µ ¶1. 218 245 8
∗ S 50
Cperpetual = (HCall − K) ∗ = (334. 919 4 − 60) =
HCall 334. 919 4
27. 100 1

The call should be exercised when the stock price reaches 334. 919 4
The call premium is 27. 1001

d. The higher the volatility, the more valuable an option is. As σ goes
up from 0.4 to 0.5, we expect both the barrier and the call option premium will
go up.

1 2
σ h (h − 1) + (r − δ) h − r = 0
2
1
× 0.52 h (h − 1) + (0.06 − 0.03) h − 0.06 = 0
2
h = 1. 170 189 9 h = −0.410 189 9
Use the bigger h for call and the smaller h for put.
∗ hcall 1. 170 189 9
HCall = K= × 60 = 412. 547 36
hcall − 1 1. 170 189 9 − 1
µ ¶hcall µ ¶1. 170 189 9
∗ S 50
Cperpetual = (HCall − K) ∗ = (412. 547 36 − 60) =
HCall 412. 547 36
29. 835 5

The call should be exercised when the stock price reaches 412. 547 36
The call premium is 29. 835 5

Problem 12.19.

S = 50 K = 60 r = 0.06 σ = 0.4 δ = 0.03

1 2
a.Solve for h. σ h (h − 1) + (r − δ) h − r = 0
2

www.myactuaryexam.com c
°Yufeng Guo 120
CHAPTER 12. BLACK-SCHOLES FORMULA

1
× 0.42 h (h − 1) + (0.06 − 0.03) h − 0.06 = 0
2
h = −0.608 182 5 h = 1. 233 182 5
Use the bigger h for call and the smaller h for put.
hput −0.608 182 5
HP∗ ut = K= × 60 = 22. 690 80
hcall − 1 −0.608 182 5 − 1
µ ¶hput µ ¶−0.608 182 5
S 50
Pperpetual = (K − HP∗ ut ) = (60 − 22. 690 80) =
HP∗ ut 22. 690 80
23. 074 7

The put should be exercised when the stock price reaches 22. 690 80.
The put premium is 23. 074 7

b. Now δ = 0.04 (instead of δ = 0.03). Everything else is the same as in a.


Higher dividend yield means that the stock price will decrease more quickly.
Since the value of a put goes up if the stock price goes down, we expect that the
barrier price will go down (i.e. exercise occurs later because we want to exercise
when the stock price is low) and the put premium will go up.
1 2
Solve for h. σ h (h − 1) + (r − δ) h − r = 0
2
1
× 0.42 h (h − 1) + (0.06 − 0.04) h − 0.06 = 0
2
h = −0.568 729 3 h = 1. 318 729 3

Use the bigger h for call and the smaller h for put.
hput −0.568 729 3
HP∗ ut = K= × 60 = 21. 752 5
hcall − 1 −0.568 729 3 − 1
µ ¶hput µ ¶−0.568 729 3
S 50
Pperpetual = (K − HP∗ ut ) = (60 − 21. 752 5) =
HP∗ ut 21. 752 5
23. 824 8

The put should be exercised when the stock price reaches 21. 752 5
The put premium is 23. 824 8

c. As r goes up, people expect to get increased return from the stock. The
stock price goes up and the put option premium goes down.
We expect the barrier will go up (compared with a, meaning that exercise
occurs sooner).
1 2
σ h (h − 1) + (r − δ) h − r = 0
2
1
× 0.42 h (h − 1) + (0.07 − 0.03) h − 0.07 = 0
2
h = 1. 218 245 8 h = −0.718 245 8
Use the bigger h for call and the smaller h for put.
hput −0.718 245 8
HP∗ ut = K= × 60 = 25. 080 67
hcall − 1 −0.718 245 8 − 1

www.myactuaryexam.com c
°Yufeng Guo 121
CHAPTER 12. BLACK-SCHOLES FORMULA

µ ¶hput µ ¶−0.568 729 3


S 50
Pperpetual = (K − HP∗ ut ) = (60 − 25. 080 67) =
HP∗ ut 25. 080 67
23. 824 8

The put should be exercised when the stock price reaches 25. 080 67
The put premium is 23. 824 8

d. The higher the volatility, the more valuable an option is. As σ goes
up from 0.4 to 0.5, we expect both the barrier will go down (i.e. exercise occurs
later) and the put option premium will go up.

1 2
σ h (h − 1) + (r − δ) h − r = 0
2
1
× 0.52 h (h − 1) + (0.06 − 0.03) h − 0.06 = 0
2
h = 1. 170 189 9 h = −0.410 189 9
Use the bigger h for call and the smaller h for put.
hput −0.410 189 9
HP∗ ut = K= × 60 = 17. 452 5
hcall − 1 −0.410 189 9 − 1
µ ¶hput µ ¶−0.410 189 9
S 50
Pperpetual = (K − HP∗ ut ) = (60 − 17. 452 5) =
HP∗ ut 17. 452 5
27. 629 4

The put should be exercised when the stock price reaches 17. 452 5
The put premium is 27. 629 4

Problem 12.20.

For a and b, if you use the Black-Scholes formula, you’ll find that call and
put are both worth 17.6988.
For part c. After we switch S and K and switch r and δ, the put after
the switch and the call before the switch have the same value. This is not a
coincidence. It’s explained in my solution to Problem 10.19.

Problem 12.21.
1 2
a. σ h (h − 1) + (r − δ) h − r = 0
2
1
× 0.32 h (h − 1) + (0.08 − 0.05) h − 0.08 = 0
2
h = −1. 177 043 h = 1. 510 376
Use the bigger h for call and the smaller h for put.
∗ hcall 1. 510 376
HCall = K= × 90 = 266. 340 6
hcall − 1 1. 510 376 − 1

www.myactuaryexam.com c
°Yufeng Guo 122
CHAPTER 12. BLACK-SCHOLES FORMULA

µ ¶hcall µ ¶1. 510 376


∗ S 100
Cperpetual = (HCall − K) ∗ = (266. 340 6 − 90) =
HCall 266. 340 6
40. 158 9
The call premium is 40. 158 9

1 2
b. σ h (h − 1) + (r − δ) h − r = 0
2
1
× 0.32 h (h − 1) + (0.05 − 0.08) h − 0.05 = 0
2
h = −0.510 38 h = 2. 177 04

Use the bigger h for call and the smaller h for put.
hput −0.510 38
HP∗ ut = K= × 100 = 33. 791 5
hcall − 1 −0.510 38 − 1
µ ¶hput µ ¶−0.510 38
St 90
Pperpetual = (K − HP∗ ut ) = (100 − 33. 791 5) =
HP∗ ut 33. 791 5
40. 158 9
The put premium is 40. 158 9

c. After we switch S and K and switch r and δ, the perpetual American


put after the switch and the perpetual American call before the switch have the
same value. This is a minor fact probably not worth memorizing. We are not
going to worry about the proof.

www.myactuaryexam.com c
°Yufeng Guo 123
CHAPTER 12. BLACK-SCHOLES FORMULA

www.myactuaryexam.com c
°Yufeng Guo 124
Chapter 13

Market making and delta


hedging

Problem 13.1.
µ ¶ µ ¶
S 1 40 1 91
ln + r − δ + σ2 T ln + 0.08 − 0 + × 0.32
K 2 45 2 365
d1 = √ = r =
σ T 91
0.3
365
−0.578 25 r
√ 91
d2 = d1 − σ T = −0.578 25 − 0.3 = −0.728 04
365
N (d1 ) = 0.281 55
N (d2 ) = 0.233 29
C = Se−δT N (d1 ) − Ke−rT N (d2 ) = 40e−0(91/365) 0.281 55 − 45e−0.08(91/365)
0.233 29 = 0.971 3

A call on 100 stocks is worth 100 (0.971 3) = 97. 13


Use the formula in Appendix 12.8 to find ∆
∆ = e−δT N (d1 ) = e−0×91/365 N (d1 ) = 0.281 55 (positive delta means buy-
ing stocks)

Suppose a trader sells a call option on 100 stocks. To hedge his risk, the
trader should at t = 0

• sell the call and receive 100 (0.971 3) = 97. 13

• buy 0.281 55 (100) = 28. 155 stocks costing 40 (28. 155) = 1126. 2

• borrow 1126. 2 − 97. 13 = 1029. 07 from the bank

125
CHAPTER 13. MARKET MAKING AND DELTA HEDGING

The trader’s net position is zero at t = 0

If the stock price is 39 on Day 1.


To close his short call position, the trader can buy, from the open market,
a 45-strike call expiring in 90 days. This new (purchased) call and the original
(sold) call have the same underlying, same expiration date, same strike price.
They will cancel out each other and the trader doesn’t have any liabilities asso-
ciated either call.
The cost of theµnew call is 70.55.
¶ It’s calculated
µ as follows: ¶
S 1 2 39 1 2 90
ln + r−δ+ σ T ln + 0.08 − 0 + × 0.3
K 2 45 2 365
d1 = √ = r =
σ T 90
0.3
365
−0.753 706 r
√ 90
d2 = d1 − σ T = −0.753 706 346 − 0.3 = −0.902 675
365
N (d1 ) = 0.774 49
N (d2 ) = 0.183 349
C = Se−δT N (d1 ) −Ke−rT N (d2 ) = 39e−0(90/365) 0.225 513−45e−0.08(90/365)
0.183 349 = 0.705 5

A call on 100 stocks is worth 100 (0.705 5) = 70. 55


So the trader needs to pay 70. 55 to buy a new call to cancel out the original
call he sold.
At t = 0, the trader owns 28. 155 stocks. Now one day later, the stocks
are worth 28. 155 (39) = 1098. 045. So the trader sells out his stocks, receiving
1098. 05.
In addition, the trader needs to pay back the loan borrowed from the bank.
The future value of the loan one day later is:
1029. 07e0.08(1/365) = 1029. 30
So the trader’s net wealth at t = 1/365 is:
1098. 05 − (1029. 30 + 70. 55) = −1. 8
So the trader lost $1.8.

If you want to following the textbook calculation on Page 417 "Day 1:


Marking-to-market," here it is:
Day 1
Gain on 28. 155 shares 28. 155 (39 − 40) = −28. 155
Gain on the written call 97. 13 − ¡70. 55 = 26. 58 ¢
Interest 1029. 07 e0.08(1/365) − 1 = 0.226
Overnight profit −28. 155 + 26. 58 − 0.226 = −1. 8

If the stock price is 40.5 on Day 1.


To close his short call position, the trader can buy, from the open market,
a 45-strike call expiring in 90 days. This new (purchased) call and the original
(sold) call have the same underlying, same expiration date, same strike price.

www.myactuaryexam.com c
°Yufeng Guo 126
CHAPTER 13. MARKET MAKING AND DELTA HEDGING

They will cancel out each other and the trader doesn’t have any liabilities asso-
ciated either call.
The cost of theµ new call is 110.46.
¶ It’s calculated
µ as follows: ¶
S 1 2 40.5 1 2 90
ln + r−δ+ σ T ln + 0.08 − 0 + × 0.3
K 2 45 2 365
d1 = √ = r =
σ T 90
0.3
365
−0.500 363 r
√ 90
d2 = d1 − σ T = −0.500 363 − 0.3 = −0.649 332
365
N (d1 ) = 0.308 41
N (d2 ) = 0.258 062
C = Se−δT N (d1 )−Ke−rT N (d2 ) = 40.5e−0(90/365) 0.308 41−45e−0.08(90/365)
0.258 062 = 1. 104 65

A call on 100 stocks is worth 100 (1. 104 65) = 110. 465

One Day 1, the trader

• pays 110. 46 and buy a new call to cancel out the original call he sold
• sells out his 28. 155 stocks, receiving 28. 155 (40.5) = 1140. 28
• pays 1029. 07e0.08(1/365) = 1029. 30 to the bank to payoff the loan

So the trader’s net wealth on Day 1 is:


1140. 28 − (1029. 30 + 110. 46) = 0.52

If you want to use the textbook notation, here you go:


Day 1
Gain on 28. 155 shares 28. 155 (40.5 − 40) = 14. 077 5
Gain on the written call 97. 13 − ¡110. 465 = −13.¢335
Interest 1029. 07 e0.08(1/365) − 1 = 0.226
Overnight profit 14. 077 5 − 13. 335 − 0.226 = 0.52
So the trader gained $0.52.

Problem 13.2.
µ ¶ µ ¶
S 1 40 1 91
ln + r − δ + σ2 T ln + 0.08 − 0 + × 0.32
K 2 40 2 365
d1 = √ = r =
σ T 91
0.3
365
0.208 05 r
√ 91
d2 = d1 − σ T = 0.208 05 − 0.3 = 0.05 825
365
N (−d1 ) = 0.417 59 N (−d2 ) = 0.476 77
P = 40e−0.08(91/365) 0.476 77 − 40e−0(91/365) 0.417 59 = 1. 990 6

www.myactuaryexam.com c
°Yufeng Guo 127
CHAPTER 13. MARKET MAKING AND DELTA HEDGING

A put on 100 stocks is worth 100 (1. 990 6) = 199. 06


Use the formula in Appendix 12.8 to find ∆
∆ = −e−δT N (−d1 ) = −e−0×91/365 0.417 59 = −0.417 6 (negative delta
means short selling stocks)

Suppose a trader sells a put option on 100 stocks. To hedge his risk, the
trader should at t = 0

• sell the put and receive 100 (1. 990 6) = 199. 06

• buy −0.417 6 (100) = −41. 76 stocks (i.e. short sell 41. 76 stocks) receiving
41. 76 (40) = 1670. 4

• lend 199. 06 + 1670. 4 = 1869. 46 to a bank

The trader’s net position is zero at t = 0

If the stock price is 39.5 on Day 1.


To close his short put position, the trader can buy, from the open market,
a 40-strike put expiring in 90 days. This new (purchased) put and the original
(sold) put have the same underlying, same expiration date, same strike price.
They will cancel out each other and the trader doesn’t have any liabilities asso-
ciated either put.
The cost of the
µ new put is ¶
as follows: µ ¶
S 1 2 39 1 90
ln + r−δ+ σ T ln + 0.08 − 0 + × 0.32
K 2 40 2 365
d1 = √ = r = 0.0
σ T 90
0.3
365
3695 r
√ 90
d2 = d1 − σ T = 0.03695 − 0.3 = −0.112 02
365
N (−d1 ) = 0.485 26 N (−d2 ) = 0.544 60
P = Ke−rT N (−d2 )−Se−δT N (−d1 ) = 40e−0.08(90/365) 0.544 60−39e−0(90/365)
0.485 26 = 2. 433 4

On Day 1 a put on 100 stocks is worth 100 (2. 433 4) = 243. 34

One Day 1, the trader

• pays 243. 34, buying a new put to cancel out the original put he sold

• buys back 41. 76 stocks to close the short sale position, paying 41. 76 (39) =
1628. 64

• receives 1869. 46e0.08(1/365) = 1869. 87 from the bank

www.myactuaryexam.com c
°Yufeng Guo 128
CHAPTER 13. MARKET MAKING AND DELTA HEDGING

So the trader’s net wealth on Day 1 is:


−243. 34 − 1628. 64 + 1869. 87 = −2. 11
So the trader lost 2. 11.
If you want to use the textbook notation, here you go:
Day 1
Gain on 41. 76 shares 41. 76 (40 − 39) = 41. 76
Gain on the written put 199. 06 −¡ 243. 34 = −44.¢28
Interest income 1869. 46 e0.08(1/365) − 1 = 0.41
Overnight profit 41. 76 − 44. 28 + 0.41 = −2. 11

If the stock price is 40.5 on Day 1.


To close his short call position, the trader can buy, from the open market,
a 40-strike call expiring in 90 days. This new (purchased) call and the original
(sold) call have the same underlying, same expiration date, same strike price.
They will cancel out each other and the trader doesn’t have any liabilities asso-
ciated either call.
The cost of theµ new call is calculated
¶ as follows:
µ ¶
S 1 2 40.5 1 2 90
ln + r−δ+ σ T ln + 0.08 − 0 + × 0.3
K 2 40 2 365
d1 = √ = r =
σ T 90
0.3
365
0.290 29 r
√ 90
d2 = d1 − σ T = 0.290 29 − 0.3 = 0.141 32
365
N (−d1 ) = 0.385 80 N (−d2 ) = 0.443 81
P = Ke−rT N (−d2 )−Se−δT N (−d1 ) = 40e−0.08(90/365) 0.443 81−40.5e−0(90/365)
0.385 80 = 1. 780 8

On Day 1 a put on 100 stocks is worth 100 (1. 780 8) = 178. 08

One Day 1, the trader


• pays 178. 08, buying a new put to cancel out the original put he sold
• buys back 41. 76 stocks to close the short sale position, paying 41. 76 (40.5) =
1691. 28
• receives 1869. 46e0.08(1/365) = 1869. 87 from the bank
So the trader’s net wealth on Day 1 is:
−178. 08 − 1691. 28 + 1869. 87 = 0.51
So the trader gains 0.51.
If you want to use the textbook notation, here you go:
Day 1
Gain on 41. 76 shares 41. 76 (40 − 40.5) = −20. 88
Gain on the written put 199. 06 −¡ 178. 08 = 20. 98 ¢
Interest income 1869. 46 e0.08(1/365) − 1 = 0.41
Overnight profit −20. 88 + 20. 98 + 0.41 = 0.51

www.myactuaryexam.com c
°Yufeng Guo 129
CHAPTER 13. MARKET MAKING AND DELTA HEDGING

Next, I’m going to redo Problem 13.2 using the strike price K = 45. This
way, Problem 13.2 is similar to Problem 13.1 except that in Problem 13.1 there’s
a call and in Problem
µ 13.2 there’s
¶ a put. µ ¶
S 1 2 40 1 91
ln + r−δ+ σ T ln + 0.08 − 0 + × 0.32
K 2 45 2 365
d1 = √ = r =
σ T 91
0.3
365
−0.578 25 r
√ 91
d2 = d1 − σ T = −0.578 25 − 0.3 = −0.728 04
365
N (−d1 ) = 0.718 45
N (−d2 ) = 0.766 71
P = Ke−rT N (−d2 )−Se−δT N (−d1 ) = 45e−0.08(91/365) 0.766 71−40e−0(91/365)
0.718 45 = 5. 082 6

A put on 100 stocks is worth 100 (5. 082 6) = 508. 26


Use the formula in Appendix 12.8 to find ∆
∆ = −Se−δT N (−d1 ) = −e−0×91/365 0.718 45 = −0.718 45 (negative delta
means short selling stocks)

Suppose a trader sells a put option on 100 stocks. To hedge his risk, the
trader should at t = 0

• sell the call and receive 100 (5. 082 6) = 508. 26

• buy −0.718 45 (100) = −71. 845 stocks (i.e. short sell 71. 845 stocks) re-
ceiving 71. 845 (40) = 2873. 8
• lend 2873. 8 + 508. 26 = 3382. 06 to a bank

The trader’s net position is zero at t = 0


If the stock price is 39.5 on Day 1.
To close his short put position, the trader can buy, from the open market,
a 45-strike put expiring in 90 days. This new (purchased) put and the original
(sold) put have the same underlying, same expiration date, same strike price.
They will cancel out each other and the trader doesn’t have any liabilities asso-
ciated either put.
The cost of theµnew call is 110.46.
¶ It’s calculated
µ as follows: ¶
S 1 2 39 1 2 90
ln + r−δ+ σ T ln + 0.08 − 0 + × 0.3
K 2 45 2 365
d1 = √ = r =
σ T 90
0.3
365
−0.753 706 r
√ 90
d2 = d1 − σ T = −0.753 706 346 − 0.3 = −0.902 675
365
N (−d1 ) = 0.774 49

www.myactuaryexam.com c
°Yufeng Guo 130
CHAPTER 13. MARKET MAKING AND DELTA HEDGING

N (−d2 ) = 0.816 65
P = Ke−rT N (−d2 )−Se−δT N (−d1 ) = 45e−0.08(90/365) 0.816 65−39e−0(90/365)
0.774 49 = 5. 826 3

On Day 1 a put on 100 stocks is worth 100 (5. 826 3) = 582. 63

One Day 1, the trader

• pays 582. 63, buying a new put to cancel out the original put he sold
• buys back 71. 845 stocks to close the short sale position, paying 71. 845 (39) =
2801. 955
• receives 3382. 06e0.08(1/365) = 3382. 801 from the bank

So the trader’s net wealth on Day 1 is:


−582. 63 − 2801. 955 + 3382. 801 = −1. 784
So the trader lost 1.78.

If you want to use the textbook notation, here you go:


Day 1
Gain on 71. 845 shares 71. 845 (40 − 39) = 71. 845
Gain on the written put 508. 26 −¡ 582. 63 = −74.¢37
Interest income 3382. 06 e0.08(1/365) − 1 = 0.741
Overnight profit 71. 845 − 74. 37 + 0.741 = −1. 784

If the stock price is 40.5 on Day 1.


To close his short call position, the trader can buy, from the open market,
a 45-strike call expiring in 90 days. This new (purchased) call and the original
(sold) call have the same underlying, same expiration date, same strike price.
They will cancel out each other and the trader doesn’t have any liabilities asso-
ciated either call.
The cost of theµ new call is 110.46.
¶ It’s calculated
µ as follows: ¶
S 1 2 40.5 1 2 90
ln + r−δ+ σ T ln + 0.08 − 0 + × 0.3
K 2 45 2 365
d1 = √ = r =
σ T 90
0.3
365
−0.500 363 r
√ 90
d2 = d1 − σ T = −0.500 363 − 0.3 = −0.649 332
365
N (−d1 ) = 0.691 59
N (−d2 ) = 0.741 94
P = Ke−rT N (−d2 )−Se−δT N (−d1 ) = 45e−0.08(90/365) 0.741 94−40.5e−0(90/365)
0.691 59 = 4. 725 76

On Day 1 a put on 100 stocks is worth 100 (4. 725 76) = 472. 576
One Day 1, the trader

www.myactuaryexam.com c
°Yufeng Guo 131
CHAPTER 13. MARKET MAKING AND DELTA HEDGING

• pays 472. 576, buying a new put to cancel out the original put he sold

• buys back 71. 845 stocks to close the short sale position, paying 71. 845 (40.5) =
2909. 72

• receives 3382. 06e0.08(1/365) = 3382. 801 from the bank

So the trader’s net wealth on Day 1 is:


−472. 576 − 2909. 72 + 3382. 801 = 0.505
So the trader gains 0.505.
If you want to use the textbook notation, here you go:
Day 1
Gain on 71. 845 shares 71. 845 (40 − 40.5) = −35. 92
Gain on the written put 508. 26 −¡ 472. 576 = 35. ¢684
Interest income 3382. 06 e0.08(1/365) − 1 = 0.741
Overnight profit −35. 92 + 35. 684 + 0.741 = 0.505

Problem 13.3.

buy a 40-45 bull spread=buy a 40-strike call and sell a 45-strike put
I used a spreadsheet to calculate the following so you might not be able to
fully match my result

Day 0: Expiration T = 91/365 Stock price 40


Position Strike Call premium Delta
buy 40 −2.7804 −0.58240
sell 45 0.9710 0.28155
Net (one stock) −2.7804 + 0.9710 = −1. 809 4 −0.28155 + 0.58240 = −0.300 85
Net (100 stocks) 100 (−1. 809 4) = −180. 94 100 (−0.300 85) = −30. 085

In the above table

• positive premium means cash inflow (i.e. receiving premium)

• negative premium means cash outgo (i.e. paying premium)

• positive delta means buying stocks

• negative delta means short selling stocks

On Day 0, the trader:

• Buy a 40-strike call and sell a 45-strike call, paying 180. 94

• Short sell 30. 085 stocks, paying 30. 085 (40) = 1203. 4. The negative delta
means short selling.

www.myactuaryexam.com c
°Yufeng Guo 132
CHAPTER 13. MARKET MAKING AND DELTA HEDGING

• Deposit 1203. 4 − 180. 94 = 1022. 46 to a savings bank to earn a risk free


rate

The trader’s net position is zero.


Day 1: Expiration T = 90/365 Stock price 39
Position Strike price Call premium
Sell 40 2.2144
Buy 45 −0.7054
Net (one stock) 2.2144 − 0.7054 = 1. 509
Net (100 stocks) 100 (1. 509) = 150. 9

Day 1, the trader closes out his position

• Sell a 40-strike call to cancel out the original 40-strike call he bought; buy
a 45-strike call to cancel out the original 45-strike call he sold. The trader
receives 150. 9
• Buy 30. 085 stocks from the market to close the short sale, paying 30.
085 (39) = 1173. 315

• Receive 1022. 46e0.08(1/365) = 1022. 684 from the savings account

The trader’s net profit: 150. 9 − 1173. 315 + 1022. 684 = 0.269
If the stock price is 39 on Day 1, the trader gets 0.269 profit.

Day 1: Expiration T = 90/365 Stock price 40.5


Position Strike price Call premium
Sell 40 3.0621
Buy 45 −1.1046
Net (one stock) 3.0621 − 1.1046 = 1. 957 5
Net (100 stocks) 100 (1. 957 5) = 195. 75

Day 1, the trader closes out his position

• Sell a 40-strike call to cancel out the original 40-strike call he bought; buy
a 45-strike call to cancel out the original 45-strike call he sold. The trader
receives 195. 75

• Buy 30. 085 stocks from the market to close the short sale, paying 30.
085 (40.5) = 1218. 442 5

• Receive 1022. 46e0.08(1/365) = 1022. 684 from the savings account

The trader’s net profit: 195. 75 − 1218. 442 5 + 1022. 684 = −0.008 5 = −0.01
If the stock price is 39 on Day 1, the trader losses 0.01.

Problem 13.4.

www.myactuaryexam.com c
°Yufeng Guo 133
CHAPTER 13. MARKET MAKING AND DELTA HEDGING

Day 0: Expiration T = 91/365 Stock price 40


Position Strike Put premium Delta
buy 45 −5.0824 − (−0.71845) = 0.718 451
sell 40 1.9905 −0.41760
Net −5.0824 + 1.9905 × 2 = −1. 101 4 0.71845 − 0.41760 × 2 = −0.116 75
Net 100 (−1. 101 4) = −110. 14 100 (−0.116 75) = −11. 675

On Day 0, the trader:

• Buy a 45-strike put and sell two 45-strike put, paying 110. 14
• Short sell 11. 675 stocks, receiving 11. 675 (40) = 467
• Deposit 467 − 110. 14 = 356. 86 to a savings bank to earn a risk free rate

The trader’s net position is zero.


Day 1: Expiration T = 90/365 Stock price 39
Position Strike price Call premium
Sell 45 5.8265
Buy 40 −2.4331
Net 5.8265 − 2.4331 × 2 = 0.960 3
Net 100 (0.960 3) = 96. 03

Day 1, the trader closes out his position

• Sell a 45-strike put to cancel out the original 40-strike put he bought; buy
two 40-strike puts to cancel out the original two 40-strike puts he sold.
The trader receives 96. 03
• Buy 11. 675 stocks from the market to close the short sale, paying 11.
675 (39) = 455. 325
• Receive 356. 86e0.08(1/365) = 356. 938 2 from the savings account

The trader’s net profit: 96. 03 − 455. 325 + 356. 938 2 = −2. 36
If the stock price is 39 on Day 1, the trader loses 2.36.

Day 1: Expiration T = 90/365 Stock price 40.5


Position Strike price Call premium
Sell 45 4.7257
Buy 40 −1.7808
Net 4.7257 − 1.7808 × 2 = 1. 164 1
Net 100 (1. 164 1) = 116. 41

Day 1, the trader closes out his position


1 The delta formula in Appendix 12.B is the delta for the market maker if he sells an option.

If the market maker buys an option, add a negative sign to the formula. In this case, the delta
is -0.71845 if the trader sells the 45-strike put. However, since the trader buys a 45-strike put,
the delta is − (−0.91845) = 0.918 45

www.myactuaryexam.com c
°Yufeng Guo 134
CHAPTER 13. MARKET MAKING AND DELTA HEDGING

• Sell a 45-strike put to cancel out the original 40-strike put he bought; buy
two 40-strike puts to cancel out the original two 40-strike puts he sold.
The trader receives 116. 41

• Buy 11. 675 stocks from the market to close the short sale, paying 11.
675 (40.5) = 472. 84

• Receive 356. 86e0.08(1/365) = 356. 938 2 from the savings account

The trader’s net profit: 116. 41 − 472. 84 + 356. 938 2 = 0.51


If the stock price is 39 on Day 1, the trader gains 0.51.

Problem 13.5.

T = 91/365 r = 8% δ=0
σ = 30% K = 40

Let’s walk through Day 0 and Day 1 calculations.


Day 0 1
stock $40.00 $40.50
put $199.05 $178.08
delta −0.417596 −0.385797
Investment −1, 869.43 −$1, 740.56
Interest credited (end of the day) $0.41 $0.381 5
Capital gain (end of the day) $0.09 −$4. 245 4
Daily profit (end of the day) $0.50 −$3. 864

Day 0:
Using the Black-Scholes formula, you can verify the put premium is C0 =
$199.05 and delta per stock is −0.417596. Negative delta means buying negative
number of stocks (i.e. the trader needs to short sell stocks). The trader buys
∆0 = 100 (−0.417596) = −41. 759 6 stocks (i.e. short-sell 41. 759 6 stocks),
receiving 41. 759 6×40 = 1670. 384. In addition, the trader receives put premium
C0 =$199.05. So at t = 0 the trader’s asset (i.e. investment)
M V (0) = ∆0 S0 − C0 = (−41. 759 6) 40 − 199.05 = −1869. 434
The negative amount means that the trader receives 1869. 434
The trader lends out $1869. 434 (i.e. depositing $1869. 434) in a savings
account. Now his net position is zero.

At the end of Day 0, the stock price goes up from S0 = 40 to S1 = 40.5.


The put liability goes down from C0 = $199.05 to C1 = $178.08. The trader’s
asset before he rebalances his portfolio is (BR stands for before rebalance):
M V BR (1) = ∆0 S1 − C1 = (−41. 759 6) 40.5 − 178.08 = −1869. 343 8
The trader’s profit at the end of Day 0 is:
M V BR (1)−M V (0) erh = −1869. 343 8−(−1869. 434) e0.08(1/365) = 0.499 98 =
0.5

www.myactuaryexam.com c
°Yufeng Guo 135
CHAPTER 13. MARKET MAKING AND DELTA HEDGING

To find the capital gain and the interest earned at the end of Day 0, we just
need to break down the profit at the end of Day 0 into two parts:
¡ ¢
M V BR (1) − M V (0) erh = M V BR (1) − M V (0) + −M V (0) erh − 1
| {z } | {z }
capital gain interest earned
Capital gain at the end of Day 0: M V BR (1) − M V (0) = −1869. 343 8 −
(−1869. 434) = 0.090 2
¡ ¢ ¡ ¢
Interest earned at the end of Day 0: −M V (0) erh − 1 = − (−1869. 434) e0.08/365 − 1 =
0.409 78 = 0.41
The investment at the beginning of Day 0 is: M V (0) = 1869. 434
Day 1:
In the beginning of Day 1, the trader starts from a clean slate. He buys sells
∆1 = 100 × (−0.385797) = −38. 579 7 stocks (i.e. short sell stocks) and receives
the put premium C1 = $178.08.
His asset (or investment) is M V (1) = ∆1 S1 − C1 = (−38. 579 7) 40.5 −
178.08 = −1740. 557 85
The trader’s asset at the end of Day 1 before he rebalances the portfolio (i.e.
before he starts over from a clean slate the next day) is:
M V BR (2) = ∆1 S2 − C2 = (−38. 579 7) 39.25 − 230.55 = −1744. 803 225
The trader’s profit at the end of Day 1 is:
M V BR (2) − M V (1) erh = −1744. 803 225 − (−1740. 557 85) e0.08(1/365) =
−3. 864

To find the capital gain and the interest earned at the end of Day 1, we just
need to break down the profit at the end of Day 1 into two parts:
¡ ¢
M V BR (2) − M V (1) erh = M V BR (2) − M V (1) + −M V (1) erh − 1
| {z } | {z }
capital gain interest earned
The capital gain credited at the end of Day 1 is:

M V BR (2) − M V (1) = −1744. 803 225 − (−1740. 557 85) = −4. 245 4
The interest
¡ earned¢ at the end of Day 1¡is: ¢
−M V (1) erh − 1 = − (−1740. 557 85) e0.08(1/365) − 1 = 0.381 5

I’ll omit the calculations for the other days. Here is the result for all days:
Day 0 1 2
stock $40.00 $40.50 $39.25
put $199.05 $178.08 $230.55
delta −0.417596 −0.385797 −0.46892
Investment −1, 869.43 −$1, 740.56 −$2, 071.07
Interest credited (end of the day) $0.41 $0.381 5 $0.45
Capital gain (end of the day) $0.09 −$4. 245 4 −$0.05
Daily profit (end of the day) $0.50 −$3. 864 $0.40

www.myactuaryexam.com c
°Yufeng Guo 136
CHAPTER 13. MARKET MAKING AND DELTA HEDGING

Day 3 4 5
stock $38.75 $40.00 $40.00
put $254.05 $195.49 $194.58
delta −0.50436 −0.41940 −0.41986
Investment −$2, 208.46 −$1, 873.10 −$1, 874.02
Interest credited (end of the day) $0.48 $0.41
Capital gain (end of the day) −$4.48 $0.91
Daily profit (end of the day) −$4.00 $1.32

Problem 13.6.
Day 0 1 2
stock $40.00 $40.642 40.018
put $199.05 172.6644 196.5319
delta −0.41760 −0.37684 −0.41731
Investment −$1, 869.43 −$1, 704.22 −$1, 866.51
Interest credited (end of the day) $0.41 $0.37 $0.41
Capital gain (end of the day) −$0.42 −$0.35 −$0.40
Daily profit (end of the day) −$0.01 $0.02 $0.01

Day 3 4 5
stock 39.403 $38.80 39.420
put 222.5962 250.8701 220.0727
delta −0.45918 −0.50202 −0.45940
Investment −$2, 031.89 −$2, 198.56 −$2, 031.02
Interest credited (end of the day) $0.45 $0.48
Capital gain (end of the day) −$0.45 −$0.48
Daily profit (end of the day) $0.00 $0.00

Problem 13.7.

If SOA tests this type of problems in the exam, they’ll need to give you at
least Γ and θ. You can calculate the option premium V and delta ∆. Once you
have Greeks, just use the formula:
1 2
V (St+h , T − t − h) ≈ V (St , T − t) + ∆t + θh + Γt (Textbook 13.6)
2
I’m not going to do all the parts in the problem. I’m just going to show you
some examples.

First, you’ll need to get Greeks. You can use the Black-Scholes formula and
find the option premium V and delta ∆. To find Γ and θ, you’ll need to use the
spreadsheet attached to the textbook.
σ = 0.3 r = 8% δ=0 K = 40

www.myactuaryexam.com c
°Yufeng Guo 137
CHAPTER 13. MARKET MAKING AND DELTA HEDGING

Day 0 1
time t 0 h = 1/365
Expiration T − t 180/365 = 0.493 150 684 9 179/365 = 0.490 410 958 9
stock price St 40 44
Call price Vt 4.1217 6.8991
Delta ∆t 0.6151 0.7720
Gamma Γt 0.0454 0.0327
Theta θt −0.0134 −0.0137
V (St , T − t) = V (40, 180/365
¡ − 0) = 4.1217 ¢
V (St+h , T − t − h) = V S0+1/365 , 180/365 − 0 − 1/365 = 6.8991
= St+h − St = 44 − 40 = 4
∆t = 0.6151
θt = −0.0134 × 365 (The spreadsheet gives the per-day theta; we need to
annualize it.)
Γt = 0.0454
1
V (St , T − t) + ∆t + θt h + Γt 2
2
1 1
= 4.1217 + 0.6151 × 4 − 0.0134 × 365 × + × 0.0454 × 42
365 2
= 6. 931 9
The true value is V (St+h , T − t − h) = 6.8991
The error percentage is:
6. 931 9 − 6.8991
= 0.4754 %
6.8991

σ = 0.3 r = 8% δ=0 K = 40

Day 0 5
time t 0 h = 5/365
Expiration T − t 180/365 = 0.493 150 684 9 175/365 = 0.479 452 054 8
stock price St 40 44
Call price Vt 4.1217 6.8440
Delta ∆t 0.6151 0.7726
Gamma Γt 0.0454 0.0330
Theta θt −0.0134 −0.0138

V (St , T − t) = V (40, 180/365


¡ − 0) = 4.1217 ¢
V (St+h , T − t − h) = V S0+1/365 , 180/365 − 0 − 5/365 = 6.8440
= St+h − St = 44 − 40 = 4
∆t = 0.6151 θt = −0.0134 × 365 Γt = 0.0454
1
V (St , T − t) + ∆t + θt h + Γt 2
2
5 1
= 4.1217 + 0.6151 × 4 − 0.0134 × 365 × + × 0.0454 × 42
365 2
= 6. 878 3
The true value is V (St+h , T − t − h) = 6.8440
The error percentage is:

www.myactuaryexam.com c
°Yufeng Guo 138
CHAPTER 13. MARKET MAKING AND DELTA HEDGING

6. 878 3 − 6.8440
= 0.5 01%
6.8440

Problem 13.8.

I’m not going to do all the parts in the problem. I’m just going to show you
some examples.
σ = 0.3 r = 8% δ=0 K = 40
Day 0 1
time t 0 h = 1/365
Expiration T − t 180/365 = 0.493 150 684 9 179/365 = 0.490 410 958 9
Stock price St 40 44
Put price Vt 2.5744 1.3602
Delta ∆t −0.3849 −0.2280
Gamma Γt 0.0454 0.0327
Theta θt −0.0050 −0.0053

V (St , T − t) = V (40, 180/365


¡ − 0) = 2.5744 ¢
V (St+h , T − t − h) = V S0+1/365 , 180/365 − 0 − 1/365 = 1.3602
= St+h − St = 44 − 40 = 4
∆t = −0.3849 θt = −0.0050 × 365 Γt = 0.0454
1 2
V (St , T − t) + ∆t + θt h + Γt
2
1 1
= 2.5744 − 0.3849 × 4 − 0.0050 × 365 × + × 0.0454 × 42
365 2
= 1. 393
The true value is V (St+h , T − t − h) = 1.3602
The error percentage is:
1. 393 − 1.3602
= 0.2 41%
1.3602

σ = 0.3 r = 8% δ=0 K = 40

Day 0 5
time t 0 h = 5/365
Expiration T − t 180/365 = 0.493 150 684 9 175/365 = 0.479 452 054 8
Stock price St 40 44
Put price Vt 2.5744 1.3388
Delta ∆t −0.3849 −0.2274
Gamma Γt 0.0454 0.0330
Theta θt −0.0050 −0.0054
V (St , T − t) = V (40, 180/365
¡ − 0) = 2.5744 ¢
V (St+h , T − t − h) = V S0+1/365 , 180/365 − 0 − 1/365 = 1.3602
= St+h − St = 44 − 40 = 4
∆t = −0.3849 θt = −0.0050 × 365 Γt = 0.0454
1 2
V (St , T − t) + ∆t + θt h + Γt
2

www.myactuaryexam.com c
°Yufeng Guo 139
CHAPTER 13. MARKET MAKING AND DELTA HEDGING

5 1
= 2.5744 − 0.3849 × 4 − 0.0050 × 365 × + × 0.0454 × 42
365 2
= 1. 373
The true value is V (St+h , T − t − h) = 1.3602
The error percentage is: 1.3388
1. 373 − 1.3388
= 2. 55%
1.3388

Problem 13.9.

I’m going to do one set of calculation assuming the stock price one day later
is $30. However, I’m not going to produce a graph.

σ = 0.3 S = 40 r = 8% δ=0 K = 40

Day 0 1
time t 0 h = 1/365
Expiration T − t 91/365 90/365
Stock price St 40 30
Call price Vt 2.7804 0.0730
Delta ∆t 0.5824 0.0423
Gamma Γt 0.0652 0.0202
Theta θt −0.0173 −0.9134
a. The price of a 40-strike, 90 day to expiration call is worth 0.0730
b. Use delta approximation:
= 30 − 40 = −10
0
V = V0 + ∆t = 2.7804 + 0.5824 (−10) = −3. 043 6

We get a nonsense value of −3. 043 6. This is because the delta approxima-
tion is good when is small. Here we have a large change of = −10.
c. Use delta-gamma approximation:
0 1
V = V0 + ∆t + Γt 2
2
1
= 2.7804 + 0.5824 (−10) + × 0.0652 × (−10)2 = 0.216 4
2
d. Use delta-gamma-theta approximation:
0 1
V = V0 + ∆t + Γt 2 + θt h
2
1 2 1
= 2.7804 + 0.5824 (−10) + × 0.0652 × (−10) − 0.0173 × 365 ×
2 365
= 0.199 1
We see in a, b, c, and d, the approximation is not good. This is because the
approximation is good when is small. Here we have a large change of = −10.

Problem 13.10.

www.myactuaryexam.com c
°Yufeng Guo 140
CHAPTER 13. MARKET MAKING AND DELTA HEDGING

I’m not going to produce a graph. I’ll do one set of calculation assuming the
stock price one day later is $41.

σ = 0.3 S = 40 r = 8% δ=0 K = 40

Day 0 1
time t 0 h = 1/365
Expiration T − t 1 364/365
Stock price St 40 30
Call price Vt 6.2845 6.9504
Delta ∆t 0.6615 0.6909
Gamma Γt 0.0305 0.0287
Theta θt −0.0104 −0.0106
a. The price of a 40-strike, 364 day to expiration call is worth 6.9504
b. Use delta approximation:
= 41 − 40 = 1
0
V = V0 + ∆t = 6.2845 + 0.6615 (1) = 6. 946

c. Use delta-gamma approximation:


0 1
V = V0 + ∆t + Γt 2
2
1
= 6.2845 + 0.6615 (1) + × 0.0305 × (1)2 = 6. 961 3
2
d. Use delta-gamma-theta approximation:
0 1
V = V0 + ∆t + Γt 2 + θt h
2
1 1
= 6.2845 + 0.6615 (1) + × 0.0305 × (1)2 − 0.0104 × 365 × = 6. 950 9
2 365
We see in a, b, c, and d, the approximation is good. This is because is
small.

Problem 13.11.
I’m going to do one set of calculation assuming the stock price one day later
is $30.

σ = 0.3 S = 40 r = 8% δ=0 K = 40

Day 0 1
time t 0 h = 1/365
Expiration T − t 91/365 90/365
Stock price St 40 30
Put price Vt 1.9905 9.2917
Delta ∆t −0.4176 −0.9577
Gamma Γt 0.0652 0.0202
Theta θt −0.0088 0.0061
a. The price of a 40-strike, 90 day to expiration put is worth 9.2917

www.myactuaryexam.com c
°Yufeng Guo 141
CHAPTER 13. MARKET MAKING AND DELTA HEDGING

b. Use delta approximation:


= 30 − 40 = −10
0
V = V0 + ∆t = 1.9905 − 0.4176 (−10) = 6. 166 5
c. Use delta-gamma approximation:
0 1
V = V0 + ∆t + Γt 2
2
1
= 1.9905 − 0.4176 (−10) + × 0.0652 × (−10)2 = 9. 426 5
2
d. Use delta-gamma-theta approximation:
0 1
V = V0 + ∆t + Γt 2 + θt h
2
1 1
= 1.9905 − 0.4176 (−10) + × 0.0652 × (−10)2 − 0.0088 × 365 ×
2 365
= 9. 417 7

Problem 13.12.

I’m going to do one set of calculation assuming the stock price one day later
is $41.

σ = 0.3 S = 40 r = 8% δ=0 K = 40

Day 0 1
time t 0 h = 1/365
Expiration T − t 1 364/365
Stock price St 40 41
Put price Vt 3.2092 2.8831
Delta ∆t −0.3385 −0.3091
Gamma Γt 0.0305 0.0287
Theta θt −0.0023 −0.0025
a. The price of a 40-strike, 364 day to expiration put is worth 2.8831
b. Use delta approximation:
= 41 − 40 = 1
0
V = V0 + ∆t = 3.2092 − 0.3385 (1) = 2. 870 7
c. Use delta-gamma approximation:
0 1
V = V0 + ∆t + Γt 2
2
1 2
= 3.2092 − 0.3385 (1) + × 0.0305 × (1) = 2. 885 95
2
d. Use delta-gamma-theta approximation:
0 1
V = V0 + ∆t + Γt 2 + θt h
2
1 1
= 3.2092 − 0.3385 (1) + × 0.0305 × (1)2 − 0.0023 × 365 ×
2 365
= 2. 883 65

Problem 13.13.

www.myactuaryexam.com c
°Yufeng Guo 142
CHAPTER 13. MARKET MAKING AND DELTA HEDGING

Please note that in DM 13.9, θ is annualized. However, in Table 13.1, θ is


a per-day theta. (DM page 411 says that if the expiry is measured in years,
then theta is the annualized change in option value; to get the per-day theta,
divide by 365. The θ in Table 13.1 and in the author’s Excel spreadsheet is
daily µ
theta.) So we need to annualize θ before using DM 13.9. ¶
1 2 2
− × 0.3 × 40 × 0.0652 − 0.0173 × 365 + 0.08 (0.5824 × 40 − 2.7804) ×
2
1
= −5. 79 4 × 10−5 ≈ 0
365

Problem 13.14.

We’ll use the Excel spreadsheet attached in the DM textbook and calculate
the following:
Inputs:
Stock Price 40
Exercise Price 45
Volatility 30%
Risk-free interest rate 8%
Time to Expiration (years) 0.5
Dividend Yield 0%

Outputs:
Black-Scholes (European)
Call Put
Price 2.1304 5.3659
Delta 0.3972 −0.6028
Gamma 0.0454 0.0454
Vega 0.1091 0.1091
Theta −0.0120 −0.0025
Rho 0.0688 −0.1474
Psi −0.0794 0.1206
Elasticity 7.4578 −4.4936

DMµ 13.9 is: ¶


1
− × 0.32 × 402 × 0.0454 − 0.0025 × 365 + 0.08 (−0.6028 × 40 − 5.3659) ×
2
1
= 5. 29 × 10−6 ≈ 0
365

Problem 13.15.

Use DM’s spreadsheet. The inputs are:

www.myactuaryexam.com c
°Yufeng Guo 143
CHAPTER 13. MARKET MAKING AND DELTA HEDGING

Stock Price 40
Exercise Price 45
Volatility 30%
Risk-free interest rate 8%
Time to Expiration (years) = 180/365
Dividend Yield 0%

We’ll get the following outputs:

Black-Scholes (European)
40-strike call 45-strike call
Price 4.1217 2.1004
Delta 0.6151 0.3949
Gamma 0.0454 0.0457

We bought one 45-strike call. Suppose we need to sell (i.e. write) X unit
of 40-strike call. The Gamma of the bought 45-strike call is −0.0457 (negative
because we bought a call). The Gamma of the sold X unit of 40-strike call is
0.0454X. The total Gamma of our portfolio is 0.0454X − 0.0457. To Gamma
hedge, set 0.0454X − 0.0457 = 0. This gives us X = 1. 006 6. The total Delta
of our portfolio is 0.6151 × 1. 006 6 − 0.3949 = 0.224 3. To delta hedge, we need
to buy 0.224 3 share of the underlying stock.
This is our final portfolio at time zero:
Transactions at time zero Cost
Buy a 45-strike call 2.1004
Sell 1. 006 6 unit of 40-strike call (to Gamma hedge) −1. 006 6 × 4.1217 = −4. 148 9
Buy 0.224 3 share of the stock (to Delta hedge) 0.224 3 × 40 = 8. 972
Borrow 2.1004 − 4. 148 9 + 8. 972 = 6. 923 5 −6. 923 5
Total 2.1004 − 4. 148 9 + 8. 972 − 6. 923 5 = 0

One day later, you close out your position:


Transactions (one day later) Revenue
0
Sell a 45-strike call C1
0
Buy 1. 006 6 unit of 40-strike call −1. 006 6C2
0
Sell 0.224 3 share of the stock 0.224 3S
Repay the borrowed amount 6. 923 5e0.08/365
0 0 0
Total C1 − 1. 006 6C2 + 0.224 3S − 6. 923 5e0.08/365
0
C1 is the price of a 45-strike call with 179 day to expiration.
0
C2 is the price of a 40-strike call with 179 day to expiration.
0
S is the price of the stock one day later.
0 0
If you want to draw the diagram of the overnight profit C1 − 1. 006 6C2 +
0 0
0.224 3S − 6. 923 5e0.08/365 , you can get different profit by changing S . For
example,

www.myactuaryexam.com c
°Yufeng Guo 144
CHAPTER 13. MARKET MAKING AND DELTA HEDGING

0
assume S = 50. Also assume that the stock volatility and risk free interest

rate are not changed. Using the Black-Scholes formula (use T = 179/365), we
get:
0 0
C1 = 8.1511 C2 = 12.0043
The overnight profit is:
8.1511 − 1. 006 6 × 12.0043 + 0.224 3 × 50 − 6. 923 5e0.08/365 = 0.36
0
If S = 50, then the overnight profit is 0 − 1. 006 6 × 0 + 0.224 3 × 0 − 6.
923 5e0.08/365 = −6. 93.
0
By changing S , you’ll get different profits. Then you draw a graph on how
0
the overnight profit varies by S . Since it’s time-consuming to calculate the
0
overnight profit by changing S , I’m not going to do it.

Problem 13.16.
Use DM’s spreadsheet. The inputs are:

Stock Price 40
Exercise Price 45
Volatility 30%
Risk-free interest rate 8%
Time to Expiration (years) = 180/365
Dividend Yield 0%

We’ll get the following outputs:

Black-Scholes (European)
40-strike call 45-strike put
Price 4.1217 5.3596
Delta 0.6151 −0.6051
Gamma 0.0454 0.0457
We sell one 45-strike put and buy X unit of 40-strike call. The total Gamma
is 0.0457 − 0.0454X. To Gamma hedge, set 0.0457 − 0.0454X = 0. This gives
us X = 1. 006 6.

The delta of 1. 006 6 unit of 40-strike call is −0.6151 × 1. 006 6.


The total Delta is −0.6151 × 1. 006 6 − 0.6051 = −1. 224 3, meaning that we
need to short sell 1. 224 3 unit of the underlying stock.

Final portfolio at time zero:


Transactions at time zero Revenue
Sell a 45-strike put 5.3596
Buy 1. 006 6 unit of 40-strike call (to Gamma hedge) −1. 006 6 × 4.1217 = −4. 148 9
Short sell 1. 224 3 share of the stock (to Delta hedge) 1. 224 3 × 40 = 48. 972
Lend 5.3596 − 4. 148 9 + 48. 972 = 50. 182 7 −50. 182 7
Total 5.3596 − 4. 148 9 + 48. 972 − 50. 182 7 = 0

www.myactuaryexam.com c
°Yufeng Guo 145
CHAPTER 13. MARKET MAKING AND DELTA HEDGING

One day later, you close out your position:


Transactions (one day later) Revenue
0
Buy a 45-strike put −P1
0
Sell 1. 006 6 unit of 40-strike call 1. 006 6C2
0
Buy 1. 224 3 share of the stock 1. 224 3S
Collect loan 50. 182 7e0.08/365
0 0 0
Total −P1 + 1. 006 6C2 − 1. 224 3S + 50. 182 7e0.08/365
0
P1 is the price of a 45-strike put with 179 day to expiration.
0
C2 is the price of a 40-strike call with 179 day to expiration.
0
S is the price of the stock one day later.

Problem 13.17.
Assume the butterfly spread is asymmetric.
K3 − K2 45 − 40
λ= = = 0.5
K3 − K1 45 − 35

Written butterfly spread consists of selling one 35-strike call, buying two
40-strike calls, and selling one 45-strike call.
Inputs: σ = 0.3 S = 40 r = 8% δ=0 T = 91/365
strike K 35 40 45 butterfly spread
call price 6.1315 2.7804 0.9710 6.1315 − 2 (2.7804) + 0.9710 = 1. 541 7
Delta 0.8642 0.5824 0.2815 0.8642 − 2 (0.5824) + 0.2815 = −0.019 1
Gamma 0.0364 0.0652 0.0563 0.0364 − 2 (0.0652) + 0.0563 = −0.037 7
Theta −0.0134 −0.0173 −0.0134 −0.0134 − 2 (−0.0173) − 0.0134 = 0.007 8

Inputs: σ = 0.3 S = 40 r = 8% δ=0 T = 180/365


strike K 49
call price 4.1217
Delta 0.6151
Gamma 0.0454
Theta −0.0134

Suppose we sell X units of 180 day to expiration call. The total Gamma of
the written butterfly spread and written X units of 180 day to expiration call
is −0.037 7 + 0.0454X.
To Gamma hedge, set −0.037 7 + 0.0454X = 0, X = 0.830 4. The total
Delta of the written butterfly and the written X units of 180 day to expiration
call is −0.019 1 + 0.830 4 × 0.6151 = 0.491 7. So we need to buy 0.491 7 share of
the underlying stock.

Transactions at time zero Cost (negative cost means receiving money)


Sell the spread and buy 0.491 7 stock −1. 541 7 + 0.491 7 × 40 = 18. 126 3
Borrow 18. 126 3 from a bank −18. 126 3
Total 0

www.myactuaryexam.com c
°Yufeng Guo 146
CHAPTER 13. MARKET MAKING AND DELTA HEDGING

One day later, we close out our position:


Transactions (one day later) Revenue
0
Buy a 35-strike 90 day to expiry call −C1
0
Sell two 40-strike 90 day to expiry calls 1. 006 6 2C2
0
Buy a 45-strike 90 day to expiry call −C3
0
Sell 0.491 7 stock 0.491 7S
Repay the borrowed amount −18. 126 3e0.08/365
0 0 0 0
Total −C1 + 2C2 − C3 + 0.491 7S − 18. 126 3e0.08/365

0 0 0 0
So the total overnight profit is −C1 +2C2 −C3 +0.491 7S −18. 126 3e0.08/365 .

Problem 13.18.

inputs: σ = 0.3 S = 40 r = 8% δ=0 T = 91/365


strike K 40 45 ratio spread
put price 1.9905 5.0824 2 × 1.9905 − 5.0824 = −1. 101 4
Delta −0.4176 −0.7185 2 × (−0.4176) − (−0.7185) = −0.116 7
Gamma 0.0652 0.0563 2 × 0.0652 − 0.0563 = 0.074 1
Theta −0.0088 −0.0037 2 × (−0.0088) − (−0.0037) = −0.013 9

in puts: σ = 0.3 S = 40 r = 8% δ=0 T = 180/365


strike K 40
call price 4.1217
Delta 0.6151
Gamma 0.0454
Theta −0.0134

We buy X unit of 40-strike 180-day to expiration call. The total Gamma


of the ratio spread and bought X unit of 40-strike 180-day to expiration call
is 0.074 1 + 0.0454X. To Gamma hedge, set 0.074 1 + 0.0454X = 0, X = −1.
632 2. So we need to short sell 1. 632 2 shares of the underlying stock.

Transactions at time zero Revenue


Enter the ratio spread and short 1. 632 2 stock 1. 101 4 + 1. 632 2 × 40 = 66. 389 4
Deposit 66. 389 4 in a savings account −66. 389 4
Total 0

One day later, we close out our position:


Transactions (one day later) Revenue
0
Buy two 40-strike 90 day to expiry puts −2P1
0
Sell a 45-strike 90 day to expiry put P2
0
Buy 1. 632 2 stock −1. 632 2S
Receive money from the bank 66. 389 4e0.08/365
0 0 0
Total −2P1 + P2 − 1. 632 2S + 66. 389 4e0.08/365

www.myactuaryexam.com c
°Yufeng Guo 147
CHAPTER 13. MARKET MAKING AND DELTA HEDGING

0 0 0
So the total overnight profit is −2P1 + P2 − 1. 632 2S + 66. 389 4e0.08/365 .
Skip Problem 13.19 and 13.20. Vega hedge and rho hedge are far outside
the scope of the Exam MFE syllabus.

www.myactuaryexam.com c
°Yufeng Guo 148
Chapter 14

Exotic options: I

Problem 14.1.
For n non-negative numbers (such as stock prices) S1 , S2 , ..., Sn , the arith-
metic mean can never be less than the geometric mean:
S1 + S2 + ... + Sn 1/n
≥ (S1 S2 ...Sn )
n
If and only if S1 = S2 = ... = Sn , we have:
S1 + S2 + ... + Sn
= (S1 S2 ...Sn )1/n
n
The proof can be found at Wikipedia:
http://en.wikipedia.org/wiki/Inequality_of_arithmetic_and_geometric_
means
For example, this is the proof for n = 2.
(S1 − S2 )2 ≥ 0
→ S12 + S22 ≥ 2S1 S2
2
(S1 + S2 ) = S12 + S22 + 2S1 S2 ≥ 4S1 S2
µ ¶2
S1 + S2 S1 + S2 √
≥ S1 S2 ≥ S1 S2
2 2

Problem 14.2.
Arithmetic average:
5+4+5+6+5
= 5.0
5
Geometric average:
(5 × 4 × 5 × 6 × 5)1/5 = 4. 959

Ignore the question "What happens to the difference between the two mea-
sures of the averages as the standard deviation of the observations increase?"
This question is vague. I’m not sure what the author is after.

149
CHAPTER 14. EXOTIC OPTIONS: I

Problem 14.3.
√ √
u = e(r−δ)h+σ √h = e(0.08−0)0.5+0.3√0.5 = 1. 286 765 9
u = e(r−δ)h−σ h = e(0.08−0)0.5−0.3 0.5 = 0.841 868 0
e(r−δ)h − d e(0.08−0)0.5 − 0.841 868
πu = = = 0.447 165
u−d 1. 286 766 − 0.841 868
πd = 1 − π u = 1 − 0.447 165 = 0.552 835

t=0 t = 0.5 t=1


Suu = 165.57665
Su = 128.67659
S = 100 Sud = 108.32871
Sd = 84.18680
Sdd = 70.87417

The four arithmetic averages are:


128.67659 + 165.57665
= 147. 126 62 (The path is uu or up up)
2

128.67659 + 108.32871
= 118. 502 65 (ud)
2
84.18680 + 108.32871
= 96. 257 8 (du)
2

84.18680 + 70.87417
= 77. 530 485 (dd)
2

Path Average asset Call Payoff Risk Neutral Prob


uu 147. 126 62 47. 126 62 π 2u
ud 118. 502 65 18. 502 65 πu πd
du 96. 257 8 0 πu πd
dd 77. 530 485 0 π 2d

The price of the Asian arithmetic average price call:


e−0.08 (47. 126 62 × 0.447 1652 + 18. 502 65 × 0.447 165 × 0.552 835
+0 × 0.447 165 × 0.552 835 + 0 × 0.552 8352 ) = 12. 921

If we were to calculate the price of the Asian arithmetic average asset put,
then
Path Average asset Put Payoff Risk Neutral Prob
uu 147. 126 62 0 π2u
ud 118. 502 65 0 πu πd
du 96. 257 8 3. 742 2 πu πd
dd 77. 530 5 22. 469 5 π2d

The price of the Asian arithmetic average price put:

www.myactuaryexam.com c
°Yufeng Guo 150
CHAPTER 14. EXOTIC OPTIONS: I

e−0.08 (0 × 0.447 1652 + 0 × 0.447 165 × 0.552 835


+3. 742 2 × 0.447 165 × 0.552 835 + 22. 469 5 × 0.552 8352 = 7. 193

The
√ four geometric averages are:
128.67659 × 165.57665 = 145. 965 2 (uu)


128.67659 × 108.32871 = 118. 065 1 (ud)

84.18680 × 108.32871 = 95. 497 9 (du)

84.18680 × 70.87417 = 77. 244 2 (dd)

Path Average asset Call Payoff Risk Neutral Prob


uu 145. 965 2 45. 965 2 π2u
ud 118. 065 1 18. 065 1 πu πd
du 95. 497 9 0 πu πd
dd 77. 244 2 0 π2d

The price of the Asian geometric average price call:


e−0.08 (45. 965 2 × 0.447 1652 + 18. 065 1 × 0.447 165 × 0.552 835
+0 × 0.447 165 × 0.552 835 + 0 × 0.552 8352 ) = 12. 607

If we were asked to calculate the price of the Asian average price put, then:
Path Average asset Call Payoff Risk Neutral Prob
uu 145. 965 2 0 π2u
ud 118. 065 1 0 πu πd
du 95. 497 9 4. 502 1 πu πd
dd 77. 244 2 22. 755 8 π2d

The price of the Asian geometric average price put:


e−0.08 (0 × 0.447 1652 + 0 × 0.447 165 × 0.552 835
+4. 502 1 × 0.447 165 × 0.552 835 + 22. 755 8 × 0.552 8352 ) = 7. 447

Problem 14.4.
a. Asian arithmetic average strike call:
Path ST K Call Payoff Risk Neutral Prob
uu 165.57665 147. 126 62 18. 450 03 π 2u
ud 108.32871 118. 502 65 0 πuπd
du 108.32871 96. 257 8 12. 070 91 πuπd
dd 70.87417 77. 530 485 0 π 2d

The price of the Asian arithmetic average strike call:


e−0.08 (18. 450 03 × 0.447 1652 + 0 × 0.447 165 × 0.552 835
+12. 070 91 × 0.447 165 × 0.552 835 + 0 × 0.552 8352 ) = 6. 160

www.myactuaryexam.com c
°Yufeng Guo 151
CHAPTER 14. EXOTIC OPTIONS: I

If we were to calculate the price of the Asian arithmetic average strike put,
then
Path ST K Put Payoff Risk Neutral Prob
uu 165.57665 147. 126 62 0 π 2u
ud 108.32871 118. 502 65 10. 173 94 πu πd
du 108.32871 96. 257 8 0 πu πd
dd 70.87417 77. 530 485 76. 656 315 π 2d

The price of the Asian arithmetic average strike put:


e−0.08 (0 × 0.447 1652 + 10. 173 94 × 0.447 165 × 0.552 835
+0 × 0.447 165 × 0.552 835 + 6. 656 315 × 0.552 8352 ) = 4. 20

b. The price of the Asian geometric average strike call:


Path ST K Call Payoff Risk Neutral Prob
uu 165.57665 145. 965 2 19. 611 45 π2u
ud 108.32871 118. 065 1 0 πu πd
du 108.32871 95. 497 9 12. 830 81 πu πd
dd 70.87417 77. 244 2 0 π2d

The price of the Asian geometric average strike call:


e−0.08 (19. 611 45 × 0.447 1652 + 0 × 0.447 165 × 0.552 835
+12. 830 81 × 0.447 165 × 0.552 835 + 0 × 0.552 8352 ) = 6. 548

If we were to calculate the price of the Asian geometric average strike put,
then
Path ST K Put Payoff Risk Neutral Prob
uu 165.57665 145. 965 2 0 π 2u
ud 108.32871 118. 065 1 9. 736 39 πu πd
du 108.32871 95. 497 9 0 πu πd
dd 70.87417 77. 244 2 6. 370 03 π 2d

The price of the Asian geometric average strike put:


e−0.08 (0 × 0.447 1652 + 9. 736 39 × 0.447 165 × 0.552 835
+0 × 0.447 165 × 0.552 835 + 6. 370 03 × 0.552 8352 ) = 4. 019

Problem 14.5.

Stock price tree:


182.14179
165.57665
128.67659 128.814749
100 108.32871
84.18680 91.10067
70.87417
64.42843

www.myactuaryexam.com c
°Yufeng Guo 152
CHAPTER 14. EXOTIC OPTIONS: I

Calculate the price of the arithmetic average asset call.


Path Arithmetic Average Payoff Risk Neutral Prob
uuu 151.1369 51.1369 π 3u = 0.0953
uud 133.3612 33.3612 π 2u π d = 0.1133
udu 118.8058 18.8058 π 2u π d = 0.1133
udd 106.2345 6.2345 π u π 2d = 0.1348
duu 106.8874 6.8874 π 2u π d = 0.1133
dud 94.3160 0 π u π 2d = 0.1348
ddu 84.0221 0 π u π 2d = 0.1348
ddd 75.1314 0 π 3d = 0.1603
Total 1

The price of the arithmetic average asset call is:


e−0.08 (51.1369 × 0.0953 + 33.3612 × 0.1133
+18.8058 × 0.1133 + 6.2345 × 0.1348 + 6.8874 × 0.1133) = 11. 45

Calculate the price of the geometric average asset call.


Path Arithmetic Average Payoff Risk Neutral Prob
uuu 149.1442 49.1442 π 3u = 0.0953
uud 132.8796 32.8796 π 2u π d = 0.1133
udu 118.3887 18.3887 π 2u π d = 0.1133
udd 105.4781 5.4781 π u π 2d = 0.1348
duu 105.4781 5.4781 π 2u π d = 0.1133
dud 93.9754 0 π u π 2d = 0.1348
ddu 83.7272 0 π u π 2d = 0.1348
ddd 74.5965 0 π 3d = 0.1603
Total 1

The price of the geometric average asset call is:


e−0.08 (49.1442 × 0.0953 + 32.8796 × 0.1133
+18.3887 × 0.1133 + 5.4781 × 0.1348 + 5.4781 × 0.1133) = 10. 94

Problem 14.6.
a. Using the Black-Scholes formula, we find that the call price is $4.1293.

b and c.
knock-in call + knock-out call = ordinary call
However, the knock-out call is worthless because the barrier 44 is lower than
the strike price 45. For the call to be worth something, the stock price must
exceed the strike price 45. However, as soon as the stock reaches the barrier 44,
the call is knocked out dead (i.e. the call doesn’t exist any more). Hence this
knock-out call will never have a positive payoff.
Hence the knock-in call becomes the ordinary call. The premium of the
knock-in call is also $4.1293.
Key point to remember:

www.myactuaryexam.com c
°Yufeng Guo 153
CHAPTER 14. EXOTIC OPTIONS: I

1. A knock-out call is worthless if the barrier is less than or equal to the


strike price.

2. A knock-in call is just an ordinary call if the barrier is less than or equal
to the strike price.

Problem 14.7.

DM Chapter 14 doesn’t have any formula on the price of a barrier option. To


calculate the barrier option price, you have to use the textbook’s spreadsheet.
So this problem is out of the scope of Exam MFE. I used the textbook’s
Excel spreadsheet and calculated the following:

T BS DO BS/DO
0.25 0.9744 0.7323 1.3306
0.5 2.1304 1.2482 1.7067
1 4.1293 1.8217 2.2667
2 7.4398 2.4505 3.0360
3 10.2365 2.8529 3.5881
4 12.6969 3.1559 4.0232
5 14.9010 3.4003 4.3823
100 39.9861 5.3112 7.5286

BS: the price of a standard option using the Black-Scholes formula.


DO: Down-and-out barrier option. Since the current stock price 40 exceeds
the barrier 38, the knock-out call with 38 barrier is really a down-and-out option
(if the stock price drops to 38, the call is dead).
The greater the expiration, the greater the ratio of BS/DO. As the expira-
tion increases, both the standard option and the down-and-out option become
more valuable. However, as T increases, the value of a standard option goes up
much faster than a down-and-out option.
The value of a down-and-out option has a moderate increase. As T increase,
the stock price may be more volatile, increasing the value of the barrier option.
However, there’s also more chance that the stock price may hit the barrier.

Problem 14.8.

Out of the scope of MFE. However, I used the textbook spreadsheet and
found the following:

www.myactuaryexam.com c
°Yufeng Guo 154
CHAPTER 14. EXOTIC OPTIONS: I

T BS UO BS/U O
0.25 5.0833 3.8661 1.3149
0.5 5.3659 3.4062 1.5753
1 5.6696 2.8626 1.9806
2 5.7862 2.2233 2.6026
3 5.6347 1.8109 3.1115
4 5.3736 1.5094 3.5601
5 5.0654 1.2761 3.9695
100 0.0012 0.0001 24.4960
As T increases, the BS/U O increases too (just like in Problem 14.7). How-
ever, if T is too big, the value of a standard put and the value of an up-and-out
put both approach zero. If T is too big, we lose the time value of money (the
money we spent in buying a put could have been invested elsewhere and earned
lot of interest).

Problem 14.9.

I used the spreadsheet and calculated the following (T is expressed in months):


T BS UO BS/UO
1 0.1727 0.1727 1.0003
2 0.5641 0.5479 1.0296
3 0.9744 0.8546 1.1401
4 1.3741 1.0384 1.3234
5 1.7593 1.1243 1.5649
6 2.1304 1.1468 1.8577
7 2.4886 1.1316 2.1991
8 2.8353 1.0954 2.5885
9 3.1718 1.0482 3.0260
10 3.4991 0.9962 3.5124
11 3.8180 0.9430 4.0488
12 4.1293 0.8906 4.6365

As T goes up, the BS/UO ratio goes up too.

Problem 14.10.

With K = 0.9, the standard put is worth 0.0188. With barrier 1 or 1.05, the
up-and-out barrier is also worth 0.0188. Why? DM page 452 and 453 have an
explanation. Here is the main point. If the exchange rate never hits 1, then the
standard put and the up-and-out put with 1 or 1.5 barrier have the same value.
The only way that the standard put is more valuable than the up-and-out put
is when the exchange rate goes up from 0.9 to 1 or to 1.05 and then falls below
0.9, in which case the up-and-out is dead yet the standard put has a positive
payoff of K − ST = 0.9 − ST . However, such a scenario is rare and 0.9 − ST is

www.myactuaryexam.com c
°Yufeng Guo 155
CHAPTER 14. EXOTIC OPTIONS: I

small. Hence the standard put and the up-and-out put have roughly the same
value.
In contrast, when the strike price K = 1, the payoff of the standard put at
the above mentioned scenario is 1 − ST , which is greater than 0.9 − ST . Hence
the standard put is slightly more valuable than the up-and-out put.

Problem 14.11.
a.Using the Black-Scholes formula, we find the call premium is $9.6099.

b. If we buy the compound option, then we can, at t1 = 1, buy, for a


guaranteed price of x = 2, a call that expires at T = 2. We’ll exercise the
compound option at t1 = 1 only if the stock price at t = 1 is such that the call
we are entitled to buy is equal to or greater than the premium 2.
We’ll use the textbook Equation 14.11 to find S ∗ , the minimum price such
that exercising the compound option is worthwhile.
C (S ∗ , K, T − t1 ) = x
C (S ∗ , 40, 2 − 1) = 2
C (S ∗ , 40, 1) = 2
To solve for S ∗ , we have use the trial-and-error approach. I found that

S = 31.723.
In other words, if the stock price at t1 = 1 is 31.723, then a call option
written at t1 = 1 and expires at T = 2 is exactly worth $2.In this case, we’ll not
exercise the compound option. We’ll let the compound option expire worthless.
If the stock price at t1 = 1 is less than 31.723, then the call written at t1 = 1
and expires at T = 2 is worth less than $2 and we’ll not exercise the compound
option. We’ll let the compound option expire worthless.
Only if the stock price at t1 = 1 is greater than 31.723 should we exercise
the compound option.
c. To find the price of the compound call (i.e. call on call), we’ll use worksheet
called "Compound" provided by the DM textbook.
The inputs are:
Stock Price 40
Exercise Price to buy asset 40
Exercise Price to buy option 2
Volatility 30%
Risk-free interest rate 8%
Expiration for Option on Option (years) 1
Expiration for Underlying Option (years) 2
Dividend Yield 0%

Compound Option Prices


Call on Call 7.9482
Put on Call 0.1845
Call on Put 2.2978

www.myactuaryexam.com c
°Yufeng Guo 156
CHAPTER 14. EXOTIC OPTIONS: I

Put on Put 0.4484


Critical S for compound call 31.723
Critical S for compound put 44.3494
So the compound call premium is 7.9482. (From the output, we see that the
critical stock price for the compound call is 31.723, which matches our answer
in part b).
d. From Part c, we see that the price of the put on call is 0.1845.
Alternatively, we can use DM 14.12 to calculate the put on the call, given
we know the price of the call on call.
CallOnCall − P utOnCall + xe−rt1 = BSCall

7.9482 − P utOnCall + 2e−0.08×1 = 9.6099


P utOnCall = 0.184 5

Problem 14.12.

a. Use the Black-Scholes formula, we find the put price is 3.6956.


b. Use the "Compound" worksheet.
Inputs
Stock Price 40
Exercise Price to buy asset 40
Exercise Price to buy option 2
Volatility 30%
Risk-free interest rate 8%
Expiration for Option on Option (years) 1
Expiration for Underlying Option (years) 2
Dividend Yield 0%

Outputs:
Call on Call 7.9482
Put on Call 0.1845
Call on Put 2.2978
Put on Put 0.4484
Critical S for compound call 31.723
Critical S for compound put 44.3494

So if S1 < 44.3494, we’ll exercise the put at t = 1.

c. The put on put price is 0.4484.

Problem 14.13.

a. DM 14.15 gives you the price formula for a gap call. A gap put formula
is
P (K1 , K2 ) = K1 e−rT N (−d2 ) − Se−δT N (−d1 )

www.myactuaryexam.com c
°Yufeng Guo 157
CHAPTER 14. EXOTIC OPTIONS: I

µ ¶
S 1
ln + r − δ + σ2 T
K2 2
d1 = √
√ σ T
d2 = d1 − σ T
For foreign currency, δ = r€ , r = r$ , and S = x0 .

b. The gap put payoff is 0.8 − x if x < 1. If σ = 0, then x = x0 = 0.9. The


gap out payoff is −0.1. The gap put premium is 0.8e−0.06×0.5 − 0.9e−0.03×0.5 =
−0.110 2.
As the volatility increases, the value of the gap put increases.

Problem 14.14.

Skip.

Problem 14.15.

Skip.

Problem 14.16.

Price of a standard 40-strike call on S. The inputs to the Black Scholes


formulas are:
S = K = 40 σ = 0.3 r = 0.08 T =1 δ=0
The call price is 6.2845.

The price of an exchange option with S as underlying and 0.667Q as the


strike asset is 7. 577.
S =√ 40 K = 0.667 × 60 = 40
σ = 0.32 + 0.52 − 2 × 0.5 × 0.3 × 0.5 = 0.435 89
40e−0×1 1
ln −0.04×1
+ × 0.435 892 × 1
d1 = 40e 2 √ = 0.309 7
0.435 89 1

d2 = 0.309 7 − 0.435 89 1 = −0.126 2
N (d1 ) = 0.621 6 N (d2 ) = 0.449 8
C = 40e−0×1 × 0.621 6 − 40e−0.04×1 × 0.449 8 = 7. 577

Problem 14.17.

a. Inputs to the exchange call formula:


S = 40 δS = 0 σ S = 0.3
K = 60 δK = 0 σ K = 0.5
ρ = 0.5 T =1

www.myactuaryexam.com c
°Yufeng Guo 158
CHAPTER 14. EXOTIC OPTIONS: I

You should get: C = 2. If you change δ S = 0.1, then the call price is
C = 1.2178

In the replicating portfolio, the number of underlying stocks to hold at time


zero is e−δS T . If we increase δ S , then we hold fewer shares of the underlying
asset and the call price drops.

b. inputs to the exchange call formula:


S = 40 δS = 0 σ S = 0.3
K = 60 δK = 0 σ K = 0.5
ρ = 0.5 T =1
Once again, C = 2.
If δ K = 0.1, then C = 2.86.
If δ K goes up, then the present value of the strike price goes down, making
the call more valuable.

c. inputs to the exchange call formula:


S = 40 δS = 0 σ S = 0.3
K = 60 δK = 0 σ K = 0.5
ρ = 0.5 T =1

Once again, C = 2.
If ρ = −0.5, then C = 5.79.
p
If ρ is negative, then σ = σ 2S + σ 2K − 2ρσ S σ K is higher then if ρ is positive.
So a negative ρ increases σ, making an option more valuable.

Problem 14.18.
ρ = 1p √
σ = σ 2S + σ 2K − 2ρσ S σ K = 0.32 + 0.32 − 2 × 1 × 0.3 × 0.3 = 0
a. The call price is zero. Because V ar [ln (S/K)] = σ 2S + σ 2K − 2ρσ S σ K = 0,
S/K is a constant during [0, T ]. Since at time zero S = K, then S = K during
[0, T ]. So the payoff of the call is zero and the call is worthless.
p √
b. Now σ = σ 2S + σ 2K − 2ρσS σK = 0.42 + 0.32 − 2 × 1 × 0.4 × 0.3 =
0.1. Using DM 14.16, we find that the exchange call premium is 1.60.
Skip the remaining problems (Problem 14.19 through 14.22).

www.myactuaryexam.com c
°Yufeng Guo 159
CHAPTER 14. EXOTIC OPTIONS: I

www.myactuaryexam.com c
°Yufeng Guo 160
Chapter 20

Brownian motion and Ito’s


lemma

Problem 20.1.
I don’t want you to memorize Ito’s lemma. For problems related to It’s
lemma, all you need to know is two things:
2
1. Unlike a deterministic random variable X where (dX) = 0, for a sto-
chastic random variable Z, the term (dZ)2 is not zero and hence can’t be
ignored. In fact, the textbook and my study guide have explained that
2 3
(dZ) = dt. However, you can ignore higher order such as (dZ) .
2. Ito’s lemma is just the stochastic counterpart of the Taylor series. To
derive Ito’s lemma, first write the Taylor series. For a stochastic random
variable X and a function y = f (t, X), first write the Taylor expansion:
∂f ∂f 1 ∂2f 1 ∂2f
dy = d f (t, X) = dt+ dX + 2
(dt)2 + (dX)2 +....Here we
∂t ∂X 2 ∂t 2 ∂X 2
3 3
ignored the higher order terms (dt) , (dX) , and above. Next, throw away
the term (dt)2 since t is a deterministic random variable and (dt)2 → 0.
∂f ∂f 1 ∂2f
Now we have dy = d f (t, X) = dt + dX + (dX)2 . This is
∂t ∂X 2 ∂X 2
Ito’s lemma.
With this point in mind, let’s solve the problem.

a. If the stock price S follows the textbook Equation 20.8, then:

dS (t) = αdt + σdZ (t) (DM 20.8)

dS (t) is a linear function of dZ (t). Since [dZ (t)]2 = dt, we should keep
2
(dS) in the Taylor expansion but throw away all other higher order terms:

161
CHAPTER 20. BROWNIAN MOTION AND ITO’S LEMMA

∂ ln S ∂ ln S 1 ∂ 2 ln S
→ d ln S = dt + dS + (dS)2
∂t ∂S 2 ∂S 2
∂ ln S ∂ ln S 1
Since ln S doesn’t contain t, = 0. In addition, = and
∂t ∂S S
2
∂ ln S ∂ 1 1
= =− 2
∂S 2 ∂S S S
∂ ln S 1 ∂ 2 ln S 2 1 1 1 2
→ d ln S = dS + 2
(dS) = dS − (dS)
∂S 2 ∂S S 2 S2

(dS)2 = (αdt + σdZ)2


= α2 (dt)2 + 2ασ (dZ) (dt) + σ 2 (dZ)2
Next, use the multiplication rule:

dZ × dt = 0 (DM 20.17a)
2
(dt) = 0 (DM 20.17b)

2
(dZ) = dt (DM 20.17c)
2 2 2
→ (dS) = (αdt + σdZ) = σ 2 (dZ) = σ 2 dt
1 1 1 2
→ d ln S = (αdt + σdZ) − σ dt
µ S ¶ 2 S2
1 1 1 2 σ
= α− σ dt + dZ
S 2 S2 S

b.If the stock price S follows the textbook Equation 20.9, then:

dS (t) = λ (α − S) dt + σdZ (DM 20.9)


∂ ln S 1 ∂ 2 ln S 2 1 1 1 2
→ d ln S = dS + (dS) = dS − (dS)
∂S 2 ∂S 2 S 2 S2

(dS)2 = [λ (α − S) dt + σdZ]2
= λ2 (α − S)2 (dt)2 + 2λ (α − S) dt × σdZ + σ2 (dZ)2 = σ 2 dt
1 1 1 2
→ d ln S = dS − (dS)
S 2 S2
λ (α − S) dt + σdZ 1 σ 2 dt
= −
∙ S ¸ 2 S2
λ (α − S) 1 σ 2 σ
= − dt + dZ
S 2 S2 S

c.If the stock price S follows the textbook Equation 20.27, then:
∙ ¸
∧ ∧ ∧
dS (t) = α (S, t) − δ (S, t) dt + σ (S, t) dZ (t) (DM 20.8)

When S (t) follows a geometric Brownian motion,

www.myactuaryexam.com c
°Yufeng Guo 162
CHAPTER 20. BROWNIAN MOTION AND ITO’S LEMMA

∧ ∧ ∧
α (S, t) = αS (t) δ (S, t) = δS (t) σ (S, t) = σS (t)
Hence DM Equation 20.8 becomes:

dS (t) = S (α − δ) dt + σSdZ (t) (20.1)


∂ ln S 1 ∂ 2 ln S 2 1 1 1 2
d ln S = dS + (dS) = dS − (dS)
∂S 2 ∂S 2 S 2 S2

(dS)2 = [S (α − δ) dt + σSdZ (t)]2 = σ 2 S 2 (dZ)2 = σ 2 S 2 dt


1 1 1
→ d ln S = dS − (dS)2
S 2 S2
S (α − δ) dt + σSdZ (t) 1 σ2 S 2 dt
= −
µ S ¶ 2 S2
1 2
= α − δ − σ dt + σdZ
2

Problem 20.2.
∂S 2 ∂S 2 1 ∂2S2 2
dS 2 = dt + dS + (dS)
∂t ∂S 2 ∂S 2
∂S 2 1 ∂2S2
= dS + (dS)2 = 2SdS + (dS)2
∂S 2 ∂S 2
a. Under DM Equation 20.8:
dS (t) = αdt + σdZ (t)
(dS)2 = σ 2 dt
→ dS 2 = 2SdS + (dS)2 ¡ ¢
= 2S (αdt + σdZ) + σ 2 dt = 2αS + σ 2 dt + 2σSdZ

b. Under DM Equation 20.9:


dS (t) = λ (α − S) dt + σdZ
(dS)2 = σ 2 dt
2
→ dS 2 = 2SdS + (dS)
= £2S [λ (α − S) dt + ¤σdZ] + σ 2 dt
= 2Sλ (α − S) + σ 2 dt + 2SσdZ

c. Under Geometric Brownian motion


dS (t) = S (α − δ) dt + σSdZ (t)
(dS)2 = σ 2 S 2 dt
→ dS 2 = 2SdS + (dS)2
= 2S [S (α − δ) dt + σSdZ] + σ 2 S 2 dt
= 2S 2£ [(α − δ) dt + σdZ]
¤ + σ2 S 2 dt
= S 2 (α − δ) + σ dt + 2σS 2 dZ
2 2

Problem 20.3.

www.myactuaryexam.com c
°Yufeng Guo 163
CHAPTER 20. BROWNIAN MOTION AND ITO’S LEMMA

∂S −1 ∂S −1 1 ∂ 2 S −1
dS −1 = dt + dS + (dS)2
∂t ∂S 2 ∂S 2
∂S −1 ∂S −1 ∂ 2 S −1
=0 = −S −2 = 2S −3
∂t ∂S ∂S 2
→ dS −1 = −S −2 dS + S −3 (dS)2
a. Under DM Equation 20.8:

dS (t) = αdt + σdZ (t)


(dS)2 = σ 2 dt
2
→ dS −1 = −S −2 dS + S −3 (dS)
−2 −3 2
= −S
¡ (αdt + σdZ)¢ + S σ dt
= −S −2 α + S −3 σ 2 dt − S −2 σdZ

b. Under DM Equation 20.9:


dS (t) = λ (α − S) dt + σdZ
(dS)2 = σ 2 dt
2
→ dS −1 = −S −2 dS + S −3 (dS)
= ¡−S −2 (λ (α − S) dt + σdZ) −3 2
¢ + S −2
σ dt
−2 −3 2
= −S λ (α − S) + S σ dt − S σdZ

c. Under Geometric Brownian motion


dS (t) = S (α − δ) dt + σSdZ (t)
(dS)2 = σ 2 S 2 dt
2
→ dS −1 = −S −2 dS + S −3 (dS)
= −S [S (α − δ) dt + σSdZ (t)] + S −3 σ 2 S 2 dt
−2

= −S −1£ [(α − δ) dt + 2σdZ


¤ (t)] + S −1 σ 2 dt
−1 −1
=S − (α − δ) + σ dt − S σdZ (t)

Problem 20.4.
∂S 0.5 ∂S 0.5 1 ∂ 2 S 0.5 2
dS 0.5 = dt + dS + (dS)
∂t ∂S 2 ∂S 2
∂S 0.5 ∂S 0.5 ∂ 2 S 0.5
=0 = 0.5S −0.5 = −0.25S −1.5
∂t ∂S ∂S 2
→ dS 0.5 = 0.5S −0.5 dS − 0.125S −1.5 (dS)2
a. Under DM Equation 20.8:

dS (t) = αdt + σdZ (t)


(dS)2 = σ 2 dt
→ dS 0.5 = 0.5S −0.5 [αdt + σdZ (t)] − 0.125S −1.5 σ 2 dt

b. Under DM Equation 20.9:


dS (t) = λ (α − S) dt + σdZ
2
(dS) = σ 2 dt
→ dS 0.5 = 0.5S −0.5 [λ (α − S) dt + σdZ] − 0.125S −1.5 σ 2 dt

www.myactuaryexam.com c
°Yufeng Guo 164
CHAPTER 20. BROWNIAN MOTION AND ITO’S LEMMA

c. Under Geometric Brownian motion


dS (t) = S (α − δ) dt + σSdZ (t)
(dS)2 = σ 2 S 2 dt
→ dS 0.5 = 0.5S −0.5 [S (α − δ) dt + σSdZ (t)] − 0.125S −1.5 σ 2 S 2 dt
= 0.5S 0.5 [(α − δ) dt + σdZ (t)] − 0.125S 0.5 σ 2 dt

Problem 20.5.

dS (t) = S (αs − δ s ) dt + σ s SdZS (DM 20.37)

dQ (t) = Q (αQ − δ Q ) dt + σ Q QdZQ (DM 20.38)

dSdQ = ρdt (DM 20.17d)


¡ 2 0.5 ¢ ¡ 2 0.5 ¢ 2
¡ 2 0.5 ¢ ¡ ¢
¡ 2 0.5 ¢ ∂ S Q ∂ S Q ∂ S Q 1 ∂ 2 S 2 Q0.5
d S Q = dS+ dQ+ dSdQ+ (dS)2 +
¡ ¢ ∂S ∂Q ∂S∂Q 2 ∂S 2
1 ∂ 2 S 2 Q0.5
(∂Q)2
2 ∂Q2
¡ ¢ ¡ ¢ ¡ ¢
∂ S 2 Q0.5 ∂ S 2 Q0.5 ∂ 2 S 2 Q0.5
= 2SQ0.5 = 0.5Q−0.5 S 2 =
∂S ∂Q ∂S∂Q
SQ−0.5
¡ ¢ ¡ ¢
∂ 2 S 2 Q0.5 0.5
∂ 2 S 2 Q0.5
= 2Q = −0.25S 2 Q−1. 5
∂S 2 ∂Q2
¡ ¢
→ d S 2 Q0.5 =
2SQ0.5 [S (αs − δ s ) dt + σ s SdZS ]
+0.5Q−0.5 S 2 [Q (αQ − δ Q ) dt + σ Q QdZQ ]
+SQ−0.5 ρdt
+Q0.5 (σ Q Q)2 dt
2
−0.125S 2 Q−1. 5 [Q (αQ − δ Q ) dt + σ Q QdZQ ]

Problem 20.6.
From Problem
µ 20.1.c, we ¶
have:
1 2
d ln S = αS − δ S − σ S dt + σ S dZS
µ 2 ¶
1
d ln Q = αQ − δ Q − σ2Q dt + σQ dZQ
2 µ ¶ µ ¶
1 2 1 2
→ d ln (SQ) = d ln S+d ln Q+ αS − δ S − σ S dt+σ S dZS + αQ − δ Q − σ Q dt+
2 2
σ Q dZQ

www.myactuaryexam.com c
°Yufeng Guo 165
CHAPTER 20. BROWNIAN MOTION AND ITO’S LEMMA

Problem 20.7.

dS (t)
= (α − δ) dt + σdZ
S (t)
£ ¤ 2
P
F0,T S A (T ) = e−rT S A (0) e[A(r−δ)+0.5A(A−1)σ ]T
Assume T = 1
If A £= 2 ¤ 2
P
F0,1 S 2 (1) = e−0.06×1 1002 e(2(0.06−0)+0.5×2(2−1)0.4 )1 = 12460. 77

If A £= 0.5 ¤ 2
P
F0,1 S 0.5 (1) = e−0.06×1 1000.5 e(0.5(0.06−0)+0.5×0.5(0.5−1)0.4 )1 = 9. 51

If A £= −2 ¤ 2
P
F0,1 S −2 (1) = e−0.06×1 100−2 e(−2(0.06−0)+0.5(−2)(−2−1)0.4 )1 = 1. 35 × 10−4

The textbook asks you to compare your solution to the solution to Problem
19.7. However, Problem 19.7 is out of the scope of Exam MFE. So you don’t to
do such a comparison.
Skip the remaining problems (Problem 20.8 and beyond); they are out of
the scope of Exam MFE.

www.myactuaryexam.com c
°Yufeng Guo 166
Chapter 24

Interest rate models

Problem 24.1.
a. We need to find the price of a 1-year bond issued at t = 1. The price of
this bond is just the PV of $1 discounted from t = 2 to t = 1.
P (0, 2)
P (1, 2) =
P (0, 1)
To understand this formula, notice that P (0, 2) = P (1, 2) P (0, 1). This
equation means that to calculate the PV of $1 discounted from t = 2 to t = 0,
we first discount $1 from t = 2 to t = 1 and next discount it from t = 1 to t = 0.
P (0, 2) 0.8495
P (1, 2) = = = 0.917 485 7
P (0, 1) 0.9259

b. C =Time zero cost of what you get at T ×N (d1 ) −Time zero cost of
what you
³ give at T ×N (d2 ) ´ √
Time zero cost of what you get at T 2
d1 = ln Time zero cost of what you give at T + 0.5σ T /σ T

d2 = d1 − σ T
Make sure you know the formula for σ. DM page 790 shows that
p
σ = V ar (ln (Ft,T (P [T, T + s])))
If you buy this option, then at T = 1, you can pay K = 0.9009 and buy a
1-year bond. This 1-year bond will give you $1 at time T + s = 2.

• The value of this bond at T = 0 is P (0, 2) = 0.8495, the PV of $1


discounted from T + s = 2 to T = 1
• Time zero cost of the strike price K at T = 1 is just PV of K discounted
from T = 1 to time zero. So P V (K) = 0.9009 × 0.9259

C =Time zero cost of what you get at T ×N (d1 ) −Time zero cost of what
you give at T ×N (d2 )

167
CHAPTER 24. INTEREST RATE MODELS

0.8495 2
ln 0.9009×0.9259 +0.5×0.1 ×1
→ d1 = √ = 0.232 43
√ 0.1 1 √
→ d2 = d1 − σ T = 0.232 43 − 0.1 1 = 0.132 43

N (d1 ) = 0.591 90 N (d2 ) = 0.552 68


C = 0.8495 × 0.591 90 − 0.9009 × 0.9259 × 0.552 68 = 0.0418

c.
P =Time zero cost of what you give at T ×N (−d2 ) −Time zero cost of what
you get at T ×N (−d1 )
P = 0.9009 × 0.9259 × (1 − 0.552 68) − 0.8495 × (1 − 0.591 90) = 0.0264

Alternative calculation using the put-call parity.


C + P (0, T ) K = P + P (0, T + s)
0.0418 + 0.9009 × 0.9259 = P + 0.8495
P = 0.0264

d. Let’s walk through the notations and formula.


Notation

• RT =1 (T = 1, T + s = 2), the (not annualized) interest rate agreed upon


at time T = 1 that applies to the time interval [T = 1, T + s = 2].

• Caplet. A Caplet gives the buyer the right to buy the time-T = 1 mar-
ket interest rate RT (T, T + s) = RT =1 (T = 1, T + s = 2) by paying a
fixed strike interest rate KR = 11%. If KR ≥ RT =1 (T = 1, T + s = 2),
the caplet expires worthless. The payoff of the caplet at T + s = 2 is
max [0, RT =1 (T = 1, T + s = 2) − 0.11]. The payoff of the caplet at T is
max [0, RT =1 (T = 1, T + s = 2) − 0.11]
RT =1 (T = 1, T + s = 2)

Please note that the actual interest rate during [T = 1, T + s = 2] is a ran-


dom variable. One might be tempted to think that we already know the Year 2
interest rate as:
1
− 1 = 8. 993 5%
P (1, 2)
However, this thinking is flawed. 8. 993 5% is the implied Yr 2 interest rate
based on the information available to us at t = 0. This rate can be different
from the Year 2 spot rate.

To calculate the price of the caplet, we first modify ∙the payoff: ¸


max [0, RT =1 (T = 1, T + s = 2) − 0.11] RT =1 (T = 1, T + s = 2) − 1.11
= 1.11 max 0, =
∙ RT =1 (T = 1, T + s = 2) ¸ RT =1 (T = 1, T + s = 2) 1.11
1 1
1.11 max 0, −
1.11 RT =1 (T = 1, T + s = 2)

www.myactuaryexam.com c
°Yufeng Guo 168
CHAPTER 24. INTEREST RATE MODELS

∙ ¸ ∙ ¸
1 1 1
max 0, − = max 0, 0.900 9 −
1.11 RT =1 (T = 1, T + s = 2) RT =1 (T = 1, T + s = 2)
is the payoff of a put on a bond. This put gives the buyer the right, at T = 1,
1
to sell a bond that matures at T + s = 2 for a guaranteed price = 0.900 9.
1.11
From Part c, we already know that this put price is 0.0264. Hence the caplet
price is:
1.11 × 0.0264 = 0.02 93

Problem 24.2.
P (0, 3) 0.7722
a. P (2, 3) = = = 0.909 01
P (0, 2) 0.8495

b. C = P (0, T + s) N (d1 ) − P (0, T ) KN (d2 )


P (0, T + s) = P (0, 2 + 1) = P (0, 3) = 0.7722
P (0, T ) = P (0, 2) = 0.8495

→ C = 0.7722N (d1 ) − 0.8495 × 0.9N (d2 )

P (0, T + s) 0.7722
ln + 0.5σ2 T ln + 0.5 × 0.1052 × 2
P (0, T ) K 0.8495 × 0.9
d1 = √ = √ = 0.141 29
σ T 0.105 × 2
√ √
d2 = d1 − σ T = 0.141 29 − 0.105 × 2 = −0.007 2
N (d1 ) = 0.556 18
N (d2 ) = 0.497 13

→ C = 0.7722 × 0.556 18 − 0.8495 × 0.9 × 0.497 13 = 0.04 94

c. P = P (0, T ) KN (−d2 ) − P (0, T + s) N (−d1 )


= 0.8495 × 0.9 × (1 − 0.497 13) − 0.7722 × (1 − 0.556 18)
= 0.04175 1

Alternative calculation using the put-call parity.


C + P (0, T ) K = P + P (0, T + s)
0.04 94 + 0.8495 × 0.9 = P + 0.7722
P = 0.041 75
d. To calculate the price of the caplet, we first modify
∙ the payoff: ¸
max [0, RT =2 (T = 2, T + s = 3) − 0.11] 1 1
= 1.11 max 0, −
∙ RT =2 (T = 2, T + s = 3) ¸ ∙ 1.11 RT =2 (T = 2, T + s = 3)¸
1 1 1
max 0, − = max 0, 0.900 9 −
1.11 RT =2 (T = 2, T + s = 3) RT =2 (T = 2, T + s = 3)
is the payoff of a put on a bond. This put gives the buyer the right, at T = 2,
1
to sell a bond that matures at T + s = 3 for a guaranteed price = 0.900 9.
1.11

www.myactuaryexam.com c
°Yufeng Guo 169
CHAPTER 24. INTEREST RATE MODELS

1
We can estimate the put price. Since = 0.900 9 is close to 0.9, from Part
1.11
c we know that P = 0.041 75. Hence the price of the caplet is 1.1×0.041 75 = 0.0
459
Or we can calculate the put price.

P = P (0, T ) KN (−d2 ) − P (0, T + s) N (−d1 ) = 0.8495 × 0.900 9N (−d2 ) −


0.7722N (−d1 )
P (0, T + s) 0.7722
ln + 0.5σ 2 T ln + 0.5 × 0.1052 × 2
P (0, T ) K 0.8495 × 0.900 9
d1 = √ = √ =
σ T 0.105 × 2
0.134 56 √ √
d2 = d1 − σ T = 0.134 56 − 0.105 × 2 = −1. 393 2 × 10−2
N (−d1 ) = 0.446 48
N (−d2 ) = 0.505 56
P = 0.8495 × 0.900 9 × 0.505 56 − 0.7722 × 0.446 48 = 0.04214
Hence the price of the caplet is 1.1 × 0.04214 = 0.0464

Problem 24.3.
Make sure you understand that this problem is different from SOA May 2007
#9. In SOA May 2007 #9, the first cash flow occurs at t = 1.The first year
market rate 6% is below the cap rate 7.5%. At the end of Year 1 (at t = 1), we
get nothing from the cap .
In this problem, the author wants us to repeatedly use DM Equation 24.36
to calculate the cap price. So the author wants the first cash flow to occur at
t = 2.
The cap contract is signed at t = 0. During Yr 2, the cap rate 11.5% is com-
pared with the actual Yr 2 interest. The payoff at t = 2 is the max (0, 11.5% − Yr 2 rate).
The PV of this payoff at t = 1 is:
µ ¶
max (0, 11.5% − Yr 2 rate) 1 1
= 1.115 max 0, −
1 + Yr 2 rate 1.115 1 + Yr 2 rate
µ ¶
1 1
max 0, − is the payoff of a put. This put gives the
1.115 1 + Yr 2 rate
buyer the right at T = 1 to buy a one-year bond maturing at T + s = 2. You
can verify that the time zero cost of this put is 0.0248.

P = P (0, 1) KN (−d2 ) − P (0, 2) N (−d1 )


0.8495
P (0, 2) ln + 0.5 × 0.12 × 1
ln 2
+ 0.5σ T 1
P (0, 1) K 0.9259 ×
d1 = √ = 1.115 √ = 0.277 36
σ T 0.1 × 1
√ √
d2 = d1 − σ T = 0.277 36 − 0.1 × 1 = 0.177 36

www.myactuaryexam.com c
°Yufeng Guo 170
CHAPTER 24. INTEREST RATE MODELS

1
P = 0.9259 × N (−0.177 36) − 0.8495N (−0.277 36)
1.115
1
= 0.9259 × × 0.429 61 − 0.8495 × 0.390 75 = 0.0248
1.115
So the time zero cost of the payoff at t = 2 is 1.115 × 0.0248.

Similarly, the time zero cost of the payoff at t = 3 is 1.115 × 0.0404

P = P (0, 2) KN (−d2 ) − P (0, 3) N (−d1 )


The put gives the buyer the right at T = 2 to buy a bond maturing at
T +s=3
0.7722
P (0, 3) ln + 0.5 × 0.1052 × 2
ln 2
+ 0.5σ T 1
P (0, 2) K 0.8495 ×
d1 = √ = 1.115 √ = 0.164 82
σ T 0.105 × 2
√ √
d2 = d1 − σ T = 0.164 82 − 0.105 × 2 = 0.01633
1
P = 0.8495 × N (−0.01633) − 0.7722N (−0.164 82)
1.115
1
= 0.8495 × × 0.493 49 − 0.7722 × 0.434 54 = 0.040 4
1.115
The time zero cost of the payoff at t = 4 is 1.115 × 0.0483

P = P (0, 2) KN (−d2 ) − P (0, 3) N (−d1 )


The put gives the buyer the right at T = 3 to buy a bond maturing at
T +s=4
0.7020
P (0, 4) ln + 0.5 × 0.112 × 3
ln + 0.5σ 2 T 1
P (0, 3) K 0.7722 ×
d1 = √ = 1.115 √ = 0.166 35
σ T 0.11 × 3
√ √
d2 = d1 − σ T = 0.166 35 − 0.11 × 3 = −0.02417
1
P = 0.7722 × N (0.02417) − 0.7020N (−0.166 35)
1.115
1
= 0.7722 × × 0.509 64 − 0.7020 × 0.433 94 = 0.04832 8
1.115
The time zero cost of the cap is:
1.115 × (0.0248 + 0.0404 + 0.0483) = 0.126 6

Problem 24.4.
At time zero, we

• But a 3-year bond. Cost: P (t, T2 ) = e−0.08×3 = 0.786 63


(T2 − t) P (t, T2 ) 1 × e−0.08×3
• To hedge, we buy N = − =− = −0.635 62
(T1 − t) P (t, T1 ) 2 × e−0.08×6
unit of 6-year bond (i.e. sell 0.635 62 unit of 6-year bond), receiving
0.635 62e−0.08×6 = 0.393 31

www.myactuaryexam.com c
°Yufeng Guo 171
CHAPTER 24. INTEREST RATE MODELS

• Borrow 0.786 63 − 0.393 31 = 0.393 32 from a bank at 8% interest rate.

• Our net position is 0.786 63 − (0.393 31 + 0.393 32) = 0

If at the end of the day the risk-free rate is 8.25%, we’ll close off our position.

• Sell the (3 − 1/365)-year bond, receiving e−0.0825×(3−1/365) = 0.780 93

• Buy back the (6 − 1/365)-year bond, paying 0.635 62e−0.0825×(6−1/365) =


0.387 54

• Pay off the loan, paying 0.393 32e0.08×1/365 = 0.393 41 (we borrow the
loan for only one day).

• Our net position: 0.780 93 − 0.387 54 − 0.393 41 = −0.000 02. So we lose


0.000 02

Why do we lose money? Because of the convexity mismatch.


We buy the 3-year bond. The convexity is (see DM 7.14 and DM Example
7.9):
3×4
= 10. 288 07
1.082

We sell 0.635 62 unit of the 6-year bond. The convexity is:


6×7
0.635 62 × = 22. 887 55
1.082
Even though we hedged the duration, we didn’t hedge convexity. The two
portfolios have different convexities. As explained in my study guide, the high-
convexity bond has a better value. To arbitrage, we should have bought the
high-convexity bond and sold the low-convexity bond. So to arbitrage, we need
to reverse our position (i.e. at t = 0, sell one unit of 3-year bond and buy
0.635 62 unit of 6-year bond). Then we’ll earn 0.000 02 free money during Day
1.
How to reverse our position. At time zero, we

• Sell a 3-year bond, receiving P (t, T2 ) = e−0.08×3 = 0.786 63

• To hedge, we buy 0.635 62 unit of 6-year bond, paying 0.635 62e−0.08×6 =


0.393 31

• Lend 0.786 63 − 0.393 31 = 0.393 32 at 8% interest rate.

• Our net position is 0.786 63 − (0.393 31 + 0.393 32) = 0

At the end of the day, the risk-free rate is 8.25%. We close off our position.

• Buy the (3 − 1/365)-year bond, paying e−0.0825×(3−1/365) = 0.780 93

• Sell the (6 − 1/365)-year bond, receiving 0.635 62e−0.0825×(6−1/365) = 0.387 54

www.myactuaryexam.com c
°Yufeng Guo 172
CHAPTER 24. INTEREST RATE MODELS

• Close the loan, receiving 0.393 32e0.08×1/365 = 0.393 41.


• Our net position: 0.387 54 + 0.393 41 − 0.780 93 = 0.000 02. So we gain
0.000 02

At time zero, our cost is zero. At the end of the day, we receive 0.000 02
profit. This is arbitrage.

If at the end of the day the risk-free rate is 7.75%, we’ll close off our position.

• Sell the (3 − 1/365)-year bond, receiving e−0.0775×(3−1/365) = 0.792 72


• Buy back the (6 − 1/365)-year bond, paying 0.635 62e−0.0775×(6−1/365) =
0.399 34
• Pay off the loan, paying 0.393 32e0.08×1/365 = 0.393 41 (we borrow the
loan for only one day).
• Our net position: 0.792 72 − 0.399 34 − 0.393 41 = −0.000 03. So we lose
0.000 03

To arbitrage, we reverse our position and gain 0.000 03 free money.


This example shows that the assumption of the parallel shift of a flat yield
curve leads to arbitrage. To build a good model for bond price, we need to
throw away this bad assumption.

Problem 24.5.
a. 4-year 5% annual
¡ coupon bond with yield 6% (Bond 1)¢
Price: P1 = 0.05 e−0.06×1 + e−0.06×2 + e−0.06×3 + e−0.06×4 + e−0.06×4 =
0.959 16
Macaulay duration:
¡ ¢
0.05 e−0.06×1 + 2e−0.06×2 + 3e−0.06×3 + 4e−0.06×4 + 4e−0.06×4
D1 = = 3.
0.959 16
716 7

8-year 7% annual coupon bond with yield 6% (Bond 2)


Price: ¡ ¢
P2 = 0.07 e−0.06×1 + e−0.06×2 + e−0.06×3 + e−0.06×4 + ... + e−0.06×8 +e−0.06×8 =
1. 050 3
Macaulay duration:
¡ ¢
0.07 e−0.06×1 + 2e−0.06×2 + 3e−0.06×3 + ... + 8e−0.06×8 + 8e−0.06×8
D2 = =
1. 050 3
6. 433 2

b. Use DM 7.13:
D1 B1 (y1 ) (1 + y1 )
N =−
D2 B2 (y2 ) (1 + y2 )

www.myactuaryexam.com c
°Yufeng Guo 173
CHAPTER 24. INTEREST RATE MODELS

Please also note that DM 24.7 is the special case of DM 7.13. If the two
bonds (1) each have one cash flow, and (2) have the same yield (i.e. y1 = y2 ),
then DM 7.13 becomes DM 24.7.

In this problem, the yield is the same for the two bonds. So y1 = y2 .
D1 B1 (y1 ) (1 + y1 ) D1 P1 (y) 3. 716 7 × 0.959 16
N =− =− =− = −0.527 6
D2 B2 (y2 ) (1 + y2 ) D2 P2 (y) 6. 433 2 × 1. 050 3

So we need to buy −0.527 6 (i.e. sell 0.527 6) unit of Bond 2. At time zero,
we

• sell 0.527 6 unit of Bond 2, receiving 0.527 6 × 1. 050 3 = 0.554 14


• buy one Bond 1, paying 0.959 16
• borrow the difference 0.959 16 − 0.554 14 = 0.405 02 from a bank

0.959 16 − 0.527 6 × 1. 050 3 = 0.405 021 72


Our net position is zero.

If at the end of the day, the yield is 6.25%:


(1) Back back 0.527 6 unit of Bond 2, which has (8 − 1/365) year to maturity.
Now we are¡ standing
¡ at t = 1/365. Bond 2’s price now is: ¢ ¢
P2 = 0.07 e−0.0625×1 + e−0.0575×2 + e−0.0625×3 + ... + e−0.0625×8 + e−0.0625×8 e0.0625/365
= 1. 034 39

(2) Sell Bond 1, which has (4 − 1/365) year to maturity


The price
¡ is¡ ¢ ¢
P1 = 0.05 e−0.0.0625×1 + e−0.0625×2 + e−0.0625×3 + e−0.0625×4 + e−0.0625×4 e0.0625/365 =
0.953 48

(3) Repay the bank: 0.405 02e0.06/365 = 0.405 09


The net profit at the end of Day 1 is: 0.953 48−0.527 6×1. 034 39−0.405 09 =
0.002 65

If at the end of the day, the yield is 5.75%:


(1) Back back 0.527 6 unit of Bond 2, which has (8 − 1/365) year to maturity.
Now we are standing
£ at t = 1/365. Bond 2’s price now is: ¤
P2 = 0.07 e−0.0575×(1−1/365) + e−0.0575×(2−1/365) + ... + e−0.0575×(8−1/365) +
e−0.0575×(8−1/365)
¡ ¡ ¢ ¢
= 0.07 e−0.0575×1 + e−0.0575×2 + e−0.0575×3 + ... + e−0.0575×8 + e−0.0575×8 e0.0575/365
= 1. 067 54

(2) Sell Bond 1, which has (4 − 1/365) year to maturity


The price is

www.myactuaryexam.com c
°Yufeng Guo 174
CHAPTER 24. INTEREST RATE MODELS

¡ ¡ ¢ ¢
P1 = 0.05 e−0.0575×1 + e−0.0575×2 + e−0.0575×3 + e−0.0575×4 + e−0.0575×4 e0.0575/365 =
0.968 27

(3) Repay the bank: 0.405 02e0.06/365 = 0.405 09

The net profit at the end of Day 1 is: 0.968 27−0.527 6×1. 067 54−0.405 09 =
−0.000 05

If we reserve our position, we can have 0.000 05 at the end of Day 1. Refer
to Problem 24.4 to see how to reverse our position.
Summary: If the yield moves up to 6.25% in Day 1, we can make 0.002 65
profit; if the yield moves down to 5.75% in Day 1, we can make 0.000 05 profit.
Once again, the assumption of the flat yield curve leads to arbitrage.

Problem 24.6.
Vasicek model:
a. Zero risk premium means that α (r, t, T ) = r.

α (r, t, T ) − r
Hence φ (r, t) = =0
q (r, t, T )
σ2 0.12
DM 24.26: r = b − 0.5 2 = 0.1 − 0.5 × = −0.025
α 0.22
2-year bond:
1 − e−α(T −t) 1 − e−0.2(2)
B (T − t = 2) = aT −t|α = = = 1. 648 4
α 0.2
aT −t|α is a T − t year continuous annuity with the force of interest α.
2 2 2 2
A (T − t = 2) = er[B(2)−2]−B (2)σ /4α = e−0.025(1. 648 4−2)−1. 648 4 ×0.1 /(4×0.2) =
0.975 14
The 2-year bond is worth:
P (0, 2) = A (2) e−B(2)r = 0.975 14e−1. 648 4×0.05 = 0.897 99
The delta is:
d d
P (0, 2) = A (2) e−B(2)r = −B (2) A (2) e−B(2)r = −B (2) P (0, 2) =
dr dr
−1. 648 4 × 0.897 99 = −1. 480 2
The gamma is:
d2 d
2
P (0, 2) = −B (2) P (0, 2) = B 2 (2) P (0, 2) = 1. 648 42 × 0.897 99 = 2.
dr dr
44

10-year bond:
1 − e−0.2×10
B (10) = a10|0.2 = = 4. 323 3
0.2 2
2 2 2
A (10) = er[B(10)−10]−B (10)σ /4α = e−0.025(4. 323 3−10)−4. 323 3 ×0.1 /(4×0.2) =
0.912 36

www.myactuaryexam.com c
°Yufeng Guo 175
CHAPTER 24. INTEREST RATE MODELS

The 10-year bond is worth:


P (0, 10) = A (10) e−B(10)r = 0.912 36e−4. 323 3×0.05 = 0.735
The delta is:
d
P (0, 10) = −B (10) P (0, 10) = −4. 323 3 × 0.735 = −3. 177 6
dr
The gamma is:
d2
P (0, 10) = B 2 (10) P (0, 10) = 4. 323 32 × 0.735 = 13. 738
dr2
The delta is:
d d
P (0, 2) = A (2) e−B(2)r = −B (2) A (2) e−B(2)r = −B (2) P (0, 2) =
dr dr
−1. 648 4 × 0.897 99 = −1. 480 2
The gamma is:
d2 d
2
P (0, 2) = −B (2) P (0, 2) = B 2 (2) P (0, 2) = 1. 648 42 × 0.897 99 = 2.
dr dr
44

b. If we buy a 2-year bond, to duration hedge our risk, we need to buy N


unit of 10-year bond

2P (0, 2) 2 × 0.897 99
N =− =− = −0.244 35
10P (0, 10) 10 × 0.735
So we need to sell 0.244 35 unit of 10-year bond.

At t = 0, we

• buy a 2-year bond, paying P (0, 2) = 0.897 99


• sell 0.244 35 unit of 10-year bond, receiving 0.244 35 × 0.735 = 0.179 60
• borrow 0.897 99 − 0.179 60 = 0.718 39 at 5%
• Our net position is zero.

p
The onepstandard deviation of the interest rate under Vasicek is r±σ 1/365 =
0.05 ± 0.1 1/365 p
ru = 0.05 + 0.1p 1/365 = 0.055
rd = 0.05 − 0.1 1/365 = 0.045
p
Under ru = 0.05 + 0.1 1/365 = 0.055
2 − 1/365 year bond:
1 − e−α(T −t) 1 − e−0.2(2−1/365)
B (2 − 1/365) = = = 1. 647
α 0.2 2 2
A (2 − 1/365) = e−0.025(1. 646 7−2+1/365)−1. 647 ×0.1 /(4×0.2) = 0.975 2

The 2 − 1/365-year bond is worth:


P (0, 2 − 1/365) = 0.975 2e−1. 6467×0.055 = 0.890 8

www.myactuaryexam.com c
°Yufeng Guo 176
CHAPTER 24. INTEREST RATE MODELS

10 − 1/365-year bond:
1 − e−0.2×(10−1/365)
B (10 − 1/365) = = 4. 323 0
0.2
r[B(10−1/365)]−B 2 (10−1/365)σ 2 /4α 2 2
A (10 − 1/365) = e = e−0.025(4. 323 −10+1/365)−4. 323 ×0.1 /(4×0.2) =
0.912 34
The 10 − 1/365-year bond is worth:
P (0, 10 − 1/365) = A (10 − 1/365) e−B(10−1/365)r = 0.912 34e−4. 323 ×0.055 =
0.719 3

At the end of the day, we close our position:

• sell a 2 − 1/365-year bond, receiving 0.890 8

• buy 0.244 35 unit of 10−1/365-year bond, paying 0.244 35 × 0.719 3 =


0.175 8

• pay back the loan, paying 0.718 39e0.05/365 = 0.718 5

• our net position is: 0.890 8 − 0.175 8 − 0.718 5 = −0.003 5

• So we lose 0.003 5

p
Under rd = 0.05 − 0.1 1/365 = 0.045
2 − 1/365 year bond:
1 − e−α(T −t) 1 − e−0.2(2−1/365)
B (2 − 1/365) = = = 1. 647
α 0.2 2 2
A (2 − 1/365) = e−0.025(1. 646 7−2+1/365)−1. 647 ×0.1 /(4×0.2) = 0.975 2

The 2 − 1/365-year bond is worth:


P (0, 2 − 1/365) = 0.975 2e−1. 6467×0.045 = 0.905 6

10 − 1/365-year bond:
1 − e−0.2×(10−1/365)
B (10 − 1/365) = = 4. 323 0
0.2 2 2 2 2
A (10 − 1/365) = er[B(10−1/365)]−B (10−1/365)σ /4α = e−0.025(4. 323 −10+1/365)−4. 323 ×0.1 /(4×0.2) =
0.912 34
The 10 − 1/365-year bond is worth:
P (0, 10 − 1/365) = A (10 − 1/365) e−B(10−1/365)r = 0.912 34e−4. 323 ×0.045 =
0.751 1

At the end of the day, we close our position:

• sell a 2 − 1/365 year bond, receiving 0.905 6

• buy 0.244 35 unit of 10 − 1/365 year bond, paying 0.244 35 × 0.751 1 =


0.183 5

www.myactuaryexam.com c
°Yufeng Guo 177
CHAPTER 24. INTEREST RATE MODELS

• pay back the loan, paying 0.718 39e0.05/365 = 0.718 5


• our net position is: 0.905 6 − 0.183 5 − 0.718 5 = 0.003 6
• So we gain 0.003 6

Delta hedge. At t = 0, we buy a 2-year bond and buy N units of 10-year


bond. We already calculated the following:

• The delta of the 2-year bond is −1. 480 2


• The delta of the 10-year bond is −3. 177 6

The total delta of the portfolio is: −1. 480 2 + N (−3. 177 6).
To delta hedge, set −1. 480 2 + N (−3. 177 6) = 0 → N = −0.465 8

So at t = 0, we

• buy a 2-year bond, paying P (0, 2) = 0.897 99


• sell 0.465 8 unit of 10-year bond, receiving 0.465 8 × 0.735 = 0.342 36
• borrow 0.897 99 − 0.342 36 = 0.555 63 at 5%
• Our net position is zero.
p
Under ru = 0.05 + 0.1 1/365 = 0.055
The 2 − 1/365-year bond is worth: P (0, 2 − 1/365) = 0.890 8
The 10 − 1/365-year bond is worth: P (0, 10 − 1/365) = 0.719 3

At the end of the day, we close our position:

• sell a 2 − 1/365-year bond, receiving 0.890 8


• buy 0.465 8 unit of 10−1/365-year bond, paying 0.465 8×0.719 3 = 0.335 05
• pay back the loan, paying 0.555 63e0.05/365 = 0.555 71
• our net position is: 0.890 8 − 0.335 05 − 0.555 71 = 0.000 04
• So we gain 0.000 04

p
Under rd = 0.05 − 0.1 1/365 = 0.045
The 2 − 1/365-year bond is worth: P (0, 2 − 1/365) = 0.905 6
The 10 − 1/365-year bond is worth: P (0, 10 − 1/365) = 0.751 1

At the end of the day, we close our position:

• sell a 2 − 1/365 year bond, receiving 0.905 6

www.myactuaryexam.com c
°Yufeng Guo 178
CHAPTER 24. INTEREST RATE MODELS

• buy 0.465 8 unit of 10−1/365 year bond, paying 0.465 8×0.751 1 = 0.349 86
• pay back the loan, paying 0.555 63e0.05/365 = 0.555 71

• our net position is: 0.905 6 − 0.349 86 − 0.555 71 = 0.000 03

• So we gain 0.000 03

CIR:
a. Zero risk premium means that α (r, t, T ) = r.
α (r, t, T ) − r
Hence φ (r, t) = =0
q (r, t, T )

DM 24.28: φ (r, t) = φ r/σ. Hence φ = 0

DM q24.29: q
γ = (a + φ)2 + 2σ 2 = (0.2 + 0)2 + 2 × 0.447212 = 0.663 32
a + φ + γ = 0.2 + 0 + 0.663 32 = 0.863 32

2-year bond:
" # 2ab
2γe(a+φ+γ )(T −t)/2 σ2
A (T − t = 2) = ¡ ¢¡ ¢
a + φ + γ eγ(T −t) − 1 + 2γ

2 × 0.2 × 0.1
∙ 0.863 32×2/2
¸
2 × 0.663 32e 0.447212
= = 0.967 18
0.863 32 (e0.663 32×2 − 1) + 2 × 0.663 32
¡ ¢
2 eγ(T −t) − 1
B (T − t = 2) = ¡ ¢¡ ¢
a + φ + γ eγ(T −t) − 1 + 2γ
¡ ¢
2 e0.663 32×2 − 1
= = 1. 489 72
0.863 32 (e0.663 32×2 − 1) + 2 × 0.663 32

P (0, 2) = A (2) e−B(2)r = 0.967 18e−1. 489 72×0.05 = 0.897 76


The delta is:
d d
P (0, 2) = A (2) e−B(2)r = −B (2) P (0, 2) = −1. 489 72 × 0.897 76 =
dr dr
−1. 337 41
The gamma is:
d2 d
2
P (0, 2) = −B (2) P (0, 2) = B 2 (2) P (0, 2) = 1. 489 722 × 0.897 76 =
dr dr
1. 992 37

www.myactuaryexam.com c
°Yufeng Guo 179
CHAPTER 24. INTEREST RATE MODELS

10-year bond:
2 × 0.2 × 0.1
∙ 0.863 32×10/2
¸
2 × 0.663 32e 0.447212
A (T − t = 10) = =
0.863 32 (e0.663 32×10 − 1) + 2 × 0.663 32
0.685 54
¡ ¢
2 e0.663 32×10 − 1
B (T − t = 10) = = 2. 311 96
0.863 32 (e0.663 32×10 − 1) + 2 × 0.663 32

P (0, 10) = A (10) e−B(10)r = 0.685 54e−2. 311 96×0.05 = 0.610 70

The delta is:


d
P (0, 10) = −B (10) P (0, 10) = −2. 311 96 × 0.610 70 = −1. 411 91
dr
The gamma is:
d2
P (0, 10) = B 2 (10) P (0, 10) = 2. 311 962 × 0.610 70 = 3. 264 29
dr2
b. If we buy a 2-year bond, to duration hedge our risk, we need to buy N
units of 10 year bond.

2P (0, 2) 2 × 0.897 76
N =− =− = −0.294 01
10P (0, 10) 10 × 0.610 70
So we need to sell 0.294 01 unit of 10-year bond.

At t = 0, we
• buy a 2-year bond, paying P (0, 2) = 0.897 76
• sell 0.294 01 unit of 10-year bond, receiving 0.294 01 × 0.610 70 = 0.179 55
• borrow 0.897 76 − 0.179 55 = 0.718 21 at 5%
• Our net position is zero.


Notice that under CIR, dr = a (b − r) dt√+p σ rdz. The one standard√ devia-
tion
p of the interest rate under CIR is r ± σ r 1/365 = 0.05 ± 0.44721 0.05 ×
1/365 p

0.44721 0.05 × 1/365 = 0.005 234 2
ru = 0.05 + 0.005 234 2 = 0.055 23
rd = 0.05 − 0.005 234 2 = 0.044 77

Under ru = 0.055 23
The 2 − 1/365-year bond is worth:

" # 2 × 0.2 × 0.1


2 × 0.663 32e0.863 32×(2−1/365)/2 0.447212
A (T − t = 2 − 1/365) = ¡ ¢ =
0.863 32 e0.663 32×(2−1/365) − 1 + 2 × 0.663 32
0.967 26

www.myactuaryexam.com c
°Yufeng Guo 180
CHAPTER 24. INTEREST RATE MODELS

¡ ¢
2 e0.663 32×(2−1/365) − 1
B (T − t = 2 − 1/365) = ¡ ¢ =
0.863 32 e0.663 32×(2−1/365) − 1 + 2 × 0.663 32
1. 488 40

→ P (0, 2 − 1/365) = A (2 − 1/365) e−B(2−1/365)r = 0.967 26e−1. 488 40×0.055 23 =


0.890 93

The 10 − 1/365-year bond is worth:

" # 2 × 0.2 × 0.1


2 × 0.663 32e 0.863 32×(10−1/365)/2 0.447212
A (T − t = 10 − 1/365) = ¡ ¢ =
0.863 32 e0.663 32×(10−1/365) − 1 + 2 × 0.663 32
0.685 63 ¡ ¢
2 e0.663 32×(10−1/365) − 1
B (T − t = 10 − 1/365) = ¡ ¢ =
0.863 32 e0.663 32×(10−1/365) − 1 + 2 × 0.663 32
2. 311 95

→ P (0, 10 − 1/365) = 0.685 63e−2. 311 95×0.055 23 = 0.603 44

At the end of the day, we close our position:

• sell a 2 − 1/365-year bond, receiving 0.890 93


• buy 0.294 01 unit of 10 − 1/365-year bond, paying 0.294 01 × 0.603 44 =
0.177 42
• pay back the loan, paying 0.718 21e0.05/365 = 0.718 31
• our net position is: 0.890 93 − 0.177 42 − 0.718 31 = −0.004 8
• So we lose 0.004 8

Under rd = 0.044 77

P (0, 2 − 1/365) = A (2 − 1/365) e−B(2−1/365)r = 0.967 26e−1. 488 40×0.044 77 =


0.904 91
P (0, 10 − 1/365) = 0.685 63e−2. 311 95×0.044 77 = 0.618 21

At the end of the day, we close our position:

• sell a 2 − 1/365-year bond, receiving 0.904 91


• buy 0.294 01 unit of 10 − 1/365-year bond, paying 0.294 01 × 0.618 21 =
0.181 76
• pay back the loan, paying 0.718 21e0.05/365 = 0.718 31

www.myactuaryexam.com c
°Yufeng Guo 181
CHAPTER 24. INTEREST RATE MODELS

• our net position is: 0.904 91 − 0.181 76 − 0.718 31 = 0.004 84

• So we gain 0.004 84

Delta hedge:

−1. 337 41 + N (−1. 411 91) = 0 N = −0.947 23


So we need to sell 0.947 23 unit of the 10-year bond.

At t = 0, we

• buy a 2-year bond, paying P (0, 2) = 0.897 76

• sell 0.947 23 unit of 10-year bond, receiving 0.947 23 × 0.610 70 = 0.578 47

• borrow 0.897 76 − 0.578 47 = 0.319 29 at 5%

• Our net position is zero.

One day later, under ru = 0.055 23


P (0, 2 − 1/365) = A (2 − 1/365) e−B(2−1/365)r = 0.967 26e−1. 488 40×0.055 23 =
0.890 93
P (0, 10 − 1/365) = 0.685 63e−2. 311 95×0.055 23 = 0.603 44

At the end of the day, we close our position:

• sell a 2 − 1/365-year bond, receiving 0.890 93

• buy 0.947 23 unit of 10 − 1/365-year bond, paying 0.947 23 × 0.603 44 =


0.571 60

• pay back the loan, paying 0.319 29e0.05/365 = 0.319 33

• our net position is: 0.890 93 − 0.571 60 − 0.319 33 = 0

• So we gain 0

Under rd = 0.044 77
P (0, 2 − 1/365) = 0.967 26e−1. 488 40×0.044 77 = 0.904 91

P (0, 10 − 1/365) = 0.685 63e−2. 311 95×0.044 77 = 0.618 21

At the end of the day, we close our position:

• sell a 2 − 1/365-year bond, receiving 0.904 91

www.myactuaryexam.com c
°Yufeng Guo 182
CHAPTER 24. INTEREST RATE MODELS

• buy 0.947 23 unit of 10 − 1/365-year bond, paying 0.947 23 × 0.618 21 =


0.585 59

• pay back the loan, paying 0.319 29e0.05/365 = 0.319 33

• our net position is: 0.904 91 − 0.585 59 − 0.319 33 ≈ 0

• So we gain 0

Problem 24.7.
t=0 t=1 t=2 t=3
0.16
0.04
0.12
0.12
0.12
0.10
0.08
0.10
0.12
0.10
0.08
0.08
0.08
0.06
0.04
Price of 1-Yr bond: P (0, 1) = e−0.1 = 0.904 84
The yield for 1-Yr bond is 0.1.

Price of 2-Yr bond:


Path Prob Price
0→u 0.5 e−0.12 e−0.1
0→d 0.5 e−0.08 e−0.1

P (0, 2) = 0.5e−0.12 e−0.1 + 0.5e−0.08 e−0.1 = 0.818 89


The yield for 2-Yr bond is y
ln 0.818 89
e−2y = 0.818 89 y=− = 9.990 3%
2

Price of 3-Yr bond:


Path Prob Price
0 → u → uu 0.25 e−0.14 e−0.12 e−0.1
0 → u → ud 0.25 e−0.1 e−0.12 e−0.1
0 → d → du 0.25 e−0.1 e−0.08 e−0.1
0 → d → dd 0.25 e−0.06 e−0.08 e−0.1

www.myactuaryexam.com c
°Yufeng Guo 183
CHAPTER 24. INTEREST RATE MODELS

¡ ¢
P (0, 5) = 0.25 e−0.14 e−0.12 e−0.1 + e−0.1 e−0.12 e−0.1 + e−0.1 e−0.08 e−0.1 + e−0.06 e−0.08 e−0.1 =
0.741 56
1
Yield for the 3-Yr bond: − ln 0.741 56 = 9. 966 6%
3
Price of 4-Yr bond:
Path Prob Price
0 → u → uu → uuu 1/8 e−0.16 e−0.14 e−0.12 e−0.1 = 0.594 52
0 → u → uu → uud 1/8 e−0.12 e−0.14 e−0.12 e−0.1 = 0.618 78
0 → u → ud → udu 1/8 e−0.12 e−0.10 e−0.12 e−0.1 = 0.644 04
0 → u → ud → udd 1/8 e−0.08 e−0.10 e−0.12 e−0.1 = 0.670 32
0 → d → du → duu 1/8 e−0.12 e−0.10 e−0.08 e−0.1 = 0.670 32
0 → d → du → dud 1/8 e−0.08 e−0.10 e−0.08 e−0.1 = 0.697 68
0 → d → dd → ddu 1/8 e−0.08 e−0.06 e−0.08 e−0.1 = 0.726 15
0 → d → dd → ddd 1/8 e−0.04 e−0.06 e−0.08 e−0.1 = 0.755 78

0.594 52 + 0.618 78 + 0.644 04 + 0.670 32 + 0.670 32 + 0.697 68 + 0.726 15 + 0.755 78


P (0, 4) = =
8
0.672 20
1
Yield for the 4-Yr bond: − ln 0.672 20 = 9. 930 0%
4
Yields decline with maturity. This is explained in DM page 796. The average
of the exponential interest rates is less than the exponentiated average.

Problem 24.8.
Instead of working path by path, we can work backwards.

At t = 3, the bond price is:


Vuuu = 0.8331 Vuud = 0.8644 Vddu = 0.8906 Vddd = 0.9123
At t = 2, the bond price is:
0.8331 + 0.8644
Vuu = 0.8321 × = 0.706 24
2
0.8644 + 0.8906
Vud = Vdu = 0.8798 × = 0.772 02
2
0.8906 + 0.9123
Vdd = 0.9153 × = 0.825 10
2
At t = 1, the bond price is:
0.706 24 + 0.772 02
Vu = 0.8832 × = 0.652 80
2
0.772 02 + 0.825 10
Vd = 0.9023 × = 0.720 54
2
At t = 0, the bond price is:
0.652 80 + 0.720 54
Vu = 0.9091 × = 0.624 25 = 0.6243
2

www.myactuaryexam.com c
°Yufeng Guo 184
CHAPTER 24. INTEREST RATE MODELS

Problem 24.9.
Next year, the bond price is either Vu = 0.652 80 with a yield of 0.652 80−1/3 −
1 = 0.152 76 or Vd = 0.720 54 with a yield of 0.720 54−1/3 − 1 = 0.115 44.
0.152 76
Using DM 24.48, the yield volatility is 0.5 ln = 0.140 06 = 0.14
0.115 44

Problem 24.10.
See DM page 805.

Problem 24.11.
P (0, 3) 0.7118
DM Page 806 and 807 explain that rA = −1 = −1 = 0.140 16.
P (0, 4) 0.6243
Since the interest rate cap applies during the interval [t = 3, t = 4], the fair cap
rate must be the implied forward rate rA = 0.140 16 during [t = 3, t = 4] . We
can verify that rA = 0.140 16 using the binomial tree.
Let r (3, 4) represent the actual interest rate during the interval [t = 3, t = 4].
The reference rate rA is the 1-year forward rate 3 years hence. So rA is also
the rate during [t = 3, t = 4]. This is the difference between r (3, 4) and rA .
r (3, 4) is the actual interest rate observed in the market during Year 3; rA is
rate agreed upon at t = 0 that applies to Year 3.
Set the notional principal to $100. At t = 4, the payoff is 100 [r (3, 4) − rA ]
= 100r (3, 4) − 100rA . We need to find rA such that the PV of the payoff is
zero.
First, we calculate PV of 100r (3, 4). At t = 4, 100r (3, 4) has 4 possible
values: 20.03,15.68,12.28, and 9.62. Discounting these 4 values to t = 3, we get
the 4 values:
100 × 0.2003 100 × 0.1568
= 16. 687 = 13. 555
1 + 0.2003 1 + 0.1568

100 × 0.1228 100 × 0.0962


= 10. 937 = 8. 775 8
1 + 0.1228 1 + 0.0962

We have 3 values at t = 2 (using 0.5 as the risk-neutral probability of up or


down) µ ¶
16. 687 13. 555
Vuu = 0.5 + = 12. 583
1 + 0.2017 1 + 0.2017
µ ¶
13. 555 10. 937
Vud = 0.5 + = 10. 774
1 + 0.1366 1 + 0.1366
µ ¶
10. 937 8. 775 8
Vdd = 0.5 + = 9. 021 9
1 + 0.0925 1 + 0.0925

The value at t = 1:

www.myactuaryexam.com c
°Yufeng Guo 185
CHAPTER 24. INTEREST RATE MODELS

µ ¶
12. 583 10. 774
Vu = 0.5 + = 10. 315
1 + 0.1322 1 + 0.1322
µ ¶
10. 774 9. 021 9
Vd = 0.5 + = 8. 931 6
1 + 0.1082 1 + 0.1082

The value
µ at t = 0: ¶
10. 315 8. 931 6
V = 0.5 + = 8. 748 5
1 + 0.1 1 + 0.1

So PV of 100r (3, 4) is 8. 748 5.


PV of 100rA is 100rA × P (0, 4) = 100rA × 0.6243
P (0, 4) = 0.6243 is from DM Table 24.2.

100rA × 0.6243 = 8. 748 5 rA = 0.140 13

Problem 24.12.

I just solve for Tree #1. Once you understand the logic, you can do Tree
#2.
1-Yr bond price:

1
P (0, 1) = = 0.925 93
1.08

2-Yr bond price:


1 1
Vd = Vu =
1 + 0.07676 1 + 0.10363
µ ¶
Vu + Vd 1 1 1
V = 0.5 × = 0.5 × + = 0.849 45
1.08 1.08 1 + 0.07676 1 + 0.10363

3-Yr bond price:


1 1 1
Vdd = Vud = Vuu =
1 + 0.08170 1 + 0.10635 1 + 0.13843
µ ¶
1 1 1
Vd = 0.5 × + = 0.849
1 + 0.07676 µ 1 + 0.08170 1 + 0.10635 ¶
1 1 1
Vu = 0.5 × + = 0.807 46
1 + 0.10363 1 + 0.10635 1 + 0.13843
1
V = 0.5 × (0.849 + 0.807 46) = 0.766 88
1.08

4-Yr bond price:


Value at t = 3
1 1 1 1
1 + 0.07943 1 + 0.09953 1 + 0.12473 1 + 0.15630

www.myactuaryexam.com c
°Yufeng Guo 186
CHAPTER 24. INTEREST RATE MODELS

Value at t = 2
µ ¶
1 1 1
Vdd = 0.5 × + = 0.848 62
1 + 0.08170 µ 1 + 0.07943 1 + 0.09953 ¶
1 1 1
Vud = 0.5 × + = 0.812 84
1 + 0.10635 µ 1 + 0.09953 1 + 0.12473 ¶
1 1 1
Vuu = 0.5 × + = 0.770 33
1 + 0.13843 1 + 0.12473 1 + 0.15630

Value at t = 1
1
Vd = 0.5 × (0.848 62 + 0.812 84) = 0.771 51
1 + 0.07676
1
Vu = 0.5 × (0.812 84 + 0.770 33) = 0.717 26
1 + 0.10363

Value at t = 0

1
V = 0.5 × (0.771 51 + 0.717 26) = 0.689 25
1.08

5-Yr bond price:


value at t = 4
1 1 1
1 + 0.07552 1 + 0.09084 1 + 0.10927
1 1
1 + 013143 1 + 0.15809
value at t = 3 µ ¶
1 1 1
0.5 × + = 0.855 32
1 + 0.07943 µ 1 + 0.07552 1 + 0.09084 ¶
1 1 1
0.5 × + = 0.826 82
1 + 0.09953 µ 1 + 0.09084 1 + 0.10927 ¶
1 1 1
0.5 × + = 0.793 67
1 + 0.12473 µ 1 + 0.10927 1 + 0.13143 ¶
1 1 1
0.5 × + = 0.755 57
1 + 0.15630 1 + 0.13143 1 + 0.15809

value at t = 2
1
Vdd = 0.5 × (0.855 32 + 0.826 82) = 0.777 54
1 + 0.08170
1
Vud = 0.5 × (0.826 82 + 0.793 67) = 0.732 36
1 + 0.10635
1
Vuu = 0.5 × (0.793 67 + 0.755 57) = 0.680 43
1 + 0.13843

value at t = 1
1
Vd = 0.5 × (0.777 54 + 0.732 36) = 0.701 13
1 + 0.07676
1
Vu = 0.5 × (0.732 36 + 0.680 43) = 0.640 07
1 + 0.10363

www.myactuaryexam.com c
°Yufeng Guo 187
CHAPTER 24. INTEREST RATE MODELS

Value at t = 0
1
V = 0.5 × (0.701 13 + 0.640 07) = 0.620 93
1.08

Problem 24.13.
I’m going to solve for only Tree #1.

DM Page 799 states that the volatility in Year 1 is the standard deviation
of the natural log of the yield for that bond 1 year hence. So the volatility in
Yr 1 for an n-year bond is the standard deviation of the natural log of the yield
of an (n − 1)-year bond issued at t = 1.
Yr-1 yield for 1-yr bond is unknown.

Yr-1 yield for 2-yr bond . The up yield of an 2 − 1 = 1 year bond issued at
t = 1 is ru = 0.10362; the down yield is 0.07676.Using DM 24.48, we get:
ru 0.10362
0.5 ln = 0.5 ln = 0.150 02
rd 0.07676

Yr-1 yield for a 3-yr bond. We first calculate the price of a 2-yr bond issued
at t = 1.

From the previous problem


µ regarding the price of the¶ 3-year bond, we know
1 1 1
Vd = 0.5 × + = 0.849
1 + 0.07676 µ 1 + 0.08170 1 + 0.10635 ¶
1 1 1
Vu = 0.5 × + = 0.807 46
1 + 0.10363 1 + 0.10635 1 + 0.13843
−1/2
ru = 0.807 46 − 1 = 0.112 86
rd = 0.849−1/2 − 1 = 0.08529 1

ru 0.112 85
Volatility: 0.5 ln = 0.5 ln = 0.140 00
rd 0.08529 1

Yr-1 yield for a 4-yr bond. We first calculate the price of a 3-yr bond issued
at t = 1.
From the previous problem about the price of a 4-yr bond,
1
Vd = 0.5 × (0.848 62 + 0.812 84) = 0.771 51
1 + 0.07676
−1/3
rd = 0.771 51 − 1 = 9. 031 7 × 10−2
1
Vu = 0.5 × (0.812 84 + 0.770 33) = 0.717 26
1 + 0.10363
rd = 0.717 26−1/3 − 1 = 0.117 14

ru 0.117 14
Volatility: 0.5 ln = 0.5 ln = 0.130 02
rd 9. 031 7 × 10−2
Yr-1 yield for a 5-yr bond. We first calculate the price of a 4-yr bond issued
at t = 1.
From the previous problem about the price of a 5-yr bond,

www.myactuaryexam.com c
°Yufeng Guo 188
CHAPTER 24. INTEREST RATE MODELS

1
Vd = 0.5 × (0.777 54 + 0.732 36) = 0.701 13
1 + 0.07676
−1/4
rd = 0.701 13 − 1 = 9. 282 4 × 10−2
1
Vu = 0.5 × (0.732 36 + 0.680 43) = 0.640 07
1 + 0.10363
ru = 0.640 07−1/4 − 1 = 0.118
ru 0.118
Volatility: 0.5 ln = 0.5 ln = 0.119 99 = 0.12
rd 9. 282 4 × 10−2
Don’t worry about the question "Can you unambiguously say that rates in
one tree are more volatile than the other?"

Problem 24.14.
Skip. This problem is not worth your time.

Problem 24.15.
I’ll solve for only Tree #1.
time 0 1 2 3 4
0.15809
0.15630
0.13843 0.13143
0.10362 0.12473
0.08 0.10635 0.10927
0.07676 0.09953
0.08170 0.09084
0.07943
0.07552

1
Set the notional amount to $1. The payoff at each node is max (0, r − 0.105)
1+r
Payoff:
time 0 1 2 3 4
0.15809 − 0.105
1 + 0.15809
0.15630 − 0.105
1 + 0.15630
0.13843 − 0.105 0.13143 − 0.105
1 + 0.13843 1 + 0.13143
0.12473 − 0.105
0
1 + 0.12473
0.10635 − 0.105 0.10927 − 0.105
0
1 + 0.10635 1 + 0.10927
0 0
0 0
0
0

www.myactuaryexam.com c
°Yufeng Guo 189
CHAPTER 24. INTEREST RATE MODELS

µ ¶
0.15630 − 0.105 1 0.15809 − 0.105 0.13143 − 0.105
Vuuu = + 0.5 × + 0.5 × =
1 + 0.15630 1 + 0.15630 1 + 0.15809 1 + 0.13143
7. 428 977 9 × 10−2
µ ¶
0.12473 − 0.105 1 0.13143 − 0.105 0.10927 − 0.105
Vuud = + 0.5 × + 0.5 × =
1 + 0.12473 1 + 0.12473 1 + 0.13143 1 + 0.10927
2. 963 786 7 × 10−2
µ ¶
1 0.10927 − 0.105
Vdud = 0 + 0.5 × + 0.5 × 0 = 1. 750 465 4 ×
1 + 0.09953 1 + 0.10927
10−3
0.13843 − 0.105 1 ¡ ¢
Vuu = + 0.5 × 7. 428 977 9 × 10−2 + 0.5 × 2. 963 786 7 × 10−2 =
1 + 0.13843 1 + 0.13843
7. 501 016 6 × 10−2

0.10635 − 0.105 1 ¡ ¢
Vud = + 0.5 × 2. 963 786 7 × 10−2 + 0.5 × 1. 750 465 4 × 10−3 =
1 + 0.10635 1 + 0.10635
1. 540 576 3 × 10−2

1 ¡ ¢
Vdd = 0.5 × 1. 750 465 4 × 10−3 + 0.5 × 0 = 8. 091 270 2×10−4
1 + 0.08170
1 ¡ ¢
Vu = 0.5 × 7. 501 016 6 × 10−2 + 0.5 × 1. 540 576 3 × 10−2 =
1 + 0.10362
4. 096 334 3 × 10−2

1 ¡ ¢
Vd = 0.5 × 1. 540 576 3 × 10−2 + 0.5 × 8. 091 270 2 × 10−4 =
1 + 0.07676
7. 529 482 0 × 10−3

1 ¡ ¢
V = 0.5 × 4. 096 334 3 × 10−2 + 0.5 × 7. 529 482 0 × 10−3 = 2.
1 + 0.08
245 038 2 × 10−2

If the notional amount is $1, the interest cap is worth 2. 245 038 2 × 10−2 at
t = 0. Since the notional amount is 250 million, the interest cap is worth at
t=0

250 × 2. 245 038 2 × 10−2 = 5. 612 595 5 (million)

www.myactuaryexam.com c
°Yufeng Guo 190

You might also like